You are on page 1of 384

CHUYN :

PHNG TRNH - H PHNG TRNH




















THNG 6/2012

Din n MATHSCOPE

Niels Henrik Abel
(1802-1829)
Gerolamo Cardano
(1501-1576)
variste Galois
(1811-1832)
Din n MATHSCOPE
PHNG TRNH
H PHNG TRNH
Ch bin: Nguyn Anh Huy
26 - 7 - 2012
Mc lc
Li ni u . . . . . . . . . . . . . . . . . . . . . . . . . . . . . . . . . . . . . . . 6
Cc thnh vin tham gia chuyn . . . . . . . . . . . . . . . . . . . . . . . . 8
1 I CNG V PHNG TRNH HU T 10
Phng trnh bc ba . . . . . . . . . . . . . . . . . . . . . . . . . . . . . . . . . . . 10
Phng trnh bc bn . . . . . . . . . . . . . . . . . . . . . . . . . . . . . . . . . . 16
Phng trnh dng phn thc . . . . . . . . . . . . . . . . . . . . . . . . . . . . . . 23
Xy dng phng trnh hu t . . . . . . . . . . . . . . . . . . . . . . . . . . . . . . 27
Mt s phng trnh bc cao . . . . . . . . . . . . . . . . . . . . . . . . . . . . . . 29
2 PHNG TRNH, H PHNG TRNH C THAM S 32
Phng php s dng o hm . . . . . . . . . . . . . . . . . . . . . . . . . . . . . 32
Phng php dng nh l Lagrange - Rolle . . . . . . . . . . . . . . . . . . . . . . 42
Phng php dng iu kin cn v . . . . . . . . . . . . . . . . . . . . . . . . . 46
Phng php ng dng hnh hc gii tch v hnh hc phng . . . . . . . . . . . . . 55
Hnh hc khng gian v vic kho st h phng trnh ba n . . . . . . . . . . . . . 76
Mt s bi phng trnh, h phng trnh c tham s trong cc k thi Olympic . . . 81
3 CC PHNG PHP GII PHNG TRNH 93
Phng php t n ph . . . . . . . . . . . . . . . . . . . . . . . . . . . . . . . . . 93
Mt s cch t n ph c bn . . . . . . . . . . . . . . . . . . . . . . . . . . 93
t n ph a v phng trnh tch . . . . . . . . . . . . . . . . . . . . . . . 94
t n ph a v phng trnh ng cp . . . . . . . . . . . . . . . . . . . . 101
Phng php t n ph khng hon ton . . . . . . . . . . . . . . . . . . . . 103
Phng php s dng h s bt nh . . . . . . . . . . . . . . . . . . . . . . . 108
t n ph a v h phng trnh . . . . . . . . . . . . . . . . . . . . . . . . 109
Phng php lng gic ha . . . . . . . . . . . . . . . . . . . . . . . . . . . . 117
Phng php bin i ng thc . . . . . . . . . . . . . . . . . . . . . . . . . . . . 121
Phng php dng lng lin hp . . . . . . . . . . . . . . . . . . . . . . . . . . . . 124
Phng php dng n iu hm s . . . . . . . . . . . . . . . . . . . . . . . . . . 138
Phng php dng bt ng thc . . . . . . . . . . . . . . . . . . . . . . . . . . . . 146
Mt s bi ton chn lc . . . . . . . . . . . . . . . . . . . . . . . . . . . . . . . . . 154
3
4
4 PHNG TRNH M-LOGARIT 158
L thuyt . . . . . . . . . . . . . . . . . . . . . . . . . . . . . . . . . . . . . . . . . 158
Phng php t n ph . . . . . . . . . . . . . . . . . . . . . . . . . . . . . . . . . 158
Phng php dng n iu hm s . . . . . . . . . . . . . . . . . . . . . . . . . . 166
Phng php bin i ng thc . . . . . . . . . . . . . . . . . . . . . . . . . . . . 170
Bi tp tng hp . . . . . . . . . . . . . . . . . . . . . . . . . . . . . . . . . . . . . 173
5 H PHNG TRNH 177
Cc loi h c bn . . . . . . . . . . . . . . . . . . . . . . . . . . . . . . . . . . . . 177
H phng trnh hon v . . . . . . . . . . . . . . . . . . . . . . . . . . . . . . . . . 184
Phng php t n ph trong gii h phng trnh . . . . . . . . . . . . . . . . . . 206
Phng php bin i ng thc . . . . . . . . . . . . . . . . . . . . . . . . . . . . 213
Phng php dng n iu hm s . . . . . . . . . . . . . . . . . . . . . . . . . . 222
Phng php h s bt nh . . . . . . . . . . . . . . . . . . . . . . . . . . . . . . . 231
K thut t n ph tng - hiu . . . . . . . . . . . . . . . . . . . . . . . . . . . . . 240
Phng php dng bt ng thc . . . . . . . . . . . . . . . . . . . . . . . . . . . . 246
Tng hp cc bi h phng trnh . . . . . . . . . . . . . . . . . . . . . . . . . . . . 258
H phng trnh hu t . . . . . . . . . . . . . . . . . . . . . . . . . . . . . . . 258
H phng trnh v t . . . . . . . . . . . . . . . . . . . . . . . . . . . . . . . . 277
6 SNG TO PHNG TRNH - H PHNG TRNH 297
Xy dng mt s phng trnh c gii bng cch a v h phng trnh . . . . 297
S dng cng thc lng gic sng tc cc phng trnh a thc bc cao . . . . 307
S dng cc hm lng gic hyperbolic . . . . . . . . . . . . . . . . . . . . . . . . . 310
Sng tc mt s phng trnh ng cp i vi hai biu thc . . . . . . . . . . . . . 312
Xy dng phng trnh t cc ng thc . . . . . . . . . . . . . . . . . . . . . . . . 318
Xy dng phng trnh t cc h i xng loi II . . . . . . . . . . . . . . . . . . . 321
Xy dng phng trnh v t da vo tnh n iu ca hm s. . . . . . . . . 324
Xy dng phng trnh v t da vo cc phng trnh lng gic. . . . . . . . 328
S dng cn bc n ca s phc sng to v gii h phng trnh. . . . . . . 331
S dng bt ng thc lng gic trong tam gic . . . . . . . . . . . . . . . . 338
S dng hm ngc sng tc mt s phng trnh, h phng trnh. . . . . 345
Sng tc h phng trnh . . . . . . . . . . . . . . . . . . . . . . . . . . . . . . . . 349
Kinh nghim gii mt s bi h phng trnh . . . . . . . . . . . . . . . . . . . . . 353
7 Ph lc 1: GII TON BNG PHNG TRNH - H PHNG TRNH 362
8 Ph lc 2: PHNG TRNH V CC NH TON HC NI TING 366
Lch s pht trin ca phng trnh . . . . . . . . . . . . . . . . . . . . . . . . . . . 366
C my cch gii phng trnh bc hai? . . . . . . . . . . . . . . . . . . . . . 366
Cuc thch chn ng th gii ton hc . . . . . . . . . . . . . . . . . . . . 368
Nhng vinh quang sau khi qua i . . . . . . . . . . . . . . . . . . . . . . . 372
5
Tu s mt s nh ton hc ni ting . . . . . . . . . . . . . . . . . . . . . . . . . 376
Mt cuc i trn bia m . . . . . . . . . . . . . . . . . . . . . . . . . . . . . 376
Ch v l sch qu hp! . . . . . . . . . . . . . . . . . . . . . . . . . . . . . . . 376
Hai gng mt tr . . . . . . . . . . . . . . . . . . . . . . . . . . . . . . . . . 377
Sng hay cht . . . . . . . . . . . . . . . . . . . . . . . . . . . . . . . . . . . . 378
9 Ti liu tham kho 381
Li ni u
Phng trnh l mt trong nhng phn mn quan trng nht ca i s v c nhng ng
dng rt ln trong cc ngnh khoa hc. Sm c bit n t thi xa xa do nhu cu tnh
ton ca con ngi v ngy cng pht trin theo thi gian, n nay, ch xt ring trong Ton
hc, lnh vc phng trnh c nhng ci tin ng k, c v hnh thc (phng trnh hu t,
phng trnh v t, phng trnh m - logarit) v i tng (phng trnh hm, phng trnh
sai phn, phng trnh o hm ring, . . . )
Cn Vit Nam, phng trnh, t nm lp 8, l mt dng ton quen thuc v c
yu thch bi nhiu bn hc sinh. Ln n bc THPT, vi s h tr ca cc cng c gii tch
v hnh hc, nhng bi ton phng trnh - h phng trnh ngy cng c trau chut, tr
thnh nt p ca Ton hc v mt phn khng th thiu trong cc k thi Hc sinh gii, thi
i hc.
c rt nhiu bi vit v phng trnh - h phng trnh, nhng cha th cp mt
cch ton din v nhng phng php gii v sng to phng trnh. Nhn thy nhu cu c
mt ti liu y v hnh thc v ni dung cho c h chuyn v khng chuyn, Din n
MathScope tin hnh bin son quyn sch Chuyn phng trnh - h phng trnh m
chng ti hn hnh gii thiu n cc thy c gio v cc bn hc sinh.
Quyn sch ny gm 6 chng, vi cc ni dung nh sau:
Chng I: i cng v phng hu t cung cp mt s cch gii tng qut phng
trnh bc ba v bn, ngoi ra cn cp n phng trnh phn thc v nhng cch xy dng
phng trnh hu t.
Chng II: Phng trnh, h phng trnh c tham s cp n cc phng php
gii v bin lun bi ton c tham s ,cng nh mt s bi ton thng gp trong cc k thi
Hc sinh gii.
Chng III: Cc phng php gii phng trnh ch yu tng hp nhng phng
php quen thuc nh bt ng thc, lng lin hp, hm s n iu, . . . vi nhiu bi ton
m rng nhm gip bn c c cch nhn tng quan v phng trnh.
Chng ny khng cp n Phng trnh lng gic, v vn ny c trong chuyn
Lng gic ca Din n.
Chng IV: Phng trnh m logarit a ra mt s dng bi tp ng dng ca hm
s logarit, vi nhiu phng php bin i a dng nh t n ph, dng ng thc, hm n
iu, ...
Chng V: H phng trnh l phn trng tm ca chuyn . Ni dung ca chng
7
bao gm mt s phng php gii h phng trnh v tng hp cc bi h phng trnh hay
trong nhng k thi hc sinh gii trong nc cng nh quc t.
Chng VI: Sng to phng trnh - h phng trnh a ra nhng cch xy dng mt bi
hay v kh t nhng phng trnh n gin bng cc cng c mi nh s phc, hm hyperbolic,
hm n iu, . . .
Ngoi ra cn c hai phn Ph lc cung cp thng tin ng dng phng trnh, h phng
trnh trong gii ton v v lch s pht trin ca phng trnh.
Chng ti xin ng li cm n ti nhng thnh vin ca Din n chung tay xy dng
chuyn . c bit xin chn thnh cm n thy Chu Ngc Hng, thy Nguyn Trng Sn,
anh Hong Minh Qun, anh L Phc L, anh Phan c Minh v h tr v ng gp nhng
kin qu gi cho chuyn , bn Nguyn Trng Thnh v gip ban bin tp kim tra cc
bi vit c mt tuyn tp hon chnh.
Nim hi vng duy nht ca nhng ngi lm chuyn l bn c s tm thy nhiu iu
b ch v tnh yu ton hc thng qua quyn sch ny. Chng ti xin n nhn v hoan nghnh
mi kin xy dng ca bn c chuyn c hon thin hn. Mi gp xin vui lng
chuyn n anhhuy0706@gmail.com
Thnh ph H Ch Minh, ngy 11 thng 7 nm 2012
Thay mt nhm bin son
Nguyn Anh Huy
Cc thnh vin tham gia chuyn
hon thnh c cc ni dung trn, chnh l nh s c gng n lc ca cc thnh vin ca
din n tham gia xy dng chuyn :
Ch bin: Nguyn Anh Huy (10CT THPT chuyn L Hng Phong - TP HCM)
Ph trch chuyn : Nguyn Anh Huy (10CT THPT chuyn L Hng Phong - TP HCM),
Nguyn An Vnh Phc (TN Ph thng Nng khiu- TP HCM)
i cng v phng trnh hu t: Hunh Phc Trng (THPT Nguyn Thng Hin
TP HCM), Phm Tin Kha (10CT THPT chuyn L Hng Phong - TP HCM)
Phng trnh, h phng trnh c tham s: thy Nguyn Trng Sn (THPT Yn M A
Ninh Bnh), V Trng Hi (12A6 THPT Thi Phin - Hi Phng), nh V Bo Chu
(THPT chuyn L Qu n - Vng Tu), Hong B Minh ( 12A6 THPT chuyn Trn
i Ngha - TP HCM), Nguyn Hong Nam (THPT Phc Thin - ng Nai), Ong Th
Phng (11 Ton THPT chuyn Lng Th Vinh - ng Nai)
Phng php t n ph: thy Mai Ngc Thi (THPT Hng Vng - Bnh Phc), thy
Nguyn Anh Tun (THPT L Qung Ch -H Tnh), Trn Tr Quc (11TL8 THPT
Nguyn Hu - Ph Yn), H c Khnh (10CT THPT chuyn Qung Bnh), on Th
Ho (10A7 THPT Long Khnh - ng Nai)
Phng php dng lng lin hp: Ninh Vn T (THPT chuyn Trn i Ngha -
TPHCM) , inh V Bo Chu (THPT - chuyn L Qu n, Vng Tu), on Th
Ha (THPT Long Khnh - ng Nai)
Phng php dng bt ng thc: Nguyn An Vnh Phc (TN Ph thng Nng khiu-
TP HCM), Phan Minh Nht, L Hong c (10CT THPT chuyn L Hng Phong - TP
HCM), ng Hong Phi Long (10A10 THPT Kim Lin H Ni), Nguyn Vn Bnh
(11A5 THPT Trn Quc Tun - Qung Ngi),
Phng php dng n iu: Nguyn Anh Huy (10CT THPT chuyn L Hng Phong
- TP HCM), Hong Kim Qun (THPT Hng Thi H Ni), ng Hong Phi Long
(10A10 THPT Kim Lin H Ni)
Phng trnh m logarit: V Anh Khoa, Nguyn Thanh Hoi (i hc KHTN- TP
HCM), Nguyn Ngc Duy (11 Ton THPT chuyn Lng Th Vinh - ng Nai)
Cc loi h c bn: Nguyn Anh Huy (10CT THPT chuyn L Hng Phong - TP HCM)
9
H phng trnh hon v: thy Nguyn Trng Sn (THPT Yn M A Ninh Bnh),
Nguyn Anh Huy (10CT THPT chuyn L Hng Phong TP HCM), Nguyn nh Hong
(10A10 THPT Kim Lin - H Ni)
Phng php bin i ng thc: Nguyn nh Hong (10A10 THPT Kim Lin - H
Ni), Trn Vn Lm (THPT L Hng Phong - Thi Nguyn), Nguyn c Hunh (11
Ton THPT Nguyn Th Minh Khai - TP HCM)
Phng php h s bt nh: L Phc L (i hc FPT TP HCM), Nguyn Anh Huy,
Phan Minh Nht (10CT THPT chuyn L Hng Phong TP HCM)
Phng php t n ph tng - hiu: Nguyn Anh Huy (10CT THPT chuyn L Hng
Phong TP HCM)
Tng hp cc bi h phng trnh: Nguyn Anh Huy (10CT THPT chuyn L Hng Phong
TP HCM), Nguyn Thnh Thi (THPT chuyn Nguyn Quang Diu ng Thp), Trn
Minh c (T1K21 THPT chuyn H Tnh H Tnh), V Hu Thng (11 Ton THPT
Nguyn Th Minh Khai TP HCM)
Sng to phng trnh: thy Nguyn Ti Chung (THPT chuyn Hng Vng Gia Lai),
thy Nguyn Tt Thu (THPT L Hng Phong - ng Nai), Nguyn L Thu Linh (10CT
THPT chuyn L Hng Phong TP HCM)
Gii ton bng cch lp phng trnh: Nguyn An Vnh Phc (TN Ph thng Nng khiu-
TP HCM)
Lch s pht trin ca phng trnh: Nguyn An Vnh Phc (TN Ph thng Nng khiu-
TP HCM), Nguyn Hong Nam (THPT Phc Thin - ng Nai)
CnucNo I: I CNO V PHNO TRNH
HU T
PHNG TRNH BC BA
Mt s phng php gii phng trnh bc ba
Phng php phn tch nhn t:
Nu phng trnh bc ba ax
3
+bx
2
+cx +d = 0 c nghim x = r th c nhn t (x r) do
c th phn tch
ax
3
+bx
2
+cx +d = (x r)[ax
2
+ (b +ar)x + c + br +ar
2
]
T ta a v gii mt phng trnh bc hai, c nghim l
b ra

b
2
4ac 2abr 3a
2
r
2
2a
Phng php Cardano:
Xt phng trnh bc ba x
3
+ax
2
+ bx + c = 0 (1).
Bng cch t x = y
a
3
, phng trnh (1) lun bin i c v dng chnh tc:
y
3
+py +q = 0(2)
Trong : p = b
a
2
3
, q = c +
2a
3
9ab
27
Ta ch xt p, q = 0 v p = 0 hay q = 0 th a v trng hp n gin.
t y = u +v thay vo (2), ta c:
(u +v)
3
+p(u + v) + q = 0 u
3
+v
3
+ (3uv + p)(u +v) + q = 0 (3)
Chn u, v sao cho 3uv +p = 0 (4).
Nh vy, tm u v v, t (3) v (4) ta c h phng trnh:

u
3
+ v
3
= q
u
3
v
3
=
p
3
27
Theo nh l Viete, u
3
v v
3
l hai nghim ca phng trnh:
X
2
+qX
p
3
27
= 0(5)
t =
q
2
4
+
p
3
27
10
11
Khi > 0, (5) c nghim:
u
3
=
q
2
+

, v
3
=
q
2

Nh vy, phng trnh (2) s c nghim thc duy nht:


y =
3

q
2
+

+
3

q
2

Khi = 0, (5) c nghim kp: u = v =


3

q
2
Khi , phng trnh (2) c hai nghim thc, trong mt nghim kp.
y
1
= 2
3

q
2
, y
2
= y
3
=
3

q
2
Khi < 0, (5) c nghim phc.
Gi u
3
0
l mt nghim phc ca (5), v
3
0
l gi tr tng ng sao cho u
0
v
0
=
p
3
.
Khi , phng trnh (2) c ba nghim phn bit.
y
1
= u
0
+v
0
y
2
=
1
2
(u
0
+v
0
) + i

3
2
(u
0
v
0
)
y
3
=
1
2
(u
0
+v
0
) i

3
2
(u
0
v
0
)
Phng php lng gic ho - hm hyperbolic:
Mt phng trnh bc ba, nu c 3 nghim thc, khi biu din di dng cn thc s lin quan
n s phc. V vy ta thng dng phng php lng gic ho tm mt cch biu din
khc n gin hn, da trn hai hm s cos v arccos
C th, t phng trnh t
3
+ pt + q = 0 () ta t t = ucos v tm u c th a () v
dng
4 cos
3
3 cos cos 3 = 0
Mun vy, ta chn u = 2

p
3
v chia 2 v ca () cho
u
3
4
c
4 cos
3
3 cos
3q
2p
.

3
p
= 0 cos 3 =
3q
2p
.

3
p
Vy 3 nghim thc l
t
i
= 2

p
3
. cos

1
3
arccos

3q
2p
.

3
p

2i
3

vi i = 0, 1, 2.
Lu rng nu phng trnh c 3 nghim thc th p < 0 (iu ngc li khng ng) nn cng
thc trn khng c s phc.
Khi phng trnh ch c 1 nghim thc v p = 0 ta cng c th biu din nghim bng cng
thc hm arcosh v arsinh:
t =
2[q[
q
.

p
3
cosh

1
3
.arcosh

3[q[
2p
.

3
p

nu p < 0 v 4p
3
+ 27q
2
> 0.
12
t = 2

p
3
. sinh

1
3
.arsinh

3q
2p
.

3
p

nu p > 0
Mi phng php trn u c th gii quyt phng trnh bc ba tng qut. Nhng mc ch
ca chng ta trong mi bi ton lun l tm li gii ngn nht, p nht. Hy cng xem qua
mt s v d:
Bi tp v d
Bi 1: Gii phng trnh x
3
+x
2
+x =
1
3
Gii
Phng trnh khng c nghim hu t nn khng th phn tch nhn t. Trc khi ngh ti
cng thc Cardano, ta th quy ng phng trnh:
3x
3
+ 3x
2
+ 3x + 1 = 0
i lng 3x
2
+3x+1 gi ta n mt hng ng thc rt quen thuc x
3
+3x
2
+3x+1 = (x+1)
3
.
Do phng trnh tng ng:
(x + 1)
3
= 2x
3
hay
x + 1 =
3

2x
T suy ra nghim duy nht x =
1
1 +
3

2
.
Nhn xt: V d trn l mt phng trnh bc ba c nghim v t, v c gii nh kho lo
bin i ng thc. Nhng nhng bi n gin nh th ny khng c nhiu. Sau y ta s i
su vo cng thc Cardano:
Bi 2: Gii phung trnh x
3
3x
2
+ 4x + 11 = 0
Gii
t x = y + 1 . Th vo phng trnh u bi, ta c phng trnh:
y
3
+ 1.y + 13 = 0
Tnh = 13
2
+
4
27
.1
3
=
4567
27
0
p dng cng thc Cardano suy ra:
y =
3

13 +

4567
27
2
+
3

13

4567
27
2
Suy ra x =
3

13 +

4567
27
2
+
3

13

4567
27
2
+ 1.
Nhn xt: V d trn l mt ng dng c bn ca cng thc Cardano. Tuy nhin cng
thc ny khng h d nh v ch c dng trong cc k thi Hc sinh gii. V th, c l chng
ta s c gng tm mt con ng hp thc ha cc li gii trn. l phng php lng
gic ho. u tin xt phng trnh dng x
3
+ px +q = 0 vi p < 0 v c 1 nghim thc:
13
Bi 3: Gii phng trnh x
3
+ 3x
2
+ 2x 1 = 0
Gii
u tin t x = y 1 ta a v phng trnh y
3
y 1 = 0 (1). n y ta dng lng gic
nh sau:
Nu [y[ <
2

3
suy ra

3
2
y

< 1. Do tn ti [0, ] sao cho

3
2
y = cos .
Phng trnh tng ng:
8
3

3
cos
3

2

3
cos 1 = 0
hay
cos 3 =
3

3
2
(v nghim)
Do [y[
2

3
. Nh vy lun tn ti t tho y =
1

3
(t +
1
t
) (). Th vo (1) ta c phng
trnh
t
3
3

3
+
1
3

3t
3
1 = 0
Vic gii phng trnh ny khng kh, xin dnh cho bn c. Ta tm c nghim:
x =
1

1
2

23

+
1
3

1
2

23

1 2
Nhn xt: Cu hi t ra l: S dng phng php trn nh th no?. Mun tr li, ta cn
lm sng t 2 vn :
1) C lun tn ti t tho mn cch t trn?
p n l khng. Coi () l phng trnh bc hai theo t ta s tm c iu kin [y[
2

3
.
Tht ra c th tm nhanh bng cch dng AM-GM:
[y[ =

t +
1
t

=
1

[t[ +
1
[t[

3
Vy trc ht ta phi chng minh (1) khng c nghim [y[ <
2

3
.
2) V sao c s
2

3
?
tng ca ta l t phng trnh x
3
+px+q = 0 a v mt phng trnh trng phng theo
t
3
qua cch t x = k

t +
1
t

. Khai trin v ng nht h s ta c k =

p
3
Sau y l phng trnh dng x
3
+px +q = 0 vi p < 0 v c 3 nghim thc:
Bi 4: Gii phng trnh x
3
x
2
2x + 1 = 0
Gii
t y = x
1
3
. Phng trnh tng ng:
y
3

7
3
y +
7
27
= 0()
14
Vi [y[ <
2

7
3
th

3y
2

< 1. Do tn ti [0, ] sao cho cos =


3y
2

7
hay y =
2

7 cos
3
.
Th vo (), ta c:
cos 3 =

7
14
y l phng trnh lng gic c bn. D dng tm c ba nghim ca phng trnh ban
u:
x
1
=
2

7
3
cos

arccos

7
14

+
1
3
x
2,3
=
2

7
3
cos

arccos

7
14

3
+
2
3

+
1
3
Do phng trnh bc ba c ti a ba nghim phn bit nn ta khng cn xt trng hp
[y[
2

7
3
. Bi ton c gii quyt.
Nhn xt: Ta cng c th chng minh phng trnh v nghim khi [y[
2

7
3
bng cch t
y =

7
3
(t +
1
t
) ging nh bi 3, t dn ti mt phng trnh trng phng v nghim.
Tng kt li, ta dng php t n ph y =

p
3

t +
1
t

() nh sau:
Nu phng trnh c 1 nghim thc, chng minh phng trnh v nghim khi [y[ < 2

p
3
,
trng hp cn li dng () a v phng trnh trng phng theo t.
Nu phng trnh c 3 nghim thc, chng minh phng trnh v nghim khi [y[ 2

p
3
bng php t () (a v phng trnh trng phng v nghim theo t). Khi [y[ 2

p
3
th
t
[y[
2

p
3
= cos , t tm , suy ra 3 nghim y.
Cn khi p > 0 khng kh chng minh phng trnh c nghim duy nht:
Bi 5: Gii phng trnh x
3
+ 6x + 4 = 0
Gii
tng: Ta s dng php t x = k

t
1
t

a v phng trnh trng phng.


php t ny khng cn iu kin ca x, v n tng ng k(t
2
1) xt = 0. Phng trnh
trn lun c nghim theo t.
Nh vy t phng trnh u ta c
k
3

t
3

1
t
3

3k
3

t
1
t

+ 6k

t
1
t

+ 4 = 0
15
Cn chn k tho 3k
3
= 6k k =

2
Vy ta c li gii bi ton nh sau:
Li gii:
t x =

t
1
t

ta c phng trnh
2

t
3

1
t
3

+ 4 = 0 t
6
1 +

2t
3
= 0 t
1,2
=
3

2
Lu rng t
1
.t
2
= 1 theo nh l Viete nn ta ch nhn c mt gi tr ca x l
x = t
1
+t
2
=

1 +

2
+
3

. 2
Bi 6: Gii phng trnh 4x
3
3x = m vi [m[ > 1
Gii
Nhn xt rng khi [x[ 1 th [V T[ 1 < [m[ (sai) nn [x[ 1. V vy ta c th t
x =
1
2

t +
1
t

.
Ta c phng trnh tng ng:
1
2

t
3
+
1
t
3

= m
T :
t =
3

m
2
1 x =
1
2

m+

m
2
1 +
3

m
2
1

.
Ta chng minh y l nghim duy nht.
Gi s phng trnh c nghim x
0
th x
0
[1, 1] v [x
0
[ > 1. Khi :
4x
3
3x = 4x
3
0
3x
0
hay
(x x
0
)(4x
2
+ 4xx
0
+ 4x
2
0
3) = 0
Xt phng trnh:
4x
2
+ 4xx
0
+ 4x
2
0
3 = 0
c

= 12 12x
2
0
< 0 nn phng trnh bc hai ny v nghim.
Vy phng trnh u bi c nghim duy nht l
x =
1
2

m+

m
2
1 +
3

m
2
1

.
Bi tp t luyn
Bi 1: Gii cc phng trnh sau:
a) x
3
+ 2x
2
+ 3x + 1 = 0
b) 2x
3
+ 5x
2
+ 4x + 2 = 0
c) x
3
5x
2
+ 4x + 1 = 0
16
d) 8x
3
+ 24x
2
+ 6x 10 3

6 = 0
Bi 2: Gii v bin lun phng trnh:
4x
3
+ 3x = m vi m R
Bi 3: Gii v bin lun phng trnh:
x
3
+ax
2
+bx +c = 0
PHNG TRNH BC BN
[1] Phng trnh dng ax
4
+bx
3
+ cx
2
+bkx + ak
2
= 0 (1)
Ta c
(1) a(x
4
+ 2x
2
.k +k
2
) + bx(x
2
+k) + (c 2ak)x
2
= 0
a(x
2
+k)
2
+bx(x
2
+k) + (c 2ak)x
2
= 0
n y c hai hng gii quyt:
Cch 1: a phng trnh v dng A
2
= B
2
:
Thm bt, bin i v tri thnh dng hng ng thc dng bnh phng ca mt tng, chuyn
cc hng t cha x
2
sang bn phi.
Cch 2: t y = x
2
+ k y k
Phng trnh (1) tr thnh ay
2
+ bxy + (c 2ak)x
2
= 0
Tnh x theo y hoc y theo x a v phng trnh bc hai theo n x.
V d: Gii phng trnh: x
4
8x
3
+ 21x
2
24x + 9 = 0 (1.1)
Cch 1:
(1.1) (x
4
+ 9 + 6x
2
) 8(x
2
+ 3) + 16x
2
= 16x
2
21x
2
+ 6x
2
(x
2
4x + 3)
2
= x
2

x
2
4x + 3 = x
x
2
4x + 3 = x

x
2
5x + 3 = 0
x
2
3x + 3 = 0

x =
5

13
2
x =
5 +

13
2
Cch 2:
(1.1) (x
4
+ 6x
2
+ 9) 8x(x
2
+ 3) + 15x
2
= 0 (x
2
+ 3)
2
8x(x
2
+ 3) + 15x
2
= 0
t y = x
2
+ 3. (1.1) tr thnh: y
2
8xy + 15x
2
= 0 (y 3x)(y 5x) = 0

y = 3x
y = 5x
Vi y = 3x: Ta c x
2
+ 3 = 3x: Phng trnh v nghim
Vi y = 5x: Ta c x
2
+ 3 = 5x x
2
5x + 3 = 0

x =
5

13
2
x =
5 +

13
2
Vy phng trnh (1.1) c tp nghim: S =

5 +

13
2
;
5

13
2

Nhn xt: Mi phng php gii c li th ring. Vi cch gii 1, ta s tnh c trc tip m
17
khng phi thng qua n ph. Vi cch gii 2, ta s c nhng tnh ton n gin hn v t b
nhm ln.
Bi tp t luyn
Gii cc phng trnh sau:
1) x
4
13x
3
+ 46x
2
39x + 9 = 0
2) 2x
4
+ 3x
3
27x
2
+ 6x + 8 = 0
3) x
4
3x
3
6x
2
+ 3x + 1 = 0
4) 6x
4
+ 7x
3
36x
2
7x + 6 = 0
5) x
4
3x
3
9x
2
27x + 81 = 0
[2] Phng trnh dng (x + a)(x +b)(x +c)(x +d) = ex
2
(2) vi ad = bc = m
Cch 1: a v dng A
2
= B
2
(2) (x +px + m)(x
2
+nx + m) = ex
2
(ad = bc = m, p = a +d, n = b + c)

x
2
+
p + n
2
x +m
n p
2
x

x
2
+
p + n
2
x +m+
n p
2
x

= ex
2

x
2
+
p + n
2
x +m

2
=

n p
2

2
+e

x
2
Cch 2: Xt xem x = 0 c phi l nghim ca phng trnh khng.
Trng hp x = 0:
(2)

x +
m
x
+p

x +
m
x
+n

= e
t u = x +
m
x
. iu kin: [u[ 2

[m[
(2) tr thnh (u + p)(u +n) = e. n y gii phng trnh bc hai theo u tm x.
V d: Gii phng trnh: (x + 4)(x + 6)(x 2)(x 12) = 25x
2
(2.1)
Cch 1:
(2.1) (x
2
+ 10x + 24)(x
2
14x + 24) = 25x
2
(x
2
2x + 24 + 12x)(x
2
2x + 24 12x) = 25x
2
(x
2
2x + 24)
2
= 169x
2

x
2
2x + 24 = 13x
x
2
2x + 24 = 13x

x
2
15x + 24 = 0
x
2
+ 11x + 24 = 0

x = 8
x = 3
x =
15

129
2
Cch 2:
(2.1) (x
2
+ 10x + 24)(x
2
14x + 24) = 25x
2
Nhn thy x = 0 khng phi l nghim ca phng trnh.
18
x = 0 : (2.1)

x +
24
x
+ 10

x +
24
x
14

= 25
t y = x +
24
x
[y[ 4

6. (2.1) tr thnh:
(y + 10)(y 14) = 25 (y + 11)(y 15) = 0

y = 11
y = 15
Vi y = 11: Ta c phng trnh:
x +
24
x
= 11 x
2
+ 11x + 24 = 0

x = 3
x = 8
Vi y = 15: Ta c phng trnh:
x +
24
x
= 15 x
2
15x + 24 = 0 x =
15

129
2
Phng trnh (2.1) c tp nghim S =

3; 8;
15

129
2
;
15 +

129
2

Nhn xt: Trong cch gii 2, c th ta khng cn xt x = 0 ri chia m c th t n ph


y = x
2
+m thu c phng trnh bc hai n x, tham s y hoc ngc li.
Bi tp t luyn
Gii cc phng trnh sau:
1) 4(x + 5)(x + 6)(x + 10)(x + 12) = 3x
2
2) (x + 1)(x + 2)(x + 3)(x + 6) = 168x
2
3) (x + 3)(x + 2)(x + 4)(x + 6) = 14x
2
4) (x + 6)(x + 8)(x + 9)(x + 12) = 2x
2
5) 18(x + 1)(x + 2)(x + 5)(2x + 5) =
19
4
x
2
[3] Phng trnh dng (x +a)(x +b)(x +c)(x + d) = m (3) vi a +b = c + d = p
Ta c (3) (x
2
+ px +ab)(x
2
+px +cd) = m
Cch 1:
(3)

x
2
+ px +
ab +cd
2
+
ab cd
2

x
2
+px +
ab +cd
2

ab cd
2

= m

x
2
+ px +
ab +cd
2

2
= m+

ab cd
2

2
Bi ton quy v gii hai phng trnh bc hai theo x.
Cch 2:
t y = x
2
+px iu kin: y
p
2
4
. (3) tr thnh:(y +ab)(y + cd) = m
Gii phng trnh bc 2 n y tm x.
V d: Gii phng trnh: x(x + 1)(x + 2)(x + 3) = 8 (3.1)
Cch 1:
19
Ta c
(3.1) (x
2
+ 3x)(x
2
+ 3x + 2) = 8
(x
2
+ 3x + 1 1)(x
2
+ 3x + 1 + 1) = 8
(x
2
+ 3x + 1)
2
= 9

x
2
+ 3x + 1 = 3
x
2
+ 3x + 1 = 3

x
2
+ 3x 2 = 0
x
2
+ 3x + 4 = 0
x =
3

17
2
Cch 2:
(3.1) (x
2
+ 3x)(x
2
+ 3x + 2) = 8
t y = x
2
+ 3x y
9
4
(3.1) tr thnh:
y(y + 2) = 8 y
2
+ 2y 8 = 0

y = 2
y = 4(loi)
y = 2
Vi y = 2: Ta c phng trnh:
x
2
+ 3x 2 = 0 x =
3

17
2
Phng trnh (3.1) c tp nghim: S =

3 +

17
2
;
3

17
2

Bi tp t luyn
Gii cc phng trnh sau:
1. (x + 2)(x + 3)(x 7)(x 8) = 144
2. (x + 5)(x + 6)(x + 8)(x + 9) = 40
3.

x +
1
4

x +
3
5

x +
1
20

x +
4
5

=
39879
40000
4. (6x + 5)
2
(3x + 2)(x + 1) = 35
5. (4x + 3)
2
(x + 1)(2x + 1) = 810
Nhn xt: Nh dng (2), ngoi cch t n ph trn, ta c th t mt trong cc dng
n ph sau:
t y = x
2
+px +ab
t y = x
2
+px +cd
t y =

x +
p
2

2
t y = x
2
+px +
ab + cd
2
[4] Phng trnh dng (x + a)
4
+ (x + b)
4
= c (c > 0) (4)
t x = y
a +b
2
. (4) tr thnh:

y +
a b
2

4
+

y
a b
2

4
= c
20
S dng khai trin nh thc bc 4, ta thu c phng trnh:
2y
4
+ 3(a b)
2
y
2
+ 2

a b
2

4
= c
Gii phng trnh trng phng n y tm x.
V d: Gii phng trnh: (x + 2)
4
+ (x + 4)
4
= 82 (4.1)
t y = x + 3. Phng trnh (4.1) tr thnh:
(y + 1)
4
+ (y 1)
4
= 82
(y
4
+ 4y
3
+ 6y
2
+ 4y + 1) + (y
4
4y
3
+ 6y
2
4y + 1) = 82
2y
4
+ 12y
2
80 = 0 (y
2
4)(y
2
+ 10) = 0
y
2
= 4 y = 2
Vi y = 2, ta c x = 1
Vi y = 2, ta c x = 5
Vy phng trnh c tp nghim: S = 1; 5
Bi tp t luyn
Gii cc phng trnh sau:
1. (x + 2)
4
+ (x + 8)
4
= 272
2. (x +

2)
4
+ (x + 1)
4
= 33 + 12

2
3. (x + 10)
4
+ (x 4)
4
= 28562
4. (x + 1)
4
+ (x 3)
4
= 90
[5] Phng trnh dng x
4
= ax
2
+bx +c (5)
a (5) v dng A
2
= B
2
:
(5) (x
2
+ m)
2
= (2m+a)x
2
+bx +c +m
2
Trong , m l mt s cn tm.
Tm m f(x) = (2m + a)x
2
+ bx + c + m
2
c = 0. Khi , f(x) c dng bnh phng ca
mt biu thc.
Nu 2m+a < 0 : (5) (x
2
+m)
2
+g
2
(x) = 0 (vi f(x) = g
2
(x))

x
2
+m = 0
g(x) = 0
Nu2m+a > 0 : (5) (x
2
+m)
2
= g
2
(x) (vi f(x) = g
2
(x))

x
2
+ m = g(x)
x
2
+ m = g(x)
21
V d: Gii phng trnh: x
4
+x
2
6x + 1 = 0 (5.1)
Ta c:
(5.1) x
4
+ 4x
2
+ 4 = 3x
2
+ 6x + 3 (x
2
+ 2)
2
= 3(x + 1)
2

x
2
+ 2 =

3(x + 1)
x
2
+ 2 =

3(x + 1)

x
2

3x + 2

3 = 0
x
2
+

3 + 2 +

3 = 0

x =

3 5
2
x =

3 +

3 5
2
Phng trnh (5.1) c tp nghim: S =

3 5
2
;

3 +

3 5
2

Bi tp t luyn
Gii cc phng trnh sau:
1. x
4
19x
2
10x + 8 = 0
2. x
4
= 4x + 1
3. x
4
= 8x + 7
4. 2x
4
+ 3x
2
10x + 3 = 0
5. (x
2
16)
2
= 16x + 1
6. 3x
4
2x
2
16x 5 = 0
Nhn xt: Phng trnh dng x
4
= ax + b c gii theo cch tng t.
Phng trnh = 0 l phng trnh bc ba vi cch gii c trnh by trc. Phng
trnh ny c th cho 3 nghim m, cn la chn m sao cho vic tnh ton l thun li nht. Tuy
nhin, d dng nghim m no th cng cho cng mt kt qu.
[6] Phng trnh dng af
2
(x) + bf(x)g(x) + cg
2
(x) = 0 (6)
Cch 1: Xt g(x) = 0, gii tm nghim v th li vo (6).
Trng hp g(x) = 0: a

f(x)
g(x)

2
+b.
f(x)
g(x)
+c = 0
t y =
f(x)
g(x)
, gii phng trnh bc hai ay
2
+by + c = 0 ri tm x.
Cch 2: t u = f(x), v = g(x), phng trnh tr thnh
au
2
+buv + cv
2
= 0 (6)
Xem (6) l phng trnh bc hai theo n u, tham s v. T tnh u theo v.
V d: Gii phng trnh: 20(x 2)
2
5(x + 1)
2
+ 48(x 2)(x + 1) = 0 (6.1)
22
t u = x 2, v = x + 1. Phng trnh (6.1) tr thnh:
20u
2
+ 48uv 5v
2
= 0 (10u v)(2u + 5v) = 0

10u = v
2u = 5v
Vi 10u = v, ta c: 10(x 2) = x + 1 x =
7
3
Vi 2u = 5v, ta c: 2(x 2) = 5(x + 1) x =
1
7
Vy phng trnh (6.1) c tp nghim: S =

7
3
;
1
7

Nhn xt: Nu chn y =


f(x)
g(x)
Vi f(x) v g(x) l hai hm s bt k (g(x) = 0), ta s to
c mt phng trnh. Khng ch l phng trnh hu t, m cn l phng trnh v t.
Bi tp t luyn
Gii cc phng trnh sau:
1. (x 5)
4
12(x 2)
4
+ 4(x
2
7x + 10)
2
= 0
2. (x 2)
4
+ 3(x + 3)
4
4(x
2
+ x 6)
2
= 0
3. 4(x
3
1) + 2(x
2
+ x + 1)
2
4(x 1)
2
= 0
4. 2(x
2
x + 1)
2
+ 5(x + 1)
2
+ 14(x
3
+ 1) = 0
5. (x 10)
4
15(x + 5)
4
+ 4(x
2
5x 50)
2
= 0
[7] Phng trnh bc bn tng qut ax
4
+bx
3
+ cx
2
+dx + e = 0 (7)
Phn tch cc hng t bc 4, 3, 2 thnh bnh phng ng, cc hng t cn li chuyn sang v
phi:
(7) 4a
2
x
4
+ 4bax
3
+ 4cax
2
+ 4dax + 4ae = 0
(2ax
2
+bx)
2
= (b
2
4ac)x
2
4adx 4ae
Thm vo hai v mt biu thc 2(2ax
2
+bx)y +y
2
(y l hng s) v tri thnh bnh phng
ng, cn v phi l tam thc bc hai theo x:
f(x) = (b
2
4ac 4ay)x
2
+ 2(by 2ad)x 4ae + y
2
Tnh y sao cho v phi l mt bnh phng ng. Nh vy, ca v phi bng 0. Nh vy ta
phi gii phng trnh = 0. T ta c dng phng trnh A
2
= B
2
quen thuc.
V d: Gii phng trnh x
4
16x
3
+ 66x
2
16x 55 = 0 (7.1)
(7.1) x
4
16x
3
+ 64x
2
= 2x
2
+ 16x + 55
(x
2
8x)
2
+ 2y(x
2
8x) + y
2
= (2y 2)x
2
+ (16 16y)x + 55 + y
2
Gii phng trnh = 0 (8 8y)
2
(55 + y
2
)(2y 2) = 0 tm c y = 1, y = 3, y = 29.
Trong cc gi tr ny, ta thy gi tr y = 3 l thun li nht cho vic tnh ton.
23
Nh vy, chn y = 3, ta c phng trnh:
(x
2
8x + 3)
2
= 4(x 4)
2

x
2
8x + 3 = 2(x 4)
x
2
8x + 3 = 2(x 4)

x
2
10x + 11 = 0
x
2
6x 5 = 0

x = 3

14
x = 5

14
Phng trnh (7.1) c tp nghim S =

3 +

14; 3

14; 5 +

14; 5

14

Nhn xt: V d trn cho ta thy phng trnh = 0 c nhiu nghim. C th chn y = 1
nhng t ta c phng trnh (x
2
8x+1)
2
= 56 th khng thun li lm cho vic tnh ton,
tuy nhin, kt qu vn nh nhau.
Mt cch gii khc l t phng trnh x
4
+ax
3
+bx
2
+cx+d = 0 t x = t
a
4
, ta s thu c
phng trnh khuyt bc ba theo t, ngha l bi ton quy v gii phng trnh t
4
= at
2
+bt +c.
Bi tp t luyn
1. x
4
14x
3
+ 54x
2
38x 11 = 0
2. x
4
16x
3
+ 57x
2
52x 35 = 0
3. x
4
6x
3
+ 9x
2
+ 2x 7 = 0
4. x
4
10x
3
+ 29x
2
20x 8 = 0
5. 2x
4
32x
3
+ 127x
2
+ 38x 243 = 0
PHNG TRNH DNG PHN THC
[1] Phng trnh cha n mu c bn
t iu kin xc nh cho biu thc mu. Quy ng ri gii phng trnh.
V d: Gii phng trnh:
1
2 x
+
x
2x 1
= 2 (1.1)
iu kin: x = 2; x =
1
2
.
(1.1)
2x 1 + x(2 x)
(2 x)(2x 1)
= 2 2x 1 + 2x x
2
= 2(4x 2 2x
2
+x)
3x
2
6x + 3 = 0 x = 1(tha iu kin)
Vy phng trnh (1.1) c tp nghim S = 1
[2] Phng trnh dng x
2
+
a
2
x
2
(x + a)
2
= b (2)
Ta c:
(2)

x
ax
(x + a)

2
+ 2x.
ax
x +a
= b

x
2
x +a

2
+ 2a.
x
2
x +a
+a
2
= b + a
2
24
t y =
x
2
x +a
. Gii phng trnh bc hai theo y tm x.
V d: Gii phng trnh: x
2
+
9x
2
(x + 3)
2
= 7 (2.1)
iu kin: x = 3.
(2.1)

x
3x
x + 3

2
+ 6.
x
2
x + 3
= 7

x
2
x + 3

2
+ 6.
x
2
x + 3
= 7
t y =
x
2
x + 3
. Ta c phng trnh
y
2
+ 6y 7 = 0

y = 1
y = 7
Nu y = 1: Ta c phng trnh x
2
= x + 3 x =
1

13
2
Nu y = 7: Ta c phng trnh x
2
+ 7x + 21 = 0 (v nghim)
Vy phng trnh (2.1) c tp nghim: S =

1 +

13
2
;
1

13
2

Nhn xt: Da vo cch gii trn, ta c th khng cn phi t n ph m thm bt hng


s to dng phng trnh quen thuc A
2
= B
2
Bi tp t luyn
Gii cc phng trnh sau:
1. x
2
+
4x
2
(x + 2)
2
= 12
2. x
2
+
25x
2
(x + 5)
2
= 11
3. x
2
+
9x
2
(x 3)
2
= 14
4.
25
x
2

49
(x 7)
2
= 1
5.
9
4(x + 4)
2
+ 1 =
8
(2x + 5)
2
[3] Phng trnh dng
x
2
+ nx +a
x
2
+mx +a
+
x
2
+qx + a
x
2
+px + a
= b (3)
iu kin:

x
2
+mx +a = 0
x
2
+px +a = 0
Xt xem x = 0 c phi l nghim phng trnh khng.
25
Trng hp x = 0:
(2)
x +
a
x
+ n
x +
a
x
+m
+
x +
a
x
+q
x +
a
x
+p
= b
t y = x +
a
x
. iu kin: [y[ 2

[a[ ta c phng trnh


y +n
y + m
+
y +q
y +p
= b
Gii phng trnh n y sau tm x.
V d: Gii phng trnh:
x
2
3x + 5
x
2
4x + 5

x
2
5x + 5
x
2
6x + 5
=
1
4
(3.1)
iu kin: x = 1, x = 5.
x = 0 khng phi l nghim ca phng trnh.
Xt x = 0 :
(3.1)
x +
5
x
3
x +
5
x
4

x +
5
x
5
x +
5
x
6
=
1
4
t y = x +
5
x
[y[ 2

5, y = 6. Phng trnh (3.1) tr thnh:


y 3
y 4

y 5
y 6
=
1
4

2
y
2
10y + 24
=
1
4
y
2
10y + 16 = 0

y = 2 (loi)
y = 8
T ta c phng trnh
x +
5
x
= 8 x
2
8x + 5 = 0 x = 4

11
Vy phng trnh (3.1) c tp nghim: S =

4 +

11; 4

11

Nhn xt: Cc dng phng trnh sau c gii mt cch tng t:


Dng 1:
mx
ax
2
+bx +d
+
nx
ax
2
+cx +d
= p
Dng 2:
ax
2
+mx + c
ax
2
+nx + c
+
px
ax
2
+qx +c
= b
Bi tp t luyn
Gii cc phng trnh sau:
1)
4x
4x
2
8x + 7
+
3x
4x
2
10x + 7
= 1
2)
2x
2x
2
5x + 3
+
13x
2x
2
+x + 3
= 6
3)
3x
x
2
3x + 1
+
7x
x
2
+x + 1
= 4
4)
x
2
10x + 15
x
2
6x + 15
=
4x
x
2
12x + 5
5)
x
2
+ 5x + 3
x
2
7x + 3
+
x
2
+ 4x + 3
x
2
+ 5x + 3
= 7
26
Tng kt
Qua cc dng phng trnh trn, ta thy phng trnh hu t thng c gii bng mt trong
cc phng php:
[1.] a v phng trnh tch
[2.] t n ph hon ton
[3.] t n ph a v h phng trnh
[4.] a v ly tha ng bc (thng l dng A
2
= B
2
)
[5.] Chia t v mu cho cng mt s
[6.] Thm bt to thnh bnh phng ng
Tuy nhin, c mt s dng phng trnh c nhng phng php gii c trng. Nhng phng
trnh ny s c trnh by c th hn nhng phn khc.
Bi tp tng hp phn phng trnh hu t
Phn 1:
1) x
3
3x
2
+ 18x 36 = 0
2) 8x
2
6x =
1
2
3) x
3
4x
2
4x + 8 = 0
4) x
3
21x
2
+ 35x 7 = 0
5) x
3
6x
2
+ 8 = 0
Phn 2:
1) 6x
5
11x
4
11x + 6 = 0
2) (x
2
6x)
2
2(x 3)
2
= 81
3) x
4
+ (x 1)(3x
2
+ 2x 2) = 0
4) x
4
+ (x + 1)(5x
2
6x 6) = 0
5) x
5
+x
2
+ 2x + 2 = 0
6) (x
2
16)
2
= 16x + 1
7) (x + 2)
2
+ (x + 3)
3
+ (x + 4)
4
= 2
8) x
3
+
1
x
3
= 13

x +
1
x

9)

x 1
x

2
+

x 1
x 2

2
=
40
9
10)
x(3 x)
x 1

x +
3 x
x 1

= 15
11)
1
x
+
1
x + 2
+
1
x + 5
+
1
x + 7
=
1
x + 1
+
1
x + 3
+
1
x + 4
+
1
x + 6
27
XY DNG PHNG TRNH HU T
Bn cnh vic xy dng phng trnh t h phng trnh, vic xy dng phng trnh t nhng
ng thc i s c iu kin l mt trong nhng phng php gip ta to ra nhng dng
phng trnh hay v l. Di y l mt s ng thc n gin.
4.1 T ng thc (a +b +c)
3
= a
3
+b
3
+ c
3
+ 3(a +b)(b +c)(c + a) (1) :
V d: Gii phng trnh: (x 2)
3
+ (2x 4)
3
+ (7 3x)
3
= 1 (1.1)
Nhn xt: Nu t a = x 2, b = 2x 4, c = 7 3x. Khi ta c phng trnh: a
3
+b
3
+c
3
=
(a +b +c)
3
. T ng thc (1), d dng suy ra (a +b)(b + c)(c +a) = 0. T , ta c li gii:
(1) (x 2)
3
+ (2x 4)
3
+ (7 3x)
3
= [(x 2) + (2x 4) + (7 3x)]
3
(3x 6)(3 x)(5 2x) = 0

x = 2
x = 3
x =
5
2
Vy phng trnh (1.1) c tp nghim: S =

2; 3;
5
2

Vi bi ton trn, cch t nhin nht c l l khai trin ri thu v phng trnh bc ba. Tuy
nhin, vic khai trin c th khng cn hiu qu vi bi ton sau:
V d: Gii phng trnh: (x
2
4x + 1)
3
+ (8x x
2
+ 4)
3
+ (x 5)
3
= 125x
3
(1.2)
4.2 T mnh
1
a
+
1
b
+
1
c
=
1
a +b +c
(a +b)(b + c)(c +a) = 0 (2) :
V d: Gii phng trnh:
1
x 8
+
1
2x + 7
+
1
5x + 8
=
1
8x + 7
(2.1)
iu kin: x = 8, x =
7
2
, x =
8
5
, x =
7
8
T bi ton (2), ta c:
(2.1) (x 8 + 2x + 7)(x 8 + 5x + 8)(2x + 7 + 5x + 8) = 0

x =
1
3
x = 0
x =
15
7
Phng trnh c tp nghim: S =

1
3
; 0;
15
7

4.3 T ng thc a
3
+b
3
+c
3
3abc = (a +b +c)(a
2
+b
2
+c
2
ab bc ca) (3) :
V d: Gii phng trnh 54x
3
9x +

2 = 0 (3.1)
Ta tm cch vit v tri ca phng trnh di dng x
3
+ a
3
+ b
3
3abx. Nh vy th a, b l
nghim ca h phng trnh

a
3
+b
3
=

2
54
a
3
.b
3
=
1
18
3
28
a
3
, b
3
l nghim ca phng trnh
t
2

2
54
t +
1
18
2
= 0 t =
1
54

2
a = b =
1
3

2
Khi phng trnh cho tng ng vi
(x + a + b)

x
2
+a
2
+ b
2
a x bx ab

= 0

x +
2
3

x
2

2
3

2
x +
1
18

= 0

x =
2
3

2
x =
1
3

2
Vy (3.1) c tp nghim S =

2
3

2
;
1
3

4.4 T bi ton Nu xyz = 1 v x+y +z =


1
x
+
1
y
+
1
z
th (x1)(y 1)(z 1) = 0 (4) :
V d: Gii phng trnh:
1
10x
2
11x + 3
=
1
2x 1
+
1
5x 3
+ 10x
2
18x + 7 (4.1)
iu kin: x =
3
5
, x =
1
2
Nhn xt: Nu t a = 2x 1, b = 5x 3, c =
1
10x
2
11x + 3
Th ta c: abc = 1 v a +b +c =
1
a
+
1
b
+
1
c
T (3.1) (2x 1 1)(5x 3 1)

1
10x
2
11x + 3
1

= 0

x = 1
x =
4
5
10x
2
11x + 2 = 0

x = 1
x =
4
5
x =
11

41
20
Phng trnh (3.1) c tp nghim: S =

1;
4
5
;
11

41
20

Qua cc v d trn, ta c th hnh dung c bn vic s dng ng thc xy dng phng


trnh. Hi vng da vo vn hiu bit v kh nng sng to ca mnh, bn c c th to ra
nhng phng trnh p mt v c o hn na. Sau y l mt s bi tp t luyn t cc
ng thc khc.
Bi tp t luyn
1. (x 2)
6
+ (x
2
5x + 4)
3
= (2x
2
+ 3)
3
+ (5 9x)
3
2. (x
2
2x + 3)
5
+ (8x x
2
+ 7)
5
= (9x + 5)
5
+ (5 3x)
5
3. (x
3
5x+4)
2
(127x5x
2
)+(x
3
+2x
2
+7)(7x
2
+12x9) = (x
3
+7x
2
+7x5)
2
(2x
2
+5x+3)
4. (x 5)
4
(4x x
2
) + (2x + 7)
4
(x
2
3x + 12) = (x
2
2x + 7)
4
(12 + x)
5.
(x
2
5x + 3)
3
(4 9x)(x
2
12x + 7)
+
(x
2
+ 4x 1)
3
(9x 4)(x
2
3x + 3)
+
(7x 4)
3
(12x 7 x
2
)(x
2
+ 3x 3)
= 7
29
6.
1
x
2
+ 4x + 3
+
1
x
2
+ 8x + 15
+
1
x
2
+ 12x + 35
+
1
x
2
+ 16x + 63
= 5
7. (x
2
8x + 5)
7
+ (7x 8)
7
= (x
2
x 3)
7
MT S PHNG TRNH BC CAO
Nh ton hc Abel chng minh rng khng c cng thc nghim tng qut cho phng
trnh bc cao (> 4). y cng khng phi l dng ton quen thuc ph thng. V th bi
vit ny ch cp n mt s phng trnh bc cao c bit, c th gii bng bin i s cp.
Bi 1: Gii phng trnh x
5
x
4
x
3
11x
2
+ 25x 14 = 0
Gii
Phng trnh cho tng ng
(x
5
2x
4
) + (x
4
2x
3
) + (x
3
2x
2
) + (9x
2
+ 18x) + (7x 14) = 0
(x 2)(x
4
+x
3
+ x
2
9x + 7) = 0
x = 2 x
4
+x
3
+ x
2
9x + 7 = 0 ()
Xt (*) ta c
() (x
4
+x
3
+x
2
9x + 6) + 1 = 0
(x
4
x
3
+ 2x
3
2x
2
+ 3x
2
3x 6x + 6) + 1 = 0
(x 1)
2
(x
3
+ 3x + 6) + 1 = 0 (v nghim)
Vy phng trnh c tp nghim S = 2 2
Bi 2: Gii phng trnh x
6
7x
2
+

6 = 0 ()
Gii
R rng ta khng th on nghim ca phng trnh ny v bc cao v h s xu. Mt cch t
nhin ta t

6 = a. Lu rng ta hi vng c th a (*) v phng trnh bc hai theo a, do


ta phn tch 7 = a
2
+ 1. Cng vic cn li l gii phng trnh ny:
t

6 = a, khi
() x
6
x
2
(a
2
+ 1) + a = 0 a
2
x
2
a +x
2
x
6
= 0 ()
() c: = 1 + 4x
2
(x
6
x
2
) = (2x
4
1)
2
nn c nghim:

a
1
= x
2
a
2
=
1 x
4
x
2
30
Vy ()

x
2
=

6
1 x
4
=

6x
2

x =
4

6
x =

5
2

3
2
Vy phng trnh c tp nghim S =

6;

5
2

3
2

. 2
Bi 3: Gii phng trnh x
6
15x
2
+

68 = 0 ()
Gii
Do x = 0 khng tho (*) nn x = 0. Vit li (*) di dng
x
3
+

68
x
3
=
15
x
x
3
+
2

17
x
3
=
17 2
x
t a =

17 > 0 ta c phng trnh


x
3
+
2a
x
3
=
a
2
2
x
x
2
a
2
2a x
6
2x
2
= 0 ()
Coi (**) l phng trnh n a ta tm c nghim

a = x
2
(loi do a > 0)
a =
2 + x
4
x
2
Vy ta c

17 =
2 + x
4
x
2
x
4

17x
2
+ 2 = 0 x
2
=

17 3
2
x =

17 3
2
Kt lun: (*) c tp nghim S =

17 3
2

2
Bi 4: Chng minh phng trnh x
5
5x
4
+ 30x
3
50x
2
+ 55x 21 = 0 c nghim duy
nht
x = 1 +
5

2
5

4 +
5

8
5

16
Gii
t f(x) = x
5
5x
4
+ 30x
3
50x
2
+ 55x 21.
Ta c f

(x) = 5x
4
20x
3
+ 90x
2
100x + 55 = 5(x
2
2x + 3)
2
+ 10(2x 1)
2
> 0 x, do
phng trnh f(x) = 0 c khng qu 1 nghim.
Ta s chng minh nghim l x = 1 +
5

2
5

4 +
5

8
5

16:
t
x = 1 +
5

2
5

4 +
5

8
5

16

2x =
5

2 +
5

4
5

8 +
5

16 2
x +
5

2x = 2
5

2 1 x + 1 =
5

2(2 x)
(x + 1)
5
= 2(2 x)
5
Khai trin biu thc trn, sau rt gn, ta c phng trnh:
x
5
5x
4
+ 30x
3
50x
2
+ 55x 21 = 0
31
Vy ta c iu phi chng minh. 2
Bi 5: Chng minh phng trnh sau c 7 nghim thc:
g(x) = x
9
9x
7
+ 3x
6
+ 27x
5
18x
4
27x
3
+ 27x
2
1 = 0 ()
Gii
t f(x) = x
3
3x + 1 th g(x) = f(f(x)). Ta s tm nghim ca g

(x) = f

(x).f

(f(x)):
Nghim ca f

(x) l: f

(x) = 0 3x
2
3x = 0 x = 1.
tm nghim ca f

(f(x)) ta tm nghim ca f(x) = 1 v f(x) = 1:


f(x) = 1 x
3
3x + 2 = 0 x 2; 1
f(x) = 1 x
3
3x = 0 x 0;

3
Nh vy tp nghim ca phng trnh g

(x) = 0 l 2;

3; 1; 0; 1;

3.
Suy ra g(x) c ti a 7 nghim. Li c:

g(x) khi x
g(2) = 3 > 0 g(x) c 1 nghim trong (; 2)
g(

3) = 1 < 0 g(x) c 1 nghim trong (2;

3)
g(1) = 19 > 0 g(x) c 1 nghim trong (

3; 1)
g(0) = 1 < 0 g(x) c 1 nghim trong (1; 0)
g(1) = 3 > 0 g(x) c 1 nghim trong (0; 1)
g(

3) = 1 < 0 g(x) c 1 nghim trong (1;

3)
g(x) + khi x + g(x) c 1 nghim trong (

3; +)
Nh vy g(x) = f(f(x)) c 7 nghim thc. 2
CnucNo II: PHNO TRNH. H PHNO
TRNH C THAM S
PHNG PHP S DNG O HM
L thuyt
i vi bi ton tm iu kin ca tham s phng trnh f(x) = g(m) c nghim min D ta
da vo tnh cht: phng trnh c nghim khi v ch khi hai th ca hai hm s y = f(x)
v y = g(m) ct nhau. Do bi ton ny ta tin hnh theo cc bc sau:
Bc 1: Lp bng bin thin ca hm s y = f(x) .
Bc 2: Da vo bng bin thin ta xc nh m ng thng y = g(m) ct th hm s
y = f(x).
Ch : Nu hm s y = f(x) lin tc trn D v m = Min
D
f(x) , M = Max
D
f(x) th phng
trnh: k = f(x) c nghim m k M
Sau y l mt s bi tp v d:
Bi tp v d
Bi 1:Tm m phng trnh

x
2
+x + 1

x
2
x + 1 = m c nghim.
Gii
Xt hm s f(x) =

x
2
+x + 1

x
2
x + 1 vi x R c
f

(x) =
2x + 1
2

x
2
+ x + 1

2x 1
2

x
2
x + 1
Ta s tm nghim ca f

(x):
f

(x) = 0
2x + 1
2

x
2
+x + 1

2x 1
2

x
2
x + 1
= 0
(2x + 1)

x
2
x + 1 = (2x 1)

x
2
+x + 1

x +
1
2

x
1
2

2
+
3
4

=(x
1
2
)
2

x +
1
2

2
+
3
4

x +
1
2

2
=

x
1
2

2
x = 0
Th li ta thy x = 0 khng l nghim ca f

(x). Suy ra f

(x) khng i du trn R, m


f

(0) = 1 > 0 f

(x) > 0x R . Vy hm s f(x) ng bin trn R.


Mt khc: lim
x+
f(x) = lim
x+
2x

x
2
+x + 1 +

x
2
x + 1
= 1 v lim
x
f(x) = 1.
32
33
Da vo bng bin thin ta thy phng trnh cho c nghim khi v ch khi m (1; 1) 2
Bi 2: Tm m phng trnh
4

x
2
+ 1

x = m c nghim
Gii
KX: x 0
Xt hm s f(x) =
4

x
2
+ 1

x vi x 0 ta c f(x) lin tc trn [0; +)


Li c: f

(x) =
x
2
4

(x
2
+ 1)
3

1
2

x
<
x
2
4

x
6

1
2

x
= 0 x > 0.
Suy ra hm s f(x) nghch bin trn [0; +)
Mt khc: lim
x+
f(x) = 0
Do phng trnh cho c nghim khi v ch khi m (0; 1] 2 .
Nhn xt: i khi ta phi tm cch c lp m a phng trnh v dng trn.
Bi 3: Tm m phng trnh sau c nghim:
4

x
4
13x +m+x 1 = 0 (*)
Gii
Ta c:
()
4

x
4
13x +m+x1 = 0

1 x 0
x
4
13x +m = (1 x)
4

1 x
4x
3
6x
2
9x = 1 m
Xt hm s f(x) = 4x
3
6x
2
9x vi x 1
Ta c: f

(x) = 12x
2
12x 9, f

(x) = 0

x =
3
2
x =
1
2
.
Da vo bng bin thin suy ra phng trnh c nghim khi v ch khi m
3
2
. 2
Bi 4: Tm m phng trnh sau c nghim: x

x +

x + 12 = m(

5 x +

4 x)
Gii
KX: x [0; 4].
Khi phng trnh tng ng vi
(x

x +

x + 12)(

5 x

4 x) = m
Xt hm s f(x) = (x

x +

x + 12)(

5 x

4 x) lin tc trn on [0; 4].


Ta c: f

(x) =

3
2

x +
1
2

x + 12

1
2

4 x

1
2

5 x

> 0 x [0; 4]
Vy f(x) l hm ng bin trn [0; 4].
Suy ra phng trnh c nghim khi v ch khi 2

3(

5 2) m 12 2 .
Bi 5: Tm m h sau c nghim: ()

2
x
2

1
2

45x
3x
2
mx

x + 16 = 0
34
Gii
Ta c:
()

x
2
4 + 5x
3x
2
mx

x + 16 = 0

x [1; 4]
3x
2
mx

x + 16 = 0
m =
3x
2
+ 16
x

x
Xt f(x) =
3x
2
+ 16
x

x
vi x [1; 4]. Ta c:
f

(x) =
6x
2

x
3
2

x(3x
2
+ 16)
x
3
=
3

x(x
2
16)
2x
3
0x [1; 4]
Nh vy m = f(x) nghch bin trn [1; 4], do f(4) m f(1) 8 m 19
Vy h c nghim khi v ch khi m [8; 19] 2
Nhn xt: Khi gp h phng trnh trong mt phng trnh ca h khng cha tham s
th ta s i gii quyt phng trnh ny trc. T phng trnh ny ta s tm c tp nghim
(i vi h mt n) hoc s rt c n ny qua n kia. Khi nghim ca h ph thuc vo
nghim ca phng trnh th hai vi kt qu ta tm c trn.
Bi 6: Tm m h sau c nghim:

7
2x+

x+1
7
2+

x+1
+ 2007x 2007
x
2
(m+ 2)x + 2m+ 3 = 0
Gii
Ta c:
7
2x+

x+1
7
2+

x+1
+ 2007x 2007 7
2+

x+1
(7
2x+2
1) 2007(1 x) ()
Nu x > 1 7
2+

x+1
(7
2x2
1) > 0 > 2007(1 x).Suy ra (*) v nghim.
Nu x 1 7
2+

x+1
(7
2x2
1) 0 2007(1 x). Suy ra (*) ng .
Suy ra h c nghim khi v ch khi phng trnh x
2
(m + 2)x + 2m + 3 = 0 c nghim vi
x [1; 1], hay phng trnh m =
x
2
2x + 3
x 2
c nghim vi x [1; 1].
Xt hm s f(x) =
x
2
2x + 3
x 2
vi x [1; 1], c
f

(x) =
x
2
4x + 1
(x 2)
2
= 0 x = 2

3
Da vo bng bin thin suy ra h c nghim khi v ch khi m 2

3.
Bi 7: Tm m h phng trnh sau c nghim: ()

x y +m = 0
y +

xy = 2
Gii
35
KX: xy 0. T h ta cng c y = 0.
()

x y + m = 0

xy = 2 y

x y + m = 0
x =
y
2
4y + 4
y
y 2
m = y
y
2
4y + 4
y
=
4y 4
y
(y 2)
Xt hm s f(y) =
4y 4
y
(y 2) ta c f

(y) =
4
y
2
> 0 y = 0, suy ra hm s f(y) ng bin
trn cc khong (; 0) v (0; 2].
Mt khc, lim
y
f(y) = 4, lim
y0
+
f(y) = ; lim
y0

f(y) = +
Suy ra h c nghim khi v ch khi m (; 2] (4; +) 2
Bi 8: Tm m phng trnh c ng hai nghim phn bit:

x
4
4x
3
+ 16x +m+
4

x
4
4x
3
+ 16x +m = 6 ()
Gii
Ta c:
()
4

x
4
4x
3
+ 16x +m = 2 m = x
4
+ 4x
3
16x + 16
Xt hm s f(x) = x
4
+ 4x
3
16x + 16 vi x R. Ta c:
f

(x) = 4x
3
+ 12x
2
16; f

(x) = 0

x = 1
x = 2
Da vo bng bin thin suy ra phng trnh c hai nghim phn bit khi v ch khi m < 27.
Bi 9: Tm m phng trnh m

x
2
+ 2 = x + m () c ba nghim phn bit.
Gii
T (*) ta c: () m =
x

x
2
+ 2 1
Xt hm s: f(x) =
x

x
2
+ 2 1
vi x R.
Ta c: f

(x) =
2

x
2
+ 2

x
2
+ 2(

x
2
+ 2 1)
2
; f

(x) = 0 x =

2 .
Da vo bng bin thin phng trnh c nghim khi v ch khi

2 < m <

2 2
Bi 10: Tm m phng trnh: mx
2
+ 1 = cos x () c ng mt nghim x

0;

2

Gii
Ta thy (*) c nghim th m 0. Khi
mx
2
+ 1 = cos x m =
cos x 1
x
2
2m =
sin
2
x
2
x
2
4
36
Xt hm s f(t) =
sin
2
t
t
2
vi t

0;

4

Ta c
f

(t) =
2t
2
sin t cos t 2tsin
2
t
t
4
=
2 sin t(t cos t sin t)
t
3
=
sin 2t(t tan t)
t
3
< 0 t

0;

4

Suy ra hm s f(t) nghch bin trn (0;



4
).
Da vo bng bin thin suy ra phng trnh c ng mt nghim trn (0;

4
) khi v ch khi
8

2
< 2m < 1
1
2
< m <
4

2
2
Bi 11: Tm m h phng trnh c ba cp nghim phn bit:
()

3(x + 1)
2
+ y m = 0
x +

xy = 1
Gii
iu kin xy 0
Ta c
()

3(x + 1)
2
+ y = m

xy = 1 x

3(x + 1)
2
+y = m
y =
x
2
2x + 1
x
y 1
m = 3(x + 1)
2
+
x
2
2x + 1
x
m3 = 3x
2
+ 6x +
x
2
2x + 1
x
Xt hm s: f(x) = 3x
2
+ 6x +
x
2
2x + 1
x
(x 1) ta c
f

(x) =
6x
3
+ 7x
2
1
x
2
= 0

x = 1
x =
1
2
x =
1
3
Da vo bng bin thin ta thy h phng trnh c ba nghim phn bit khi
m [4;
15
4
] [
20
3
; 12] 2
Nhn xt: Khi t n ph ta phi tm min xc nh ca n ph v gii quyt bi ton n
ph trn min xc nh va tm. C th:
* Khi t t = u(x)(x D), ta tm c t D
1
v phng trnh f(x, m) = 0 (1) tr thnh
g(t, m) = 0 (2). Khi (1) c nghim x D (2) c nghim t D
1
.
* tm min xc nh ca t ta c th s dng cc phng trnh tm min gi tr (v min xc
nh ca t chnh l min gi tr ca hm u(x)).
* Nu bi ton yu cu xc nh s nghim th ta phi tm s tng ng gia x v t, tc l mi
gi tr t D
1
th phng trnh t = u(x) c bao nhiu nghim x D.
Bi 12: Tm m phng trnh m(

x 2 + 2
4

x
2
4)

x + 2 = 2
4

x
2
4 c nghim
37
Gii
KX; x 2
Ta thy x = 2 khng l nghim ca phng trnh nn ta chia hai v phng trnh cho
4

x
2
4:
m

x 2
x + 2
+ 2

x + 2
x 2
= 2 ()
t t =
4

x + 2
x 2
(t > 1). Khi (*) tr thnh: m

1
t
+ 2

t = 2 m =
t
2
+ 2t
2t + 1
.
Xt hm s f(t) =
t
2
+ 2t
2t + 1
(t > 1) ta c f

(t) =
2t
2
+ 2t + 2
(2t + 1)
2
> 0 t > 1.
Vy hm s f(t) ng bin trn (1; +).
Vy phng trnh c nghim khi v ch khi m > 1 2
Nhn xt: Trong cc bi ton trn sau khi t n ph ta thng gp kh khn khi xc nh
min xc nh ca t . tm c iu kin ca n ph t, chng ta c th dng cng c hm
s, bt ng thc, lng gic ha. . .
Bi 13: Tm m phng trnh log
2
3
x +

log
2
3
x + 1 2m1 = 0 c nghim trn

1; 3

Gii
t t =

log
2
3
x + 1. Do x

1; 3

1 t 2
Phng trnh cho tr thnh: t
2
+t = 2m+ 2
Xt hm s f(t) = t
2
+t vi 1 t 2, ta thy f(t) l hm ng bin trn [1; 2]
Vy phng trnh c nghim khi v ch khi 2 2m+ 2 6 0 m 2 2
Bi 14: Xc nh m h sau c 2 nghim phn bit

log

3
(x + 1) log

3
(x 1) > log
3
4
log
2
(x
2
2x + 5) mlog
(x
2
2x+5)
2 = 5
Gii
iu kin : x > 1. T bt phng trnh th nht ca h ta c: log

3
x + 1
x 1
> log

3
2 x (1; 3).
t t = log
2
(x
2
2x + 5)(t (2; 3))v phng trnh th hai ca h tr thnh
t +
m
t
= 5 t
2
5t = m
T cch t t ta c: Vi mi gi tr t (2; 3) th cho ta ng mt gi tr x (1; 3). Suy ra h c
2 nghim phn bit khi v ch khi phng trnh t
2
5t = m c 2 nghim phn bit t (2; 3).
Xt hm s f(t) = t
2
5t vi t (2; 3). Ta c f

(t) = 2t 5; f

(t) = 0 t =
5
2
. Da vo bng
bin thin ta c, h phng trnh c 2 nghim phn bit khi v ch khi m (6;
25
4
) 2
Bi 15:( thi H khi B - 2004) Tm m phng trnh c nghim:
m(

1 + x
2

1 x
2
+ 2) = 2

1 x
4
+

1 + x
2

1 x
2
()
38
Gii
iu kin: 1 x 1
Trc tin, ta nhn thy rng: (1 x
2
) (1 +x
2
) = 1 x
4
v 1 x
2
+1 +x
2
= 2 nn ta c php
t n ph nh sau:
t t =

1 + x
2

1 x
2
Phng trnh cho tr thnh:
m(t + 2) = t
2
+ t + 2
t
2
+t + 2
t + 2
= m (1)
Do

1 + x
2

1 x
2
t 0
Mt khc: t
2
= 2 2

1 x
4
2 t

2
Ta xt hm s: f (t) =
t
2
+ t + 2
t + 2
, t

0;

, ta c:f

(t) =
t
2
4t
(t + 2)
2
0
Vy hm f(t) nghch bin trn on

0;

. M hm s lin tc trn

0;

nn phng trnh
cho c nghim x khi phng trnh (1) c nghim t

0;

.
iu ny tng ng vi:
min f (t) m max f (t) t

0;

m f (0)
Vy cc gi tr m cn tm l

2 1 m 1 2.
Bi 16: Tm m phng trnh sau c nghim:
x

x +

x + 12 = m

5 x +

4 x

Gii
Cng ging nh nhng bi ton trc, bi ny ta ngh ngay l phi a bi ton v dng
f (x) = m ri s dng tng giao gia 2 th v suy ra iu kin m.
Ta gii bi ton nh sau:
iu kin: 0 x 4
Phng trnh cho tng ng vi:
x

x +

x + 12

5 x +

4 x
= m
t f (x) =
x

x +

x + 12

5 x +

4 x
x [0; 4]
Tuy nhin, nu n y ta kho st hm s ny th c v kh phc tp v di dng. V vy ta
s gii quyt theo mt hng khc. rng, ta c tnh cht sau:
Vi hm s y =
f (x)
g (x)
. Nu y = f (x) ng bin v y = g (x) nghch bin th y =
f (x)
g (x)
ng
bin.
Ta vn dng tnh cht trn nh sau:
Xt hm s g (x) = x

x +

x + 12 ta c:
g

(x) =
3
2

x +
1
2

x + 12
> 0 nn hm g(x) l ng bin.
Xt hm s h(x) =

5 x +

4 x ta c:
h

(x) =
1
2

5 x

1
2

4 x
< 0 nn hm h(x)l nghch bin
39
Vy, theo tnh cht trn ta c hm y =
g (x)
h(x)
ng bin x [0; 4].
Do phng trnh f(x) = m c nghim khi v ch khi
f(0) m f(4) 2

15

12

m 12 2.
Bi 17: Gii v bin lun phng trnh sau theo m:

x
2
2x + m
2
= [x 1[ m (1)
Gii
Xt x 1 ta c:
(1)

x
2
2x +m
2
= x 1 m

x 1 m 0
x
2
2x +m
2
= (x 1 m)
2

x 1 + m
2mx = 2m+ 1
Nu m = 0: h trn v nghim
Nu m = 0 x =
2m+ 1
2m
Ta c x 1 + m
2m+ 1
2m
1 + m
2m
2
+ 1
2m
0 m

2
2
0 < m <

2
2
Li c x 1
2m+ 1
2m
1 m > 0
Kt hp iu kin trn ta c 0 < m <

2
2
.
- Xt x < 1:
(1)

x
2
2x +m
2
= 1 x m

1 x m 0
x
2
2x +m
2
= (1 x m)
2

2mx = 2m1
x 1 m
Kt lun:
Nu m = 0: h v nghim
Nu 0 < m <
1
3
m > 3: phng trnh c 2 nghim:x =
1 + m

3m
2
+ 10m3
1 m
Nu m = 3: phng trnh c nghim duy nht x = 1
Nu m =
1
3
: phng trnh c nghim duy nht x = 1.2
Bi 18: Tm m phng trnh sau c nghim
x
2
+

x
2
+x + 1

2
=

x
2
+ 1

2
+m

x
2
x + 1

2
Gii
rng: (x
2
+ 1)
2
x
2
= (x
2
+x + 1) (x
2
x + 1)
Do , phng trnh cho tng ng vi:

x
2
+x + 1

2
=

x
2
+x + 1

x
2
x + 1

+m

x
2
x + 1

2
Do x
2
x + 1 > 0, x nn chia hai v ca phng trnh cho x
2
x + 1 ta c:

x
2
+ x + 1
x
2
x + 1

x
2
+x + 1
x
2
x + 1
= m
40
t t =
x
2
+x + 1
x
2
x + 1
,
1
3
t 3
Phng trnh trn tr thnh: t
2
t = m
y l mt phng trnh bc hai n gin nn vic kho st xin dnh cho bn c.
iu lu y l iu kin ca t. Thc cht y ta tm gi tr ln nht v nh nht
ca biu thc
x
2
+x + 1
x
2
x + 1
(c th dng phng php min gi tr).
Bi 19: Tm m h sau c nghim thc:

x +
1
x
+ y +
1
y
= 5
x
3
+
1
x
3
+y
3
+
1
y
3
= 15m10
Gii
Cch 1:
Nhn vo h ny, ta thy ngay c hng i l phi t n ph v gia cc i lng x +
1
x
v x
3
+
1
x
3
dng nh c mi lin h vi nhau. Vi tng , ta c php t nh sau:
t

a = x +
1
x
b = y +
1
y
, ([a[ 2, [b[ 2).
Ta c

x
3
+
1
x
3
=

x +
1
x

3
3

x +
1
x

= a
3
3a
y
3
+
1
y
3
=

y +
1
y

3
3

y +
1
y

= b
3
3b
Khi , h cho tr thnh:

a +b = 5
a
3
+ b
3
3 (a + b) = 15m10

a +b = 5
ab = 8 m
D thy a, b l nghim ca phng trnh X
2
5X + 8 m = 0 X
2
5x + 8 = m (1) Xt
hm s f (X) = X
2
5X + 8, [X[ 2, ta c:
f

(X) = 2X 5 f

(X) = 0 X =
5
2
T , k bng bin thin v ch rng h cho c nghim thc khi v ch khi phng trnh
(1) c nghim [X[ 2. Ta tm c:
7
2
m 2 m 22.
Cch 2:
Ta nhn thy rng phng trnh th nht khng c cha tham s nn ta s xut pht t
phng trnh ny. Khai trin phng trnh ny ra, ta c:
x
3
y
3
+ 4 (x y) = 9y + 8 3x
2
+ 6y
2
x
3
+ 3x
2
+ 4x = y
3
+ 6y
2
+ 13y + 8
(x + 1)
3
+ (x + 1) = (y + 2)
3
+ (y + 2)
Xt hm s f (t) = t
3
+t, d thy l hm s ny ng bin nn
f (x + 1) = f (y + 2) x + 1 = y + 2 x = y + 1
41
T y, thay x = y + 1 vo phng trnh th hai ta c:

15 + 2y y
2
= 2m+

4 y
2

(5 y) (y + 3)

4 y
2
= 2m
n y tng r, ta ch cn chuyn v tng giao gia hai th.
Bi 20: Tm m h sau c nghim thc:

x
3
+ (y + 2) x
2
+ 2xy = 2m3
x
2
+ 3x + y = m
Gii
tng: h ny ta quan st thy bi ton cn cha r ng li no v c hai phng
trnh trong h u cha n tham s m. V vy i n hng gii quyt tt ta nn bt u
phn tch hai v tri trong hai phng trnh trong h. C th ta c:
x
3
+ (y + 2) x
2
+ 2xy = x
3
+yx
2
+ 2x
2
+ 2xy = x
2
(x + y) + 2x (x +y) = (x +y)

x
2
+ 2x

Mt khc:
x
2
+ 3x +y = x
2
+ 2x +x +y
R rng bc phn tch ny ta tm ra li gii cho bi ton ny chnh l t n ph.
Li gii:
t:

a = x
2
+ 2x 1
b = x +y
ta c h phng trnh

a +b = m
ab = 2m3

a
2
3 = (a + 2) m(1)
b = ma
T phng trnh (1) trong h ta c:
a
2
3
a + 2
= m (2)
H cho c nghim khi v ch khi phng trnh (2) c nghim a 1.
Xt hm s: f (x) =
x
2
3
x + 2
vi x 1
n y ta ch cn lp bng bn thin. Cng vic tip theo xin dnh cho bn c.
Bi tp t luyn
Bi 1: Tm m phng trnh tan
2
x + cot
2
x +m(cot x + tan x) = 3 c nghim
Bi 2: Tm m phng trnh

x +

x + 9 =

9x x
2
+m c nghim
Bi 3: Tm m phng trnh

3 + x +

x + 6

18 + 3x x
2
= m c nghim
Bi 4: Tm m phng trnh x
3
4mx
2
+ 8 = 0 c 3 nghim phn bit.
Bi 5: Tm m phng trnh x
3
+ 3x
2
+ (3 2m) x + m + 1 = 0 c ng mt nghim ln
hn 1.
Bi 6: Tm m phng trnh sau c ng 2 nghim thc phn bit:
4x
2
2mx + 1 = 3

8x
3
+ 2x
42
PHNG PHP DNG NH L LAGRANGE-ROLLE
L thuyt
nh l Rolle: : Nu f(x) l hm lin tc trn on [a; b], c o hm trn khong (a; b)
v f(a) = f(b) th tn ti c (a; b) sao cho f

(c) = 0.
T ta c 3 h qu:
H qu 1: Nu hm s f(x) c o hm trn (a; b) v f(x) c n nghim (n l s nguyn
dng ln hn 1) trn (a; b) th f

(x) c t nht n - 1 nghim trn (a; b).


H qu 2: Nu hm s f(x) c o hm trn (a; b) v f

(x) v nghim trn (a; b) th f(x)c


nhiu nht 1 nghim trn (a; b).
H qu 3: Nu f(x)c o hm trn (a; b) v f

(x) c nhiu nht n nghim (n l s nguyn


dng) trn (a; b) th f(x) c nhiu nht n + 1 nghim trn (a; b).
Cc h qu trn vn ng nu cc nghim l nghim bi (khi f(x) l a thc) v cho ta
tng v vic chng minh tn ti nghim cng nh xc nh s nghim ca phng trnh, v
nu nh bng mt cch no ta tm c tt c cc nghim ca phng trnh th ngha l
khi phng trnh c gii.
nh l Lagrange: : Nu f(x)l hm lin tc trn on [a; b], c o hm trn khong
(a; b)th tn ti c (a; b) sao cho f

(c) =
f(b) f(a)
b a
.
Sau y l mt s ng dng ca hai nh l trn:
Bi tp v d
Dng nh l Lagrange - Rolle bin lun phng trnh:
Bi 1: Chng minh rng phng trnh a cosx + b cos 2x + c cos 3x = 0 lun c nghim vi
mi a, b, c.
Gii
Xt f(x) = a sin x +
b sin 2x
2
+
c sin 3x
3
ta c f

(x) = a cosx +b cos 2x +c cos 3x


M f(0) = f() = 0 x
0
(0; ) : f

(x
0
) = 0, suy ra iu phi chng minh. 2
Bi 2: Cho a + b + c = 0. Chng minh rng phng trnh sau c t nht 4 nghim thuc
[0; ]: a sin x + 9b sin3x + 25c sin 5x = 0
Gii
chng minh f(x) c t nht n nghim ta chng minh F(x) c t nht n+1 nghim vi F(x)
l mt nguyn hm ca f(x) trn (a; b) (c th phi p dng nhiu ln)
Xt hm s f(x) = a sin x b sin 3x c sin 5x, ta c:
f

(x) = a cos x 3b cos 3x 5c cos 5x; f

(x) = a sin x + 9b sin 3x + 25c sin 5x


43
Ta c f(0) = f(

4
) = f(
3
4
) = f() = 0
Suy ra x
1
(0;

4
), x
2
(

4
;
3
4
), x
3
(
3
4
; ) : f(0) = f

(x
1
) =

f(x
2
) = f

(x
3
) = 0
x
4
(x
1
; x
2
), x
5
(x
2
; x
3
)[f

(x
4
) = f

(x
5
) = 0
m f

(0) = f

() = 0, suy ra iu phi chng minh. 2


Bi 3: Chng minh phng trnh x
5
5x + 1 = 0 c ng ba nghim thc.
Gii
t f(x) = x
5
5x + 1 th f(2) = 21 < 0, f(0) = 1 > 0, f(1) = 3 < 0, f(2) = 23 > 0
nn t y suy ra phng trnh cho c ba nghim thc
Gi s c nghim th t ca phng trnh. p dng nh l Rolle ta c:
f

(x) = 0 5x
4
5 = 0. Phng trnh ny c hai nghim thc nn suy ra mu thun.
Vy ta c iu phi chng minh 2.
Bi 4: Cho a thc P(x) v Q(x) = aP(x) + bP

(x) trong a, b l cc s thc, a =0.


Chng minh rng nu Q(x) v nghim th P(x) v nghim.
Gii
Ta c deg P(x) = deg Q(x)
V Q(x) v nghim nn deg Q(x) chn. Gi s P(x) c nghim, v deg P(x) chn nn P(x) c
t nht 2 nghim.
Khi P(x) c nghim kp x = x
0
ta c x
0
cng l mt nghim ca P

(x) suy ra Q(x) c


nghim.
Khi P(x) c hai nghim phn bit x
1
< x
2
:
Nu b = 0 th hin nhin Q(x) c nghim.
Nu b =0 : Xt f(x) = e
a
b
x
P(x) th f(x)c hai nghim phn bit x
1
< x
2
v
f

(x) =
a
b
e
a
b
x
P(x) + e
a
b
x
P

(x) =
1
b
e
a
b
x
[aP(x) + bP

(x)] =
1
b
e
a
b
x
Q(x)
V f(x) c hai nghim suy ra f

(x) c t nht 1 nghim hay Q(x) c nghim.


Tt c trng hp u mu thun vi gi thit Q(x) v nghim. Vy khi Q(x) v nghim th
P(x) v nghim 2
Bi 5: Gi s phng trnh sau c n nghim phn bit:
a
0
x
n
+ a
1
x
n1
+... +a
n1
x +a
n
= 0, (a
0
= 0)
Chng minh (n 1)a
2
1
> 2na
0
a
2
Gii
t a
0
x
n
+ a
1
x
n1
+... +a
n1
x +a
n
= f(x)
Nhn xt f kh vi v hn trn R nn suy ra
f

(x) c n 1 nghim phn bit.


f

(x) c n 2 nghim phn bit.


. . . ..
f
[n2]
(x) =
n!
2
a
0
x
2
+ (n 1)!a
1
x + (n 2)!a
2
c 2 nghim phn bit.
44
Nhn thy f
[n2]
(x) c > 0 nn ((n 1)!a
1
)
2
2n!a
0
(n 2)!a
2
> 0
Suy ra iu phi chng minh. 2
Bi 6: (VMO 2002) Xt phng trnh
n

i=1
1
i
2
x 1
=
1
2
, vi n l s nguyn dng. Chng
minh rng vi mi s nguyn dng n, phng trnh nu trn c mt nghim duy nht ln
hn 1; k hiu nghim l x
n
Gii
Xt f
n
(x) =
n

i=1
1
i
2
x 1

1
2
, ta c: f
n
(x) lin tc v nghch bin trn (1; +)
M lim
x1
+
f
n
(x) = +, lim
x+
f
n
(x) =
1
2
f
n
(x) = 0 c mt nghim duy nht ln hn 1. 2
Dng nh l Lagrange -Rolle gii phng trnh:
Bi 7: Gii phng trnh 3
x
+ 5
x
= 2.4
x
()
Gii
Nhn xt: x = 0; x = 1 l nghim ca phng trnh (*).
Gi x
0
l nghim khc ca phng trnh cho. Ta c:
3
x
0
+ 5
x
0
= 2.4
x
0
5
x
0
4
x
0
= 4
x
0
3
x
0
()
Xt hm s f(t) = (t + 1)
x
0
t
x
0
, ta c () f(4) = f(3)
V f(t) lin tc trn [3; 4] v c o hm trong khong (3; 4), do theo nh l Rolle tn ti
c (3; 4) sao cho
f

(c) = 0 x
0
[(c + 1)
x
0
1
c
x
0
1
]=0

x
0
= 0
x
0
= 1
(loi)
Vy phng trnh (*) c tp nghim S = 0; 1. 2
Bi 8: Gii phng trnh 5
x
3
x
= 2x ()
Gii
Nhn xt: x = 0; x = 1 l nghim ca phng trnh (2). Gi x
0
l nghim khc ca phng
trnh cho, ta c:
5
x
0
5x
0
= 3
x
0
3x
0
(2a)
Xt hm s: f(t) = t
x
0
tx
0
, khi : (2a) f(5) = f(3)
V f(t) lin tc trn [3; 5] v c o hm trn (3; 5), do theo nh l Lagrange lun tn ti
c (3; 5) sao cho
f

(c) = 0 x
0
(c
x
0
1
1)=0

x
0
= 0
x
0
= 1
(loi)
Vy phng trnh (*) c tp nghim S = 0; 1 2
Bi 9: Gii phng trnh: 3
x
+ 2.4
x
= 19x + 3 ()
45
Gii
Ta c
() 3
x
+ 2.4
x
19x 3 = 0
Xt hm s: y = f(x) = 3
x
+ 2.4
x
19x 3 ta c: f

(x) = 3
x
ln 3 + 2.4
x
ln 4 19 ta c
f

(x) = 3
x
(ln 3)
2
+ 2.4
x
(ln 4)
2
> 0, x R hay f

(x) v nghim, suy ra f

(x) c nhiu nht 1


nghim, suy ra f(x) c nhiu nht 2 nghim.
M f(0) = f(2) = 0 do (*) c ng hai nghim x = 0, x = 2 2
Bi 10: Gii phng trnh: (1 + cos x)(2 + 4
cos x
) = 3.4 cos x ()
Gii
t t = cos x, (t [1; 1])
Ta c: () (1 + t)(2 + 4
t
) = 3.4
t
(1 + t)(2 + 4
t
) 3.4
t
= 0
Xt hm s: f(t) = (1 +t)(2 + 4
t
) 3.4
t
ta c:
f

(t) = 2 + 4
t
+ (t 2)4
t
ln 4, f

(t) = 2.4
t
ln 4 + (t 2)4
t
ln
2
4
Li c: f

(t) = 0 t = 2 +
2
ln 4
, suy ra f

(t) c nghim duy nht.


Suy ra f

(t) c nhiu nht hai nghim, ngha l f(t) c nhiu nht ba nghim.
Mt khc d thyf(0) = f(
1
2
) = f(1) = 0, do f(t) c ba nghim t = 0,
1
2
, 1.
Kt lun: Nghim ca phng trnh (*) l: x =

2
+k2, x =

3
+k2, x = k2, k Z 2
Bi 11: Gii phng trnh 3
cos x
2
cos x
= 2
cos x
2 cos x ()
Gii
Xt hm f(t) = t
cos
t cos , f

(t) = cos (t
cos 1
1)
Ta nhn thy f(3) = f(2) v f(x) kh vi trn [2;3] nn p dng nh l Lagrange ta c:
c [2; 3] : f

(c) =
f(3) f(2)
1
cos (c
cos 1
1) = 0
T ta suy ra nghim ca phng trnh (*) l x =

2
+k, x = k2(k Z) 2
46
PHNG PHP DNG IU KIN CN V
L thuyt
Bi ton:
Cho h phng trnh (hoc h bt phng trnh) cha tham s c dng:
(I)

f(x, m) = 0
x D
x
m D
m
hoc (II)

f(x, m) 0
x D
x
m D
m
Trong x l bin s, m l tham s, D
x
, D
m
l min xc nh ca x v m.
Yu cu t ra: ta phi tm gi tr ca tham s m h (I) hc (II) tha mn mt tnh cht
no .
Phng php gii:
Bc 1 (iu kin cn): Gi s h tha mn tnh cht P no m u bi i hi. Khi
, da vo c th ca tnh cht P v dng ca phng trnh ta s tm c mt rng buc no
i vi tham s m v rng buc y chnh l iu kin cn c tnh cht P. iu c ngha
l: nu vi m
0
khng tha mn rng buc trn th chc chn ng vi m
0
, h khng c tnh cht P.
Bc 2 (iu kin ): Ta tm xem trong cc gi tr ca m va tm c, gi tr no lm
cho h tha mn tnh cht P. bc ny ni chung ta cng ch cn gii nhng h c th khng
cn tham s. Sau khi kim tra, ta s loi i nhng gi tr khng ph hp v nhng gi tr cn
li chnh l p s ca bi ton.
Nh vy, tng ca phng php ny kh r rng v n gin. Trong rt nhiu bi ton v
bin lun th phng php ny li th hin u th r rt. Tuy nhin, thnh cng ca phng
php cn nm ch ta phi lm th no pht hin iiu kin cn mt cch hp l v chn
iu kin mt cch ng n.
Bi tp v d
S dng tnh i xng ca cc biu thc c mt trong bi ton
Bi 1: Tm m phng trnh sau c nghim duy nht
4

x +
4

1 x +

x +

1 x = m (1)
Gii
iu kin cn:
Gi s (1) c nghim duy nht x =
D thy nu (1) c nghim x = th (1) cng c nghim x = 1 . V nghim l duy nht
47
nn = 1 =
1
2
Thay =
1
2
vo (1) ta tm c m =

2 +
4

8.
iu kin :
Gi s m =

2 +
4

8, khi (1) c dng sau:


4

x +
4

1 x +

x +

1 x =

2 +
4

8 (2)
Theo bt ng thc AM-GM ta c:

x +

1 x

2 v
4

x +
4

1 x
4

8
Do (2) x = 1 x x =
1
2
.
Vy (1) c nghim duy nht th iu kin cn v l m =

2 +
4

8 2
Bi 2: Tm a v b phng trnh sau c nghim duy nht
3

(ax +b)
2
+
3

(ax b)
2
+
3

a
2
x
2
b
2
=
3

b (1)
Gii
iu kin cn:
Gi s (1) c nghim duy nht x = x
0
, khi d thy x = x
0
cng l nghim ca (1). Do
t gi thit ta suy ra x
0
= 0. Thay x
0
= 0 vo (1) ta c :
3

b
2
=
3

b = 0
b = 1
iu kin :
Khi b = 0, (1) c dng:
3

a
2
x
2
+
3

a
2
x
2
+
3

a
2
x
2
= 0 a
2
x
2
= 0
Do (1) c nghim duy nht khi v ch khi a = 0
Khi b = 1, (1) c dng:
3

(ax + 1)
2
+
3

(ax 1)
2
+
3

a
2
x
2
1 = 1 ()
t u =
3

ax + 1; v =
3

ax 1, ta thy:
()

u
3
v
3
= 2
u
2
+uv +v
2
= 1

u v = 2
u
2
+uv + v
2
= 1

u = 1
v = 1

ax + 1 = 1
ax 1 = 1
ax = 0
Vy (*) c nghim duy nht khi v ch khi a = 0
Tm li, phng trnh (1) c nghim duy nht th iu kin cn v l

a = 0; b = 0
b = 1
2
Bi 3: Tm m h sau c nghim duy nht:

7 + x +

11 x 4 = m

4 3

10 3m

7 + y +

11 y 4 = m

4 3

10 3m
48
Gii
iu kin: 7 x, y 11;
74
27
m
10
3
Tr theo v hai phng trnh ta c:

x + 7

11 x =

y + 7

11 y
Xt hm s: f (t) =

t + 7

11 t; 7 t 11 ta c:
f

(t) =
1
2

t + 7
+
1
2

11 t
> 0 Vy hm s ng bin, suy ra: f (x) = f (y) x = y.
Thay vo mt trong hai phng trnh ca h ta c:

7 + x +

11 x 4 = m

4 3

10 3m ()
iu kin cn:
Ta thy l nu x
0
l mt nghim ca phng trnh th 4x
0
cng l nghim ca phng trnh.
Nn h cho c nghim duy nht khi v ch khi
x
0
= 4 x
0
x
0
= 2
Thay vo phng trnh (*) ta c:

4 3

10 m = m2 ()
Gii phng trnh (**) ta tm c m = 3.
iu kin :
Vi m = 3, ta thu c h phng trnh:

7 + x +

11 x = 6

7 + y +

11 y = 6
V x = y nn ta ch vic gii phng trnh

7 + x +

11 x = 6 x = 2
Vy m = 3 l gi tr cn tm h cho c nghim duy nht. 2
Bi 4: Tm a,b h sau c nghim duy nht:

xyz +z = a
xyz
2
+z = b
x
2
+y
2
+z
2
= 4
Gii
iu kin cn:
Gi s (x
0
; y
0
; z
0
) l nghim ca h phng trnh cho th (x
0
; y
0
; z
0
) cng l nghim. Do
tnh duy nht nn x
0
= x
0
; y
0
= y
0
x
0
= y
0
= 0 Thay tr li vo h , ta c:

z
0
= a
z
0
= b
z
2
0
= 4
T y ta suy ra a = b = 2 hoc a = b = 2
iu kin :
49
Xt hai trng hp sau:
Nu a = b = 2: Khi h c dng:

xyz +z = 2 (1)
xyz
2
+ z = 2 (2)
x
2
+y
2
+z
2
= 4 (3)
Ly (1) (2) ta c xyz(1 z) = 0, t (1) li c z = 0 do xy(1 z) = 0
* Nu x = 0 z = 2 y = 0
* Nu y = 0 z = 2 x = 0
* Nu z = 1

x
2
+y
2
= 3
xy = 1
.
H trn c nghim (x
1
; y
1
) = (0; 0).V vy ngoi nghim (0, 0, 2), h cn c nghim khc
(x
1
; y
1
; 1) do h khng c nghim duy nht. Trng hp ny khng tha mn.
Nu a = b = 2:
Khi h c dng:

xyz = 2
xyz
2
2 + z = 2
x
2
+y
2
+z
2
= 4
Tin hnh lm nh trng hp trn ta i n:
* Nu x = 0 z = 2 y = 0
* Nu y = 0 z = 2 x = 0
* Nu z = 1

x
2
+y
2
= 3
xy = 3
Ta thy t h phng trnh trn, ta suy ra x
2
+y
2
< 2[xy[ nn h v nghim.
Vy trong trng hp ny h c duy nht nghim (x; y; z) = (0, 0, 2)
Vy iu kin cn v h phng trnh cho c nghim duy nht l a = b = 2 2
S dng im thun li
Bi 5: Tm a phng trnh sau nghim ng vi mi x:
log
2

a
2
x
2
5ax
2
+

6 a

= log
2+x
2

a 1

()
Gii
iu kin cn:
Gi s (*) ng vi mi x. Vi x = 0 ta c log
2

6 a = log
2
(3

a 1)
Li c:

1 a 6

a 1 < 3

a 1 +

6 a = 3
a 2; 5
iu kin :
50
Nu a = 2 th () log
2
(2 12x
2
) = log
2+x
2 2 ()
R rng (**) khng ng vi mi x, v log
2
(2 12x
2
) c ngha th phi c 12x
2
< 2
Nu a = 5 th () log
2
1 = log
2+x
2 1 (lun ng)
Vy a = 5 l iu kin cn v (*) ng vi mi x 2
Bi 6: Tm a h phng trnh n (x; y) c nghim vi mi b:

2
bx
+ (a + 1)by
2
= a
2
(a 1)x
3
+y
2
= 1
Gii
iu kin cn:
Gi s h c nghim vi mi b, thay b = 0 ta c

a
2
= 1
(a 1)x
3
+y
2
= 1
Do iu kin cn l a = 1
iu kin :
Nu a = 1: ta c h

2
bx
+ 2by
2
= 1
y
2
= 1
Khi b >
1
2
h v nghim. Vy trng hp ny loi.
Nu a = 1: ta c h

2
bx
= 1
2x
3
+ y
2
= 1
H trn lun c nghim (x; y) = (0; 1)
Vy a = 1 l iu kin cn v h phng trnh c nghim vi mi b 2
Bi 7: Tm a h phng trnh n (x; y) c nghim vi mi b:

(x
2
+ 1)
a
+ (b
2
+ 1)
y
= 2
a +bxy +x
2
y = 1
Gii
iu kin cn:
Gi s h c nghim vi mi b, thay b = 0 ta c
()

(x
2
+ 1)
a
= 1
a +x
2
y = 1

a = 0; x
2
y = 1
x
2
+ 1 = a +x
2
y = 1
a 0; 1
iu kin :
Nu a = 0 : ta c

(b
2
+ 1)
y
= 1 (1)
bxy +x
2
y = 1 (2)
51
Nu b = 0 b
2
+ 1 = 1 nn t (1) c y = 0, nhng khng tho (2). Vy trng hp ny loi.
Nu a = 1: ta c

x
2
+ (b
2
+ 1)
y
= 1
bxy +x
2
y = 0
H trn lun c nghim x = y = 0.
Vy a = 1 l iu kin cn v h cho c nghim vi mi b 2
Bi 8: Tm iu kin ca a, b, c, d, e, f hai phng trnh n (x; y) sau l tng ng:

ax
2
+ bxy +cy
2
+dx +ey +f = 0 (1)
x
2
+ y
2
= 1 (2)
Gii
iu kin cn:
Ta thy (x; y) = (0; 1) , (1; 0) ,

2
;
1

2
;
1

l nghim ca (2). Do (1)


cng phi c cc nghim trn.
Nh vy

c + e +f = c e +f = a + d +f = a d + f = 0
a +b +c +

2d +

2e + 2f
2
=
a +b +c

2d

2e + 2f
2
= 0
Gii h trn ta tm c iu kin cn ca bi ton l ()

b = d = e = 0
a = c = f = 0
iu kin :
D thy vi (*) th (2) trng vi (1).
Vy (*) l iu kin cn v (1) (2) 2
Bi 9: Cho phng trnh x
3
+ax +b = 0 (1)
Tm a, b phng trnh trn c ba nghim phn bit x
1
< x
2
< x
3
cch u nhau.
Gii
iu kin cn:
Gi s phng trnh (1) c 3 nghim khc nhau x
1
, x
2
, x
3
tha gi thit x
1
+ x
3
= 2x
2
Theo nh l Viete vi phng trnh bc 3 ta c: x
1
+ x
2
+x
3
= 0 3x
2
= 0 x
2
= 0
Thay x
2
= 0 vo (1) ta c b = 0
iu kin :
Gi s b = 0 , khi (1) tr thnh:
x
3
+ax = 0 x(x
2
+a) = 0 (2)
Ta thy (2) c 3 nghim phn bit nu a < 0. Khi cc nghim ca (2) l

x
1
=

a
x
2
= 0
x
3
=

a
52
Cc nghim trn cch u nhau nn iu kin cn v (1) c nghim tha mn bi l
b = 0, a < 0 2
Bi 10: Cho phng trnh
x
3
3x
2
+ (2m2) x + m3 = 0
Tm m phng trnh c ba nghim x
1
, x
2
, x
3
sao cho x
1
< 1 < x
2
< x
3
.
Gii
iu kin cn:
t f (x) = x
3
3x
2
+ (2m2) x +m3
Gi s phng trnh f (x) = 0 c 3 nghim x
1
, x
2
, x
3
tha mn x
1
< 1 < x
2
< x
3
Ta c: f (x) = (x x
1
) (x x
2
) (x x
3
), suy ra f (x) > 0 khi x
1
< x < x
2
Suy ra f (1) > 0 hay m5 > 0 m < 5.
iu kin :
Gi s m < 5.
Do lim
x
f (x) = nn tn ti < 1 m f () < 0
Li c: f (1) = m5 > 0 v f (x) lin tc nn ta c < x
1
< 1 sao cho f (x
1
) = 0
Ta c: f (0) = m3 < 0 ( do m < 5)
Vy tn ti 1 < x
2
< 0 sao cho f (x
2
) = 0
Mt khc, do lim
x+
f (x) = + nn phi c > 0 sao cho f () > 0.
T , tn ti x
3
m 0 < x
3
< sao cho f (x
3
) = 0.
Nh vy, phng trnh f (x) = 0 khi m < 5 c 3 nghim x
1
, x
2
, x
3
tha mn x
1
< 1 < x
2
<
0 < x
3
. Vy m < 5 chnh l iu kin cn v tha mn bi. 2
Bi 11: Tm m phng trnh sau c nghim:
20x
2
+ 10x + 3
3x
2
+ 2x + 1
= x
2
+ 2 (2m3) x + 5m
2
16m+ 20
Gii
iu kin cn:
Gi s phng trnh cho c mt nghim l x
0
.
t f (x) =
20x
2
+ 10x + 3
3x
2
+ 2x + 1
v g (x) = x
2
+ 2 (2m3) x + 5m
2
16m+ 20
Khi d thy rng:
max f (x) f (x
0
) = g (x
0
) min g (x)
Do min g (x) = g ((2m3)) = m
2
4m+ 11
Gi y
0
l gi tr ty ca f (x), khi ta c:
y
0
=
20x
2
+ 10x + 3
3x
2
+ 2x + 1
(20 3y
0
) x
2
+ 2 (5 y
0
) x + 3 y
0
= 0 (1)
Suy ra (1) c nghim khi:

= 2y
0
2
+ 19y
0
35 0
5
2
y
0
7 Vy max f (x) = 7
Nh vy, t iu kin max f (x) min g (x) ta c:
7 m
2
4m+ 11 (m2)
2
0 m = 2
53
iu kin :
Gi s m = 2, khi phng trnh cho tr thnh
20x
2
+ 10x + 3
3x
2
+ 2x + 1
= x
2
+ 2x + 8 (2)
Ta nhn thy rng g (x) = x
2
+ 2x + 8

= 7 x = 1
> 7 x = 1
Mt khc: f (x) =
20x
2
+ 10x + 3
3x
2
+ 2x + 1
7 vi x v f (1) = 6, 5. T y suy ra (2) v nghim.
Vy khng tn ti gi tr no ca m tha mn gi thit. 2
Bi 12: Tm m phng trnh x
2
3x + m = 0 (1) c mt nghim gp i nghim ca
phng trnh x
2
x +m = 0 (2)
Gii
iu kin cn: Gi s tn ti m tho mn iu kin u bi, tc l phng trnh (2) c
nghim x
0
, cn (1) c nghim 2x
0
. Vy ta c

4x
2
0
6x0 + m = 0
x
2
0
x0 + m = 0
Tr v vi v ca hai phng trnh trn cho nhau ta s tm c x
0
= 0 hoc x
0
=
5
3
Thay hai gi tr x
0
vo mt trong hai phng trnh trn ta c m = 0 v m =
10
9
y cng chnh l iu kin cn ca bi ton. iu kin :
Xt khi m = 0 v m =
10
9
ta s ln lt gii cc phng trnh

x
2
3x = 0
x
2
x = 0
x
2
3x +
10
9
= 0
x
2
x +
10
9
= 0
D thy nghim ca 2 cp phng trnh ny tho gi thit.
Vy m

0;
10
9

l iu kin cn v tha mn bi. 2


Bi 13: Tm a,b sao cho vi mi c phng trnh sau c khng qu hai nghim dng:
x
3
+ax
2
+ bx + c = 0
Gii
gii bi ny, ta s dng phn b. Ngha l ta tm a, b sao cho tn ti c phng trnh
x
3
+ax
2
+ bx +c = 0 c 3 nghim dng.
iu kin cn:
Gi s tn ti c phng trnh f(x) = x
3
+ax
2
+bx+c = 0 c 3 nghim dng x
1
< x
2
< x
3
.
Khi f(x) = (x x
1
)(x x
2
)(x x
3
)
Vy hm s f(x) c cc tr ti , > 0 (vi x
1
< < x
2
< < x
3
)
54
Phng trnh f

(x) = 3x
2
+ 2ax +b = 0 (1) c hai nghim dng.
Nh vy

= a
2
3b > 0
P =
b
3
> 0
S =
2a
3
> 0

a
2
> 3b
b > 0
a < 0
()

b > 0
a <

3b
(*) l iu kin cn ca bi ton.
iu kin :
Gi s a, b tho mn (*) th r rng phng trnh 3x
2
+ 2ax + b = 0 c 2 nghim dng
0 < <
Suy ra hm s f(x) = x
3
+ax
2
+bx + c c cc i ti x = v cc tiu ti x = .
Do > 0 nn tm c x
1
(c; ) sao cho f() < f(x
1
) < f() phng trnh f(x) = f(x
1
)
c ba nghim dng. t c = f(x
1
) th phng trnh x
3
+ ax
2
+ bx + c = 0 c 3 nghim
dng.
Vy (*) l iu kin cn v tn ti c sao cho phng trnh x
3
+ ax
2
+ bx + c = 0 c 3
nghim dng.
T , ta suy ra: b 0 hoc a

3b l iu kn cn v sao cho phng trnh


x
3
+ax
2
+bx +c = 0 c khng qu 2 nghim dng. 2
Bi tp t luyn
Bi 1: Tm m hai phng trnh sau l tng ng x
2
+ (m
2
5m+ 6) x = 0 v x
2
+
2 (m3) x +m
2
7m+ 12 = 0
Bi 2: Tm m phng trnh sau c nghim duy nht:
[x + m[
2
+[x + 1[ = [m+ 1[
Bi 3: Tm m phng trnh sau c nghim duy nht:

x + 3 +

6 x

(3 + x) (6 x) = m
Bi 4: Tm a h sau c ng mt nghim:

x
2
+ y
2
1 a

x + y 1

= 1
x +y = xy + 1
Bi 5: Tm a h sau c ng mt nghim:

x
2
+ 3 +[y[ = a

y
2
+ 5 +[x[ =

x
2
+ 5 +

3 a
Bi 6: Cho h phng trnh

(x +y)
4
+ 13 = 6x
2
y
2
+ m
xy

x
2
+ y
2

= m
Tm m h c nghim duy nht.
Bi 7: Tm s thc m sao cho h:

x
3
my
3
=
1
2
(m+ 1)
2
x
3
+mx
2
y +xy
2
= 1
c nghim (x; y) tho x + y = 0
55
PHNG PHP NG DNG HNH HC GII TCH
V HNH HC PHNG
L thuyt
Trong mt phng ta Oxy, cho cc vector:

u = (x
1
, y
1
),

v = (x
1
, y
2
) khi ta c:
[

u +

v [ [

u [ +[

v [

(x
1
+x
2
)
2
+ (y
1
+ y
2
)
2

x
2
1
+y
2
1
+

x
2
2
+y
2
2
Du bng xy ra khi v ch khi hai vector

u v

v cng hng
x
1
x
2
=
y
1
y
2
= k 0.
Vi hai vector

u ,

v bt k trong khng gian th

u .

v = [

u [ . [

v [ . cos (

u ,

v ) [

u [ . [

v [
(nh l cosin trong tam gic) Cho a, b, c l ba cnh ca tam gic ABC v A l gc
nh A ca tam gic khi :
a
2
= b
2
+c
2
2bc cos A
Cho tam gic ABC c ba gc nhn v im M ty trong mt phng khi gi T l
im nhn cc cnh BC, CA, AB di cng mt gc 120
0
th vi mi im M trn mt
phng ta c:
MA +MB +MC TA + TB + TC
T c gi l im Torricelli ca tam gic ABC.
Sau y l mt s dng ton thng gp:
Kho st h phng trnh cha dng tuyn tnh v phn tuyn tnh
Bi 1: Bin lun s nghim ca h

x
2
+y
2
= r
ax + by = c
(r>0)
Phn tch: H cho gm phng trnh ng trn (C) v phng trnh ng thng
d. Nh vy ta cn kho st s giao im ca (C) v d.
Cch 1: Lp cng thc tnh khong cch t gc to n d. Bin lun s giao im ca
ng thng v ng trn bng cch so snh khong cch vi bn knh ca (C).
Cch 2: Tm di mt phng P hoc min gc

Q nh nht cha (C). Bin lun v tr ca
ng thng d i vi cc min phng . Nu pht hin ng thng i qua mt im trong
ng trn th h phng trnh lun c nghim phn bit.
56
Bi 2: Bin lun s nghim ca h phng trnh

(x p)
2
+ (y q)
2
= r
ax +by = c
(r>0)
Phn tch: t u = x p; v = y p a v bi ton 1.
Bi 3: Bin lun s nghim ca h phng trnh

px
2
+qy
2
= r
ax +by = c
(p, q, r > 0)
Phn tch: H cho gm phng trnh ng elip (E) v phng trnh ng thng d.
Nh vy ta cn kho st s giao im ca d v (E). S dng php co - dn bin (E) thnh
ng trn (C) v bin d thnh d

, ta a v kho st s im chung ca d

v (C). Ta bit
rng s im chung khng ln hn 2.
Phng php: t u = x; v =

q
p
y h cho tr thnh
()

u
2
+v
2
=
r
p
au +

p
q
v

= c
T y lm tip nh bi 1. Nu pht hin ng thng i qua mt im nm trong (E) th h
lun c 2 nghim phn bit.
Bi 4: Bin lun s nghim ca h phng trnh

p(x k)
2
+q(y h)
2
= r
ax + by = c
(p, q, r > 0)
Phng php: t u = x k; v = y h a v bi ton 3.
Bi 5: Bin lun s nghim ca h phng trnh vi p, q, r > 0; k = 1:

px
2
+ kxy +qy
2
= r
ax +by = c
Phng php: Bin i phng trnh u v dng mu
2
+nv
2
= r a v bi ton 4.
Bi 6: Kho st tnh cht nghim ca h

x
2
+y
2
= r
ax + by = c
(r > 0)
Phng php: Coi mi nghim ca h l mt im vi to l cp s . Bin i h
phng trnh v iu kin ca h thnh ngha hnh hc. iu kin bi ton thng lin quan
n mt s tnh cht nh tp hp im thuc phn chung ca cc na mt phng hoc min
trn, min elip hoc khong cch gia 2 im, tch v hng ca 2 vect c lp ra t cc
im , s o gc to bi 2 vect hoc 2 ng thng.
Bi 7: Kho st tnh cht nghim ca h phng trnh

px
2
+qy
2
= r
ax +by = c
(p, q, r > 0)
57
Phng php: t u =

px; v =

qy h cho tng ng

u
2
+ v
2
= r
a

u +b

v = c

Trong a

=
a

p
; b

=
b

q
. Bi ton a v bi 6.
a phng trnh v t mt n sang h phng trnh hai n
Bi 1: Kho st nghim ca phng trnh

a bx
2
= kx +m vi a, b > 0
Phng php: t y =

a
b
x
2
khi ta c h
()

x
2
+y
2
=
a
b
kx

by = m
y 0
Bi ton a v kho st nghim ca h ().
Bi 2: Kho st nghim ca phng trnh

a b(x +c)
2
= kx + m vi a, b > 0
Phng php: t z = x +c; y =

a
2
+bz
2
v a v bi 1.
Bi 3: Kho st nghim ca phng trnh a sin t +b cos t = c vi t [; ]
Phng php: t x = sin t; y = cos t ta c h
()

ax +by = c
x
2
+y
2
= 1
Bi ton a v kho st nghim ca h () vi x, y tho iu kin xc nh.
Bi 4: Kho st s nghim ca phng trnh p

a + bx+q

c +dx = m vi bd < 0; q = 1
Phng php: Coi b > 0 d < 0. t y =

a
b
+x; z =

c
d
x ta c h
()

y
2
+z
2
=
a
b

c
d
p

by + q

dz = m
y 0
z 0
58
Bi ton a v kho st s nghim ca h () gm cung trn

AnB nm trong min gc P xc
nh bi cc min x 0, y 0 v ng thng d.
Bi tp v d
Cc bi tp v h tuyn tnh v phn tuyn tnh
Bi 1: Bin lun s nghim ca h

(x 1)
2
+ (y 2)
2
= 5
x +my = m+ 1
Gii
t u = x 1; v = y 2 ta c h cho tng ng:

u
2
+ v
2
= 5
u +mv = m
(C)
y
x
O
A(0,-1)
H trn gm phng trnh ng trn (C) : x
2
+ y
2
= 5 tm l gc ta v bn knh l

5
v phng trnh ng thng d : x +my +m = 0 ph thuc m. Hn na, ng thng d lun
i qua im c nh A(0; 1) nm trong (C) nn d lun ct (C) ti 2 im phn bit vi mi
m.2
Bi 2: Tm m h c nghim x > 0; y > 1:
()

x
2
+y
2
= 4
2x y = m
Gii
59
d3 d2
d1
y = 1
y
x
1
n
A( 3, 1)
M(
m
2
; 0)
M2 M1
B(0; 2)
O
iu kin ca bi ton xc nh min gc P vi x > 0; y > 1. () gm phng trnh ng trn
(C) : x
2
+y
2
= 4 v phng trnh ng thng d : 2x y = m ph thuc m. Phn chung ca
(C) v P l cung

AnB c cc u mt A(

3; 1) v B(0; 2). Yu cu bi ton l tm m d v


cung

AnB c t nht 1 im chung.
Phng trnh ng thng d
1
, d
2
i qua A, B v cng phng vi d l 2x y = 2

3 1 v
2x y = 2. Cc ng thng ct Ox ti M
1
(

3
1
2
; 0) v M
2
(1; 0). Cc im nm
trong (C) nn giao im th hai ca d
1
, d
2
vi (C) thuc na mt phng P
1
= (x; y) : y < 0.
V vy d
1
, d
2
ch c mt im chung vi

AnB l A v B.
Gi Q l di mt phng bin d
1
, d
2
, khi Q cha

AnB. ng thng d ct on thng
M
1
M
2
ti M(
m
2
; 0) khc M
1
, M
2
.
T suy ra 2 < m < 2

3 1 l tp hp cc gi tr m tho gi thit. 2
Bi 3: Tm m h c 2 nghim phn bit tho x
1
x
2
+y
1
y
2
> 0:
()

x
2
+ y
2
= 4
x +y = m
Gii
y
x
d
H
O
B
A
Gi s h c hai nghim phn bit l (x
1
, y
1
) v (x
2
, y
2
), khi ta c: Gi A(x
1
; y
1
), B(x
2
; y
2
),
60
th do

OA.

OB = OA.OB. cos

OA,

OB

= x
1
x
2
+y
1
y
2
Nn suy ra:
cos

OA,

OB

> 0

AOB <

2
Hn na do (x
1
, y
1
) v (x
2
, y
2
) u l nghim ca h nn cc im A v B u nm trn ng
trn (C) : x
2
+y
2
= 4 v ng thng d : x +y = m, nh vy AB l mt dy ca ng trn
(C), ng thi AB cng chnh l ng thng d.
Nh vy h c nghim tha iu kin x
1
x
2
+ y
1
y
2
> 0 th AB phi ct ng trn (C) ti
hai im phn bit, hn na phi tha mn

AOB <

2
AB ct (C) ti hai im phn bit d
O/d
< 2
[m[

2
< 2
Gi H l trung im ca dy AB khi OH AB. Ta c:

AOB <

2


AOH <

4

AH < OH. V OH
2
+AH
2
= 4 nn suy ra

AOB <

2
OH >

2.
Nh vy h c hai nghim phn bit tha x
1
x
2
+y
1
y
2
> 0 2 >
[m[

2
>

2 2

2 > [m[ > 2 2


Bi 4: Tm m h c 2 nghim phn bit tho (x
1
x
2
)
2
+ 4(y
1
y
2
)
2
= 3:
()

x
2
+ 4y
2
= 16
x my = m
Gii
H () gm phng trnh elip (E) :
x
2
16
+
y
2
4
= 1 v ng thng d : xmy = m ph thuc m.
y
x
-1 A
O
Ta c ng thng d lun i qua im c nh A(0; -1) hn na A nm trong (E). Vy d lun
ct (E) ti 2 im phn bit, do h () c 2 nghim phn bit vi mi m.
Gi B(x
1
; y
1
), D(x
2
; y
2
) v t u = x; v = 2y (1). Cng thc (1) xc nh mt php dn vi h
61
s 2 bng trc dn Ox. Php dn bin cc im B, D thnh B

(u
1
; v
1
), D

(u
2
; v
2
) v h ()
tr thnh:
()

u
2
+v
2
= 16
2u mv = 2m
To cc im B

, D

l nghim ca (). H gm phng trnh ng trn (C) v phng


trnh ng thng d

l nh ca d qua php dn .
y
x
Q'
P'
-2
O
Mt khc
B

D
2
= (u
1
u
2
)
2
+ (v
1
v
2
)
2
= (x
1
x
2
)
2
+ 4(y
1
y
2
)
2
= 3
Ngha l d

ct (C) ti 2 im B

, D

sao cho B

3 hn na (C) c tm l gc ta nn
d
2
O/d

+
B

D
2
4
= R
2
= 16 Suy ra d
O/d
=

61
2
, do d =
[2m[

4 + m
2
nn tm c m =

122
3
2
Bi 5: Tm m h c 2 nghim phn bit v (x
1
x
2
)
2
+ (y
1
y
2
)
2
t gi tr nh nht:
()

x
2
+y
2
= 4
2x +my = m+ 2
Gii
(C)
y
x
d
B
M
O
A
62
H () gm phng trnh ng trn (C) : x
2
+ y
2
= 4 v ng thng d ph thuc vo
m. Gi A, B l giao im ca d v (C) th ta cn tm m AB
2
t gi tr nh nht. Do
M(1; 1) d m v M nm trong (C) nn d lun ct (C) ti 2 im phn bit A, B.
Ta c: AB Min d OM ().
Gi

u = (m; 2) l vect ch phng ca d th
()

u .

OM = 0 2 m = 0 m = 2
Do vi m = 2 th h c hai nghim phn bit tha (x
1
x
2
)
2
+(y
1
y
2
)
2
t gi tr nh nht.
Bi 6. Bin lun s nghim ca h phng trnh sau:

[x[ + 2 [y[ = 4
(x 2a) (y a) = 0
Gii
Xt h phng trnh:

[x[ + 2 [y[ = 4 (1)


(x 2a) (ya) = 0 (2)
Ta thy rng cc im tha mn (1) l bn cnh ca hnh thoi ABCD trong :
A((4; 0) ; B(0; 2) ; C (4; 0) ; D(0; 2)
ng thi, cc im tha mn (2) nm trn hai ng thng d
1
: x = 2a v d
2
: y = a. S
nghim ca h hai phng trnh (1) v (2) chnh l s giao im ca bn cnh hnh thoi vi
hai ng thng ni trn.
Trc tin ta tm xem khi no 3 ng thng x = 2a; y = a v [x[ + 2 [y[ = 4 ng quy. Gi
(x
0
; y
0
) l im chung ca ba ng thng ny, khi ta c h sau:

x
0
= 2a
y
0
= a
[x
0
[ + 2 [y[ = 4

x
0
= 2a
y
0
= a
4 [a[ = 4

[a[ = 1
x
0
= 2; y
0
= 1
x
0
= 2; y
0
= 1
y
x
-4
-2
4
2
y = 1
x = 2
D
A
B
C O
y
x
y = a
x = 2a
-4
-2
4
2
D
A
B
C O
63
Da vo th trn ta c kt lun sau:
Nu [a[ > 2: H (1), (2) v nghim.
Nu [a[ = 2: H (1), (2) c hai nghim.
Nu [a[ < 2 v [a[ = 1: H (1), (2) c 4 nghim.
Nu [a[ = 1: H (1), (2) C 3 nghim.
a phng trnh v t v h phng trnh
Bi 7: Bin lun s nghim ca phng trnh
m

9 x
2
x + 5m = 0 (1)
Gii
t y =

5 x
2
ta c h
()

x
2
+ y
2
= 9
x my = 5m
y 0
H () gm phng trnh ng thng d ph thuc m v na ng trn (C) xc nh bi h
bt phng trnh

x
2
+y
2
= 9
y 0
Ta c A(0; 5) d m do A nm ngoi (C)
(C)
y
x
d
-3 3 5m
M
2
A
M
1
O
M
64
Cc im M
1
(3; 0), M
2
(3; 0) l u mt ca ng knh ca (C). Cc tia AM
1
, AM
2
l bin
ca min gc P. ng thng d ct M
1
M
2
ti M(5m; 0) nn vi m tho 3 5m 3
3
5

m
3
5
th d v na trn ca (C) c duy nht 1 giao im. Trng hp ny phng trnh c 1
nghim.
Vi cc gi tr cn li ca m th d v na trn ca (C) khng c giao im nn phng trnh
v nghim. 2
Bi 8: Bin lun theo m nghim ca phng trnh sau:

4 x
2
= mx + 2 m ()
Gii
Ta bit rng s nghim ca phng trnh () l s giao im ca hai ng y = mx + 2 m
v y =

4 x
2
.
Hn na v y =

4 x
2
x
2
+ y
2
= 4, y 0 nn th ca y =

4 x
2
l na ng trn
tm l gc ta bn knh bng hai v nm pha trn trc honh.
Cn y = mx + 2 m l mt h ng thng lun i qua im c nh A(1; 2). Ta nhn thy
c hai tip tuyn vi ng (C) : y =

4 x
2
, mt l ng thng y = 3 song song vi trc
honh, v tip tuyn AD.
y
x
2
-2 2 1
O
D
B
A
C
E
Gi B(2, 0) v C(2, 0) l hai u mt ca ng knh BOC, gi s m
1
, m
2
, m
3
, m
4
tng
ng l h s gc ca cc ng thng AC, AD, AB, AE th ta c cc iu sau:
m
1
= tan

ACO = 2.
m
2
= tan

DCO = tan

EAD = tan

OAE

=
4
3
.
m
3
= tan

ABO =
2
3
.
m
4
= 0.
65
T suy ra:
1. Phng trnh () c hai nghim

0 < m
2
3
2 m
4
3
2. Phng trnh () c mt nghim

m >
2
3
m < 2
m = 0
m =
4
3
3. Phng trnh () v nghim
4
3
< m < 0 2
Bi 9: Bin lun s nghim ca phng trnh sau theo a:

9 x
2
+ x

x a

= 0
Gii
t y =

9 x
2
x
2
+y
2
= 9
Phng trnh (4) tng ng:

x
2
+y
2
= 9, y 0
(ay + x)

x a

= 0
D thy phng trnh th nht biu din phn pha trn trc honh ca ng trn tm l gc
ta bn knh l 3 cn phng trnh th hai biu din hai ng thng x = a

3 v y =
1
a
.x
(nu a = 0).
S nghim ca h chnh l s giao im ca hai ng thng vi na ng trn. Ta ch cn
xt khi a > 0 (v khi a < 0 ta c kt qu tng t v khi a = 0 th (4) x = 0 v lc h
c 1 nghim).
Thy rng ng thng y =
1
a
x nn n i qua O v v vy lun ct na ng trn ti mt
im. Ta ch cn quan tm n v tr tng i ca x = a

3 vi ng trn v ng thng
y =
1
a
x.
Do y 0 nn nu ng trn v hai ng thng ng quy th phi c x = a

3 > 0 v
y =
1
a
x < 0. iu ny l v l. Do s khng c trng hp ba ng ny ng quy. Nh
th nhn vo th ta s thu c kt lun:
66
y
x
x = a 3
a > 3
y = -
1
a
x
3
O
y
x
x = a 3
0 < a < 3
y = -
1
a
x
3
O
Do ta c kt lun sau:
Phng trnh (4) c mt nghim

a = 0
[a[ >

3
Phng trnh (4) c hai nghim 0 < [a[ <

3 2
Bi 10: Bin lun s nghim ca phng trnh:

9 2x x
2
= x +m (()
Gii
Quan st mt cht ta thy bi ny c cu trc hi khc cc bi trn nhng tht ra chng nh
nhau:
()

10 (x + 1)
2
= x +m
t z = x + 1; y =

10 z
2
ta c h:

y
2
+z
2
= 10
y z = m1
y 0
Bi ton a v bin lun s im chung ca ng thng y = z + m1 v na ng trn

y
2
+z
2
= 10
y 0
n y tng t nh Bi 1, xin dnh cho bn c gii quyt tip phn cn li ca bi ton.
Bi 11: Bin lun s nghim ca phng trnh
2 sin t (m+ 3) cos t = m1, t

3
;
2
3

67
Gii
t x = sin t; y = cos t v a iu kin ca bi ton thnh:

x
2
+y
2
= 1 (i)
2x (m+ 3)y = m1 (ii)

3
2
x 1 (iii)
1
2
y
1
2
(iv)
Do hai iu kin (iii) v (iv) nn ta ch cn kho st nghim trong hnh ch nht ABCD trong
:
A

3
2
;
1
2

, B

1;
1
2

, C

1;
1
2

, D

3
2
;
1
2

Do bi ton a v kho st s giao im ca ng thng d : 2x (m + 3)y = m1 v


ng trn (C) : x
2
+y
2
= 1 nm trong hnh ch nht ABCD, h c nghim th ch c nghim
duy nht.
y
x
I
D C
B A
O
S
Cc nh A v D thuc ng trn n v. Cc nh cn li nm ngoi ng trn. Hn na
ng thng d lun i qua im c nh I(2, 1), do h c nghim th ng thng d phi
nm trong phn gii hn ca hai tia IA, ID. Li c d ct AD ti im S

3
2
,

3 + 1 m
m+ 3

Do h c nghim duy nht

1
2

3 + 1 m
m+ 3

1
2

1
2

3 + 4
m+ 3

1
2

1
2

3 + 4
m+ 3

3
2
5+2

3 5
2

3 1
3
Min gc cha tp hp im tho mn h iu kin trn c bin l cc ng thng ng vi
cc gi tr ca m =
2
3

3
1
2

v m = 2

3 + 5 2
68
Mt vi bi ton khc
Hai phn trn chng ta tip cn vi cch gii ton bng hnh hc, th, trong phn ny,
chng ta s thc s thy c v p ca vic gii ton bng hnh hc, cc bi ton trong phn
ny thng khng mu mc v c cch gii quyt khc nhau ty dng ton, ta bt u vi v
d sau:
Bi 12: Tm cc b s (x, y) dng tha mn phng trnh:

x
2
+b
2
bx

3 +

y
2
+a
2
ay

3 +

x
2
+y
2
xy

3 =

a
2
+ b
2
Trong a, b l cc s thc dng cho trc.
Gii
tng: Cc bi ton hai phn trc ch cp n bin lun nghim ca phng trnh,
h phng trnh, nhng y l mt bi v gii phng trnh, cch tip cn hon ton khc.
Ta ch n nhn xt sau: v tri l ba cn thc c dng gn ging nh l Cosin trong tam gic,
v phi l biu thc gn ging cng thc tnh ng cho ca cnh huyn (nh l Pytagore).
Hn na li c mt kt qu quen thuc trong hnh hc phng l AM +MN +NB AB. Do
ta i n li gii nh sau:
Li gii:
iu kin: x
2
+b
2
bx

3, y
2
+a
2
ay

3, x
2
+y
2
xy

3 0 Dng tam gic vung OAB c


OA = a, OB = b. Ox v Oy l hai ng chia ba cc gc ca tam gic, trn Ox ly M v trn
Oy ly N sao cho OM = y v ON = x (nh hnh v)
a
b
x
y
y
x
A
O
B
M
N
Khi ta c c AM =

OA
2
+ OM
2
2OA.OM. cos

6
=

a
2
+y
2
ay.

3.
Tng t: MN =

x
2
+y
2
xy.

3, BN =

x
2
+ b
2
bx.

3, AB =

a
2
+b
2
.
Hn na AM + MN +BN AB nn:

x
2
+b
2
bx

3 +

y
2
+a
2
ay

3 +

x
2
+y
2
xy

a
2
+b
2
Do iu kin ca bi ton nn du bng xy ra, ni cch khc M v N ln lt l giao im
ca Ox v Oy vi AB.
n y ta ch cn tm OM, ON na l c!
69
a
b
x
y
N
M
O
A
B
Ta c:
S
OAB
= S
OAM
+S
OBM
ab =
1
2
ay +

3
2
by y =
2ab
a +

3.b
Tng t ta tm c x =
2ab

3.a +b
Do nghim ca bi ton l x =
2ab

3.a +b
, y =
2ab
a +

3.b
2
Nhn xt: Bi ton trn cho thy c vic s dng hnh hc gii ton s a bi ton
v mt bi ton kh nh nhng. Ta cng n vi v d tip theo:
Bi 13: Gii phng trnh:

2x
2
2x + 1 +

2x
2
+

3 + 1

x + 1 +

2x
2

3 1

x + 1 = 3
Gii
tng: Cng ging nh trn, v tri gi ta dng nh l Cosin cho tam gic nhng khng
th c. Ta s phi s dng cch gii khc.
Phn tch t biu thc n gin nht ta c c: 2x
2
2x + 1 = x
2
+ (x 1)
2
. Do tng
ca chng ta cng l phn tch cc biu thc cn li thnh tng hai bnh phng, do ta s
c li gii sau:
Li gii:
Trong h ta Oxy ly cc im M(x, x), A(0, 1), B

3
2
,
1
2

, C

3
2
,
1
2

.
y
x
-
1
2
3
2
-
3
2
1
x
x
C B
O
A
M
70
Khi ta c:

MA =

2x
2
2x + 1
MB =

2x
2
+

3 + 1

x + 1
MC =

2x
2

3 1

x + 1
Nh vy V T = MA + MB + MC TA + TB + TC vi T l im Torricelli ca tam gic
ABC.
D c c tam gic ABC l tam gic u v T chnh l gc ta , do
MA +MB +MC 3TA = 3
Theo iu kin bi ton th du ng thc xy ra hay M T, do x = 0 2
Nhn xt: Bng cch phn tch cc biu thc nh trn ta hon ton c th gii bng cch
s dng BT Cauchy-Schwarz. Cng vic ca ta l nh gi c BT sau:

x
2
+ (x 1)
2
+

x +

3
2

2
+

x +
1
2

2
+

3
2

2
+

x +
1
2

2
3
C th thit lp bt ng thc trn bng cch chn tham s.
Mi cch lm u c u nhc im. Dng im Toricelli s cho li gii nh nhng, nhng nu
ABC khng u vic tnh ton s gp kh khn. Dng tham s c th tng qut bi ton
nhng i khi nghim nhn c rt phc tp.
Di y s l mt v d vi cch gii khc hon ton vi hai bi ton trn.
Bi 14. Gii phng trnh sau:
x

x + 1 +

3 x = 2

x
2
+ 1
Gii
tng: Bi ny r rng vic a cc biu thc cha cn thnh di mt on thng
gp kh khn v cc biu thc khng c dng bc hai. Tuy th trong bi ny phng php hnh
hc vn pht huy tc dng. Bin i phng trnh mt cht ta c:
x.

x + 1 + 1.

3 x =

x + 1

2
+

3 x

2
.

x
2
+ 1
V phi khin ta ngh n tch di cn v tri gip ta ngh n tch v hng ca hai vector,
t ta hnh thnh li gii nh sau:
Li gii:
iu kin: 1 x 3.
t

a = (x, 1);

b =

x + 1,

x + 3

ta c


a .

b = x

x + 1 +

3 x
[

a [ .

x
2
+ 1.

x + 1

2
+

3 x

2
Hn na ta li c

a .

b [

a [ .

Do ta c:
x.

x + 1 + 1.

3 x

x + 1

2
+

3 x

2
.

x
2
+ 1
71
Nh vy theo iu kin ta phi tm x trong trng hp du bng xy ra tc l ta c:

a |

b x.

3 x =

x + 1 x
3
3x
2
+x + 1 = 0
(x 1)(x
2
2x 1) = 0

x = 1
x = 1 +

2
x = 1

2 (loi)
2
Ba v d trn chng ta tip cn cch gii phng trnh bng hnh hc, sau y s l mt v
d v gii h phng trnh.
Bi 15: Gii h phng trnh:

x =

y
2
a
2
+

z
2
a
2
y =

z
2
b
2
+

x
2
b
2
z =

x
2
c
2
+

y
2
c
2
Trong a, b, c l cc s thc dng cho trc tha mn
1
a
,
1
b
,
1
c
l ba cnh ca tam gic
khng t.
Gii
tng: R rng ngay khi nhn vo bi th gi thit a, b, c l cc s thc dng cho
trc tha mn
1
a
,
1
b
,
1
c
l ba cnh ca tam gic khng t khin ta cm thy kh ri. Tuy
nhin nu ch n phn tch sau th mi vic hon ton d dng:
Vi mt tam gic khng t c ba cnh l a, b, c cc ng cao tng ng l h
a
, h
b
, h
c
v din
tch l S. ta c:
ah
a
= bh
b
= ch
c
= 2S

a
2S
=
1
h
a
b
2S
= h
b
c
2S
= h
c
T ch rng
a
2S
,
d
2S
,
c
2S
l ba cnh ca tam gic khng t ng dng vi tam gic c ba
cnh l a, b, c theo t l
1
2S
do
1
h
a
,
1
h
b
,
1
h
c
l ba cnh ca tam gic khng t. Vy ta c li
gii nh sau:
Li gii:
iu kin a
2
, b
2
z
2
; b
2
, c
2
x
2
; a
2
, c
2
y
2
.
z
x
y
c
a
b
72
h phng trnh c nghim th x, y, z phi dng nn suy ra a, b z; b, c x; a, c y. Do
ta lun c th dng cc tam gic sau:
A
1
B
1
C
1
c A
1
B
1
= y, A
1
C
1
= z v ng cao ti nh A
1
bng a ng thi hai tia
A
1
B
1
v A
1
C
1
nm v hai pha i vi ng cao ti A
1
. Khi theo phng trnh th
nht d dng c c x = B
1
C
1
.
A
2
B
2
C
2
c B
2
A
2
= z, B
2
C
2
= x ng cao ti nh B
2
l b ng thi hai tia B
2
A
2
,
B
2
C
2
nm v hai pha i vi ng cao nh B
2
th y = A
2
C
2
.
A
3
B
3
C
3
c C
3
A
3
= y, C
3
B
3
= x ng cao ti nh C
3
bng c ng thi hai tia C
3
A
3
,
C
3
B
3
nm v hai pha i vi ng cao ti nh C
3
, khi z = A
3
B
3
.
Do ba tam gic A
1
B
1
C
1
, A
2
B
2
C
2
, A
3
B
3
C
3
c ba cnh bng nhau nn chng bng nhau do
din tch ca chng bng nhau suy ra ax = by = cz = 2S.
p dng cng thc Heron ta c: S =

p(p a)(p b)(p c), trong p =


x +y +z
2
Ta c: x =
2S
a
, y =
2S
b
, z =
2S
c
, p =
x + y +z
2
=
2S

1
a
+
1
b
+
1
c

2
. Do suy ra:
S = S
2

1
a
+
1
b
+
1
c

1
a
+
1
b
+
1
c

1
a

1
b
+
1
c

1
a
+
1
b

1
c

Do :
S =
1

1
a
+
1
b

1
c

1
a

1
b
+
1
c

1
a
+
1
b

1
c

T ta c c: x =
2S
a
, y =
2S
b
, z =
2S
c
2.
Bi 16: Gii h phng trnh n (a; b; c; d) sau:

a
2
+b
2
= 1
c + d = 3
ac +bd +cd =
9 + 6

2
4
Gii
tng: R rng bi ton trn l mt bi ton khng h mu mc, ch c ba phng trnh
nhng li c ti 4 n, hn na phng trnh th 3 kh phc tp. Kh c th bin i i s,
nn chng ta ngh ti phng php hnh hc.
Vi nhng g chng ta phn tch trn, tng hnh hc cn bn ca ta s l xt ng
trn (C) : x
2
+y
2
= 1 khi im M(a, b) s thuc ng trn ny. Cn phng trnh th 2
ta xt ng thng (d) : x + y = 3 th im N(c, d) s thuc ng thng ny.
73
Qu tch nhng im M(a, b) tha phng trnh th nht l ng trn (O; 1).
Qu tch nhng im N(c, d) tha mn phng trnh th hai l ng thng (d) : x +y = 3.
Hn na ta li c:
2(ac +bd +cd) = 2ac + 2bd + (c +d)
2
c
2
d
2
= 2ac + 2db + 10(a
2
+b
2
+c
2
+d
2
)
= 10 (a c)
2
(b d)
2
= 10 MN
2
Suy ra 10 MN
2
=
9 + 6

2
2
MN
2
=
11 6

2
2
MN =
3

2
Do ta s tm hai im M, N nm trn ng trn (O; 1) v ng thng (d) : x +y = 3 sao
cho khong cch gia chng l
3

2
.
Li gii:
1
1 3
3
M
0
N
0
O
M
N
Gi N
0
l hnh chiu ca O ln MN, v M
0
v M

0
l giao im ca ON
0
vi ng trn n
v (O; 1).
Nhn xt rng nu h phng trnh c nghim th ta phi c NM
0
MN NM

0
, do ta
s tnh N
0
M
0
v N
0
M

0
.
Ta c: M
0
N
0
= ON
0
1 =
3

2
1 =
3

2
= MN
T suy ra M M
0
v N N
0
.
D dng tnh c M
0

2
2
,

2
2

v N
0
=

3
2
,
3
2

T suy ra nghim ca h 2
Nhn xt: Bi ton trn rt c th v may mn l MN t gi tr nh nht. Trong mt
v trng hp s
9 + 6

2
4
ca ta thay bng s khc m vn tha mn h c nghim v MN
khng t cc tr th vic lm bi ton s rt kh khn v h s c v s nghim, do thng
thng cc bi ton dng ny s l v nghim hay c nghim duy nht.
Bi tp t luyn
74
Bi 1: Bin lun s nghim theo m ca h

x
2
xy + 4y
2
= m
x +

3
7
y = 1
Bi 2: Tm m h

x
2
+y
2
= 8
x +y = m
c nghim x, y > 0
Bi 3: Tm m h

x
2
+y
2
= 4
2x my = m+ 2
c nghim x 0 y
Bi 4: Bin lun theo m s nghim ca phng trnh

5 2(x + m)
2
= x 2
Bi 5: Bin lun theo m s nghim ca phng trnh 4 sin t 3 cos t = 2m1 vi t [0; ]
Bi 6: Bin lun theo m s nghim ca phng trnh

5 4x +

m+ 9x = 1
Bi 7: Cho h bt phng trnh:

(x + 1)
2
+ y
2
a
9 + (y + 1)
2
a
Tm a h c nghim duy nht.
Bi 8: Cho h phng trnh:

x
2
+y
2
= 2 (1 +a)
(x +y)
2
= 4
Tm a h c hai nghim.
Bi 9: Cho h:

x
2
+ (5a + 2) x + 4a
2
+ 2a < 0
x
2
+a
2
= 4
Tm a h c nghim.
Bi 10: Cho h phng trnh:

x +mx m = 0
x
2
+y
2
x = 0
1. Bin lun s nghim ca h theo m.
2. Khi h c hai nghim (x
1
, y
1
), (x
2
, y
2
). Xt i lng:
d = (x
2
x
1
)
2
+ (y
2
y
1
)
2
.
Tm m d t gi tr ln nht.
Bi 11: Cho h:

2ax +y = 1
x + 2y > a
Tm m h c t nht mt nghim m (tc l c t nht mt nghim (x, y) trong x < 0 v
y < 0)
75
Bi 12: Bin lun theo m s nghim ca cc phng trnh sau:
a.

16 4x
2
= x m
b.

1 9x
2
= m3x
c.

2x x
2
= 2mx
d.

4 x
2
=

2mx x
2
e.

1 x
2
=

1 m
2
+ 2mx x
2
f.

4x
2
16 = m2x
Bi 13: Tm m bt phng trnh sau c nghim:
a.

1 x
2
x m
b.

x
2
9 > x m
Bi 14: Tm m bt phng trnh sau c nghim duy nht:

12 3x
2
x m
Bi 15: Phng trnh sau c bao nhiu nghim: 4x
3
3x =

1 x
2
Bi 16: Tm m cc h sau c nghim:
a.

1 x
2
= y
3mx 3y = 5m
b.

x + 1 +

y + 2 = m
x + y = 3m
c.

4 x
2
= y
mx 3y = 2m
Bi 17: Tm m cc h sau c nghim:
a.

x
2
+ x 2 + x < 1
x
2
2(m+ 2)x + 1 > 0
b.

x + y 1
x + y

2xy +m
c.

x +y 2
x +y +

2x(y 1) + a = 2
Bi 18: Tm m cc h sau c nghim duy nht:
a.

x + y +

2xy + m 1
x + y 1
b.

x +

y = 1
x +y m
Bi 19: Bin lun theo a s nghim ca cc phng trnh:
a. [x[ (3 4x
2
) = a.
b.

1
3
[x[ 1

([x[ 1)
2
= a.
c. x
2
2x 2 =
a
[x 1[
d.
x
2
+ 2 [x[ 1
[x[ + 1
= a.
e.
x
2
+ 2
[x[ 1
= a
76
f. (x + 1)
2
a [x + 2[ = 0
Bi 20: Cho cc s thc dng x, y, z tha mn h phng trnh

x
2
+xy +
y
2
3
= 25
y
2
3
+z
2
= 9
z
2
+zx + x
2
= 16
Tm gi tr biu thc P = xy + 2yz + 3xz
Bi 21: Cho a, b, c l cc s thc dng tha ng thc:

a
2

2ab +b
2
+

b
2

2bc +c
2
=

a
2
+c
2
Tm gi tr nh nht ca
a + c
b
Bi 22: Cho p, q l cc s thc sao cho phng trnh x
2
+ px + q = 0 c hai nghim thc c
gi tr tuyt i khng vt qu 1. Tm gi tr ln nht v nh nht ca biu thc:
P = p
2
(p
2
+ 2q
2
3) + q
2
(q
2
3)
Bi 23: Kho st s nghim ca h phng trnh theo a:

x
2
y
2
(x a)
2
+ y
2
= 1
.
HNH HC KHNG GIAN V VIC KHO ST H
PHNG TRNH 3 N
Trong nhng bi ton sau ta quy c (x; y; z) l n cn a, b, c l tham s.
Bi tp v d:
Bi 1: Bin lun s nghim ca h phng trnh vi r > 0:
()

x
2
+ y
2
+z
2
= r
ax +by +cz = m
a

x +b

y +c

z = m

tng:
H (*) gm mt phng trnh ca mt mt cu (W) vi tm l gc ta v mt ng thng
d. Ta giao im ca d v (W) l nghim ca (*). V th, bi ton c a v dng bin
lun s giao im ca d v (W).
Phng php:
Vit phng trnh mt phng (P) i qua O(0; 0; 0) v vung gc vi ng thng
77
d :

ax +by +cz = m
a

x +b

y +c

z = m

()
Tm giao im K ca (P) v d. Xt v tr tng i ca K so vi (W).
* Nu K nm trong (W) th d ct (W) ti hai im phn bit v h (*) c hai nghim phn
bit.
* Nu K nm trn (W) th d v (W) c mt im chung v h (*) c nghim duy nht
* Nu K nm ngoi (W) th h (*) v nghim.
V d 1: Bin lun s nghim ca h phng trnh sau theo tham s m:

x
2
+y
2
+z
2
= 22
x + y + 2z = m
x z = 2.
Gii
Phng trnh mt phng (P) i qua O v vung gc vi d l: x 3y + z = 0.
Gi K l giao im ca (P) v d. Khi , ta ca K l nghim ca h

x +y + 2z = m
x z = 2
z 3y +z = 0
Gii h ny ta c ta im K

3m
11
;
2m
11
;
3m
11

.
* Nu K nm trong (W) khi v ch khi
2m
2
11
< 22 11 m 11. Khi , h cho c hai
nghim phn bit.
* Nu K nm trn (W) khi v ch khi m = 11 m = 11. Khi , h cho c nghim duy
nht.
* Nu K nm ngoi (W) khi v ch khi m > 11 m < 11. Khi , h cho v nghim. 2
V d 2: Bin lun s nghim ca h phng trnh sau:
()

x
2
+ y
2
+ z
2
= 16
x +my +z = 2m4
x z = 2
Gii
Trong h (*) th phng trnh x + my + z = 2m 4 thuc chm ng thng c trc l

x +z = 4
y 2 = 0
.
Do ng thng d c phng trnh

x +my + z = 2m4
x z = 2
V d i qua im c nh K (1; 2; 3).
Ta thy rng K nm trong mt cu (W): x
2
+y
2
+z
2
= 16 do d ct (W) ti hai im phn
bit vi mi m. Vy h cho lun c hai nghim phn bit vi mi m.
M rng:
78
R rng l h phng trnh bi ton 1. cho vi mt cu nhn gc ta lm tm. Tuy nhin,
ta cng c th tng qut phng php trn ln vi h c phng trnh mt cu c tm khng
phi l gc ta :

(x a)
2
+ (y b)
2
+ (z c)
2
= r
a
1
x +b
1
y +c
1
z = m
1
a
2
x +b
2
y +c
2
z = m
2
(r > 0)
Khi , nu ta t: u = x a, v = y b, w = z c th bi ton trn s a v bi ton 1.
V d 3: Bin lun s nghim ca h phng trnh sau:

(x m)
2
+ (y 2)
2
+ (z + 1)
2
= 16
x +y +z = 1
x z = 2m
Gii
t u = x m, v = y 2, w = z + 1 h phng trnh cho tng ng vi h sau:
()

u
2
+v
2
+w
2
= 16
u + v + w = m
u w = m1
n y ta ch cn bin lun s nghim ca h (*) nh phng php bi ton 1.
Bi 2: Bin lun nghim ca h phng trnh:

r x
2
y
2
= px + qy +k
ax + by = c
Phng php:
Bng cch t z =

r x
2
y
2
th h cho tng ng vi h:

x
2
+ y
2
+ z
2
= r
z 0
ax +by = c
px +qy z + k = 0
H ny gm phng trnh ca na mt cu (W) v ng thng d. Bi ton cng chuyn v
dng bi 1.
V d 1: Bin lun s nghim ca h phng trnh sau

6 x
2
y
2
= 2x +m
x y = 2
Gii
79
t z =

6 x
2
y
2
. H phng trnh cho tr thnh:
()

x
2
+y
2
+z
2
= 6
z 0
2x z = m
x y = 2
H (*) gm na mt cu (W) v ng thng d cha tham s m.
Gi (P) l mt phng i qua tm mt cu (W) v vung gc vi d. Phng trnh mt phng
(P) : x +y + 2z = 0
Gi H l giao im ca d v (P), ta H l nghim ca h phng trnh

x +y + 2z = 0
2x z = m
x y = 2
T ta tm c H

1 m
3
;
m5
3
;
m+ 2
3

Vy ng thng d ct na mt cu (W) ti hai im phn bit khi v ch khi

m+ 2
3
> 0
m
2
4
+

m
2
+ 2

2
6
(m1)
2
+ (m+ 5)
2
+ (m+ 2)
2
9
< 6
2

3 1

> m 2

2 1

ng thng d v na mt cu (W) c mt im chung duy nht khi v ch khi:

m+ 2
3
> 0
(m1)
2
+ (m+ 5)
2
+ (m+ 2)
2
9
= 6
m = 2

3 1

Kt lun:
* Khi m = 2

3 1

th h cho c nghim duy nht


* Khi 2

3 1

> m 2

2 1

th h c hai nghim phn bit


* Mi gi tr cn li ca m lm h v nghim. 2
Nhn xt: Cng bng cch lm nh m rng ca bi ton 1, ta c th m rng cho bi
ton 2. vi h c phng trnh mt cu khng nhn gc ta lm tm:

r (x m)
2
(y n)
2
= px +qy +k
ax +by = c
By gi ta s m rng thm vi h gm hai phng trnh mt cu.
80
Bi 3: Bin lun nghim ca h phng trnh:
()

(x a
1
)
2
+ (y b
1
)
2
+ (z c
1
)
2
= r
1
(x a
2
)
2
+ (y b
2
)
2
+ (z c
2
)
2
= r
2
ax +by +cz = m
tng:
Ta nhn thy h (*) c hai phng trnh mt cu v mt phng trnh mt phng. Nu hai
mt cu khng c im chung th h v nghim. Nu hai mt cu c im chung th h hoc
v nghim hoc c nghim ph thuc vo im chung thuc hoc khng thuc mt phng
ax + by +cz = m.
Phng php:
Tr hai v ca hai phng trnh u ta thu c mt phng trnh mt phng c dng:
(a
1
a
2
) x + (b
1
b
2
) y + (c
1
c
2
) z = m

. Ta gi mt phng ny l mt ng phng ca hai


mt cu. D dng nhn thy mt ng phng ny cha cc im chung ca hai mt cu v
vung gc vi ng thng i qua tm ca hai mt cu.
* Nu hai mt cu tip xc nhau ti K th mt ng phng l tip din chung ca hai mt
cu ti K.
* Nu hai mt cu ct nhau th giao tuyn ca chng l ng trn nm trong mt ng
phng.
Nh vy, vic kho st h phng trnh trn chuyn v bi ton kho st h tng ng l:

(x a
1
)
2
+ (x b
1
)
2
+ (x c
1
)
2
= r
1
(a
1
a
2
) x + (b
1
b
2
) y + (c
1
c
2
) z = m

ax +by + cz = m
Vic kho st phng trnh ny chnh l ni dung ca bi ton 1.
Bi tp t luyn:
Bi 1: Bin lun s nghim ca cc h phng trnh sau:
a)

x
2
+y
2
+z
2
= 2m
2
2
x +y + z = 0
x z = m+ 4
b)

45 x
2
y
2
= m+ 2x
x y = 1
c)

x
2
+y
2
+z
2
= 29
mx +y + z = 3m
x z = 2
d)

6 (x 2)
2
(y + 2)
2
= 2x + m
x y = 2
Bi 2: Tm tt c cc gi tr ca m h phng trnh sau c hai nghim phn bit (x
1
; y
1
; z
1
)
81
v (x
2
; y
2
; z
2
) biu thc (x
1
x
2
)
2
+ (y
1
y
2
)
2
+ (z
1
z
2
)
2
t gi tr nh nht:

x
2
+y
2
+z
2
= 12
mx +y +z = m2
x + y = 2
MT S BI PHNG TRNH, H PHNG TRNH
C THAM S TRONG CC K THI OLYMPIC
Bi 1: Gii v bin lun h phng trnh sau theo m,n,p,k:
(I)

x m
2
+

y m
2
= k

y n
2
+

z n
2
= k

z p
2
+

x p
2
= k
Trong m, n, p, k > 0 v tha m+n +p =
k

3
2
Gii
Trc ht ta s chng minh (I) nu c nghim th l nghim duy nht. Tht vy: Gi s
(x
1
; y
1
; z
1
) v (x
2
; y
2
; z
2
) l 2 nghim phn bit ca (I). Do vai tr ca x,y,z nh nhau nn ta
gi s x
1
> x
2
Khi t phng trnh th nht ca (I) ta c:

x
1
m
2
+

y
1
m
2
=

x
2
m
2
+

y
2
m
2
Suy ra y
2
> y
1
Hon ton tng t t phng trnh th hai ca h ta chng minh c z
1
> z
2
T phng trnh th 3 ta suy ra c x
2
> x
1
, iu ny mu thun vi gi thit.
Vy (I) c nghim duy nht.
Li c

m, n, p, k > 0
m+n +p = k

3
2
Nn tn ti mt ABC u cnh k v mt im I nm trong tam gic sao cho khong cch
t I n cc cnh l m, n, p.
Xt FIA c IA
2
IF
2
= AF
2

IA
2
m
2
= AF
Tng t ta c

IB
2
m
2
= BF
Do

IA
2
m
2
+

IB
2
m
2
= AF +BF = k.
T phng trnh 1 ca h ta suy ra x = IA
2
, y = IB
2
x = IC
2
Vy (I) c nghim:

x = IA
2
y = IB
2
z = IC
2
82
Vy vi mi m, n, p, k tho bi th (I)

x = IA
2
y = IB
2
z = IC
2
2
Bi 2: Tm tt c gi tr nguyn dng ca n sao cho h phng trnh sau c nghim
dng:

x
1
+x
2
+... +x
n
= 9
1
x
1
+
1
x
2
+... +
1
x
n
= 1
Gii
Gi s h cho c nghim x
1
; x
2
; ...; x
n
. Theo bt ng thc Cauchy ta c:
(x
1
+x
2
+... +x
n
)

1
x
1
+
1
x
2
+ ... +
1
x
n

n
2
n
2
9 n 3
Vi n = 3 th h

x
1
+x
2
+x
3
= 9
1
x
1
+
1
x
2
+
1
x
3
= 1
c nghim dng x
1
= x
2
= x
3
= 3.
Vi n = 2, xt h:

x
1
+x
2
= 9
1
x
1
+
1
x
2
= 1

x
1
+ x
2
= 9
x
1
x
2
= 9
Ta thy phng trnh t
2
9t + 9 = 0 c 2 nghim phn bit dng. Vy n = 2 tha mn yu
cu bi.
Vi n = 1, ta c h:

x
1
= 9
1
x
1
= 1
D thy h trn v nghim.
Nh vy n 2; 3 l gi tr cn tm. 2
Bi 3: Tm tt c gi tr ca tham s m h sau c 5 nghim thc:
(I)

x
3
mx y = 0
y
3
my +x = 0
Gii
Nhn thy h cho lun c nghim x = y = 0 m
Xt x = 0, y = 0 ta c (I)

x
4
mx
2
xy = 0
y
4
my
2
+xy = 0
T ta c x
4
+y
4
= m(x
2
+y
2
) m > 0.
Bi ton cho tr thnh: tm m > 0 h c 4 nghim (x; y) = (0; 0)
Th y = x
3
mx vo phng trnh th hai ca h, ta c:
x
2
(x
2
m)
3
m(x
2
m) + 1 = 0 (1)
t t = x
2
m, (1) tng ng vi: ()

t
4
+mt
3
mt + 1 = 0
t > m
t u = t
1
t
, (*) tr thnh: u
2
mu + 2 = 0 (2)
83
Ta phi c 0 m 2

2. Khi (2) c 2 nghim

u
1
=
m+

m
2
8
2
u
2
=
m

m
2
8
2
Xt hai phng trnh sau:

t
1
t
= u
1
f(t) = 2t
2
(

m
2
8 m)t 2 = 0 (3)
t
1
t
= u
2
g(t) = 2t
2
+ (

m
2
8 + m)t 2 = 0 (4)
Ta c f(m).f(0) < 0 v g(m).g(0) < 0
Nn (3) c 2 nghim phn bit m < t
1
< 0 < t
2
V (4) c 2 nghim phn bit m < t
3
< 0 < t
4
ng vi mi t s c hai gi tr ca x =

t +m.
Vy h cho c 4 nghim ta phi c t
1
= t
3
, t
2
= t
4
u
1
= u
2
m = 2

2
Th li thy ng. Vy m = 2

2 l gi tr cn tm. 2
Bi 4: Tm iu kin ca cc s a,b h phng trnh sau y c t nht mt nghim vi
x
1
> 0, x
2
> 0, x
3
> 0, x
4
> 0 :

x
1
x
2
= a
x
3
x
4
= b
x
1
+ x
2
+x
3
+x
4
= 1
Gii
Gi s h cho c t nht mt nghim x
1
> 0, x
2
> 0, x
3
> 0, x
4
> 0.
Ta c [a[ = [x
1
x
2
[ < [x
1
[ +[x
2
[ = x
1
+ x
2
[b[ = [x
3
x
4
[ < [x
3
[ +[x
4
[ = x
3
+x
4
T suy ra [a[ +[b[ < x
1
+ x
2
+x
3
+ x
4
= 1
Ngc li nu c [a[ +[b[ < 1 th c = 1 ([a[ +[b[) > 0
Khi ty theo a; b m hay dng m h cho c nghm: x
1
= [a[ +
c
4
, x
2
=
c
4
, x
3
=
[b[ +
c
4
, x
4
=
c
4
x
1
=
c
4
, x
2
= [a[ +
c
4
, x
3
= [b[ +
c
4
, x
4
=
c
4
x
1
= [a[ +
c
4
, x
2
=
c
4
, x
3
=
c
4
, x
4
= [b[ +
c
4
Cc nghim ny u tho yu cu bi ton. 2
Bi 5: Cho a, b, c = 0. Gii v bin lun h sau theo a, b, c:
(I)

x y + z.

x + y z = x

yz
b

x +y z.

y +z x = y

xz
c

y + z x.

x + y z = z

xy
Gii
84
Xt hai trng hp sau y:
Nu c t nht mt trong ba s a, b, c < 0. Ta gi s a < 0. Khi t phng trnh u ca
h ta suy ra c hai v u phi bng 0, tc l ta c:

yz = 0

x y +z.

x +y z = 0

xyz = 0
(x y + z)(x +y z) = 0
T n iu kin cho biu thc trong cn c ngha th tho mn phng trnh th
nht ca h ta phi c: x = 0, y = z 0 hoc y = 0, x = z 0 hoc z = 0, x = y 0
Thay li vo hai phng trnh sau thy ng. chnh l cc nghim ca h phung trnh
trong trng hp ny.
Nu a, b, c > 0: Khi h cho tng ng vi h
(II)

a
2
(x y +z)(x + y z) = x
2
yz
b
2
(x +y z)(y +z x) = y
2
xz
c
2
(y +z x)(x + z y) = z
2
xy
R rng (II) lun c nghim (0; 0; 0). Ta th tm nghim khc ca h:
Nhn v vi v ca ba phng trnh trong h trn ri ly cn bc hai, ta c:
abc(x + y z)(x + z y)(y +z x) = x
2
y
2
z
2
T ta i n h phng trnh sau:

bc(y + z x) = ayz
ac(x +y z) = bxz
ab(x +z y) = cyx

abc(y +z x) = a
2
yz
abc(x + y z) = b
2
xz
abc(x + z y) = c
2
yx

a
2
y +b
2
x = 2abc
b
2
z + c
2
y = 2abc
c
2
x +a
2
z = 2abc

x =
a
b
2
+ c
2
a
2
y =
b
a
2
+c
2
b
2
z =
c
a
2
+b
2
c
2
x, y, z > 0, ta phi c: a
2
+b
2
> c
2
, b
2
+c
2
> a
2
, c
2
+a
2
> b
2
Tm li ta i n kt lun sau:
Nu c t nht mt trong ba s a, b, c < 0 th nghim ca h l: (0; ; ), (; ; 0), (; 0; )
vi > 0 tu .
Nu c ba s a, b, c u dng th:
* Nu a
2
+b
2
> c
2
, b
2
+c
2
> a
2
, c
2
+a
2
> b
2
th nghim ca h l (0; 0; 0) v

a
b
2
+c
2
a
2
;
b
a
2
+ c
2
b
2
;
c
a
2
+ b
2
c
2

* Nu c t nht mt trong cc bt ng thc sau: a


2
+ b
2
c
2
, b
2
+ c
2
a
2
, c
2
+ a
2
b
2
th
nghim ca h ch l (0; 0; 0) 2
Bi 6: Gi s
a
m+ 2
+
b
m+ 1
+
c
m
= 0. Chng minh rng phng trnh ax
2
+ bx + c = 0
c nghim x (0; 1)
Gii
85
Xt hm s f(x) =
a
m+ 2
x
m+2
+
b
m+ 1
x
m+1
+
c
m
x
m
, x [0, 1]
f

(x) = ax
m+1
+bx
m
+cx
m1
= x
m1
(ax
2
+ bx +c)
Ta li c: f(0) = 0 v f(1) =
a
m+ 2
+
b
m+ 1
+
c
m
= 0
Do theo nh l Lagrange th tn ti x
0
(0, 1)sao cho f(1) f(0) = (1 0)f

(x
0
)
Nn f

(x
0
) = 0 x
0
m1
(ax
0
2
+bx
0
+ c) = 0
ax
0
2
+ bx
0
+c = 0
Vy ta c iu phi chng minh. 2
Bi 7: Cho phng trnh
x
3
2002x
2
+ 2001bx 2000a = 0
Tm gi tr ln nht ca a sao cho tn ti b phng trnh trn c 3 nghim trn [
2002, 2002]
Gii
Gi s x
1
, x
2
, x
3
l 3 nghim ca phng trnh. Khi theo nh l Viete cho phng trnh bc
ba ta c:
(I)

x
1
+x
2
+ x
3
= 2002
x
1
x
2
+ x
2
x
3
+x
3
x
1
= 2001b
x
1
x
2
x
3
= 2000a
Xy ra cc trng hp sau:
Nu x
2
< 0 hoc x
3
< 0 khi x
1
+ x
2
+ x
3
< x
3
2002. iu ny mu thun vi phng
trnh th nht ca (I)
Nu x
1
< 0 x
2
x
3
, t phng trnh ba ca (I) suy ra a 0.
Nu 0 x
1
x
2
x
3
, theo bt ng thc Cauchy ta c:
2002 = x
1
+x
2
+ x
3
3
3

x
1
x
2
x
3
=
3

2000a a
2002
3
54000
Du = xy ra khi x
1
= x
2
= x
3
=
2002
3
v khi y b =
2002
2
6003
By gi ta xt bi ton trong trng hp a =
2002
3
54000
v b =
2002
2
6003
Khi phng trnh cho tr thnh:
x
3
2002x
2
+
2002
2
3
x
2002
2
3
3
= 0 (x
2002
3
)
3
= 0
Phng trnh trn c 3 nghim x
1
= x
2
= x
3
=
2002
3
[-2002,2002]
Vy a
max
=
2002
3
54000
l gi tr cn tm 2
Bi 8: Tm tt c gi tr ca tham s a sao cho phng trnh sau c ng hai nghim phn
bit:

x + 4 [y[ = [x[
[y[ +[x a[ = 1
86
Gii
Ta xt hai trng hp sau:
Nu x 0 th y = 0. Khi :
Vi a > 1 th h c hai nghim phn bit vi x 0 l (a 1; 0), (a + 1; 0).
Vi 1 a 1 h ch c mt nghim vi x 0 l (a + 1; 0).
Vi a < 1 th h cho v nghim.
Nu x < 0: Ta c [y[ =
x
2
v [x a[ = 1 +
x
2
Hay

1 +
x
2
0

x a = 1 +
x
2
x a = 1
x
2

x 2

x = 2(1 +a)
x =
2(a 1)
3
Mt khc 2 2(a + 1) < 0 2 a < 1 v 2
2
3
(a 1) < 0 2 a < 1.
R rng nu (x; y) l mt nghim ca h vi x < 0 th y = 0 v (x; y) cng l mt nghim
ca h. Do :
Khi a < 2 th h v nghim.
Khi a = 2 ta c x = 2(a + 1) = 2 v x =
2
3
(a 1) = 2 nn h c ng hai nghim
phn bit (2; 1).
Khi 2 < a < 1 th 2(a + 1) =
2
3
(a 1) v hai gi tr ny cho h c 4 nghim.
Khi 1 < a < 1 th h c 3 nghim.
Tm li h cho c ng hai nghim khi

a 1
a = 2
2
Bi 9: Tm a, b, c, d sao cho phng trnh sau c nghim x R:
(2x 1)
40
(ax +b)
40
= (x
2
+cx +d)
20
(1)
Gii
V (1) ng vi mi x nn cng ng khi x =
1
2
Vi x =
1
2
, (1)

a
2
+ b

40
+

1
4
+
c
2
+ d

20
= 0

a = 2b
c
2
+ d =
1
4
(2)
Thay a = 2b vo (1) ta c:
(2x 1)
40
= (2bx + b)
40
+ (x
2
+cx +d)
20
x
87
H s x
40
ca hai v l 2
40
v 2
40
b
40
+ 1 do suy ra 2
40
= 2
40
b
40
+ 1 b =
1
2
40

2
40
1
(1) tr thnh
(2x 1)
40
= b
40
(2x + 1)
40
+ (x
2
+ cx + d)
20
x
(2x 1)
40
(1 b
40
) = (x
2
+cx +d)
20
x
(2x 1)
40
1
2
40
= (x
2
+cx +d)
20
x

x
1
2

40
= (x
2
+cx +d)
20
x

x
1
2

2
= (x
2
+ cx + d) x
Xt khi

x
1
2

2
= x
2
+cx +d x c = 1 c = 1 v d =
1
4
(tho (2))
Xt khi

x
1
2

2
= (x
2
+ cx + d) x: khng c c, d tha mn yu cu bi ton.
Vy a = 2b, b =
1
2
40

2
40
1, c = 1 v d =
1
4
l cc gi tr cn tm 2.
Bi 10: Cho h phng trnh sau (n x, y):

x = y
2
+a
y = x
2
+b
vi a, b l cc tham s thc. Bit rng h phng trnh ny c nghim duy nht (x
0
, y
0
).
Tnh gi tr ca tch P = x
0
y
0
.
Gii
u tin ta cng xem qua li gii sau:
Gi s vi cc gi tr a, b no , h cho c nghim duy nht l (x
0
, y
0
). t x
0
y
0
= k R.
Nu x
0
= 0 th y
0
= b v b
2
+a = 0, suy ra a 0.
Trong h cho, thay y t phng trnh th hai ln phng trnh th nht, ta c
(x
2
+ b)
2
+a = x x
4
+ 2bx
2
+b
2
+a = x x(x
3
+ 2bx 1) = 0 ()
Ta xt phng trnh x
3
+ 2bx 1 = 0v t f(x) = x
3
+ 2bx 1, x R.
Ta thy hm ny lin tc trn R v f(0) = 1 < 0, lim
x+
f(x) = + nn f(x) = 0 hay phng
trnh x
3
+ 2bx 1 = 0 c t nht mt nghim dng x = x
1
> 0.
D thy rng iu kin x tha mn bi l x = y
2
+ a a nn c hai nghim nu
x = x
0
= 0, x = x
1
ca phng trnh () u tha mn. Mi gi tr x cho ta ng mt gi tr
y nn trong trng hp ny h c t nht hai nghim phn bit, khng ng vi bi.
Nu y
0
= 0 th cng xt tng t nh trn.
Nu x
0
, y
0
= 0. Thay y
0
=
k
x
0
vo hai phng trnh ca h cho, ta c:

x
0
=
k
2
x
2
0
+a
k
x
0
= x
2
0
+b

x
3
0
= k
2
+ ax
2
0
k = x
3
0
+bx
0
k = k
2
+ax
2
0
+bx
0
k
2
k + (ax
2
0
+bx
0
) = 0
88
H c nghim duy nht nn phng trnh bc hai theo k cng c nghim duy nht, tc l
k =
1
2
. Vy nu h c nghim duy nht th tch P = x
0
y
0
=
1
2
.
Nhn xt: Li gii trn tuy nhn hp l nhng tht ra c mt sai lm rt tinh vi, l
khi cho rng h cho v phng trnh thu c sau php th, cn gi l phng trnh h qu,
l tng ng vi nhau. y, r rng nu h phng trnh ban u c nghim duy nht th
cha hn phng trnh cui theo bin k k
2
k +(ax
2
0
+bx
0
) = 0 c nghim duy nht, trong
vn c th c mt nghim khng tha mn bi. Cch bin i i s dng cho bi ny kh
c th tm ra c kt qu chnh xc.
Li gii ng l nh sau:
Trong h phng trnh cho, thay y bi x
2
+b vo phng trnh trn, ta c:
x = (x
2
+ b)
2
+ a
D thy cc nghim ca phng trnh ny u tha mn bi v x a, m mi nghim nh
th cho ta mt gi tr tng ng ca y nn iu kin h cho c nghim duy nht l
phng trnh n x trn c nghim duy nht.
Phng trnh bc 4 c nghim duy nht khi n c nghim kp, iu c ngha l nghim
cng l nghim ca phng trnh o hm bng 0.
Gi x
0
l nghim (tng ng vi gi tr y
0
) th ta c h sau

x
0
= (x
2
0
+ b)
2
+a
4x
0
(x
2
0
+b) = 1
Hn na, do x
0
l nghim ca h ban u nn x
2
0
+b = y
0
, suy ra x
0
y
0
=
1
4
. Vy nu h cho
c nghim duy nht th tch P = x
0
y
0
=
1
4
.
M RNG:
gii quyt trn vn bi ton, chng ta s lm r hn cc vn sau:
Vn 1: Nu phng trnh bc 4 c nghim duy nht khi n c nghim kp, iu c ngha
l nghim cng l nghim ca phng trnh o hm bng 0.
Li gii:
Xt a thc P(x) = x
4
+ax
3
+bx
2
+cx +d, gi s a thc ny c nghim duy nht l x = x
0
.
Khi , theo nh l Bezut, ta c th phn tch thc cho thnh P(x) = (xx
0
)Q(x) vi
Q(x) l a thc bc 3 a thc bc ba Q(x) lun c t nht mt nghim nn nghim cng
phi l x = x
0
. Ta li phn tch a thc ban u thnh P(x) = (x x
0
)
2
R(x) vi R(x) l a
thc bc 2
Suy ra P

(x) = 2(x x
0
)R(x) + (x x
0
)
2
R

(x) = (x x
0
) [2R(x) + (x x
0
)R

(x)].
a thc ny cng c nghim l x = x
0
nn ta c pcm. 2
Vn 2: Tm iu kin cn ca hai tham s a, b h cho c nghim duy nht.
Li gii:
89
T cch gii phn tch trn, ta thy rng a, b phi tha mn iu kin:

x = (x
2
+b)
2
+ a
4x(x
2
+ b) = 1

x = (x
2
+b)
2
+a
x
2
+ b =
1
4x

x
2
+b
4x
+a = x 3x
2
4ax b = 0(**)
Suy ra x
2
=
4ax + b
3
. Thay vo phng trnh th hai ca h trn, ta c
4x

4ax + 4b
3

= 1 4x

4ax + 4b
3

= 1 16ax
2
+ 16bx 3 = 0 x
2
=
3 16bx
16a
Do
3 16bx
16a
=
4ax +b
3
x =
9 16ab
64a
2
+ 48b
Thay vo phng trnh (), ta c
3

9 16ab
64a
2
+ 48b

2
4a

9 16ab
64a
2
+ 48b

b = 0
3(9 16ab)
2
4a(64a
2
+ 48b)(9 16ab) b(64a
2
+ 48b)
2
= 0
256(a
3
+b
3
+ a
2
b
2
) + 288ab = 27 256(a +b
2
)(b +a
2
) = 27 32ab
Biu thc ny i xng gia a v b nn nu lp li qu trnh bin i trn vi y th ta cng c
kt qu tng t. Do , iu kin cn ca hai tham s a, b h cho c nghim duy nht
l a, b phi tha mn ng thc sau
256(a + b
2
)(b +a
2
) = 27 32ab
Mt cp gi tr p tha mn ng thc trn l (a, b) =

1
4
,
1
4

v tng ng vi nghim duy


nht ca h l (x
0
, y
0
) =

1
2
,
1
2

2.
Vn 3: Bi ton m rng.
Bi ton trn c th m rng ra cho bc cao hn ca x v y nh sau:
Bi 10*: Cho h phng trnh sau (n x, y) vi a, b l cc tham s thc:

x = y
2n
+ a
y = x
2n
+ b
, n N

Bit rng h phng trnh ny c nghim duy nht (x


0
, y
0
). Hy tnh tch P
n
= x
0
y
0
.
Li gii:
Thay y = x
2n
+b vo x = y
2n
+ a, ta c x = (x
2n
+ b)
2n
+a.
a thc ny bc chn nn chng minh tng t nhn xt (1) nu trn, ta thy a, b phi tha

x
0
= (x
2n
0
+b)
2n
+a
(2n)
2
x
2n1
0
(x
2n
0
+ b)
2n1
= 1
Li thay y
0
= x
2n
0
+b vo ng thc trn, ta c
(2n)
2
x
2n1
0
y
2n1
0
= 1 x
0
y
0
= (4n
2
)
12n
90
Vy nu h cho c nghim duy nht th P
n
= (4n
2
)
12n
2.
Bi ton ban u l trng hp c bit khi n = 1.
Ch rng ta cho bc y chn x
0
y
0
nhn mt gi tr, bi ton hon ton c th m rng
vi s m ty nhng trong trng hp bc l th c th c 2 gi tr ca x
0
y
0
.
Ngoi ra kt qu th v t bu thc iu kin cn trn cng cho ta mt bi Bt ng thc
hay:
Ta li thy rng nu xt gi thit a +b 0 th c th chng minh rng ab
1
16
. Tht vy, nu
c mt s m, mt s dng th iu ny ng. Ngc li th ch cn dng BT AM-GM kt
hp vi n ph nh gi.
R rng iu kin a +b 0 c th c thay th bng a +b c vi c
1
8
. y c th chn
c =
1
2012
.
Chuyn h cho v

x
2
y = b
y
2
x = a
R rng phng trnh trn chuyn c thnh phng trnh di thng qua mt php hon
i v tr gia hai bin.
Nh vy ta c bi ton sau:
Bi 10**: Cho a thc P(x, y) = x y
2
vi x, y R`0.
Xt a, b R sao cho tn ti ng mt cp s thc x
0
, y
0
R`0 tha mn P(x
0
, y
0
) = b v
P(y
0
, x
0
) = a.
Chng minh rng nu a +b
1
2012
th 4ab x
0
y
0
.
Sau y l mt bi ton v h bt phng trnh:
Bi 11: Tm a h bt phng trnh c nghim:

x
2
+ 2xy 7y
2

1 a
1 + a
()
3x
2
+ 10xy 5y
2
2
Gii
iu kin cn: Gi s h bt phng trnh c nghim (x
0
; y
0
) v a
0
l mt trong cc gi tr
cn tm. Vy th:

x
0
2
+ 2x
0
y
0
7y
0
2

1 a
0
1 + a
0
3x
0
2
+ 10x
0
y
0
5y
0
2
2

2x
0
2
4x
0
y
0
+ 14y
0
2
2
4
1 + a
0
(1)
3x
0
2
+ 10x
0
y
0
5y
0
2
2 (2)
T (1) v (2) suy ra:
x
0
2
+ 6x
0
y
0
+ 10y
0
2

4
1 + a
0
(x
0
+ 3y
0
)
2

4
1 + a
0
1 + a
0
< 0 a
0
< 1
91
Vy iu kin cn h c nghim l a < 1.
iu kin : Vi a < 1 th
1a
1+a
< 1.
Do nu h phng trnh (I)

x
2
+ 2xy 7y
2
= 1
3x
2
+ 10xy 5y
2
= 2
c nghim th h bt phng trnh
cho c nghim.
Ta xt h (I):
(I)

x
2
+ 2xy 7y
2
= 1
(x + 3y)
2
= 0

x =
3
2
; y =
1
2
x =
3
2
; y =
1
2
H (I) c nghim nn h bt phng trnh cho c nghim.
Tm li, h bt phng trnh ban u c nghim khi v ch khi a < 1 2
Nhn xt: y l mt bi ton tng i kh, tng nm trong k thi i hc. iu c bit
ca li gii trn nm iu kin khi chuyn vic chng minh h bt phng trnh c nghim
v vic gii h phng trnh ng cp bc hai. Tuy nhin, mu cht ca bi ton li nm
iu kin cn. Cc bn bao gi ngh, lm cch no a c h bt phng trnh u
bi v dng f
2
(x
0
, y
0
) g(a) cha? Hay ni cch khc, ti sao ngh ti vic nhn s (2) vo
bt phng trnh (*)? Chng ta hy cng xem qua tng sau y:
Trc ht, ta thy rng:
ax
2
+bx +c 0 x R

a = b = 0
c 0

a > 0
0
ax
2
+bx +c 0 x R

a = b = 0
c 0

a < 0
0
ax
2
+ bx +c = f
2
(x)x R

a = b = 0
c 0

a > 0
= 0
(vi f(x) = x +)
ax
2
+ bxy +cy
2
= f
2
(x, y) x, y R

a = b = 0
c 0

a, c > 0
b
2
4ac = 0
(vi f(x, y) = x + y)
Mc tiu ca chng ta l a h bt phng trnh v dng 0 f
2
(x
0
, y
0
) g(a), chnh v l
do t bt phng trnh (*) ta cn nhn vi mt s < 0. Ta c:

x
2
+ 2xy 7y
2

1 a
1 + a
3x
2
+ 10xy 5y
2
2

x
2
+ 2xy 7y
2

1 a
1 + a
3x
2
+ 10xy 5y
2
2
( + 3)x
2
+ 2( + 5)xy (7 + 5)y
2

1 a
1 + a
2 (3)
92
By gi, n v tri ca (3), ta cn bin i v tri ny v bnh phng ca mt biu thc
vi mi x, y. iu ny ch tng ng vi iu kin sau:

+ 3 > 0
( + 5)
2
y
2
+ ( + 3)(7 + 5)y
2
= 0

+ 3 > 0
( + 5)
2
+ ( + 3)(7 + 5) = 0

+ 3 > 0
8
2
+ 36 + 40 = 0

= 2
=
5
2
Tuy nhin, bi ton ny, ta li thy c hai gi tr ca . Ta cn chn la s no s thch
hp.
Ta c:

5(1 a)
2(1 + a)
2 [
2(1 a)
1 + a
2] =
2(1 a)
1 + a

5(1 a)
2(1 + a)
=
1 a
2(1 + a)
> 0 ( do
1 a
1 + a
< 0)
5(1 a)
2(1 + a)
2 >
2(1 a)
1 + a
2
Nh vy, t bt phng trnh (3) ta suy ra cn nhn vo hai v ca bt phng trnh (*) vi
s = 2.
T ta cng c th sng to nhiu bi ton mi, v d nh bi sau:
Bi 11*: Tm a cc h bt phng trnh sau c nghim:
a)

5x
2
4xy + 2y
2
3
7x
2
+ 4xy + 2y
2

2a 1
2a + 5
b)

5x
2
+ 7xy + 2y
2

3a + 1
a + 2
3x
2
+xy +y
2
1
c)

3x
2
8xy 8y
2
2
x
2
4xy + 2y
2

a + 1
2a + 1
CnucNo III: CAC PHNO PHAP OII PHNO
TRNH
PHNG PHP T N PH
Mt s cch t n ph c bn
Dng 1: Phng trnh c dng ax
2
+ bx + c =

px
2
+qx +r trong
a
p
=
b
q
Phng php: t t =

px
2
+qx +r, t 0 a v phng trnh theo t.
Dng 2: Phng trnh c dng
P(x) + Q(x) + (

P(x)

Q(x)) 2

P(x).Q(x) + = 0 ( R)
Phng php: t t =

P(x)

Q(x) t
2
= P(x) + Q(x) 2

P(x).Q(x).
T a v phng trnh theo t.
Bi 1: Gii phng trnh 1 +
2
3

x x
2
=

x +

1 x
Gii
KX: 0 x 1
t t =

x +

1 x th

x x
2
=
t
2
1
2
. Phng trnh u tr thnh
1 +
t
2
1
3
= t t
2
3t + 2 = 0 t = 1; t = 2
Nu t = 2

x +

1 x = 2 (v nghim)
Nu t = 1

x +

1 x = 1 x 0; 1
Vy phng trnh c tp nghim S = 0; 1. 2
Bi tp t luyn
Gii cc phng trnh sau:
1/(H Ngoi Thng-2000) (x + 5)(2 x) = 3

x
2
+ 3x
2/(H Ngoi ng 1998) (x + 4)(x + 1) 3

x
2
+ 5x + 2 = 6
3/(H Cn Th 1999)

(x + 1)(2 x) = 1 + 2x 2x
2
4/ 4x
2
+ 10x + 9 = 5

2x
2
+ 5x + 3
5/ 18x
2
18x + 5 = 3

9x
2
9x + 2
6/ 3x
2
+ 21x + 18 + 2

x
2
+ 7x + 7 = 2
7/(HVKTQS-1999)

3x 2 +

x 1 = 4x 9 + 2

3x
2
5x + 2
93
94
8/

2x + 3 +

x + 1 = 3x + 2

2x
2
+ 5x + 3 16
9/

4x + 3 +

2x + 1 = 6x +

8x
2
+ 10x + 3 16
10/(CSPHN-2001)

x 2

x + 2 = 2

x
2
4 2x + 2
t n ph a v phng trnh tch
Xut pht t mt s hng ng thc c bn khi t n ph:
x
3
+ 1 = (x + 1)(x
2
x + 1)
x
4
+ 1 = (x
2

2x + 1)(x
2
+

2x + 1)
x
4
+x
2
+ 1 = (x
4
+ 2x
2
+ 1) x
2
= (x
2
+x + 1)(x
2
x + 1)
4x
4
+ 1 = (2x
2
2x + 1)(2x
2
+ 2x + 1)
Mc tiu ca ta sau khi t n ph l a v nhng dng c bn nh
u +v = 1 +uv (u 1)(v 1) = 0
hay
au +bv = ab +vu (u b)(v a) = 0
i vi phng trnh ng cp bc hai ax
2
+bxy +cy
2
= 0 c th t t =
x
y
(sau khi xt y = 0)
a v phng trnh bc hai theo t : at
2
+bt + c = 0.
Xt bi 1 trn, ta c cch a v phng trnh tch nh sau:
Bi 1*: Gii phng trnh 1 +
2
3

x x
2
=

x +

1 x ()
Gii
tng: Ta thy (

x)
2
+ (

1 x)
2
= 1(**), m t phng trnh u ta rt c mt cn
thc qua cn thc cn li. Vy ta c li gii nh sau:
Li gii:
Xt (*) ta c: ()

x =
3

1 x 3
2

1 x 3
Do t t =

1 x

x =
3t 3
2t 3
Thay vo (**) ta bin i thnh
t(t 1)(2t
2
4t + 3) = 0 t 0; 1
T suy ra x 0; 1. 2
Ta xt tip v d sau:
Bi 2: Gii phng trnh
3

x + 1 +
3

x + 2 = 1 +
3

x
2
+ 3x + 2
95
Gii
Ta thy (x + 1)(x + 2) = x
2
+ 3x + 2 nn t u =
3

x + 1; v =
3

x + 2 c phng trnh
u +v = 1 +uv (u 1)(v 1) = 0
T tm c nghim x 0; 1. 2
Bi 3: Gii phng trnh
3

x
2
+ 3x + 2(
3

x + 1
3

x + 2) = 1
Gii
tng: Vi hng i nh bi 2, ta chn n mi l
3

x + 1 = a;
3

x + 2 = b. Vic cn li
l phn tch VP theo a, b. D thy 1 = (x + 2) (x + 1) nn ta c li gii sau:
Li gii:
Phng trnh cho tng ng vi
(x + 1) (x + 2) +
3

x
2
+ 3x + 2(
3

x + 1
3

x + 2) = 0
t
3

x + 1 = a; b =
3

x + 2 ta c phng trnh
a
3
+ b
3
ab(a +b) = 0 (a +b)(a b)
2
= 0 a = b

x + 1 =
3

x + 2 x =
3
2
Vy phng trnh c nghim duy nht x =
3
2
. 2
Chng ta cng c bi ton tng t:
Bi 4: Gii phng trnh (x + 2)(

2x + 3 2

x + 1) +

2x
2
+ 5x + 3 1 = 0
Gii
KX: x 1
t

2x + 3 = a

x + 1 = b
a; b 0

x + 2 = a
2
b
2

2x
2
+ 5x + 3 = ab
1 = a
2
2b
2
Phng trnh cho tr thnh
(a
2
b
2
)(a 2b) + ab = a
2
2b
2
(a
2
b
2
)(a 2b) + b(a +b) (a
2
b
2
) = 0
(a b)(a 2b) (a 2b) = 0 (do a + b > 0)
(a 2b)(a b 1) = 0
Vi a = b + 1

2x + 3 =

x + 1 + 1 (v nghim)
Via = 2b

2x + 3 = 2

x + 1 x =
1
2
(chn)
Vy phng trnh c tp nghim S =

1
2

. 2
96
Bi 5: Gii phng trnh
2 +

2 +

2 +

x
+
2

x
=

2
Gii
Thot nhn ta a ra nh gi 2 +

x + 2

x = 4 nn ta t

2 +

x = a;

x = b
Suy ra ()

a b 0
ab =

4 x
a
2
+b
2
= 4
Ta vit li phng trnh nh sau:
a
2

2 + a
+
b
2

2 b
=

2
a
2

2 a
2
b + b
2

2 + ab
2
=

2(2 b

2 + a

2 ab)

2(a
2
+b
2
+ab 2) ab(a b) = 2(a b)

2(ab + 2) = (a b)(ab + 2)
a b =

2 (do a, b 0 ab + 2 > 0)
Kt hp (*) ta c h phng trnh

a
2
+b
2
= 4
a b =

2
ab = 1

4 x = 1 x = 3
Vy phng trnh c nghim duy nht x = 3. 2
Bi 6: Gii phng trnh (13 4x)

2x 3 + (4x 3)

5 2x = 2 + 8

16x 4x
2
15
Gii
Nhn xt: D thy rng (2x 3)(5 2x) = 16x 4x
2
15, nhng cn cc nh thc ngoi
cn ta khng th biu din ht theo mt n ph c, do ta t hai n ph v c a v
phng trnh tch.
Li gii:
K
3
2
x
5
2
t

u =

2x 3
v =

5 2x

u
2
+v
2
= 2
uv =

16x 4x
2
15
Phng trnh cho tr thnh
(2v
2
+ 3)u + (2u
2
+ 3)v = 2 + 8uv = u
2
+v
2
+ 8uv
2uv(u +v) + 3(u + v) = (u +v)
2
+ 6uv
(u + v 3)(2uv u v) = 0
Nu u +v = 3 :

16x 4x
2
15 =
7
2
(v nghim)
Nu u +v = 2uv :

16x 4x
2
15 = 1 x = 2 (chn)
97
Vy phng trnh cho c nghim duy nht x = 2. 2
Bi 7: Gii phng trnh x
2
+

x + 1 = 1 ()
Gii
XK: x 1
t

x + 1 = t; t 0 ta c phng trnh
(t
2
1)
2
+t = 1 t(t 1)(t
2
+t 1) = 0
Vi t = 0 th x = 1.
Vi t = 1 th x = 0.
Vi t =
1 +

5
2
th x =
1

5
2
Phng trnh c tp nghim S = 1; 0;
1

5
2
. 2
Bi 8: Gii phng trnh x
4
+

x
2
+ 3 = 3
Gii
t x
2
= a, a 0 ta c a
2
+

a + 3 = 3 (), ta s a v phng trnh tch nh sau:


() a
2
(a+3)+(a+

a + 3) = 0 (a+

a + 3)(a

a + 3+1) = 0 a+1 =

a + 3 (do a > 0)
T tm c a = 1, suy ra x = 1 l nghim ca phng trnh. 2
Bi 9: Gii phng trnh (x
2
+ 2)
2
+ 4(x + 1)
3
+

x
2
+ 2x + 5 = (2x 1)
2
+ 2 ()
Gii
Nhn xt: Bi ny c ly tha bc cao nht l 4, v c c cn bc 2 nn ta s c nhm cc
biu thc ly tha ging trong cn c th t n ph.
Li gii:
() x
4
+ 4x
2
+ 4 + 4(x
3
+ 3x
2
+ 3x + 1) +

x
2
+ 2x + 5 = 4x
2
4x + 3
(x
2
+ 2x)
2
+ 8(x
2
+ 2x) +

x
2
+ 2x + 5 + 5 = 0
t t =

x
2
+ 2x + 5

t 2
t
2
5 = x
2
+ 2x
Ta vit li phng trnh di dng
(t
2
5)
2
+ 8(t
2
5) + t + 5 = 0
t
4
2t
2
+t 10 = 0 (t 2)(t
3
+ 2t
2
+ 2t + 5) = 0
t = 2 (do t 2 t
3
+ 2t
2
+ 2t + 5 > 0)
98
Suy ra

x
2
+ 2x + 5 = 2 x = 1
Vy phng trnh c nghim duy nht x = 1. 2
Bi 10: Gii phng trnh

x
2
2x + 5 +

x 1 = 2
Gii
KX: x 1
Vit li phng trnh cho di dng

(x 1)
2
+ 4 = 2

x 1
t t =

x 1

t 0
t
2
= x 1
Ta c phng trnh

t
4
+ 4 = 2 t (t 2) t
4
t
2
+ 4t = 0 t = 0 (do t [0; 2])
T suy ra x = 1 l nghim ca phng trnh. 2
Bi 11: Gii phng trnh vi tham s y
5
2
: (4x
2
+ 1)x + (y 3)

5 2y = 0
Gii
t a = 2x v b =

5 2y (b 0) ta c phng trnh vit li thnh
a
3
+a
2
+
(b
3
+ b)
2
= 0 a = b
Hay
2x =

5 2y x =
5 4y
2
2
Vy x =
5 4y
2
2
l nghim ca phng trnh. 2
Nhn xt: Cu hi t ra l: Lm sao c th t n ph nh trn?
Trc tin ta t

5 2y = b y 3 =
5 b
2
2
3 =
(b
2
+ 1)
2
(y 3)

5 2y =
(b
2
+ 1) b
2
Ta hi vng c

4x
2
+ 1

x =
a (a
3
+ 1)
2

4x
2
+ 1

.2x = a
3
+a
8x
3
+ 2x = a
3
+a a = 2x
Vn ny s c cp k hn trong chng Sng to phng trnh - h phng trnh.
Bi 12: Gii phng trnh

x + 2
2
1 =
3

3(x 3)
2
+
3

9(x 3)
99
Gii
iu kin x 2
t t =
3

9 (x 3) th ta c

x =
t
3
+ 27
9

x + 2
2
=

t
3
+ 45
18
3

3(x 3)
2
=
t
2
3
Phng trnh cho tr thnh

t
3
+ 45
18
1 =
t
2
3
+ t

t
3
+ 45
2
= t
2
+ 3t + 3 (1)
Ta c t
2
+ 3t + 3 =

t +
3
2

2
+
3
4
> 0 nn phng trnh (1) tng ng vi
t
3
+ 45
2
= (t
2
+ 3t + 3)
2
2t
4
+ 11t
3
+ 30t
2
+ 36t 27 = 0
(2t 1)(t + 3)(t
2
+ 3t + 9) = 0 t
1
2
; 3
Vi t =
1
2
th x =
t
3
+ 27
9
=
217
72
Vi t = 3 th x =
t
3
+ 27
9
= 0
Cc nghim trn tha mn iu kin ca bi ton.
Vy phng trnh c hai nghim x = 0 v x =
217
72
. 2
Bi 13: Gii phng trnh 5
3

x
5

x + 3
5

x
3

x = 8
Gii
Phng trnh cho tng ng vi:
5
3

x
6
+ 3
5

x
4
= 8 5
15

x
6
+ 3
15

x
4
= 8
t:y =
15

x
2
vi y 0 ta c:
5y
3
+ 3y
2
8 = 0 (y 1)(5y
2
+ 8y + 8) = 0 y 1 = 0 y = 1
Do ta c
15

x
2
= 1 x
2
= 1 x = 1.
Vy tp nghim ca phng trnh cho l:S = 1; 1 . 2
Bi 14: Gii phng trnh
5

x
4

7
5

x
2
+
6
x
= 0
Gii
100
K x = 0. Ta c phng trnh cho tng ng vi
5

x
4

7
5

x
2
+
6
5

x
5
= 0
5

x
9
7
5

x
3
+ 6 = 0 ()
t:y =
5

x
3
, y = 0, phng trnh (*) tr thnh:
y
3
7y + 6 = 0 (y 1)(y
2
+y 6) = 0

y = 1
y = 2
y = 3

x
3
= 1
5

x
3
= 2
5

x
3
= 3

x = 1
x = 2
3

4
x = 3
3

9
Vy tp nghim ca phng trnh cho l

1; 2
3

4; 3
3

. 2
Bi 15: Gii phng trnh

4x 1 +

4x
2
1 = 1
Gii
K

4x 1 0
4x
2
1 0
x
1
2
Bnh phng hai v phng trnh cho, ta c:
(4x 1) + (4x
2
1) + 2

(4x 1)(4x
2
1) = 1
2

(4x 1) (4x
2
1) = 3 4x
2
4x = 4 (2x + 1)
2
()
t y = 2x + 1 4x 1 = 2y 3, 4x
2
1 = y
2
2y
Phng trnh (*) tr thnh:
2

(2y 3)(y 2) = 4 y
2

4 y
2
0
4(2y 3)(y 2)y = (4 y
2
)
2

2 y 2

y 2 = 0
4(2y 3)y = (y + 2)
2
(y 2)

2 y 2

y = 2
y
3
6y
2
+ 8y 8 = 0 (v nghim trn [2; 2])
y = 2
Do ta c 2x + 1 = 2 x =
1
2
Vy phng trnh c nghim duy nht x =
1
2
. 2
Bi 16: Gii phng trnh

2x 1 + x
2
3x + 1 = 0
Gii
101
KX: x
1
2
t t =

2x 1 0 x =
t
2
+ 1
2
Phng trnh cho tng ng
t
4
4t
2
+ 4t 1 = 0 (t 1)
2
(t
2
+ 2t 1) = 0

t = 1
t =

2 1
t =

2 1 (loi do t 0)
Vi t = 1 x = 1
Vi t =

2 1 x = 2

2 Vy phng trnh c tp nghim S = 1; 2

2. 2
Nhn xt: i vi nhng bi c dng

ax + b + cx
2
+ dx + e = 0 th cch gii l t

ax +b = t, sau a v phng trnh bc 4, dng ng nht thc phn tch nhn t.


Nhng c mt s bi khng gii c bng cch , ta s nhc li vn ny phn sau.
Bi 17: Gii phng trnh (x 2)

x 1

2x + 2 = 0
Gii
tng: Ta thy trong cn c

x 1, nn ta s c gng thm bt v tch s c mt


phng trnh theo n mi.
Li gii:
KX: x 1. t

x 1 = t 0
Ta bin i phng trnh nh sau :
[(x 1) 1]

x 1

2[(x 1)

2]

2 = 0
t
3

2t
2
t + 2

2 = 0
(t + 1

2)(t
2
t

2) = 0
Vi t =

2 1

x 1 =

2 1 x = 4 2

2
Vi t
2
t

2 = 0, do t > 0 ta chn t =
1 +

1 + 4

2
2
x =

1 +

1 + 4

2
2

2
+ 1
Kt lun: Phng trnh c tp nghim S =

4 2

2;

1 +

1 + 4

2
2

. 2
t n ph a v phng trnh ng cp
Bi 18: Gii phng trnh 2(x
2
+ 2) = 5

x
3
+ 1
Gii
tng: i vi bi ton ny u tin ta phn tch nhn t trong cn x
3
+1 = (x+1)(x
2

x + 1) ri bin i v tri thnh tng hoc hiu ca hai tha s trong cn.
102
Li gii:
Vit li phng trnh di dng
2(x
2
+ 2) = 2(x
2
x + 1) + 2(x + 1)
t

x
2
x + 1 = a;

x + 1 = b ta c phng trnh 2a
2
+ 2b
2
= 5ab

a = 2b
a =
b
2
T tm c nghim x =
5

37
2
. 2
Bi 19: Gii phng trnh

5x
2
14x + 9

x
2
x 20 = 5

x + 1
Gii
KX: x 5.
u tin bnh phng hai v ca phng trnh:
2x
2
5x + 2 = 5

(x
2
x 20)(x + 1) ()
Ta khng th tm c hai s , sao cho
(x
2
x 20) + (x + 1) = 2x
2
5x + 2
Nn khng th t a =

x
2
x 20; b =

x + 1 nh cc v d trn c.
bi ny, ch rng x
2
x20 = (x5)(x+4) v () 2x
2
5x+2 =

(x
2
4x 5)(x + 4)
Nh vy ta s tm , tha (x
2
4x 5) + (x + 4) = 2x
2
5x + 2

= 2
= 3
Li gii:
Ta bin i li phng trnh nh sau
() 2(x
2
4x 5) + 3(x + 4) = 5

(x
2
4x 5)(x + 4)
t a =

x
2
4x 5; b =

x + 4 ta c phng trnh
2a
2
+ 3b
2
= 5ab

a = b
a =
3b
2
Vi a = b x =
5 +

61
2
(x 5)
Vi a =
3
2
b x = 8; x =
7
4
i chiu vi iu kin ta nhn x = 8; x =
5 +

61
2
l nghim ca phng trnh.2
Nhn xt: Ta cng c bi ton tng t:
Bi 19*: Gii phng trnh 3x
2
2x 2 =
6

30

x
3
+ 3x
2
+ 4x + 2
Sau y l mt v d kh hn:
Bi 20: Gii phng trnh (x
2
6x + 11)

x
2
x + 1 = 2(x
2
4x + 7)

x 2
103
Gii
Li gii: K x 2
t

x
2
x + 1 = a;

x 2 = b vi a, b 0
Ta biu din cc biu thc ngoi cn theo a v b nh sau:

x
2
6x + 11 = (

x
2
x + 1)
2
+(

x 2)
2
x
2
4x + 7 = (

x
2
x + 1)
2
+(

x 2)
2
S dng ng nht thc ta gii c

x
2
6x + 11 = a
2
5b
2
x
2
4x + 7 = a
2
3b
2
Phng trnh cho tng ng vi
(a
2
5b)a = 2(a
2
3b
2
)b
a
3
2a
2
b 5ab
2
+ 6b
3
= 0
t
3
2t
2
5t + 6 = 0 vi t =
a
b
0
(t 1)(t 3)(t + 2) = 0
Vi t = 1 a = b

x
2
x + 1 =

x 2 (VN)
Vi t = 3 a = 2b

x
2
x + 1 = 3

x 2 x = 5

6 (chn)
Vy phng trnh c tp nghim S =

5 +

6; 5

2
Bi tp t luyn
Gii cc phng trnh sau:
1/

x
2
+x 6 + 3

x 1

3x
2
6x + 19 = 0
2/ 3

x
2
+ 4x 5 +

x 3

11x
2
+ 25x + 2 = 0
3/

7x
2
+ 25x + 19

x
2
2x 35 = 7

x + 2 4/2(x
2
3x + 2) = 3

x
3
+ 8
5/2x
2
+ 5x 1 = 7

x
3
1
6/10

x
3
+ 8 = 3(x
2
x + 6)
7/10

x
3
+ 1 = 3(x
2
+ 2)
8/4x
2
2

2x + 4 =

x
4
+ 1
9/(

x + 5

x + 2)(1 +

x
2
+ 7x + 10) = 3
10/(

x + 1 +

x 2)(1

x
2
x 2) = 3
11/

x x
2
+

1 x = 1 + (1 x)

x
12/

3x
2
18x + 25 +

4x
2
24x + 29 = 6x x
2
4
Phng php t n ph khng hon ton
i vi nhiu phng trnh v t, khi khng biu din hon ton c theo n ph th c mt
cch l xem bin mi l n, bin c l tham s. Dng ton ny gi l n ph khng hon ton.
Ni dung phng php:
a phng trnh cho v dng phng trnh bc hai vi n l n ph hay l n ca phng
104
trnh cho.
a phng trnh v dng

f(x)Q(x) = f(x) + P(x)x khi :


t

f(x) = t; t 0. Phng trnh vit li thnh t


2
t.Q(x) + P(x) = 0
n y chng ta gii t theo x. Cui cng l gii quyt phng trnh

f(x) = t sau khi


n gin ha v kt lun.
Ta xt v d sau hiu r hn.
Bi 21: Gii phng trnh x
2
+ 3x + 1 = (x + 3)

x
2
+ 1
Gii
Ta thy trong cn c x
2
+1, ta t t =

x
2
+ 1. Ta s khng rt x theo t m coi x l tham s.
Tht vy, phng trnh cho tng ng
t
2
(x + 3)t + 3x = 0
Phng trnh trn c = (x + 3)
2
12x = (x 3)
2
nn c nghim t = 3; t = x + 3
Nu t = x + 3: h cho v nghim.
Nu t = 3 x = 2

2 2
Bi 22: Gii phng trnh x
2
+

x
2
+ 2

x = 1 + 2

x
2
+ 2
Gii
Phng trnh tng ng vi
x
2
+

x
2
+ 2

x = 1 + 2

x
2
+ 2
x
2
+ 3x 1 (x + 2)

x
2
+ 2 = 0
t t =

x
2
+ 2 (t

2), phng trnh vit li thnh


t
2
(x + 2) t + 3x 3 = 0
Ta c = (x 4)
2
nn phng trnh c 2 nghim l
t = x 1

x
2
+ 2 = x 1

x 1
2x 1 = 0
(v nghim)
t = 4

x
2
+ 2 = 4 x
2
= 14 x =

14
Vy phng trnh c hai nghim l x =

14 v x =

14 2
Bi 23: Gii phng trnh (3x + 1)

2x
2
1 = 5x
2
+
3
2
x 3
Gii
105
t t =

2x
2
1 (t 0)
Phng trnh vit li thnh
2t
2
(3x + 1)t + x
2
+
3
2
x 1 = 0
Ta c = (x 3)
2
suy ra phng trnh c hai nghim l
t = 2x 1

2x
2
1 = 2x 1

x
1
2
x
2
2x + 1 = 0
x = 1
t = x + 2

2x
2
1 = x + 2

x 2
x
2
4x 5 = 0
x = 1 hoc x = 5
Vy S = 1; 5; 1 2
Nhn xt: Thng thng sau khi t n ph th ta vit phng trnh cho li thnh
t
2
(3x + 1) t +3x
2
+
3
2
x2, nhng bi ton ny li c s khc bit l ta s vit phng trnh
ny li thnh 2t
2
(3x + 1) t +x
2
+
3
2
x1. Chng ta quan tm ti liu h s trc t
2
c p
tc ta mong mun phi l bnh phng ca mt s hoc mt biu thc, v iu ny s quyt
nh ti li gii s ngn gn hay phc tp. c th iu chnh c h s trc t
2
sao cho
p cc bn c th lm nh sau:
Xt phng trnh
mt
2
(3x + 1) t + (5 2m) x
2
+
3
2
x +m3 = 0 c o
Phng trnh trn c l
= (3x + 1)
2
4m

(5 2m) x
2
+
3
2
x + m3

8m
2
20m+ 9

x
2
+ (6 6m) x +

4m
2
+ 12m+ 1

Ta xt tip ca bng cch gii phng trnh sau:


2

(8m
2
20m+ 9) (4m
2
+ 12m+ 1) = 6 6m
m = 2
chnh l h s m ta cn tm.
Bi 24: Gii phng trnh 3

2x
2
+ 1 1

= x

1 + 3x + 8

2x
2
+ 1

Gii
Phng trnh tng ng vi
3

2x
2
+ 1 1

= x

1 + 3x + 8

2x
2
+ 1

3x
2
+x + 3 + (8x 3)

2x
2
+ 1 = 0
t

2x
2
+ 1 = t (t 1) ta c phng trnh
3t
2
+ (8x 2) t 3x
2
x = 0 ()
106
C = (10x 3)
2
nn nghim ca (*) l t
1
= x; t
2
= 1 3x
Nu t = x

2x
2
+ 1 = x

x 0
2x
2
+ 1 =
x
2
9
(v nghim)
Nu t = 1 3x

2x
2
+ 1 = 1 = 3x

x
1
3
7x
2
6x = 0
x = 0
Vy phng trnh c nghim l x = 0. 2
Bi 25: Gii phng trnh 2

2x + 4 + 4

2 x =

9x
2
+ 16
Gii
KX: [x[ 2
Bnh phng 2 v ta c
4(2x + 4) + 16

2(4 x
2
) + 16(2 x) = 9x
2
+ 16 8(4 x
2
) + 16

2(4 x
2
) = x
2
+ 8x
Ta thy c hai v c dng hng ng thc, v c a v A
2
= B
2
. lm iu ta s thm
16 vo 2 v c (2

2(4 x
2
) + 4)
2
= (x + 4)
2
Vit li phng trnh di dng
8(4 x
2
) + 16

2(4 x
2
) = x
2
+ 8x
t t =

2(4 x
2
) ta c phng trnh
4t
2
+ 16t x
2
8x = 0
Gii phng trnh trn theo n t ta c t
1
=
x
2
; t
2
=
x
2
4
V [x[ 2 nn t
2
khng tha iu kin.
Vi t =
x
2
th

2(4 x
2
) =
x
2
x =
4

2
3
(tho [x[ 2)
Vy phng trnh c nghim duy nht x =
4

2
3
. 2
Bi 26: Gii phng trnh (3x + 2)

2x 3 = 2x
2
+ 3x 6
Gii
iu kin x
3
2
t t =

2x 3; t 0 t
2
+ 3 = 2x
Ta s thm bt theo n ph a v phng trnh theo t v x l tham s.
Phng trnh cho tng ng
t
2
(3x + 2)t + 2x
2
+x 3 = 0 ()
(*) c = 9x
2
+ 12x + 4 4(2x
2
+x 3) = (x + 4)
2
nn c nghim t = 2x + 3 hoc t = x 1
Vi t = 2x + 3

2x 3 = 2x + 3 (v nghim)
Vi t = x 1

2x 3 = x 1 x = 2 (chn)
Vy phng trnh c nghim duy nht x = 2. 2
Bi 27: Gii phng trnh 4

x + 1 1 = 3x + 2

1 x +

1 x
2
107
Gii
KX: 1 x 1
t

1 x = t ta c phng trnh
3t
2
(2 +

1 x)t + 4(

x + 1 1) = 0 ()
(*) c = (2 +

1 x)
2
48(

x + 1 1) khng c dng bnh phng phng nn khng th


lm nh thng. y, lu rng c th phn tch 3x theo 1 x v 1 +x nn ta s tm cch
phn tch thch hp l bnh phng:
Ta s tm v sao cho
3x + 1 = (

1 x)
2
+(

1 + x)
2

= 1
= 2
Nh vy vit li (*) di dng
t
2
(2 +

x + 1)t 2(x + 1) + 4

x + 1 = 0 ()
(**) c = 9x + 13 12

x + 1 = 9(x + 1) 12

x + 1 + 4 = (3

x + 1 2)
2
T d dng tm c nghim x

3
5
; 0

. 2
Nhn xt: Vn y l phi tinh tch 3x thnh hai dng c biu thc nh trong cn,
n y bi ton mi thc s c gii quyt.
Bi 28: Gii phng trnh 2(2

1 + x
2

1 x
2
)

1 x
4
= 3x
2
+ 1
Gii
Li gii: iu kin 1 x 1
t a =

1 + x
2
; b =

1 x
2
3x
2
+ 1 = 2(1 +x
2
) (1 x
2
) = 2a
2
b
2
Khi phng trnh tr thnh
2(2a b) ab = 2a
2
b
2
2a
2
+a(b 4) + 2b b
2
= 0 ()
(*)c
a
= (b 4)
2
8(2b b
2
) = (3b 4)
2
nn suy ra a =
b
2
hoc a = 2 b
Vi a =
b
2
2

1 + x
2
=

1 x
2
(v nghim)
Vi a = 2 b

x
2
+ 1 = 2

1 x
2
Gii phng trnh ny tm c x = 0. Vy tp nghim ca phng trnh l S = 0 . 2
Bi tp t luyn
Gii cc phng trnh sau:
1/6x
2
10x + 5 (4x 1)

6x
2
6x + 5 = 0
2/(x + 3)

10 x
2
= x
2
x 12 (H Dc-1999)
3/2(1 x)

x
2
+ 2x 1 = x
2
2x 1 (H Dc 1997)
108
4/(4x 1)

x
2
+ 1 = 2x
2
+ 2x + 1
5/2(1 x)

x
2
+x + 1 = x
2
3x 1
6/(x + 1)

x
2
2x + 3 = x
2
+ 1 (Ch thm bt c chnh phng).
7/(4x 1)

x
3
+ 1 = 2x
3
+ 2x + 1
Phng php s dng h s bt nh
Bi 29: Gii phng trnh 2x
2
11x + 21 3
3

4x 4 = 0
Gii
Ta cn tm a, b, c sao cho:
2x
2
11x + 21 = a(4x 4)
2
+b(4x 4) + c
2x
2
11x + 21 = 16ax
2
+ (4b 32a)x + (16a 4b +c)
ng nht h s ta thu c a =
1
8
; b =
7
4
; c = 12
Ta vit li PT nh sau:
1
8
(4x 4)
2

7
4
(4x 4) + 12
3

4x 4 = 0
t u =
3

4x 4, khi PT tr thnh
u
6
14u
3
24u + 96 = 0 (u 2)
2
(u
4
+ 4u
3
+ 18u + 24) = 0
D thy u
4
+ 4y
3
+ 18u + 24 = 0 v nghim v:
Nu u 0 th u
6
14u
3
24u + 96 > 0
Nu u > 0 th u
4
+ 4u
3
+ 18u + 24 > 0
Vy u = 2, t tm c x = 3. 2
Bi 30: Gii phng trnh 2

1 x

x + 1 + 3

1 x
2
= 3 x
Gii
K 1 x 1
Ta tm ; sao cho x + 3 = (

1 x)
2
+(

x + 1)
2
x + 3 = ( )x + +
Gii ra ta c = 2; = 1
Ta vit li phng trnh thnh 1 + x + 2(1 x) 2

1 x +

x + 1 3

1 x
2
= 0
t u =

1 + x; v =

1 x v u, v 0, phng trnh tr thnh


u
2
+ 2v
2
2v +u 3uv = 0 u
2
+ (1 3v)u + 2v
2
2v = 0
109
Phng trnh trn c = (1 3v)
2
4(2v
2
2v) = (v + 1)
2
nn u = 2v hoc u = v 1
u = 2v

x + 1 = 2

1 x x =
5
3
u = v 1

1 x
1
2
4x
2
= 3
Nn phng trnh c 2 nghim x =
5
3
; x =

3
2
Vy tp nghim ca phng trnh l S =

5
3
;

3
2

2
Bi 31: Gii phng trnh 4

1 x = x + 6 3

1 x
2
+ 5

1 + x
Gii
t a =

1 + x v b =

1 x
Phng trnh cho tng ng
2x + 2 + 1 x + 5

1 + x 3

1 x
2
4

1 x + 3 = 0
2a
2
+b
2
3ab + 5a 4b + 3 = 0
(a b)(2a b) + 3(a b) + (2a b) + 3 = 0
(a b + 1)(2a b + 3) = 0
Nu a + 1 = b :

x + 1 + 1 =

1 x 2

x + 1 = (2x + 1) x =

3
2
Nu 2a + 3 = b ta suy ra phng trnh v nghim.
V d tng t sau xin dnh cho bn c:
Bi 32: Gii phng trnh 4 + 2

1 x = 3x + 5

x + 1 +

1 x
2
p s: Phng trnh c 3 nghim S =

0;
24
25
;

3
2

2
t n ph a v h phng trnh
Dng 1: Phng trnh c dng x
n
+ a = b
n

bx a
Cch gii: t y =
n

bx a khi ta c h i xng loi II

x
n
by +a = 0
y
n
bx +a = 0
Ta xt bi ton sau:
Bi 33: Gii phng trnh x
3
+ 1 = 2
3

2x 1
110
Gii
t y =
3

2x 1 y
3
= 2x 1
Ta c h PT sau

x
3
+ 1 = 2y
y
3
+ 1 = 2x
y l h i xng loi II, tr v theo v ta c:
x
3
y
3
= 2(y x)
(x y)(x
2
+y
2
+xy + 2) = 0
x = y
3

2x 1 = x x
3
2x + 1 = 0 (x 1)(x
2
+ x 1) = 0
Vy x = 1; x =
1

5
2
Ta c x
2
+y
2
+xy + 2 =

x +
y
2

2
+
3y
2
4
+ 2 > 0, x, y
Vy phng trnh c tp nghim S =

1;
1

5
2

. 2
Dng 2: Phng trnh c dng
n

a f(x) +
m

b +f(x) = c
Cch gii: t u =
n

a f(x); v =
m

b + f(x)
Ta c h sau

u +v = c
u
n
+v
m
= a +b
Bi 34: Gii phng trnh
4

x + 8 +
4

x 7 = 3
Gii
KX: x 7
t u =
4

x + 8 0 u
4
= x + 8 x = u
4
8
v =
4

x 7 0 v
4
= x 7 x = v
4
+ 7
Ta c h

u +v = 3
u, v 0
u
4
v
4
= 15

v = 3 u
(u
2
v
2
)(u
2
+v
2
) = 15
u, v 0

v = 3 u
(u v)(u + v)(u
2
+v
2
) = 15
u, v 0

v = 3 u
0 u 3
(u v)(u
2
+v
2
) = 5

0 u 3
(2u 3)

u
2
+ (3 u)
2

= 5

0 u 3
(2u 3)(2u
2
6u + 9) = 5

0 u 3
4u
3
18u
2
+ 36u 32 = 0

0 u 3
u = 2
111
T ta tm c

x + 8 = 2
4

x 7 = 1

x + 8 = 16
x 7 = 1
x = 8 (tho KX)
Vy phng trnh cho c nghim duy nht x = 8. 2
Bi 35: Gii phng trnh 2
3

3x 2 + 3

6 5x = 8
Gii
t u =
3

3x 2; v =

6 5x 0

u
3
= 3x 2
v
2
= 6 5x
5u
3
+ 3v
2
= 5(3x 2) + 3(6 5x) = 8(1)
Mt khc ta li c 2u + 3v 8 = 0(2)
T (1) v (2) ta c h sau:

5u
3
+ 3v
2
= 8
2u + 3v = 8
5u
3
+ 3

8 2u
3

2
= 8 15u
3
+ 4u
2
32u + 40 = 0
Phng trnh c nghim duy nht u = 2 nn
3

3x 2 = 2 x = 22
Bi 36: Gii phng trnh

1 +

1 x
2

(1 + x)
3

(1 x)
3

= 2 +

1 x
2
Gii
K 1 x 1
t

1 + x = a;

1 x = b vi a, b 0 a
2
+b
2
= 2
Ta c h sau

a
2
+b
2
= 2 (1)

1 + ab(a
3
b
3
) = 2 +ab (2)
Ta c
(1) (a +b)
2
= 2 + 2ab

1 + ab =
1

2
(a +b) (do a, b 0)
Kt hp (2) ta c
1

2
(a + b)(a b)(a
2
+b
2
+ ab) = 2 +ab
1

2
(a
2
b
2
) = 1
T ta c h

a
2
b
2
=

2
a
2
+b
2
= 2
Cng hai phng trnh v theo v ta c
2a
2
= 2 +

2 a
2
= 1 +
1

2
1 + x = 1 +
1

2
x =
1

2
112
Vy phng trnh c nghim duy nht x =
1

2
. 2
Bi 37: Gii phng trnh (x + 5)

x + 1 + 1 =
3

3x + 4
Gii
K x 1
t a =

x + 1; b =
3

3x + 4 x = a
2
1 v 3a
2
+ 1 = b
3
Thay vo phng trnh ta c h sau

(a
2
+ 4)a + 1 = b
3a
2
+ 1 = b
3
Cng v theo v ta c
a
3
+ 3a
2
+ 4a + 2 = b
3
+b (a + 1)
3
+ (a + 1) = b
3
+b ()
Xt hm s c trng f(t) = t
3
+ t
Ta c f

(t) = 3t
2
+ 1 > 0, vy hm s ng bin nn () a + 1 = b
Ta c h sau:

a + 1 = b
b
3
3a
2
= 1

a = b 1
b
3
3a
2
= 1
S dng php th ta c
b
3
3(b 1)
2
= 1 b
3
3b
2
+ 6b 4 = 0 (b 1)(b
2
b + 4) = 0
Suy ra b = 1, t x = 1.
Vy phng trnh c nghim duy nht x = 1. 2
Dng 3: Phng trnh dng

ax +b = cx
2
+dx +e
Ta gp cc dng bi ton nh

ax +b = cx + d v mt s v d nu trn bng cch


bnh phng bc 4 v ng nht h s tm c nghim, nhng i vi nhng bi ton
khng dng c phng php th sao? Chng ta cng lm r vn .
Xt v d sau:
Bi 38: Gii phng trnh 2x
2
6x 1 =

4x + 5
Gii
Li gii: K x
5
4
Ta bin i phng trnh nh sau
4x
2
12x 2 = 2

4x + 5 (2x 3)
2
= 2

4x + 5 + 11
.t 2y 3 =

4x + 5 ta c h phng trnh sau:

(2x 3)
2
= 4y + 5
(2y 3)
2
= 4x + 5
113
(x y)(x + y 1) = 0
Vi x = y 2x 3 =

4x + 5 x = 2 +

3.
Vi x + y 1 = 0 y = 1 x x = 1

22
Nhn xt:
Bi ton tng qut c dng nh sau

ax +b = cx
2
+ dx +e, (a = 0, c = 0, a =
1
c
)
Xt f(x) = cx
2
+dx + e f

(x) = 2cx + d
Gii phng trnh f

(x) = 0, khi bng php t

ax +b = 2cy + d, ta s a c v h
i xng loi II (tr mt s trng hp c bit).
Ta xt v d tip theo
Bi 39: Gii phng trnh x
2
4x 3 =

x + 5
Gii
tng: Xt f(x) = x
2
4x 3 c f

(x) = 2x 4 = 0 x = 2. Vy t

x + 5 = y 2.
Li gii:
KX: x 5
Vit li phng trnh u di dng

x + 5 = (x 2)
2
7 ()
t

x + 5 = y 2 (y 2)
2
= x + 5. Thay vo (*) ta c

(x 2)
2
= y + 5
(y 2)
2
= x + 5
Tr v theo v ta c
(x y)(x +y 3) = 0
Nu x = y

x + 5 = x 2

x 2

x =
1
2
(5 +

29)
x =
1
2
(5

29)
x =
5 +

29
2
(tho x 5)
Nu x +y = 3 1 x =

x + 5

x 1

x = 1
x = 4
x = 1 (tho x 5)
Vy phng trnh cho c tp nghim S =

1;
1
2
(5 +

29)

. 2
Bi 40: Gii phng trnh x
2
+ 5 +

3x + 1 = 13x
Gii
114
tng: Ta vit li phng trnh di dng

3x + 1 = 4x
2
+ 13x 5 v t f(x) =
4x
2
+ 13x 5
C f

(x) = 8x + 13 nhng nu ta gii ra v t bng phng php tng t s khng thu


c h i xng loi II. Vy ta hi vng tm hng i khc, v mt phng php t nhin l
h s bt nh.
Li gii:
K x
1
3
t

3x + 1 = (2y 3); y
3
2
Ta c h phng trnh sau

(2x 3)
2
= 2y +x + 1
(2y 3)
2
= 3x + 1
Tr v theo v ta c (x y)(2x + 2y 5) = 0
Vi x = y x =
15

97
8
Vi 2x + 2y 5 = 0 x =
11 +

73
8
Vy tp nghim ca phng trnh l S =

15

97
8
;
11 +

73
8

2
Nhn xt: Ta thy cch gii bi ton ny khc so vi v d trn v a v h gn i xng
loi II nhng vn gii c mt cch d dng. Bi ton ny c dng nh sau:

ax +b = r(ux + v)
2
+dx +e trong

u = ar +d
v = br +e
Cch gii: t uy + v =

ax +b khi ta c h

uy +v = r(ux +v)
2
+dx + e
ax +b = (uy +v)
2
Ta vit li phng trnh trn nh sau

3x + 1 + (2x 3)
2
x 4 = 0. D dng ta kim tra
c cc h s u tha mn, nhng khi t

3x + 1 = 2y 3 th ta thu c h phng trnh


khng d dng gii mt cht no, do chuyn v v i du s a v h gn i xng
gii c nh bi ton trn.
Tng kt li, ta c o hm p dng c khi h s d = 0, cn nu khng c th dng cch
thm bt nh trn.
Dng 4: Phng trnh dng

ax +b =
1
a
x
2
+cx+d(a = 0) v tha mn b+ad =
a
2
c
2

1 +
c
2

Cch gii: Xt hm s f(x) =


1
a
x
2
+cx +d c f

(x) =
2
a
x + c
Cho f

(x) = 0 ta thu c x =
ac
2
. T t

ax +b = y+
ac
2
ta thu c h i xng loi II.
Bi 41: Gii phng trnh 3x
2
+x
29
6
=

12x + 61
36
Gii
tng: Xt f(x) = 3x
2
+ x
29
6
f

(x) = 6x + 1 = 0 x =
1
6
Li gii:
115
t

12x + 61
36
= y +
1
6
(y
1
6
)

12x + 61
36
= y
2
+
1
3
y +
1
36
12x + 61 = 36y
2
+ 12y + 1 3y
2
+ y = x + 5
Mt khc t phng trnh u ta c 3x
2
+x
29
6
= y +
1
6
3x
2
+x = y + 5
Nn ta c h sau:

3x
2
+ x = y + 5
3y
2
+y = x + 5
Tr v theo v ta c (x y)(3x + 3y + 2) = 0 x = y y =
3x + 2
3
Vi x = y 3y
2
= 5 x = y =

5
3
; y
1
6
Vi y =
3x + 2
3
3x
2
+x =
3x + 2
3
+ 5 9x
2
+ x 13 = 0
x =
3

126
9
T y tm c y v kt lun nghim 2
Bi 42: Gii phng trnh

2 1 x +
4

x =
1
4

2
Gii
iu kin 0 x

2 1
t

2 1 x = u
4

x = v

0 u

2 1
0 v
4

2 1
Nh vy ta c h

u + v =
1
4

2
u
2
+v
4
=

2 1

u =
1
4

2
v

1
4

2
v

2
+ v
4
=

2 1
T phng trnh th hai ta c:
(v
2
+ 1)
2
=

1
4

2
+ v

2
v
2
v + 1
1
4

2
= 0 v =
1

4
4

2
3
2
(chn)
Vy phng trnh c nghim duy nht x =

4
4

2
3
2

4
. 2
Bi 43: Gii phng trnh

1 x
2
=

2
3

2
Gii
KX:

1 x
2
0
x 0

1 x 1
x 0
0 x 1
116
Vit li phng trnh di dng

1 x
2
= 1 u
4

2
3

2
= v
2
t u =

x; v =
2
3

x vi u 0, v
2
3
Do ta c h

u +v =
2
3

1 u
4
= v
2

u +v =
2
3
u
4
+v
4
= 1

u +v =
2
3
(u
2
+ v
2
)
2
2u
2
.v
2
= 1

u +v =
2
3

(u + v)
2
2u.v

2
= 1

u +v =
2
3

4
9
2u.v

2
2u
2
.v
2
= 1

u +v =
2
3
2u
2
.v
2

16
9
u.v
65
81
= 0

u +v =
2
3
u.v =
8

194
18

u +v =
2
5
u.v =
8 +

194
18
Nn u, v l nghim ca phng trnh

y
2

2
3
y +
8

194
18
= 0
y
2

2
3
y +
8 +

194
18
= 0 (v nghim)
T tm c nghim duy nht ca phng trnh l
x =
1
9

2 +

2(

194 6) +

97
2

. 2
Bi 44: Gii phng trnh 4x
2
11x + 10 = (x 1)

2x
2
6x + 2
Gii
tng: Ngoi cch dng n ph khng hon ton, ta cng c th a phng trnh v h
i xng loi II.
Li gii:
Phng trnh cho tng ng
(2x 3)
2
+ x + 1 = (x 1)

(x 1)(2x 3) x 1
t u = 2x 3; v =

(x 1)(2x 3) x 1 ta c h phng trnh

u
2
+x + 1 = (x 1)v
v
2
+x + 1 = (x 1)u
Tr v theo v ta c
u
2
v
2
= (x 1)(v u) (u v)(u + v + x 1) = 0
Nu u = v :
u
2
+x + 1 = (x 1)u (2x 3)
2
+x + 1 = (x 1)(2x 3)
117
2x
2
6x + 7 = 0 (v nghim)
Nu u + v +x 1 = 0 :
2x 3 +

2x
2
6x + 2 + x 1 = 0

2x
2
6x + 2 = 4 3x

x
4
3
7x
2
18x + 14 = 0
(v nghim)
Vy phng trnh cho v nghim.2
Nhn xt: Cch gii ca bi ton ny xut pht t 1 cch lm nu trn. Dng tng
qut l
f
n
(x) + b = a
n

af(x) b
Ta d on f(x) = (2x +c)
2
. n y ta ng nht h s tm c :
4x
2
+ 4cx +c
2
+ (11 4c)x + 10 c
2
= (x 1)

(x 1)(2x + c) (11 4c)x 10 + c


2
b = (11 4c)x + 10 c
2
i chiu vi bi ton ng nht h s suy ra x = 3
Phng php lng gic ha
Nu bi ton cha

a
2
x
2
c th
t x = [a[ sin t vi

2
t

2
hoc x = [a[ cos t vi 0 t
Nu bi ton cha

x
2
a
2
c th
t x =
[a[
sin t
vi t

2
;

2

` 0
Hoc x =
[a[
cos t
vi t [0; ] `

Nu bi ton cha

a
2
+ x
2
c th: t:x = [a[ tan t vi t

2
;

2

Hoc x = [a[ cot t vi t (0; ).


Nu bi ton cha

a +x
a x
hoc

a x
a +x
c th t x = a cos 2t.
Nu bi ton cha

(x a) (b x) c th t x = a + (b a) sin
2
t.
Li th ca phng php ny l a phng trnh ban u v mt phng trnh lng gic
c bn bit cch gii nh phng trnh ng cp, i xng ... V iu kin nhn hoc loi
nghim cng d dng hn rt nhiu. V lng gic l hm tun hon nn ta ch t iu
kin cc biu thc lng gic sao cho khi khai cn khng c du tr tuyt i, c ngha l lun
dng.
Bi 45: Gii phng trnh x
3
+

(1 x
2
)
3
= x

2(1 x
2
)
Gii
K 1 x 1
T iu kin ca bi ton ta t n ph x = cos t, khi

1 x
2
= [sin [
118
Ch cn chn m 0 khi 1 cos = x 1 v sin 0 v [sin [ = sin
Phng trnh cho bin i c v dng
cos
3
+ sin
3
=

2cossin (cos + sin) (1 cossin) =

2cossin
t u = cos + sin =

2 sin

+

4

Do 0 x

4
+

4

5
4

2
2
sin

+

4

1, ta c
1
2
u

2
Phng trnh i s vi n u c dng
u

1
u
2
1
2

2
u
2
1
2
u
3
+

2u
2
3u

2 = 0

u
2
+ 2

2u + 1

= 0

u =

2
u =

2 + 1
u =

2 1 <

2
Nu u =

2 sin

+

4

2 sin

+

4

= 1 =

4
+k2 , k Z
Nu u =

2 sin

+

4

= 1

2 v cossin =
u
2
1
2
= 1

2
Khi cos , sin l nghim ca phng trnh
X
2

X + 1

2 = 0 X =
1

2 1

2 + 3

2
Do sin 0 cho nn cos =
1

2 1

2 + 3

2
Vy phng trnh c tp nghim S =

2 1

2 + 3

2
;

2
2

. 2
Bi 46: Gii phng trnh 2x
2
+

1 x + 2x

1 x
2
= 1
Gii
K x [1; 1]
t x = cos t, t [0; ]
Phng trnh tng ng
2 cos
2
t +

2 sin
t
2
+ 2 sin t cos t = 1 cos 2t + sin 2t =

2 sin
t
2
cos

2t

4

= sin
t
2
cos

2t

4

= cos

t
2
+

2

2t

4
=
t
2
+

2
+ k2
2t

4
=
t
2


2
+k2

t =

2
+
k4
3
t =

10
+
k4
5
119
Da vo iu kin nghim ca phng trnh ta nhn 2 nghim l x = cos

2
; x = cos
7
10
Vy phng trnh c tp nghim S =

cos
7
10
; 0

2
Bi 47: Gii phng trnh

1 +

1 x
2
= x(1 + 2

1 x
2
)
Gii
KX: 1 x
2
0 1 x 1
t x = sin t vi t

2
;

2

Khi phng trnh c dng:

1 +

1 sin
2
t = sin t

1 + 2

1 sin
2
t

1 + cos t = sin t (1 + 2 cos t)

2 cos
t
2
= sin t + sin 2t

2 cos
t
2
= 2 sin
3t
2
cos
t
2

2 cos
t
2

2 sin
3t
2

= 0

cos
t
2
= 0
sin
3t
2
=

2
2

t =

6
t =

2

x =
1
2
x = 1
Vy tp nghim ca phng trnh cho l S =

1
2
; 1

. 2
Bi 48: Gii phng trnh

x
2
+ 1 +
x
2
+ 1
2x
=
(x
2
+ 1)
2
2x(1 x
2
)
Gii
iu kin

x = 1
x = 0
t x = tan t; t

2
;

2

4
; 0

Khi ta c:
x
2
+ 1 = tan
2
t + 1 =
1
cos
2
t

x
2
+ 1 =
1
cos t
sin 2t =
2 tan t
1 + tan
2
t
=
2x
x
2
+ 1

x
2
+ 1
2x
=
1
sin 2t
cos 2t =
1 tan
2
t
1 + tan
2
t
=
1 x
2
x
2
+ 1
sin 2t. cos 2t =
2x (1 x
2
)
(x
2
+ 1)
2
sin 4t =
4x (1 x
2
)
(x
2
+ 1)
2

2
sin 4t
=
(x
2
+ 1)
2
2x (1 x
2
)
Khi phng trnh c bin i v dng
1
cos t
+
1
sin 2t
=
2
sin 4t
4 sin t. cos 2t + 2 cos 2t = 2
2 sin t. cos 2t = 1 cos 2t 2 sin t. cos 2t = 2sin
2
t (cos 2t sin t) sin t = 0

1 2sin
2
t sin t

sin t = 0 (sin t + 1) (2 sin t 1) sin t = 0


sin t =
1
2
t =

6
x =
1

3
120
Vy nghim duy nht ca phng trnh cho l:x =
1

3
. 2
Bi tp t luyn
Gii cc phng trnh sau:
1/ x
3
x
2
10x 2 =
3

7x
2
+ 23x + 12
2/7x
2
13x + 8 = 2x
2 3

x(1 + 3x 3x
2
)
3/8x
2
13x + 7 =

x +
1
x

3x
2
2
4/

2x 1 + x
2
3x + 1 = 0
5/

2x + 15 = 32x
2
+ 32x 20
6/

x 1 + x
2
x 3 = 0
7/x
2
x 2004

1 + 16032x = 2004 (HSG Bc Giang 2003-2004)


8/

9x 5 = 3x
2
+ 2x + 3
9/x
2
=

2 x + 2 10/
3

x + 34
3

x 3 = 14
11/
4

97 x +
4

x = 5
12/
3

x + 2 +

x + 1 = 3
13/
4

18 x +
4

x 1 = 3
14/
4

17 x
8

2x
8
1 = 1
121
PHNG PHP BIN I NG THC
Bi 1: Gii phng trnh

x
2
+ 3x + 2

x + 2 = 2x +

x +
6
x
+ 5
Gii
KX:

x(x + 3) 0
x + 2 0
x
2
+ 5x + 6
x
0
x = 0
x > 0
Phng trnh c vit li thnh
x

x + 3
x
+ 2

x + 2 = 2x +

(x + 2)(x + 3)
x
(x

x + 2)(

x + 3
x
2) = 0

x + 2 = 0

x + 3
x
2 = 0

x = 2
x = 1
Vy tp nghim ca phng trnh l S = 1; 2 . 2
Bi 2: Gii phng trnh
1
x
2
+ 9x + 40
+
1
x
2
+ 11x + 30
+
1
x
2
+ 13x + 42
=
1
18
Gii
KX: x / 4; 5; 6; 7
Phng trnh tng ng vi:
1
x + 4

1
x + 5
+
1
x + 5

1
x + 6
+
1
x + 6

1
x + 7
=
1
18

1
x + 4

1
x + 7
=
1
18
x
2
+ 11x 26 = 0

x = 2
x = 13
Vy tp nghim ca phng trnh l S = 13; 2 . 2
Bi 3: Gii phng trnh

x
2
+x + 2 =
x
2
+ 5x + 2
2(x + 1)
Gii
122
KX: x = 1
Phng trnh tng ng vi:
(x
2
+x + 2) (2x + 2)

x
2
+ x + 2 + 4x = 0
(

x
2
+x + 2 2x)(

x
2
+x + 2 2) = 0

x
2
+x + 2 = 2x

x
2
+x + 2 = 2

x = 1
x = 2
Vy phng trnh c tp nghim S = 1; 2. 2
Bi 4: Gii phng trnh x
4
x
2
+ 3x + 5 2

x + 2 = 0
Gii
KX: x 2
Phng trnh cho tng ng vi:
(x
4
2x
2
+ 1) + (x
2
+ 2x + 1) + (x + 3 2

x + 2) = 0
(x
2
1)
2
+ (x + 1)
2
+ (

x + 2 1)
2
= 0

x
2
1 = 0
x + 1 = 0

x + 2 1 = 0
x = 1
Vy phng trnh c nghim duy nht x = 1. 2
Bi 5: Gii phng trnh x
8
x
5
+x
2
x + 1 = 0
Gii
Phng trnh tng ng vi:

x
4

x
2

2
+

x
2
1

2
+
x
2
2
= 0
T suy ra x
4

x
2
=
x
2
1 =
x
2
2
= 0. H ny v nghim nn phng trnh v nghim. 2
Bi 6: Gii phng trnh
2

x + 1 + 6

9 x
2
+ 6

(x + 1)(9 x
2
) = x
3
2x
2
+ 10x + 38
Gii
KX: 1 x 3
Phng trnh cho c vit li thnh:
(

x + 1 1)
2
+ (

9 x
2
3)
2
+ (

(x + 1)(9 x
2
) 3)
2
= 0

x + 1 1 =

9 x
2
3 =

(x + 1)(9 x
2
) 3 = 0 x = 0
T d dng thu c x = 0 l nghim duy nht ca phng trnh. 2
Bi 7: Gii phng trnh x +y
1
x

1
y
+ 4 = 2(

2x 1 +

2y 1)
123
Gii
KX: x
1
2
, y
1
2
Phng trnh trn tng ng vi:
(x
1
x
+ 2 2

2x + 1) + (y
1
y
+ 2 2

2y + 1) = 0

(x

2x 1)
2
x
+
(y

2y 1)
2
y
= 0 ()
M x, y > 0 nn V T() 0 v (*) xy ra

x =

2x 1
y =

2y 1
x = y = 1
Vy phng trnh c nghim duy nht l (x, y) = (1, 1). 2
Bi 8: Gii phng trnh 2x

x + 3 +

x = 2x
2
+ x + 2
Gii
KX: x 0
Phng trnh tng ng vi
(

x + 3 2x)
2
2
+ (

x 1)
2
= 0

x + 3 2x = 0

x 1 = 0
x = 1
Vy phng trnh c nghim duy nht x = 1. 2
Bi 9: Gii phng trnh
1
(2x 1)
2
+
1
(3x + 1)
2
=
3
(x + 2)
2
Gii
KX: x /

1
2
;
1
3
; 2

Ta c b sau, c th d dng chng minh bng cch bnh phng 2 v ng thc:


Vi a, b, c l cc s thc tha mn a +b +c = 0 th

1
a
2
+
1
b
2
+
1
c
2
=

1
a
+
1
b
+
1
c

Nh vy phng trnh cho c vit li thnh


1
(2x 1)
2
+
1
(3x 1)
2
+
1
(x + 2)
2
=
4
(x + 2)
2
(1)
V (2x 1) + (3x 1) + (x + 2) = 0 nn

1
(2x 1)
2
+
1
(3x 1)
2
+
1
(x + 2)
2
=

1
(2x 1)
+
1
(3x 1)
+
1
(x + 2)

124
Vy (1) tng ng vi

1
(2x 1)
+
1
(3x 1)
+
1
(x + 2)

=
2
[x + 2[

(3x 1)(x + 2) + (2x 1)(x + 2) + (2x 1)(3x 1)


(2x 1)(3x 1)(x + 2)

=
2
[x + 2[

7x
2
3x 3
(2x 1)(3x 1)

= 2 x =
1

5
2
Vy phng trnh c nghim x =
1

5
2
. 2
PHNG PHP DNG LNG LIN HP
L thuyt
Phng php lng lin hp l mt cch gii hiu qu i vi phng trnh, h phng trnh
v t. tng chnh ca phng php l trc cn thc v to nhn t chung. Mun vy, ta
cn phi bit c nghim v bin i kho lo.
1) Dng my tnh b ti gii nghim ca phng trnh:
V d tm nghim ca phng trnh

x
2
x + 3 +

x
2
+ x + 4 = 7:
u tin ta cn nhp vo my tnh b ti:

X
2
X + 3 +

X
2
+X + 4 7
Sau , bm Shift CALC
Sau khi my tnh gii ra nghim, ta nhn thy phng trnh trn c nghim X = 3. kim
tra phng trnh trn cn st nghim hay khng ta tip tc nhp:

X
2
X + 3 +

X
2
+X + 4 7

: (X 3)
Ri li tip tc bm Shift Solve. Ta s tm c nghim th hai l X =
143
48
.
2) Dng tnh cht s hc on nghim ca phng trnh:
Vn dng phng trnh trn lm v d, ta nhn xt v phi ca phng trnh l 1 s nguyn.
Chnh v vy m ta a ra d on cc s trong cn thc phi l s chnh phng v b hn 7.
V vy ta c th cho mi cn thc bng cc s t 1 7 v gii tng v cn thc.
V d nh:
Cho

x
2
x + 3 = 1 v

x
2
+x + 4 = 6 ri gii ln lt tng phng trnh. Nhng nu c
hai v u l cn thc th phng php ny s khng hiu qu.
3) Th th mt vi nghim vo phng trnh cho on nghim:
Da vo iu kin ca bi ton, ta s ln lt th cc nghim gn gi nh 3, 2, 1, 0, 1, 2, 3
vo phng trnh tm xem s no l nghim ca phng trnh. i khi vi mt cht nhy
125
bn, ta c th d dng nhm ngay nghim ca phng trnh nu l nghim p.
Sau y l mt s k thut dng phng php lng lin hp:
Mt s hng ng thc hay s dng:
1) x
2
y
2
= (x y) (x +y)
2) x
3
y
3
= (x y) (x
2
+xy +y
2
)
3) x
4
y
4
= (x y) (x +y) (x
2
+y
2
)
4) x
n
y
n
= (x y) (x
n1
+x
n2
y + ... + xy
n2
+y
n1
)
........
S dng nhng hng ng thc ny, ta c th quy phng trnh v t ban u v dng phng
trnh tch bng vic lm xut hin cc nhn t chung. T ta c th d dng gii quyt tip.
Ta cng c mt s hng ng thc trc cn thc:
1)

x

y =
x y

y
2)
3

x
3

y =
x y
3

x
2

xy +
3

y
2
3)
4

x
4

y =
x y
(
4

x
4

y)(

x +

y)
Bi tp v d
k thut thm bt v tch hng t
phng php ny, chng ta s thm bt hng t hoc tch hng t sn c dng phng
php lng lin hp gii phng trnh.
Bi 1: Gii phng trnh: (x + 1)

x
2
2x + 3 = x
2
+ 1 (1.1)
Gii
V x = 1 khng phi l nghim ca phng trnh trn, ta vit phng trnh di dng:

x
2
2x + 3 =
x
2
+ 1
x + 1

x
2
2x + 3 2 =
x
2
2x 1
x + 1
V

x
2
2x + 3 + 2 > 0 nn:
(1.1)

x
2
2x + 3 2

x
2
2x + 3 + 2

x
2
2x + 3 + 2
=
x
2
2x 1
x + 1

x
2
2x 1

x
2
2x + 3 + 2
=
x
2
2x 1
x + 1

x
2
2x 1 = 0
1

x
2
2x + 3 + 2
=
1
x + 1

x = 1 +

2
x = 1

2
1

x
2
2x + 3 + 2
=
1
x + 1
D thy phng trnh

x
2
2x + 3 + 2 = x + 1 v nghim.
126
Vy tp nghim ca phng trnh cho l: S =

2; 1 +

2
Nhn xt: mu cht ca li gii trn l nhn ra lng lin hp tm ra nhn t chung l

x
2
2x + 3 + 2. Vy lm cch no nhn ra c iu ny? Ta lm nh sau:
Xt phng trnh:

x
2
2x + 3 =
x
2
+ 1
x + 1
()
()

x
2
2x + 3 m =
x
2
+ 1
x + 1
m (m > 0)

x
2
2x + 3 m =
x
2
mx m+ 1
x + 1

x
2
2x + 3 m

x
2
2x + 3 + m

x
2
2x + 3 + m
=
x
2
mx m+ 1
x + 1
x
2
2x + 3 m
2

x
2
2x + 3 + m
=
x
2
mx m+ 1
x + 1
By gi ta ch cn xc nh m sao cho
x
2
2x + 3 m
2
= x
2
mx m+ 1 2 = m 3 m
2
= m+ 1 m = 2
T ta suy ra li gii nh trnh by.
Bi 2: Gii phng trnh: 3
3

x
2
+

x
2
+ 8 2 =

x
2
+ 15 (1.2)
Gii
D on c nghim x = 1, ta vit li phng trnh nh sau:
(1.2) 3

x
2
1

x
2
+ 8 3

x
2
+ 15 4

3 (x
2
1)
3

x
4
+
3

x
2
+ 1
+
x
2
1

x
2
+ 8 + 3
=
x
2
1

x
2
+ 15 + 4

x
2
= 1
1
3

x
4
+
3

x
2
+ 1
+
1

x
2
+ 8 + 3
=
1

x
2
+ 15 + 4
()
Mt khc, ta c:

x
2
+ 15 >

x
2
+ 8

x
2
+ 15 + 4 >

x
2
+ 8 + 3
1

x
2
+ 15 + 4
<
1

x
2
+ 8 + 3
Nn phng trnh () v nghim. Vy phng trnh c tp nghim S = 1; 12.
Bi 3: Gii phng trnh: 2 (x
2
+ 2) = 5

x
3
+ 1 (1.3)
Gii
iu kin: x 1
Do x = 1 khng phi l nghim phng trnh nn ta xt x > 1.
127
Khi vit li (1.3) di dng
(1.3) 2

x
2
+ 2

= 5

(x + 1) (x
2
x + 1) 2

x
2
+ 2

= 5

(x + 1)
2
(x
2
x + 1)
x + 1

2 (x
2
+ 2)
5 (x + 1)
=

x
2
x + 1
x + 1

2 (x
2
+ 2)
5 (x + 1)
2 =

x
2
x + 1
x + 1
2

2x
2
10x 6
5 (x + 1)
=

x
2
x + 1
x + 1
2

x
2
x + 1
x + 1
+ 2

x
2
x + 1
x + 1
+ 2

2x
2
10x 6
5 (x + 1)
=
x
2
5x 3
(x + 1)

x
2
x + 1
x + 1
+ 2

x
2
5x 3 = 0
5
2
=

x
2
x + 1
x + 1
+ 2 (v nghim)

x =
5 +

37
2
x =
5

37
2
Vy tp nghim ca (1.3) l S =

37
2
;
5 +

37
2

2
Bi 4: Gii phng trnh: x
3
+ 3x
2
3
3

3x + 5 = 1 3x (1.4)
Gii
tng: nh gi phng trnh, ta thy phng trnh c dng hng ng thc (x+1)
3
nn
ta s bin i phng trnh theo hng ng thc biu thc gn hn. Tht vy t phng
trnh u ta c biu thc gn hn nh sau:
(x + 1)
3
= 3
3

3x + 5 + 2
n y, ta c th d dng nhm nghim hn. Tht vy, nhm nghim ca phng trnh ta c
phng trnh c nghim l x = 1. i vi cn bc ba, ta cng lm tng t nh cn bc hai.
Li gii:
Vit li (1.4) di dng
(1.4) (x + 1)
3
8 = 3
3

3x + 5 6
(x 1)[(x + 1)
2
+ 2(x + 1) + 4] =
9(x 1)
3

(3x + 5)
2
+ 2
3

3x + 5 + 4

x = 1
[(x + 1)
2
+ 2(x + 1) + 4](
3

(3x + 5)
2
+ 2
3

3x + 5 + 4) = 9 ()
Do c cn bc ba nn vic gii (*) s c phn phc tp hn. Ta c th dng bt ng thc nh
sau:
()

(x + 2)
2
+ 3

3x + 5 + 1

2
+ 3

= 9
128
Li c (x + 2)
2
+ 3 3 v (
3

3x + 5 + 1)
2
+ 3 3 V T() 9
Du bng xy ra khi v ch khi

x = 2
3

3x + 5 + 1 = 0
x = 2
Vy tp nghim ca phng trnh l: S = 2; 1 2
Bi 5: Gii phng trnh:
3

162x
3
+ 2

27x
2
9x + 1 = 1 (1.5)
Gii
Vit li (1.5) di dng
(1.5)
3

162x
3
+ 2 2

27x
2
9x + 1 + 1 = 0

162x
3
6

162x
3
+ 2

2
+ 2
3

162x
3
+ 2 + 4

3x (3x 1)

27x
2
9x + 1 + 1
= 0
(3x 1)

2 (9x
2
+ 3x + 1)

162x
3
+ 2

2
+ 2
3

162x
3
+ 2 + 4

3x

27x
2
9x + 1 + 1

= 0
Xt phng trnh
2 (9x
2
+ 3x + 1)

162x
3
+ 2

2
+ 2
3

162x
3
+ 2 + 4

3x

27x
2
9x + 1 + 1
= 0

2 (9x
2
+ 3x + 1)

162x
3
+ 2

2
+ 2
3

162x
3
+ 2 + 4
=
3x
3

162x
3
+ 2
t a =
3

162x
3
+ 2, suy ra
2

3x +
1
3x
+ 1

= a +
4
a
+ 2 3x +
1
3x
+ 1 =
a
2
+
2
a
+ 1

3x =
a
2
3x =
2
a
x =
1
3
Vy phng trnh cho c nghim duy nht x =
1
3
2.
Bi 6: Gii phng trnh:

x
2
+ 12 + 5 = 3x +

x
2
+ 5 (1.6)
Gii
iu kin: x
5
3
tng: Ta nhn thy x = 2 l mt nghim ca phng trnh. Nh vy phng trnh
cho c th phn tch c v dng (x 2) Q(x) = 0.
Li gii:
129
Vit li (1.6) di dng
(1.6)

x
2
+ 12 4 = 3x 6 +

x
2
+ 5 3

x
2
4

x
2
+ 12 + 4
= 3 (x 2) +
x
2
4

x
2
+ 5 + 3
(x 2)

x + 2

x
2
+ 12 + 4

x + 2

x
2
+ 5 + 3
3

= 0

x = 2
x + 2

x
2
+ 12 + 4

x + 2

x
2
+ 5 + 3
3 = 0 ()
Do
1

x
2
+ 12 + 4
<
1

x
2
+ 5 + 3

x + 2

x
2
+ 12 + 4

x + 2

x
2
+ 5 + 3
< 0 nn () v nghim.
Vy phng trnh cho c nghim duy nht x = 2 2.
trn ta lm quen vi k thut thm bt v tch s, nhng nu phng trnh c nhiu
nghim th khng th dng cch . Sau y l k thut thm bt v tch n. tng ca
phng php ny l dng n nh vai tr ca s sau khi nhm nghim.
Bi 7: Gii phng trnh:

2x 1 + x
2
3x + 1 = 0 (1.7)
Gii
tng: Nhm nghim ta thy x = 1 l nghim ca phng trnh.
Ta th k thut tch s vi phng trnh ny:
(1.7)

2x 1 1 + x
2
3x + 2 = 0

2(x 1)

2x 1 + 1
+ (x 1)(x 2) = 0

x = 1
2

2x 1 + 1
= 2 x ()
Nhn thy () vn cn nghim. Vy vic thm bt s khng th tm c ht nghim. Nu
quy ng phng trnh (), ta li phi gii phng trnh kh phc tp l x = (2 x)

2x 1.
Vy ta cn suy ngh n vic dng n thay th s.
Li gii:
iu kin: x
1
2
Tht vy, t phng trnh u ta c:
(1.7)

2x 1 x + x
2
2x + 1 = 0

(x 1)
2

2x 1 + x
+ (x 1)
2
= 0

x = 1
1

2x 1 + x
= 1()
130
Nh vy ch cn gii (*):
() 1 x =

2x 1

1 2x +x
2
= 2x 1
1 x
1
2

x
2
4x + 2 = 0
1 x
1
2
x = 2

2
Kt lun: phng trnh c tp nghim S =

1; 2

Bi 8: Gii phng trnh:


3

12x
2
+ 46x 15
3

x
3
5x + 1 = 2x + 2 (1.8)
Gii
Vit li (1.8) di dng
(1.8)
3

12x
2
+ 46x 15 (2x + 1)
3

x
3
5x + 1 1 = 0

8x
3
+ 40x 16

12x
2
+ 46x 15 +
2x + 1
2

2
+
3(2x + 1)
2
4

x
3
5x + 2

x
2
5x + 1
1
2

2
+
3
4
= 0

8x
3
40x + 16
(
3

12x
2
+ 46x 15 +
2x + 1
2
)
2
+
3(2x + 1)
2
4
+
x
3
5x + 2
(
3

x
2
5x + 1
1
2
)
2
+
3
4
= 0

8(x
3
5x + 2)

12x
2
+ 46x 15 +
2x + 1
2

2
+
3(2x + 1)
2
4
+
x
3
5x + 2

x
2
5x + 1
1
2

2
+
3
4
= 0

x
3
5x + 2 = 0
8

12x
2
+ 46x 15 +
2x + 1
2

2
+
3(2x + 1)
2
4
+
1

x
2
5x + 1
1
2

2
+
3
4
= 0 ()
Do () v nghim nn(1.8) x
3
5x + 2 = 0

x = 2
x =

2 1
x =

2 1
Vy tp nghim ca phng trnh trn l S =

2 1;

2 1; 2

2
Nhn xt: Cu hi t ra l: Ti sao xut hin i lng x
3
5x + 2 = 0? Bng cch thm
bt ax + b vo phng trnh v ng nht h s, ta s lm xut hin cc i lng nh vy.
in hnh ta xt cc v d sau:
Bi 9: Gii phng trnh:

3x
2
6x 5 =

(2 x)
5
+

(2 x)(2x
2
x 10) (1.9)
Gii
tng: u tin ta c (1.9)

3x
2
6x 5 =

2 x(3x
2
5x 6)
Dng my tnh bm nghim ta thy phng trnh ra nghim xu. Chnh v th, ta s thm bt
131
n cho ph hp c th nhn lng lin hp:
Do gi thit phng trnh ny c

2 x nn ta s thm bt

2 x vo phng trnh:

3x
2
6x 5

2 x =

2 x(3x
2
5x 6 )

3x
2
x(6
2
) 5 2
2

3x
2
6x 5 +

2 x
=

2 x(3x
2
5x 6 )
c nhn t chung, ta s ng nht h s ca 2 v:

6
2
= 5
5 + 2
2
= 6 +
a = 1
Li gii:
iu kin:

x 2
3x
2
6x 5 0

x 2

x
3 + 2

6
3
x
3 2

6
3
x
3 2

6
3
Vit li (1.9) di dng
(1.9)

3x
2
6x 5

2 x =

2 x(3x
2
5x 7)

3x
2
5x 7

3x
2
6x 5 +

2 x
=

2 x(3x
2
5x 7)

3x
2
5x 7 = 0
1

3x
2
6x 5 +

2 x
=

2 x ()
Xt (*) ta c:
() 1 = 2 x +

(2 x)(3x
2
6x 5) x 1 =

(2 x)(3x
2
6x 5)
Vi iu kin x
3 2

6
3
th V T
2

6
3
< 0 V P
Vy phng trnh (1.9) tng ng: 3x
2
5x7 = 0

x =
5 +

109
6
(khng tho x < 2)
x =
5

109
6
(chn)
Vy phng trnh c tp nghim S =
5

109
6
2
Bi 10: Gii phng trnh: x
3
3x + 1 =

8 3x
2
(1.10)
Gii
iu kin:
2

6
3
x
2

6
3
tng: bi ny, tuy bit phng trnh c 2 nghim nhng li l nghim v t nn ta
khng th bit chnh xc nghim. Tuy nhin ta ch quan tm ti biu thc cn thm bt to t
2 nghim trn. Do ta s ln lt dng chc nng Shift Solve tm ra 2 nghim ca phng
trnh l: x
1
= 0, 6180339887...; x
2
= 1, 618033989... sau gn hai nghim ny vo hai bin
132
A v B.
By gi ta s th tm tam thc bc 2 to t 2 nghim trn. Ngha l ta cn tnh A+B v A.B.
Thu c A + B = 1, AB = 1.
iu chng t A, B l hai nghim ca phng trnh: X
2
X 1 = 0
V t y, ta c th d on c x
2
x 1 l mt nhn t ca phng trnh.
Ta vit phng trnh cho li thnh:
(1.10) x
3
3x + 1 (px + q)

8 3x
2
+px +q = 0
x
3
3x + 1 (px +q) +
(px +q)
2
(8 3x
2
)

8 3x
2
+px +q
= 0 (2)
x
3
(p + 3) x + 1 q +
(p
2
+ 3) x
2
+ 2pqx + q
2
8

8 3x
2
+px + q
= 0
n y, xut hin nhn t x
2
x 1 th (p
2
+ 3) x
2
+ 2pqx + q
2
8 = (x
2
x 1) vi
l mt h s. Chn = 4 th ta c mt cp (p, q) tha mn l (p, q) = (1; 2).
Li gii:
Vit li (2) di dng
x
3
2x 1 + 4
x
2
x 1

8 3x
2
+ 2 x
= 0

x
2
x 1

x + 1 +
4

8 3x
2
+ 2 x

= 0
Xt f (x) =

8 3x
2
+ 2 x, ta c: f

(x) =
3x

8 3x
2
1
f

(x) = 0
3x

8 3x
2
= 1 x =

2
3
Ta c bng bin thin:
x
2

6
3

2
3

2

6
3
f

(x) + 0
f(x)
6+2

6
3

6+4

6
3

6+2

6
3
0 < f (x)
6 + 4

6
3
x + 1 +
4

8 3x
2
+ 2 x
= x + 1 +
4
f (x)

2

6
3
+ 1 +
12
6 + 4

6
> 0
Vy phng trnh cho tng ng x
2
x 1 = 0 x =
1

5
2
2
By gi chng ta s phi hp cc k thut trn gii bi ton sau:
Bi 11: Gii phng trnh
(x 1)

2x
2
5x 15 +

2x
3
7x
2
+ 19
2
= 2x
3
7x
2
12x + 17 +

7x (1.11)
Gii
tng: Ta nhn xt thy cc h s c cng s m trong cn thc ging nhau. iu ny
gi cho ta tng thm bt n v s sao cho ph hp nghim.
khng qu ph thuc vo vic on nghim nn ta s thm bt x + tm h s tch
133
hp l. (x 1)

2x
2
5x 15 c dng bc ba ging nh trong cn thc v ngoi cn thc
(v h s ca s m ln nht trong cn v ngoi cn ging nhau) ta s thm bt (x 1). Vic
ng nht cc h s s cho ta bit nn tch

2x
3
7x
2
+ 19
2
nh th no.
(1.11) (x1)(

2x
2
5x 15)+

2x
3
7x
2
+ 19
2
= 2x
3
7x
2
(12+)x+17++

7x

(x 1)(2x
2
5x 15
2
)

2x
2
5x 15 +
+

2x
3
7x
2
+ 19
2
= 2x
3
7x
2
(12 + )x + 17 + +

7x

2x
3
7x
2
(10 +
2
)x + 15 +
2

2x
2
5x 15 +
+

2x
3
7x
2
+ 19
2
= 2x
3
7x
2
(12+)x+17++

7x
ng nht h s, ta c:

10 +
2
= 12 +
15 +
2
= 17 +

2
2 = 0

= 1
= 2
Do nhn lng lin hp vi

ax +b c nn ta nhn nghim dng. Vy ta chn = 2.


Li gii:
iu kin:

x
5 +

145
4
x
3

7
2
x
2
+
19
2
0
Vit li (1.11) di dng
(1.11)
2x
3
7x
2
14x + 19

2x
2
5x 15 + 2
+

2x
3
7x
2
+ 19
2
= 2x
3
7x
2
14x + 19 +

7x

2x
3
7x
2
14x + 19

2x
2
5x 15 + 2
+

2x
3
7x
2
+ 19
2

7x = 2x
3
7x
2
14x + 19

2x
3
7x
2
14x + 19

2x
2
5x 15 + 2
+
2x
3
7x
2
14x + 19

2x
3
7x
2
+ 19
2
+

7x
= 2x
3
7x
2
14x + 19

2x
3
7x
2
14x + 19 = 0
1

2x
2
5x 15 + 2
+
1

2x
3
7x
2
+ 19
2
+

7x
= 1()

x = 1
x =
5 +

177
4
x =
5

177
4
1

2x
2
5x 15 + 2
+
1

2x
3
7x
2
+ 19
2
+

7x
= 1()
D thy VT ca phng trnh () l hm nghch bin. Mt khc theo iu kin xc nh
x
5 +

145
4
> 1. Vy phng trnh (*) v nghim.
Xt vi iu kin xc nh. Vy phng trnh trn c nghim duy nht l x =
5 +

177
4
2
Nhn xt: C th nhm c nghim x = 1. T vic tch n s d dng hn. Nhng mt
cch tng qut ta s lm nh trn trong mi trng hp c kh nng dng lng lin hp. Tuy
nhin khuyt im ca phng php ny l nu gp phng trnh khng c bt k 1 nghim
134
hi t no v sau khi thm bt x+ ta khng tm c ; hoc gp 1 h phng trnh phc
tp trong khi ng nht h s th vic tch n dng lng lin hp kh c th t hiu qu.
Nhng lng lin hp li c th mnh trong 1 s bi ton xt iu kin phc tp. in hnh l
nhng phng trnh v t cha tr tuyt i:
Bi 12: Gii phng trnh:

3 x +

2 + x = x
3
+x
2
4x 4 +[x[ +[x 1[ (1.12)
Gii
iu kin: 2 x 3
(1.12)

3 x [x 1[

2 + x [x[

= x
3
+x
2
4x 4

x
2
+x + 2

3 x +[x 1[
+
x
2
+ x + 2

2 + x +[x[
= (x + 2) (x + 1) (x 2)
(2 x) (x + 1)

3 x +[x 1[
+
1

2 + x +[x[
+ (x + 2)

= 0

x = 1
x = 2
Vy phng trnh c tp nghim S = 1; 2 2
Nhn xt: Vi bi ny, vic xut hin thm cc a thc cha tr tuyt i tng chng nh
s gy cho ta thm kh khn trong vic gii quyt. Nhng nh s dng phng php nhn
lng lin hp, bi ton c gii quyt nhanh chng! Khi y, ta ch cn chuyn cc lng
trn v ng v tr v s dng phng php nhn lin hp l .
k thut ghp hng t
Bi 1: Gii phng trnh:

10x + 1 +

3x 5 =

9x + 4 +

2x 2 (2.1)
Gii
iu kin: x
5
3
tng: i vi k thut p dng vo mt s bi tp, vic on nghim c th khng cn
thit v ta s nhm tng hng t ca 2 v vi nhau to ra nhn t chung. Tht vy, ta c
th thy:
10x + 1 9x 4 = x 3 v 3x 5 2x + 2 = x 3
Vy ta c th d dng thy c nhn t chung cn nhm l x3 v khng cn nhm nghim.
Do ta c th gii bi phng trnh ny nh sau:
Li gii:
Vit li (2.1) di dng
(2.1)

10x + 1

9x + 4 +

3x 5

2x 2 = 0

x 3

10x + 1 +

9x + 4
+
x 3

3x 5 +

2x 2
= 0

x = 3
1

10x + 1 +

9x + 4
+
1

3x 5 +

2x 2
= 0 (v nghim)
x = 3 (chn)
135
Vy phng trnh c tp nghim S = 3 2
M RNG:
Sau y ta s xt h s ca cc cn thc. Nu thay i h s ca cc cn thc trn, ta s c
mt v d khc kh hn:
Bi 1*: Gii phng trnh

26

10x + 1 +

26
31

3x 5 =
28

26

9x + 4
5

806

2x 2 (2.1)
Gii
iu kin: x
5
3
i vi bi ny, tng ca ta l ng nht cc h s to ra nhn t chung: Quan st
phng trnh ny, ta thy c kh nng ghp i xng cc phng trnh ca 2 v to ra nhn
t chung l x3. Nhng h s y kh phc tp, v th ta cn phi ng nht h s ghp
cho ph hp.
Ta thm bt

9x + 4 v

2x 2 tm h s ; ph hp:

26

10x + 1

9x + 4+

26
31

3x 5

2x 2 =

28

26

9x + 4

806
+

2x 2
Ta d on =

26, =

26
31
th VT s xut hin nhn t chung l x 3. kim chng
li, ta s lm nh sau:
xut hin nhn t chung l x 3 th VP sau khi nhn lin hp phi xut hin x 3 hoc
3 x. Vy tm ra s ; nhanh gn, ta c th tm a, b tho

9x + 4 b

2x 2 = x 3
a

9x + 4 b

2x 2 = 3 x
Ta a v gii h phng trnh:

9a
2
2b
2
= 1
4a
2
+ 2b
2
= 3

9a
2
2b
2
= 1
4a
2
+ 2b
2
= 3

a
2
=
2
13
b
2
=
31
26

a =

2
13
b =

31
26
T tm c h s , :

28

26
=

2
13
5

806
+ =

31
26

26
=

26
31
iu ny ng nh ta d on, vy lm nh tng t nh bi 1 ta s c nghim duy nht
l x = 3. Vy ta t hi c th tm c h s bt k cho phng trnh trn hay khng? Sau y
ta s xt iu :
Bi 2*: Tm mi quan h gia cc h s phng trnh sau c nghim duy nht:
a

10x + 1 + b

3x 5 = c

9x + 4 + d

2x 2
136
Gii
Ta bin i phng trnh nh sau:
a(

10x + 1

9x + 4) + b(

3x 5

2x 2) = (c a)

9x + 4 + (d b)

2x 2
Theo nh V d 1 (*) ta s c h phng trnh sau:

(c a)
2
=
2
13
(d b)
2
=
31
26

c a =

2
13
d b =

31
26

a = c

2
13
b = d +

31
26
Ta chn h s nh trn l c th nhn lng lin hp cho a

9x + 4 v b

2x 2.
Vy

a = c

2
13
b = d +

31
26
l h s chun phng trnh c nghim duy nht. 2
Bi 2: Gii phng trnh

3x
2
5x + 1

x
2
2 =

3 (x
2
x 1)

x
2
3x + 4 (2.2)
Gii
tng:
Trc ht, kim tra ta thy c rng phng trnh cho c mt nghim x = 2 nn ta s c
gng a phng trnh trn v phng trnh tch xut hin nhn t (x 2). Ta c nhn xt
rng: (3x
2
5x + 1) (3x
2
3x 3) = 2 (x 2) v (x
2
2) (x
2
3x + 4) = 3 (x 2)
Ta i n li gii nh sau:
Li gii:
Vit li (2.2) di dng
(2.2)

3x
2
5x + 1

3 (x
2
x 1) =

x
2
2

x
2
3x + 4

2x + 4

3x
2
5x + 1 +

3 (x
2
x + 1)
=
3x 6

x
2
2 +

x
2
3x + 4
(x 2)

3x
2
5x + 1 +

3 (x
2
x 1)
+
3

x
2
2 +

x
2
3x + 4

= 0
x = 2
Vy phng trnh (2.2) c nghim duy nht x = 2. 2
Bi tp t luyn
1. Bi tp c bn
1)

x
2
3x + 3 +

x
2
3x + 6 = 3
137
2)

3 x +x
2

2 x + x
2
= 1
3)

x
2
3x + 1 +

2x
2
6x + 1 = 2
4)

1 x
x
=
2x +x
2
1 + x
2
5)
3

x 2 +
3

2x 3 = 1
6)
3

x
2
1 +

x 7 = 4
7)
4

x + 2
3

x
2
+ 7 +

x
3
+ 1 x
4
= 1
8)

x + 3 +
3

x = 3
9)

5 x +

x 1 = x
2
+ 2x + 1
10)

x
2
+ x + 1

2x
2
+ 3x 1 =

3
11)

x
2
+ 2x + 3 +

4x
2
+ 5x + 6 =

7x
2
+ 8x + 9 +

10x
2
+ 11x + 12
12)

2x 1 + x
2
3x + 1 = 0
13)

4x + 1

3x 2

= x 5
14)

x + 1 +

x + 4 +

x + 9 +

x + 16 =

x + 100
15)

4x + 1

3x 2 =
x + 1
3
16)

3x
2
7x + 3

x
2
2 =

3x
2
5x 1

x
2
3x + 4
17)

x
2
+ 12 + 5 = 3x +

x
2
+ 5
18)
3

x
2
1 + x =

x
3
1
19) x
2
3x 4 =

x 1 (x
2
4x 2)
2. Bi tp nng cao
1)
3

x
2
1 +

x 3 +

x + 1 + x =
x + 3
x
2
6
+ 5
2) x
2
+x 1 = (x + 2)

x
2
2x + 2
3)

5x 1 +
3

9 x = 2x
2
+ 3x 1
4)

3x
2
7x + 3

x
2
2 =

3x
2
5x 1

x
2
3x + 4
5)

2x
2
5 + 2x
2
5 +
3

4x
4
29x
2
+ 25 =

3x +

12x
3
9x
2
30x
6)

2x
2
+x + 6 +

x
2
+ x + 3 = 2(x +
3
x
)
7)

2x + 1 +

x =

2x
2
+ 4x 23
8) x
2
x 3 +

2x + 5 = 0
9) 2

x
2
7x + 10 = x +

x
2
12x + 20
10)
1

x 1
+
2
x
2
+
1
2x
=
7
4
11)

2 (x
2
+ 8) = 5

x
3
+ 8
12)

x
2
+ x + 1
x + 4
+
x
2
2
=
1

x
2
+ 1
+ 2
13)

2x
2
3x + 1 =
x
2
1
2x 3
14)

x 3

2x 1 1
=
1

x + 3

x 3
138
PHNG PHP DNG N IU HM S
Trong nhng bi ton gii phng trnh v t th dng n iu ca hm s l mt phng
php mnh v thng cho ta li gii p. Bi vit ny s gii thiu mt s ng dng ca phng
php trn. Mi bi ton iu c trnh by theo th t tng - Li gii Nhn xt, vi
mong mun cho bn c c mt ci nhn su hn v cch t duy v kinh nghim gii ton.
L thuyt
nh l 1: Nu hm s y = f(x) lun ng bin (hoc nghch bin) v lin tc trn D th s
nghim trn D ca phng trnh f(x) = k khng nhiu hn mt v x, y D : f(x) = f(y)
x = y.
nh l 2: Nu hm s f(x) v g(x) n iu ngc chiu v lin tc trn D th s nghim
trn D ca phng trnh f(x) = g(x) khng nhiu hn mt.
nh l 3: Nu hm s f(x) lun ng bin (hoc lun nghch bin) trn D th
f(x) > f(y) x > y ( hoc x < y)
Vn dng linh hot cc nh l trn, t mt phng trnh n x, ta s a hai v v dng
f(g(x)) = f(h(x)) vi f(t) l mt hm n iu trn min D ang xt. Thng thng ta c th
d on c h(x) v bc ca g(x), t ng nht h s tm g(x) (u tin h s nguyn).
Cc bi ton sau s lm r tng trn:
Bi tp v d
Bi 1: Gii phng trnh:

3x + 1 +

x +

7x + 2 = 4 (1.1)
Gii
tng: VT ton du cng nn ta hi vng y l mt hm ng bin theo x. Khi theo
nh l 1, (1.1) c nghim duy nht (d thy l x = 1).
Li gii:
KX: ()

x >
1
3
x +

7x + 2 > 0
t VT (1.1) l f(x). Ta c
f

(x) =
3
2

3x + 1
+ (1 +
7
2

7x + 2
).
1
2

x +

7x + 2
> 0 x tho (*)
Vy f(x) = 4 = f(1) x = 1
Th li ta thy x = 1 tho phng trnh.Vy (1.1) c tp nghim S = 1 2
Nhn xt: y l mt ng dng c bn ca phng php n iu. Ta ch vic a cc bin
v cng mt v v xt o hm. Chng ta s khng i su vo dng ny, m tp trung nhiu
hn v cch xy dng hm s.
Bi 2: Gii phng trnh vi a > 0 (x l n): x
3
b = a
3

ax + b (2.1)
Gii
139
tng: Ta ngh ti vic a hai v v dng f(g(x)) = f(h(x)) trong f(t) = mt
3
+ nt.
C th xc nh h(x) VP chnh l
3

ax +b, cn g(x) VT c bc nht nn g(x) = px +u.


tip, ta thy VT(2.1) sau khi bin i s tr thnh: m(px +u)
3
+n(px +u). Nh vy th
hng t bc ba s l mp
3
x
3
, trong khi phng trnh ban u l x
3
. Do mp
3
= 1
V u tin s nguyn nn ta ly m = p = 1. Tng t, VP th hng t bc nht l a
3

ax +b,
tng ng vi nh(x) trong f(h(x)), nn n = a.
Vy f(t) = t
3
+at. Do ta cn a (2.1) v dng
(x +u)
3
+a(x +u) = ax +b +a
3

ax +b (2.2)
Tip tc phn tch, ta thy VT khng xut hin x
2
nn c ngay u = 0, v nu u = 0 ta khng
th kh hng t 3ux
2
.
Ngha l
(2.2) x
3
+ax = ax + b + a
3

ax +b
D thy ch cn cng ax + b vo 2 v ca (2.1) l ta c (2.2). Cng vic n y tr nn n
gin.
Li gii:
(2.1) x
3
b +ax +b = ax + b + a
3

ax +b
f(x) = f(
3

ax +b) vi f(t) = t
3
+at
x =
3

ax + b x
3
= ax +b
y chnh l phng trnh bc 3 dng c bn. 2
Nhn xt: Bi ton trn cho ta mt cch nhn s lc v n iu hm s, trong phn
quan trng nht l xy dng hm v dng nhng nh gi thch hp tm ra h s. Chng
ta cng c th m rng hn mt cht:
Bi 2*: Gii phng trnh x
n
b = a
n

ax +b (n N), n l v a > 0
Sau y l mt v d kh hn:
Bi 3: Gii phng trnh 8x
3
36x
2
+ 53x 25 =
3

3x 5 (3.1)
Gii
tng: Ta cn a 2 v v biu thc dng f(g(x)) = f(h(x)) trong f(t) = mt
3
+ nt
rng hng t
3

3x 5 VP c bc thp nht nn tng ng vi nh(x) trong f(h(x)),


vy n = 1.
Nh bi 2, ta xc nh g(x) = px + u. VT sau khi bin i s l m(px + u)
3
+ n(px + u). Xt
hng t bc 3 ta c mp
3
x
3
= 8x
3
. Nh vy mp
3
= 8. Tuy nhin n y li c 2 trng hp
m ta s phi ln lt xt: m = 8, p = 1 hoc m = 1, p = 2
Nu m = 1 : f(t) = t
3
+t. Do cn a (3.1) v dng
(2x +u)
3
+ (2x +u) = 3x 5 +
3

3x 5
8x
3
+x
2
(12u) + x(6u
2
1) + u
3
+u + 5 =
3

3x 5
140
ng nht h s vi VT ca (3.1) ta c

12u = 36
6u
2
1 = 53
u
3
+u + 5 = 25
u = 3
Vy trng hp m = 1 cho kt qu, do khng cn xt m = 8.
Li gii:
(3.1) 8x
3
36x
2
+ 54x 27 + 2x 3 = 3x 5 +
3

3x 5
(2x 3)
3
+ 2x 3 = 3x 5 +
3

3x 5
f(2x 3) = f(
3

3x 5)(2.2) vi f(t) = t
3
+t
Ta cf(t) ng bin trn R do
(3.2) 2x 3 =
3

3x 5 (2x 3)
3
= 3x 5
8x
3
36x
2
+ 51x 22 = 0

x = 2
x =
5

3
4
Vy (3.1) c tp nghim S = 2;
5

3
4
2
Nhn xt: i khi ta cn tinh trong vic xy dng hm, nh trong bi trn h s bc cao
nht c th l 8 hoc 1. Mt v d khc:
Bi 3*: Gii phng trnh 4x
3
+ 18x
2
+ 27x + 14 =
3

4x + 5
Lu rng 4x
3
= 4(x
3
) =
1
2
.(2x)
3
do ta cng cn xt 2 trng hp.
Bi ton trn cng c th gii bng cch t
3

4x + 5 = 2y + 3 a v h i xng loi II.


Nhng bc phn tch trn nhn tuy di nhng khi quen ri th ta c th tnh rt nhanh.Tuy
nhin, trong mt s bi ton, hm f(t) ca ta khng ng bin trn R nhng ta c th ch cn
xt n iu trn min xc nh D.
Bi 4: Gii phng trnh 9x
2
28x + 21 =

x 1 (4.1)
Gii
tng: Ta xy dng hm f(t) = mt
2
+nt. rng hng t

x 1 VP c bc thp nht
nn tng ng vi nh(x) trong f(h(x)), do n = 1. Nh bi 3, khi xc nh g(x) = px +u
v mp
2
= 9, ta cng th xt 2 trng hp: m = 9, p = 1 hoc m = 1, p = 3.
Nu m = 9 : f(t) = 9t
2
+t. Vy cn a (3.1) v dng
9(x +u)
2
+x +u = 9(x 1) +

x 1
9x
2
+x(18u 8) + u
2
+u + 9 =

x 1
141
ng nht h s ta c

18u 8 = 28
u
2
+ u + 9 = 21

u =
10
9
u 4; 3
u
Nu m = 1 : f(t) = t
2
+t. Vy cn a (3.1) v dng
(3x +u)
2
+ 3x +u = x 1 +

x 1 9x
2
+x(6u + 2) + u
2
+ u + 1 =

x 1
ng nht h s ta c

6u + 2 = 28
u
2
+u + 1 = 21
u = 5
n y c l bi ton c gii quyt nhng tht ra chng gai cn pha trc. Th lm
tip ta s c
(4.1) x
2
30x + 25 + 3x 5 = x 1 +

x 1
f(3x 5) = f(

x 1) (4.2) vi f(t) = t
2
+t
(!) Lu rng f(t) = t
2
+ t ch ng bin trn (
1
2
; +) v nghch bin trn (;
1
2
), hn
na

x 1 0
1
2
.
Nh vy t (4.2) ta ch suy ra 3x 5 =

x 1 khi 3x 5
1
2
x
3
2
.
Cn x [1;
3
2
] th sao? Li rng hm s bc 2 cng c ci hay ca n, l t
2
= (t)
2
.
trn, da vo h s ca x
2
, ta ch mi xt g(x) = px +u vi mp
2
= 9 v p N

, nhng thc ra
vn cn trng hp m = 1, p = 3 . Ta s xt tip trng hp ny:
Cn a (4.1) v dng
(u 3x)
2
+u 3x = x 1 +

x 1 9x
2
+ x(6u 4) + u
2
+ u + 1 =

x 1
ng nht h s ta c

6u 4 = 28
u
2
+u + 1 = 21
u = 4
Kim tra li: C x <
3
2
4 3x >
1
2
. Vy chn u = 4.
n y bi ton mi thc s c gii quyt.
Li gii:
KX: x 1
Nu x
3
2
3x 5
1
2
Ta c
(4.1) (3x 5)
2
+ (3x 5) = (x 1) +

x 1
f(3x 5) = f(

x 1) (vi f(t) = t
2
+t) 3x 5 =

x 1
142

3x 5 0
(3x 5)
2
= x 1

x
5
3
x

2;
13
9

x = 2
Nu 1 x <
3
2
: 4 3x >
1
2
Ta c
(4.1) (4 3x)
2
+ 4 3x = x 1 +

x 1
f(4 3x) = f(

x 1) (vi f(t) = t
2
+ t) 4 3x =

x 1

4 3x 0
(4 3x)
2
= x 1

x
4
3
x

25

13
8

x =
25

13
18
(chn)
Vy (3.1) c tp nghim S =

2;
25

13
18

2
Nhn xt: Cn linh hot trong vic xy dng hm s, nht l i vi hm bc chn.
Ta cng c th gii bi ton trn bng cch t

x 1 = 3y 5 a v h i xng loi 2.
Bi 5: Gii phng trnh 3x
3
6x
2
3x 17 = 3
3

9(3x
2
+ 21x + 5) (5.1)
Gii
tng: Nh nhng bi trc, u tin ta th a 2 v v biu thc dng f(t) = 3t
3
+ 3t.
(5.1) tr thnh:
3(x u)
3
+ 3(x u) = 9(3x
2
+ 21x + 5) + 3
3

9(3x
2
+ 21x + 5)
3x
3
+ x
2
(9u + 27) + x(9u
2
186) + (3u
3
3u 45) = 3
3

9(3x
2
+ 21x + 5)
ng nht h s vi VT(5.1) ta c

9u + 27 = 6
9u
2
186 = 3
3u
3
3u 45 = 17
D thy h ny v nghim. Vy ta khng th xy dng hm nh bnh thng. rng nguyn
nhn dn n vic ny l v h s ca 9(3x
2
+21x +5) qu ln, cn tr vic ng nht h s.
Vy ta hy th xy dng hm theo mt hng khc:
Nhn 9 cho 2 v ca (5.1) ta c:
(5.1) 27x
3
54x
2
27x 153 = 27
3

9(3x
2
+ 21x + 5) (5.2)
By gi ta s a 2 v v biu thc dng f(t) = t
3
+ 27t (cch tm hm s tng t nhng bi
trn).
(5.2) tr thnh:
(3x u)
3
+ 27(3x u) = 9(3x
2
+ 21x + 5) + 27
3

9(3x
2
+ 21x + 5)
27x
3
+ x
2
(27u + 27) + x(9u
2
108) 27u u
3
45 = 27
3

9(3x
2
+ 21x + 5)
143
ng nht h s ta c

27u + 27 = 54
9u
2
108 = 27
27u u
3
45 = 153
u = 3
Bi ton c gii quyt.
Li gii: Nhn 9 vo 2 v ta c phng trnh:
(3x 3)
3
+ 27(3x 3) = 9(3x
2
+ 21x + 5) + 27
3

9(3x
2
+ 21x + 5)
f(3x 3) = f(
3

9(3x
2
+ 21x + 5)) (vi f(t) = t
3
+ 27t)
3x 3 =
3

9(3x
2
+ 21x + 5) (3x 3)
3
= 9(3x
2
+ 21x + 5)
3(x 1)
3
= (3x
2
+ 21x + 5) 3x
3
6x
2
12x 8 = 0
x =
2
3
(1 +
3

2)
2
(tham kho cch gii PT bc 3 tng qut)
Nhn xt: Mt cu hi t ra l: Ti sao li nhn 9 m khng phi l s khc? Tht ra iu
ny c cp n ri. Khi xy dng hm f(t) = mt
3
+3t, ta thng ngh ti g(x) = px+q
nn mp
3
= 3, do m = 3, p = 1 m qun rng cn c m =
1
9
, p = 3 (trng hp ny tht ra
him gp, tr nhng bi h s ln nh bi ny). Nh vy f(t) cng c th l
t
3
9
+ 3t (v trong
bi ny th ng l vy). Vic nhn 9 ch n gin l kh mu s.
Lu rng vi nhng dng phng trnh nh trn, ta s khai trin v ng nht h s cc bc
3, 2, 1, 0. Ngha l ta c mt 4 h phng trnh, do s n ti a c th l 4. bi trn,
ta vn c th t f(t) = mt
3
+ t, khi trong qu trnh ng nht h s s xut hin thm 2
n m v p nhng vn gii c. Ta cng xem qua bi tng t:
Bi 6: Gii phng trnh x
3
6x
2
+ 12x 7 =
3

x
3
+ 9x
2
19x + 11 (6.1)
Gii
tng: Ta a hai v v hm s f(t) = mt
3
+ t. H s bc nht l 1, v n tng ng vi
3

x
3
+ 9x
2
19x + 11 VP. VT th hng t bc 3 l x
3
nn ta ngh ti m = 1, nhng vic
ng nht h s khng thnh cng ( bi ny nguyn nhn l do VP cng c x
3
trong cn).
Vy, vi nhn xt rng VT th t = g(x) = px + u, ta s tm c p, u v m.
Cn a hai v v dng
m(px +u)
3
+ (px +u) = m(x
3
+ 9x
2
19x + 11) +
3

x
3
+ 9x
2
19x + 11
x
3
(mp
3
+m) + x
2
(3mup
2
9m) + x(3u
2
mp +p + 19m) + mu
3
+u 11m
=
3

x
3
+ 9x
2
19x + 11 (6.2)
ng nht h s VT ca (6.1) v (6.2) ta c

mp
3
+m = 1
3mpu
2
9m = 6
3u
2
mp +p + 19m = 12
mu
3
+u 11m = 7

m =
1
2
p = 1
u = 1
144
Li gii:
Ta vit (6.1) di dng
(x 1)
3
2
+ (x 1) =
x
3
+ 9x
2
19x + 11
2
+
3

x
3
+ 9x
2
19x + 11
f(x 1) = f(
3

x
3
+ 9x
2
19x + 11) (vi f(t) =
t
3
2
+t)
x 1 =
3

x
3
+ 9x
2
19x + 11 (do f(t) ng bin trn R)
(x 1)
3
= x
3
+ 9x
2
19x + 11

x = 1
x = 2
x = 3
Vy (6.1) c tp nghim S = 1; 2; 3 2
Chng ta lm quen vi mt s bi phng trnh tng. Hy xem qua nhng bi phng trnh
c tch.
Bi 7: Gii phng trnh x
3
+ 3x
2
+ 4x + 2 = (3x + 2)

3x + 1 (7.1)
Gii
tng: thot nhn th VT c bc 3, VP c bc
3
2
nn kh c th dng n iu. Nhng
nu VP ta coi y =

3x + 1 l n th VP cng l bc 3 theo y. Nh vy cn phn tch


3x + 2 = m(3x + 1) + n(), khi VP c dng my
3
+ny. D thy t (*) c ngay m = n = 1.
Cng vic cn li l a VT v dng (x u)
3
+ x u l ta c th dng n iu. ng nht
h s ta c u = 1.
Li gii:
KX: x
1
3
Ta c:
(7.1) (x + 1)
3
+x + 1 = (3x + 1 + 1)(

3x + 1) = (

3x + 1)
3
+

3x + 1
f(x + 1) = f(

3x + 1) (vi f(t) = t
3
+t)
x + 1 =

3x + 1 x 0; 1 (tho KX)
Vy (7.1) c tp nghim S = 0; 1 2.
Nhn xt: Vi nhng bi phng trnh tch cn linh hot trong vic i bin v xy dng
hm c th a hai v v hm c trng. Mt s bi nhn vo rt phc tp i hi ta phi
bnh tnh phn tch. Hy nh ta lun c gng phn tch biu thc bc ln theo biu thc bc nh.
Bi 8: Gii phng trnh 3x(2 +

9x
2
+ 3) + (4x + 2)(

1 + x +x
2
+ 1) = 0 (8.1)
Gii
tng: Nhn qua s sp xp ca bi ton, ta thy hai biu thc VT kh ging nhau v
hi vng c th tm hm c trng ca phng trnh t y.
u tin a mi biu thc v 1 v:
(8.1) (4x + 2)(

1 + x + x
2
+ 1) = 3x(2 +

9x
2
+ 3) (8.2)
145
Nh kinh nghim bi 7, ta s phn tch biu thc bc ln theo biu thc bc nh.
Ta c V P(8.2) = 3x(2 +

(3x)
2
+ 3) nn hi vng V T(8.2) cng c th a v
f(t) = t(2 +

t
2
+ 3). V xut hin s 2 trong f(t) ta bin i:
V T(8.2) = (2x + 1)(

4x
2
+ 4x + 4 + 2)
D thy 4x
2
+ 4x + 4 = (2x + 1)
2
+ 3. Vy ta xy dng hm thnh cng.
Tuy nhin hm s f(t) c f

(t) = 2 +
2t
3
+ 3t

t
4
+ 3t
2
nn c th i chiu n iu, do ta phi
c thm mt nhn xt: (8.1) ch c nghim trong [
1
2
; 0]. n y bi ton thc s c gii
quyt.
Li gii:
Nu x > 0 hoc x <
1
2
th (8.1) v nghim. Vy ta xt x [
1
2
; 0].
Ta c
(8.1) (8.2) (2x + 1)(2 +

(2x + 1)
2
+ 3) = (3x)(2 +

(3x)
2
+ 3)
f(2x + 1) = f(3x) (8.3) vi f(t) = t(2 +

t
2
+ 3)
Do f

(t) = 2 +
2t
3
+ 3t

t
4
+ 3t
2
> 0 t [
1
2
; 0] nn (8.3) 2x + 1 = 3x x =
1
5
(chn)
Vy (8.1) c tp nghim S =

1
5

2
Nhn xt: y l mt bi ton hay v kh, i hi phi c k nng bin i linh hot. Ta
cng c li gii gn gi hn, khng cn dng o hm nh sau:
(8.1) (2x + 1)(2 +

(2x + 1)
2
+ 3) = (3x)(2 +

(3x)
2
+ 3) ()
Nu x

1
2
;
1
5

: 3x < 2x 1 < 0 (3x)


2
> (2x + 1)
2
2 +

(3x)
2
+ 3 > 2 +

(2x + 1)
2
+ 3 V T() < V P()
Nu x

1
5
; 0

: Chng minh tng t ta cng c () v nghim.


Nu x =
1
5
: Ta c () nghim ng.
Vy (8.1) c tp nghim S =

1
5

2
Bi tp t luyn
Gii cc phng trnh sau
Bi 1) x
3
15x
2
+ 78x 141 = 5
3

2x 9 (1)
Bi 2) 2
x1
2
x
2
x
= (x 1)
2
(2)
Bi 3) log
3
(
2x 1
(x 1)
2
) = 3x
2
8x + 5 (3)
Bi 4) sin 2x + cos x = 1 + log
2
sin x vi x

0;

2

(4)
Bi 5) Chng minh phng trnh

x
3

18+

5 =

2 (5) c nghim duy nht.


146
PHNG PHP DNG BT NG THC
L thuyt
Ta gii phng trnh, h phng trnh bng bt ng thc da trn hai tng sau:
1) Bin i phng trnh v dng f(x) = g(x) m

f(x) a
g(x) a
hay

f(x) a
g(x) a
vi a l hng s.
Nghim ca phng trnh l cc gi tr x tha mn f(x) = g(x) = a.
2) Bin i phng trnh v dng h(x) = m ( m l hng s) m ta lun c h(x) m hoc
h(x) m th nghim ca phng trnh l cc gi tr x lm cho du ca ng thc xy ra.
Mt s phng php hay c s dng l a v bnh phng ng, s dng tnh n
iu ca hm s nh gi mt cch hp l, s dng mt s bt ng thc nh bt ng
thc AM-GM, BCS v bt ng thc Holder.
Bt ng thc AM-GM : x
1
+x
2
+.... +x
n
n
n

x
1
.x
2
....x
n
, vi x
i
0 (i = 1, n)
ng thc xy ra khi v ch khi x
1
= x
2
= .... = x
n
.
Bt ng thc Cauchy-Schwarz (BCS):

i=1
a
2
i

i=1
b
2
i

i=1
a
i
b
i

2
ng thc xy ra khi v ch khi
a
1
b
1
=
a
2
b
2
= ... =
a
n
b
n
Bt ng thc Holder: (dng m rng ca BCS): cho m b n s dng (m, n 2):
(a
11
, a
12,....,
a
1n
) (a
21
, a
22,....,
a
2n
) ; ....; (a
m1
, a
m2,....,
a
mn
)
Ta lun c:
(a
11
.a
21
....a
m1
+a
12
.a
22
....a
m2
+...a
1n
.a
2n
....a
mn
)
m
(a
11
m
+a
12
m
+... +a
1n
m
)(a
21
m
+ a
22
m
+... +a
2n
m
)...(a
m1
m
+a
m2
m
+... +a
mn
m
)
ng thc xy ra khi v ch khi cc b s t l vi nhau.
Bi tp v d
Bi 1: Gii phng trnh
13

x 1 + 9

x + 1 = 16x
Gii
tng: Ta thy VT c bc thp hn nn ngh ti vic s dng bt ng thc BCS
chng minh V T V P.
y nu k ta s cn chn im ri sao cho xut hin (16x a) t c th tip tc
s dng bt ng thc AM-GM.
Li gii:
147
iu kin: x 1
p dng bt ng thc BCS:
V T
2
= (

13.

13x 13 + 3

3.

3x + 3)
2
(13 + 27)(13x 13 + 3x + 3) = 40(16x 10)
p dng bt ng thc AM-GM:
40(16x 10) = 4.10.(16x 10) 4.(
10 + 16x 10
2
)
2
= (16x)
2
= V P
2
V T V P. ng thc xy ra

13x 13

13
=

3x + 3
3

3
10 = 16x 10
x
Vy phng trnh v nghim. 2
Nhn xt: Ta cng c bi ton tng t:
Bi 1*: Gii phng trnh 13

x
2
x
4
+ 9

x
2
+ x
4
= 16
Ta cng xem qua v d tip theo:
Bi 2: Gii phng trnh x
2
+ 4x + 5 = 2

2x + 3
Gii
tng: Nh bi trn, do VT c bc ln hn VP nn nhiu kh nng c th dng bt ng
thc AM-GM.
Li gii:
iu kin x
3
2
.
T phng trnh, p dng bt ng thc AM-GM:
(2x + 3) + 1 2

2x + 3 = x
2
+ 4x + 5
2x + 4 x
2
+ 4x + 5 x
2
+ 2x + 1 0
(x + 1)
2
0 x = 1 (chn)
Vy phng trnh c nghim duy nht x = 1 2.
Bi 3: Gii phng trnh

x
2
+x 1 +

x
2
+ x + 1 = x
2
x + 2
Gii
tng: rng VT c dng

A+

B nn ta ngh ti vic s dng bt ng thc BCS


tm gi tr ln nht. Ri sau ta s chng minh VP ln hn hoc bng gi tr ny.
Li gii:
iu kin:

x
2
+x 1 0
x
2
+x + 1 0
p dng bt ng thc BCS c:
V T

2(x
2
+ x 1 x
2
+ x + 1) = 2

x
148
Mt khc:
V P 2

x = x
2
x + 2 2

x = (x 1)
2
+ (

x 1)
2
0 V P 2

x V T
ng thc xy ra

x
2
+ x 1 = x
2
+x + 1
(x 1)
2
= 0
x = 1
x = 1 (tho KX)
Vy phng trnh c nghim x = 1 2.
Bi 4: Gii phng trnh x =

x
1
x
+

1
1
x
Gii
y l mt bi ton trong k thi v ch ton cng ha Yugoslavia (Nam T) nm 1977, a s
cc li gii thc hin php bin i tng ng hoc gii bng phng php a v h phng
trnh.Sau y l mt li gii khc s dng bt ng thc AM-GM:
iu kin x 1.
p dng bt ng thc AM-GM ta c:

x
1
x
+

1
1
x
=

(x
1
x
).(1) +

(x 1).
1
x

1
2
(x
1
x
+ 1 + x 1 +
1
x
) = x
Vy phng trnh cho tng ng:

x
1
x
= 1
x 1 =
1
x
Kt hp vi iu kin ta tm c x =
1 +

5
2
2.
Bi 5: Gii phng trnh
4

1 x
2
+
4

1 + x +
4

1 x = 3
Gii
tng: y ta thy rng VT l tng ca cc biu thc c dng
4

A, cn VP l hng
s. Do ta ngh ti vic s dng bt ng thc AM-GM cho tng biu thc VT chng
minh V T V P.
Li gii:
iu kin: 1 x 1
p dng bt ng thc AM-GM ta c :

1 x
2
=
4

(1 x)(1 + x)

1 + x +

1 x
2
(1)
4

1 + x
1 +

1 + x
2
(2)
4

1 x
1 +

1 x
2
(3)
149
Cng theo v (1), (2) v (3) ta c:
4

1 x
2
+
4

1 + x +
4

1 x 1 +

1 + x +

1 x
p dng bt ng thc BCS:

1 + x +

1 x

2(1 + x + 1 x) = 2 V T 1 + 2 = 3 = V P
ng thc xy ra

1 x = 1 +x
1 + x = 1
1 x = 1
x = 0 ( tha)
Vy phng trnh c nghim x = 0 2.
Bi 6: Gii phng trnh
4

(x 2)(4 x) +
4

x 2 +
4

4 x + 6x

3x = x
3
+ 30
Gii
tng: ta thy ngay phi p dng bt ng thc AM-GM cho
4

(x 2)(4 x) v
6x

3x v bt ng thc BCS cho (


4

x 2 +
4

4 x). y vic kh khn ch l phi dng


AM-GM 6x

3x nh hn hoc bng biu thc dng x


3
+a sao cho ph hp nht .
Li gii:
iu kin: 2 x 4
Ap dng bt ng thc AM-GM:

(x 2)(4 x)

x 2 + 4 x
2
= 1 (1)
6x

3x = 2

27x
3
x
3
+ 27 (2)
p dng bt ng thc BCS:
4

x 2 +
4

4 x

2(

x 2 +

4 x)

2.

2(x 2 + 4 x) = 2 (3)
Cng theo v (1), (2) v (3) ta c:
4

(x 2)(4 x) +
4

x 2 +
4

4 x + 6x

3x x
3
+ 30 V T V P
ng thc xy ra

x 2 = 4 x
x
3
= 27

x 2 =

4 x
x = 3 (tha).
Vy phng trnh c nghim x = 3 2.
Bi 7: Gii phng trnh
5

27x
10
5x
6
+
5

864 = 0
Gii
150
tng: H s ca bi kh xu (
5

864 = 2
5

27) nn kh c th bin i ng thc.


Vy ta s c gng s dng bt ng thc chng minh
5

27x
10
+
5

864 5x
6
.
V vic ny d dng hn ta chia c 2 v ca phng trnh cho x
6
.
Li gii:
D thy x = 0 khng phi l nghim ca phng trnh, chia 2 v ca phng trnh cho x
6
, vit
li phng trnh di dng:
5

27x
4
+
2
5

27
x
6
= 5
p dng bt ng thc AM-GM c:
V T =
5

27x
4
3
+
5

27x
4
3
+
5

27x
4
3
+
5

27
x
6
+
5

27
x
6
5
5

27
5
x
12
3
3
.x
12
= 5 = V P
ng thc xy ra
5

27x
4
3
=
5

27
x
6
x =
10

3
Vy phng trnh c nghim x =
10

3 2.
Bi 8: Gii phng trnh

2 x
2
+

2
1
x
2
= 4

x +
1
x

Gii
tng: Hng i ca ta khi nhn vo bi ton l a ht cc bin sang mt v, t dng
BCS mt cch thch hp. kh c x khi dng BCS th ta s nhm
x vi

2 x
2
v
1
x
vi

2
1
x
2
.
Li gii:
KX:

2 x
2
0
2
1
x
2
0
x = 0
Phng trnh cho tng ng vi x +

2 x
2
+
1
x
+

2
1
x
2
= 4.
p dng bt ng thc BCS:

(x +

2 x
2
)
2
2(x
2
+ 2 x
2
) = 4
(
1
x
+

2
1
x
2
)
2
2(
1
x
2
+ 2
1
x
2
) = 4

x +

2 x
2
2
1
x
+

2
1
x
2
2
x +

2 x
2
+
1
x
+

2
1
x
2
4
Du = xy ra ti x = 1 (tha iu kin).
Vy phng trnh c nghim duy nht x = 1 2.
151
Bi 9: Gii phng trnh

3x
2
1 +

x
2
x x

x
2
+ 1 =
1
2

2
(7x
2
x + 4)
Gii
tng: D on phng trnh c nghim duy nht x = 1, t ta ngh ti dng bt
ng thc v VT c nhiu cn thc phc tp.
Li gii:
iu kin x 1 hoc x
1

3
.
p dng bt ng thc BCS:

3x
2
1 +

x
2
x x

x
2
+ 1

(x
2
+ 2)(3x
2
1 + x
2
x + x
2
+ 1)

3x
2
1 +

x
2
x x

x
2
+ 1

(x
2
+ 2)(5x
2
x) (1)
p dng bt ng thc AM-GM:
1
2

2
(7x
2
x + 4) =
1
2

2
[5x
2
x + 2(x
2
+ 2)]
1
2

2
.2

(5x
2
x).2(x
2
+ 2)

1
2

2
(7x
2
x + 4)

(5x
2
x)(x
2
+ 2) (2)
T (1) v (2) ta c

3x
2
1+

x
2
xx

x
2
+ 1 =
1
2

2
(7x
2
x+4) =

(x
2
+ 2)(5x
2
x)
Du = xy ra khi x = 1 (tha iu kin).
Vy nghim ca phng trnh l x = 1 2.
Bi 10: Gii phng trnh

1 x
x
=
2x +x
2
1 + x
2
Gii
iu kin: 0 x 1
d dng hn trong vic nh gi ta vit li phng trnh:

1
x
1 = 1 +
2x 1
1 + x
2
Nhn thy x =
1
2
l mt nghim ca phng trnh. Ta chng minh nghim ny l duy nht:
* Nu 0 x <
1
2
, khi

V T > 1
V P < 1
, phng trnh v nghim.
* Nu
1
2
< x 1, khi

V T < 1
V P > 1
, phng trnh v nghim.
Vy phng trnh c nghim x =
1
2
2.
Nhn xt: li gii trn khc vi nhng bi trc, chng ta khng th chng minh V T V P
hay V T V P nu khng chia trng hp ra xt. Do i vi nhng bi phng trnh c
s dng phng php nh gi i hi ta phi bin i linh hot (nhn qua th c v VP l
hm tng cn VT l hm gim, nn nhiu kh nng PT c nghim duy nht) . K thut chia
152
trng hp ny cng rt quan trng trong gii h phng trnh.
Bi 11: Gii phng trnh
x
2
+ 4x + 5
3x
x
2
+ x + 1
= (x 1)

1
2

1 x

x
2
+x + 1

Gii
K: x 1.
Phng trnh cho tng ng vi:
(x + 2)
2
+
x
2
2x + 1
x
2
+ x + 1
= (x 1)

1
2

1 x

x
2
+ x + 1

hay
(x + 2)
2
= (1 x)

1 x

x
2
+x + 1
1

(1 x)
2
x
2
+ x + 1
t y =

1 x, z =

x
2
+x + 1 (y 0, z 0), phng trnh tr thnh:
(x + 2)
2
= y
2

2y
z
1

y
4
z
2
D thy V T 0. Ta c:
V P = y
2

2y
z
1

y
4
z
2
= y
2

y
2
z
2
+
2y
z
1

= y
2

y
z
1

2
0
T suy ra V T V P v phng trnh tng ng vi:

x + 2 = 0
y

y
z
1

= 0

x + 2 = 0
y = 0 y = z
x = 2 (chn)
Vy phng trnh c nghim duy nht x = 2 2
Bi 12: Gii phng trnh
13

(x
2
3x + 6)
2
+ (x
2
2x + 7)
2

= (5x
2
12x + 33)
2
Gii
tng: thy rng 13 = 2
2
+ 3
2
nn VT c dng (a
2
+ b
2
)(x
2
+ y
2
), t ta ngh ti
vic s dng BCS chng minh V T V P.
Li gii:
p dng bt ng thc BCS c:
V T = (2
2
+ 3
2
)

(x
2
3x + 6)
2
+ (x
2
2x + 7)
2

2(x
2
3x + 6) + 3(x
2
2x + 7)

2
= (5x
2
12x + 33)
2
= V P
ng thc xy ra

2
x
2
3x + 6
=
3
x
2
2x + 7
x = 1 x = 4
Vy phng trnh c tp nghim S = 1; 4 2
Bi 13: Gii phng trnh x
3000
+ 500x
3
+ 1500x + 1999 = 0
153
Gii
Vi x > 0, V T > 0 = V P (v l) x 0 [x[ = x
p dng bt ng thc AM-GM c:

x
3000
+ 2999 = x
3000
+ 1 + 1 +... + 1 3000
3000

x
3000
.1.1...1 = 3000 [x[ = 3000x (1)
x
3000
+ 999 = x
3000
+ 1 + 1 +... + 1 1000
1000

x
3000
.1.1...1 = 1000 [x
3
[ = 1000x
3
(2)
Ly (1)+(2) ta c:
2x
3000
+ 3998 (1000x
3
+ 3000x)
x
3000
+ 500x
3
+ 1500x + 1999 0 V T V P
ng thc xy ra

x
3000
= 1
x 0
x = 1
Vy phng trnh c nghim S = 1 2
Bi 14: Gii cc phng trnh sau:
a) 32x
4
+ (4x 1)
4
=
1
27
b)
(1 + x)
8
+ 16x
4
(1 + x
2
)
4
=
1
8
Gii
a) Ta chng minh bt ng thc ph sau: x
4
+y
4
+z
4

1
27
(x +y +z)
4
Tht vy: x
4
+y
4
+z
4

1
3
(x
2
+y
2
+z
2
)
2

1
3

1
3
(x +y +z)
2

2
=
1
27
(x +y + z)
4
x
4
+y
4
+z
4

1
27
(x +y + z)
4
p dng ta c
32x
4
+ (4x 1)
4
= (2x)
4
+ (2x)
4
+ (1 4x)
4

1
27
(2x + 2x + 1 4x) =
1
27
Do phng trnh trn tng ng 2x = 1 4x x =
1
6
Vy phng trnh c nghim duy nht x =
1
6
2.
b) Ta chng minh bt ng thc ph sau: A
4
+B
4

(A
2
+ B
2
)
2
2

(A + B)
4
8
Vi A = (x + 1)
2
v B = 2x th ta c:
(1 + x)
8
+ 16x
4
(1 + x
2
)
4

1
8
.
(1 + x
2
)
4
(1 + x
2
)
4
=
1
8
Du = xy ra khi v ch khi (x + 1)
2
= 2x x = 2 +

3
Vy phng trnh c nghim duy nht x = 2 +

3 2.
Bi tp t luyn
Bi 1: Gii phng trnh

3x
2
+ 6x + 7 +

5x
2
+ 10x + 14 = 4 2x x
2
154
Bi 2: Gii phng trnh

x
2
6x + 11 +

x
2
6x + 13 +
4

x
2
4x + 5 = 3 +

2
Bi 3: Gii phng trnh 19

x1
+ 5
4

x
2
1
+ 95
6

x
2
3x+2
= 3
Bi 4: Gii phng trnh x
2
3x + 3.5 =

(x
2
2x + 2)(x
2
4x + 5)
Bi 5: Gii phng trnh

5x
3
+ 3x
2
+ 3x 2 =
x
2
2
+ 3x
1
2
Bi 6: Gii phng trnh
(

x
2
8x + 7 +

x
2
8x 9)
x
+ (

x
2
8x + 7

x
2
8x 9)
x
= 2
x+1
Bi 7: Gii phng trnh 8x
2
+

1
x
=
5
2
Bi 8: Gii phng trnh
1
10
(3x
3
+x
2
+ 9x 7) = (x
2
+ 2)
2
+ (x
3
+ 3x 3)
2
MT S BI TON CHN LC
Bi 1: Gii phng trnh
3

7x + 1
3

x
2
x 8 +
3

x
2
8x + 1 = 2
Gii
t a =
3

7x + 1; b =
3

8 + x x
2
; c =
3

x
2
8x + 1
Ta c h

a +b +c = 2
a
3
+b
3
+ c
3
= 8

(a +b +c)
3
= 8
a
3
+b
3
+c
3
= 8
(a +b)(b + c)(c +a) = 0
a = b x = 1 x = 9
b = c x = 1
c = a x = 0 x = 1
Vy phng trnh c nghim S = 1; 1; 0; 9 . 2
Nhn xt: Ta c bi tng t:
3

3x + 1 +
3

5 x +
3

2x 9
3

4x 3 = 0
Bi 2: Gii phng trnh 5(30x
2
4x) = 1336(

30060x + 1 + 1)
Gii
KX: x
1
30060
t y =

30060x + 1 + 1
15
15y 1 =

30060x + 1 15y
2
2y = 2004x
Mt khc t phng trnh u ta c 30x
2
4x = 4008y 15x
2
2x = 2004y
Ta c h phng trnh

15x
2
2x = 2004y
15y
2
2y = 2004x
Tr v theo v ta c (x y)[15(x + y) + 2002] = 0
Vi x = y 15x 1 =

30060x + 1
x = 0 x =
2006
15
155
Vi x = 0 th phng trnh u v nghim.
Vi 15(x + y) + 2002 = 0:
Ta c 30060x + 1 0 y =

30060x + 1 + 1
15

1
15
Nn x +y
1
30060
+
1
15
> 0 15(x +y) + 2002 > 0. Vy trng hp ny loi.
Vy phng trnh c nghim duy nht x =
2006
15
. 2
Bi 3: Gii phng trnh 4

1 x = x + 3 + 3

1 x +

1 x
2
Gii
t x = cos t; t [0; ] ta c phng trnh
4

1 + cos t = cos t + 3 + 3

1 cos t + sin t
4

2 cos
t
2
= cos t + 3 + 3

2 sin
t
2
+ 2 sin
t
2
cos
t
2
4

2 cos
t
2
= 4 2 sin
2
t
2
+ 3

2 sin
t
2
+ 2 sin
t
2
cos
t
2
2 cos
t
2

2 sin
t
2

+ 2 sin
2
t
2
3

2 sin
t
2
4 = 0
2 cos
t
2

2 sin
t
2

sin
t
2
2

2 sin
t
2
+

= 0

sin
t
2
2

2 sin
t
2
2 cos
t
2
+

= 0
2 sin
t
2
2 cos
t
2
+

2 = 0 sin
t
2
cos
t
2
=
1

2
sin

t
2


4

=
1
2

t
2


4
=

6
+ k2
t
2


4
=
7
6
+k2

t =

6
t =
17
6
(l)
i chiu vi iu kin ca t, phng trnh c nghim duy nht x = cos

6
=

3
2
. 2
Bi 4: Gii phng trnh (x
3
3x + 1)

x
2
+ 21 + x
4
3x
2
+x = 21
Gii
Vit li phng trnh cho di dng
(x
2
+ 21) (x
3
3x + 1)

x
2
+ 21 (x
4
2x
2
+x) = 0
t t =

x
2
+ 21 > 0 ta c phng trnh
t
2
(x
3
3x + 1)t (x
4
2x
2
+x) = 0 ()
(*) c = (x
3
3x + 1)
2
+ 4.(x
4
2x
2
+ x) = x
6
2x
4
+ 2x
3
+x
2
2x + 1 = (x
3
x + 1)
2
nn c nghim

t = x(1)
t = x
3
2x + 1(2)
156
Xt (1) ta c:
(1)


x
2
+ 21 = x
x < 0

x
2
+ 21 = x
2
x < 0
(v nghim)
Xt (2) ta c:
(2)

x
2
+ 21 = x
3
2x + 1

x
2
+ 21 5 = x
3
2x 4

x
2
4

x
2
+ 21 + 5
= (x 2)(x
2
+ 2x + 2)

x = 2
x + 2

x
2
+ 21 + 5
= x
2
+ 2x + 2 (3)
Xt (3) ta c:
x + 2

x
2
+ 21 + 5

x + 2

x
2
+ 21 + 5

<

x + 2
5

< x
2
+ 2x + 2 = V P
Suy ra (3) v nghim.
Vy phng trnh c nghim duy nht x = 2. 2
Bi 5: Gii phng trnh x
4
+ 2x
3
+ 2x
2
2x + 1 = (x
3
+ x)

1 x
2
x
Gii
Do x
4
+ 2x
3
+ 2x
2
2x + 1 = x
2
(x + 1)
2
+ (1 x)
2
> 0 x nn iu kin ca x l 0 < x < 1
Vit li phng trnh di dng
x
2
(x + 1)
2
+ (1 x)
2
= (x
2
+ 1)

x(x + 1)(1 x) (2)


t u = x(x + 1); v = 1 x

u, v > 0
u + v = x
2
+ 1
Khi (2) tr thnh
u
2
+v
2
= (u +v)

uv (

v)
2
(u +v +

uv) = 0 u = v
Suy ra x(x + 1) = 1 x x
2
+ 2x 1 = 0 x = 1

2
i chiu iu kin ch c x = 1 +

2 l nghim ca phng trnh. 2


Bi 6: Gii phng trnh 10x
2
+ 3x + 1 = (6x + 1)

x
2
+ 3 ()
Gii
t u = 6x + 1; v =

x
2
+ 3 ta c:
V T()10x
2
+ 3x + 1 =
1
4
(6x + 1)
2
+ (x
2
+ 3)
9
4
=
u
2
4
+ v
2

9
4
(*) tr thnh:
1
4
u
2
+v
2

9
4
= uv (u 2v)
2
= 9 u 2v = 3
157
Vi u 2v = 3
1 + 6x

x
2
+ 3 = 3 3x 1 =

x
3
+ 3

3x 1 0
x
2
+ 3 = (3x 1)
2
x = 1
Vi u 2v = 3 3x + 2 =

x
2
+ 3

3x 2 0
(3x + 2)
2
= x
2
+ 3
x =

7 3
4
Vy phng trnh c tp nghim S =

1;

7 3
4

. 2
Bi 7: Gii phng trnh
8x(1 x
2
)
(1 + x
2
)
2

2

2x(x + 3)
1 + x
2
= 5

2 ()
Gii
t a =
2x
1 + x
2
; b =
1 x
2
1 + x
2
Ta tnh c
2

2x(x + 3)
1 + x
2
=

2(3a b + 1) v a
2
+b
2
= 1 nn (*) tr thnh:
4ab

2(3a b + 1) = 5

2
4ab

2(3a b) = 5 = 3 + 2a
2
+ 2b
2
2(a b)
2
+ 3

2(a 3b) = 0
2(a b)
2

2[2(a b) (a + b)] + 3 = 0
[

2(a b)]
2
2

2(a b) + 1 +

2(a +b +

2) = 0
[

2(a b) 1]
2
+

2(a + b +

2) = 0 ()
Ta cng c [a + b[

2(a
2
+b
2
) =

2 a +b

2
Vy (**) c V T 0 = V P v ng thc khng xy ra. Do phng trnh (*) v nghim. 2
Bi 8: Gii phng trnh (4x
3
x + 3)
3
x
3
=
3
2
()
Gii
t t = 4x
3
x + 3 = t 3 = t 4x
3
+ x
Phng trnh (*) tr thnh:
t
3
x
3
=
t 4x
3
+ x
2
2t
3
+ 2x
3
t x = 0
(t +x)(2t
2
2tx + 2x
2
1) = 0 t +x = 0
T tm c x =
3

3
4
. 2
CnucNo IV: PHNO TRNH M-LOOARIT
Bi vit ny s gii thiu n cc bn mt s Phng php gii phng trnh m - Logarit, vi
mong mun t nhiu s l ti liu tham kho hu ch cho bn c, qua chun b hnh trang
cho cc bn hc sinh trong cc k thi tuyn sinh vo i hc.
L THUYT
Cho phng trnh a
x
= m (a > 0, a = 1), ta c:
Nu m > 0 th phng trnh cho c mt nghim duy nht x = log
a
m.
Nu m 0 th phng trnh cho v nghim.
Hm s logarit:
a
c
= b c = log
a
b.
a
log
a
b
= b.
log
a
(b
1
.b
2
) = log
a
b
1
+ log
a
b
2
.
log
a
b

= . log
a
b.
log
a
c =
log
b
c
log
b
a
.
log
b
c = log
b
a. log
a
c.
log
a
b =
1
log
b
a
.
log
a
c =
1

. log
a
c.
Sau y l mt s phng php gii phng trnh m - logarit.
PHNG PHP T N PH
Phng php t n ph l mt phng php kh ph bin i vi cc bi ton phng trnh
v h phng trnh. Cc bi ton gii phng trnh m ta c th p dng phng php ny,
nu d, th ta s thy ngay du hiu l mt biu thc cha bin no lp i lp li nhiu
ln, cn nu kh hn, th ta cn phi c mt t bin i kho lo, ch yu l a v hnh
dng s khai ca bi ton, l mt phng trnh vi cc biu thc cha bin lp li. Cng c
trng hp, bi ton yu cu ta phi t thm nhiu n ph khc, nhm to ra mt phng
158
159
trnh hoc mt h phng trnh mi d dng gii quyt hn. Sau y, chng ta hy cng nhau
xt cc v d nh nhm lm sng t hn tng gii quyt cc bi ton dng ny.
Bi 1. Gii phng trnh 2
x
2
x
+ 2
2+xx
2
= 3
Gii
tng: Ta nhn thy rng biu thc x
2
x c lp li trong phng trnh trn. V th,
ta s c ngay tng l phi t mt n mi thay th cho biu thc ny.
Li gii:
t t = x
2
x, phng trnh cho tr thnh:
2
t
+ 2
2t
= 3 2
t
+ 4.
1
2
t
= 3 (2
t
)
2
+ 4 = 3.2
t
n y, ta li thy phng trnh thu c c biu thc 2
t
c lp li, vy ta s t thm
mt n mi.
t u = 2
t
, ta c : u
2
3u + 4 = 0. Phng trnh ny v nghim.
Do , ta c phng trnh cho v nghim 2
Bi 2. Gii phng trnh log
2
x +

10 log
2
x + 6 = 9
Gii
t t = log
2
x, phng trnh c vit li thnh:
t +

10t + 6 = 9

t 9
10t + 6 = t
2
18t + 81
t = 3 x = 8
Vy phng trnh cho c mt nhim duy nht x = 8 2
Nhn xt: hai bi ton trn, biu thc lp li c phi by ra trc mt, v chng
ta d dng nhn bit c phng php t n ph i vi bi ton. Nhng bi ton sau y,
cc biu thc cha bin lp li s b giu i, t n gin, n tinh xo, v kh ca vic tm
ra quy tc b n y, cng l nc thang quyt nh mc kh ca bi ton.
Bi 3. Gii phng trnh 3
4x+8
4.3
2x+5
+ 27 = 0 ()
Gii
tng: Vi bi ton ny, ta nhn thy rng cc hng t trong phng trnh u c ly
tha ca 3, do , u tin ta s trit tiu cc ly tha ny.
Li gii:
Ta c:
() 3
4x+8
4.3
2x+5
+ 3
3
= 0 3
4x+5
4.3
2x+2
+ 1 = 0
3.3
4x+4
4.3
2x+2
+ 1 = 0
160
n y, ta d dng c c cch t n ph hp.
t t = 3
2x+2
> 0. Phng trnh cho tr thnh:
3t
2
4t + 1 = 0

t = 1
t =
1
3

3
2x+2
= 1
3
2x+2
= 3
1

x = 1
x =
3
2
Vy phng trnh cho c tp nghim l S =

1;
3
2

2
Ta s luyn tp thm vi bi ton tip theo.
Bi 4. Gii phng trnh 2
2x+6
+ 2
x+7
17 = 0 ()
Gii
Ta c:
() 2
2x+6
+ 16.2
x+3
17 = 0
t t = 2
x+3
> 0, ta c:
t
2
+ 16t 17 = 0

t = 1
t = 17 (loi)
2
x+3
= 1 x = 3
Vy phng trnh cho c tp nghim l S = 3 2
Bi 5. Gii phng trnh

2 +

x
+

x
= 4 ()
Gii
tng: Vi bi ton ny, rng

2 +

= 1. Ta s nhn lng lin hp


lm xut hin biu thc chung.
Li gii:
Ta c:
()

2 +

x
+
1

2 +

x
= 4
t t =

2 +

x
> 0. Phng trnh tr thnh:
t +
1
t
= 4

t = 2 +

3
t = 2


2 +

x
= 2 +

2 +

x
=

2 +

x = 1
x = 1
Vy phng trnh cho c nghim l x = 1 2
Bi 6. Gii phng trnh 2.16
x
15.4
x
8 = 0
Gii
tng: Ta ch rng 16
x
= (4
x
)
2
, do , ta nh hng c ngay phng php t
n ph.
161
Li gii:
t t = 4
x
> 0, ta c:
2t
2
15t 8 = 0

t = 8
t =
1
2
(loi)
4
x
= 8 2
2x
= 2
3
x =
3
2
Vy phng trnh c mt nghim duy nht x =
3
2
2
Bi 7. Gii phng trnh

3 +

x
+ 16

x
= 2
x+3
()
Gii
tng: Vi bi ton ny, tng u tin chnh l nhn lng lin hp cho (3

5). Th
nhng, v phi ca phng trnh by gi l 2
x+3
, do , ta cn phi bin i phng trnh
lm sao cho mt i bin x hng t 2
x+3
.
Li gii:
Ta c:
()

3 +

5
2

x
+ 16.

5
2

x
= 8
n y, ta s dng lng lin hp nh bnh thng.
t t =

3 +

5
2

x
> 0
1
t
=

5
2

x
. Ta c:
t +
16
t
= 8 t = 4

3 +

5
2

x
= 4 x. ln

3 +

5
2

= ln 4 x =
ln 4
ln

3 +

ln 2
Vy phng trnh c mt nghim duy nht x =
ln 4
ln

3 +

ln 2
2
Bi 8. Gii phng trnh

7 + 4

x
3

x
+ 2 = 0
Gii
Vit li phng trnh:

2 +

2x
3

x
+ 2 = 0
t t =

2 +

x
> 0
1
t
=

x
. Ta c:
t
2

3
t
+ 2 = 0

t = 1
t
2
t + 3 = 0

2 +

x
= 1 x = 0
Vy phng trnh cho c nghim duy nht x = 0 2
Bi 9. Gii phng trnh 2.4
1
x
+ 6
1
x
= 9
1
x
Gii
162
tng: Bi ton ny yu cu chng ra cn phi bin i mt hp l to ra biu thc
thch hp cho vic t n ph.
Li gii:
Phng trnh cho tng ng vi
2 +

3
2
1
x
=

3
2
2
x
t t =

3
2
1
x
> 0, ta c:
2 + t = t
2

t = 1 (loi)
t = 2
t = 2

3
2
1
x
= 2
1
x
. ln

3
2

= ln 2 x = log
2
3 1
Vy phng trnh cho c mt nghim duy nht x = log
2
3 1 2
Bi 10. Gii phng trnh 3.16
x
+ 2.81
x
= 5.36
x
Gii
Phng trnh cho tng ng vi
3 + 2

9
4

2x
= 5

9
4

x
t t =

9
4

x
> 0. Phng trnh tr thnh:
3 + 2t
2
= 5t

t = 1
t =
3
2

x = 0
x =
1
2
Vy phng trnh c tp nghim S =

0;
1
2

2
Bi 11. Gii phng trnh 125
x
+ 50
x
= 2
3x+1
Gii
Phng trnh tng ng vi

125
8

x
+

50
8

x
= 2.
t t =

5
2

x
> 0. Ta c:
t
3
+ t
2
2 = 0 t = 1 x = 0
Vy phng trnh cho c nghim duy nht x = 0 2
Bi 12. Gii phng trnh
1
4 log x
+
2
2 + log x
= 1
163
Gii
t t = log x, suy ra

t = 4
t = 2
. Phng trnh tr thnh
1
4 t
+
2
2 + t
= 1

t = 1
t = 2

x = 10
x = 100
Vy phng trnh cho c hai nghim l x = 10 v x = 100 2
Bi 13. Gii phng trnh

1 + log
0.04
x +

3 + log
0.2
x = 1
Gii
tng: i vi cc biu thc log
a
f(x) vi a > 0, th iu kin hm logarit ny tn ti
l f(x) > 0. Khi tin hnh gii cc bi ton phng trnh logarit, ta phi c bit ch n
iu kin xc nh ca bi ton.
Li gii:
KX: log
0.2
x 2, x > 0.
t t = log
0.2
x t 2. Ta c:

1 +
1
2
t +

3 + t = 1 2

1
2
t
2
+
5
2
t + 3 = 3
3
2
t
Do t 1 nn 3
3
2
t 0, suy ra t = 2, hay x = 25 2
Bi 14. Gii phng trnh log
x
2 16 + log
2x
64 = 3
Gii
Ta nhn thy rng x = 1 v x =
1
2
khng tha mn phng trnh cho.
Vi mt s bin i nh, phng trnh tng ng vi:
2
log
2
x
+
6
1 + log
2
x
= 3
t t = log
2
x

t = 0
t = 1
. Ta c:
2
t
+
6
t + 1
= 3

t = 2
t =
1
3

x = 4
x = 2
1/3
Vy phng trnh cho c tp nghim S =

4; 2
1/3

2
Bi 15. Gii h phng trnh sau:

log
1+x
(1 2y +y
2
) + log
1y
(1 + 2x +x
2
) = 4 (1)
log
1+x
(1 + 2y) + log
12y
(1 + 2x) = 2 (2)
(H Quc gia TP HCM)
164
Gii
iu kin:

0 < 1 y = 1
0 < 1 + x = 1

x > 1
y < 1
.
Ta c:
(1) log
x+1
(1 y)
2
+ log
1y
(1 + x)
2
= 4 log
1+x
(1 y) + log
1y
(1 + x) = 2 (3)
t t = log
1+x
(1 y), suy ra log
1y
(1 + x) =
1
t
. Phng trnh (3) tr thnh:
t +
1
t
= 2 t
2
2t + 1 = 0 t = 1
Suy ra
log
1+x
(1 y) = 1 1 y = 1 +x x = y
Thay vo phng trnh (2), ta c
log
1+x
(1 2x) + log
1+x
(1 + 2x) = 2 log
1+x
(1 4x
2
) = 2
1 4x
2
= (1 +x)
2
5x
2
+ 2x = 0

x = 0 (loi)
x =
2
5
Suy ra y =
2
5
.
Th li nghim nhn c vo h phng trnh ban u, ta c c nghim

2
5
;
2
5

l nghim
duy nht ca h cho 2
Bi 16. Gii phng trnh log
3
(3
x
1). log
3
(3
x+1
3) = 6
Gii
t t = log
3
(3
x
1), phng trnh c vit li:
t
2
+t 6 = 0

t = 2
t = 3

log
3
(3
x
1) = 2
log
3
(3
x
1) = 3

x = log
3
10
x = log
3
28 3
Vy phng trnh cho c hai nghim l x = log
3
10 v x = log
3
28 3 2
Bi 17. Gii phng trnh log
4
(x

x
2
1). log
5
(x +

x
2
1) = log
20
(x

x
2
1)
Gii
t t = x

x
2
1, suy ra x +

x
2
1 =
1
t
.
Phng trnh cho tng ng vi
log
4
t. log
5
1
t
= log
20
t log
4
t. log
5
t = log
20
4. log
4
t

log
4
t = 0
log
5
t = log
20
4
165

t = 1
t = 5
log
20
4
.
Vi t = 1 suy ra x = 1.
Vi t = 5
log
20
4
suy ra

x
2
1 = 5
log
20
4
x +

x
2
1 = 5
log
20
4
x =
1
2

5
log
20
4
+ 5
log
20
4

Th li cc gi tr nhn c, phng trnh cho c hai nghim l x = 1 v x =


1
2

5
log
20
4
+ 5
log
20
4

2
Bi 18. Gii phng trnh log
2
(x
2
+ 3x + 2) + log
2
(x
2
+ 7x + 12) = 3 + log
2
3
Gii
Nghim ca phng trnh (nu c) phi tha mn iu kin xc nh

x
2
+ 3x + 2 0
x
2
+ 7x + 12 0
Phng trnh tng ng vi
log
2
(x + 1)(x + 2)(x + 3)(x + 4) = log
2
24 x + 1)(x + 2)(x + 3)(x + 4) = 24
t t = x
2
+ 5x + 4, ta thu c t(t + 2) = 24

t = 6
t = 4
Suy ra

x
2
+ 5x + 4 = 6
x
2
+ 5x + 4 = 4

x = 0
x = 5
.
Nhn thy cc gi tr ny tha mn tp xc nh ban u. Vy, phng trnh cho c hai
nghim l x = 0 v x = 5 2
Qua cc bi ton trn, chng ta rt ra c mt s kinh nghim v mt s iu ng lu khi
gii phng trnh m - logarit:
Lu n iu kin xc nh ca bi ton a ra. Cc iu kin ny c th gip ta chn
c min nghim trn phng trnh h qu, gip ta thu c nghim chnh xc hn. Cn nu
nh chng ta khng xc nh iu kin bi ton, th sau khi gii quyt xong, chng ta cn mt
bc khng th b qua l th li nghim vo bi ton.
c c cc bc bin i mu mo trong cc bi ton phng trnh nh trn, i hi
chng ta phi c kinh nghim lm bi, luyn tp vi nhiu dng bi tp khc nhau. V gip
cho cc bn luyn tp nhiu hn vi phng php ny, chng ti xin ngh vi cc bn th
luyn tp vi mt s bi tp t luyn sau y:
Bi tp t luyn:
1/ 4
3+2 cos x
7.4
1+cos x
2 = 0.
2/ log
2
2
x.log
x
(4x
2
) = 12.
3/
log
2
x
log
4
2x
=
log
8
4x
log
16
8x
.
4/ (2 log
3
x) log
9x
3
4
1 log
3
x
= 1.
5/ 2x
log
2
x
+ 2x
3log
8
x
5 = 0.
6/

7 + 4

cos x
+

7 4

cos x
=
5
2
.
7/ log
2
(9 2
x
) = 3 x.
8/ 7
log
2
25
(5x)1
x
log
5
7
= 0.
166
PHNG PHP DNG N IU HM S
Bi 1. Gii phng trnh 2
2x1
+ 3
2x
= 5
2x+1
= 2
x
+ 3
x+1
+ 5
x+2
Gii
Nhn vo bi ton ny, chng ta s thy ngay rng phng php t n ph c v nh khng
c tc dng. Chng ta s c nhng nhn xt sau y:
Nu x > 1 th 2
2x1
+ 3
2x
= 5
2x+1
> 2
x
+ 3
x+1
+ 5
x+2
Nu x < 1 th 2
2x1
+ 3
2x
= 5
2x+1
< 2
x
+ 3
x+1
+ 5
x+2
Nu x = 1 th 2
2x1
+ 3
2x
= 5
2x+1
= 2
x
+ 3
x+1
+ 5
x+2
(= 136).
Vy phng trnh c 1 nghim duy nht x = 1 2
Bi 2. Gii phng trnh 3
2x
+ 4
2x
= 2.12
x
Gii
p dng BT AM-GM, ta c: 3
2x
+ 4
2x
2.12
x
. Du ng thc xy ra, do : 3
x
= 4
x
.
Nu x > 0: 3
x
< 4
x
.
Nu x < 0: 3
x
> 4
x
.
Nu x = 0: 3
x
= 4
x
= 1.
Vy phng trnh cho c nghim duy nht x = 0 2
Bi 3. Gii phng trnh x + log(x
2
x 6) = 4 + log(x + 2)
Gii
Phng trnh tr thnh:
x + log(x + 2) + log(x 3) = 1 + log(x + 2)

x + 2 > 0
log(x 3) = 4 x

x > 3
log(x 3) = 4 x
Nu x > 4 th log(x 3) > log 1 = 0 > 4 x.
Nu 3 < x < 4 th log(x 3) < log 1 = 0 < 4 x.
Nu x = 4 th log(x 3) = log 1 = 0 = 4 x.
Vy phng trnh c nghim duy nht x = 4 2
Bi 4. Gii phng trnh (1 + 2
x
)(1 + 3
x
)(1 + 36
x
) = (1 + 6
x
)
3
Gii
Ta c BT sau:
(1 + x)(1 + y)(1 + z) (1 +
3

xyz)
3
, x, y, z > 0
p dng vo bi ton, ta c:
(1 + 2
x
)(1 + 3
x
)(1 + 36
x
) (1 +
3

2
x
3
x
36
x
) = (1 + 6
x
)
3
167
Du ng thc xy ra, do 2
x
= 3
x
= 36
x
x = 0.
Vy phng trnh c nghim duy nht x = 0 2
Bi 5. Gii phng trnh x = 2
log
5
(x+3)
Gii
KX: x + 3 > 0 x > 3. t t = log
5
(x + 3) x = 5
t
3. Ta c:
5
t
3 = 2
t

5
3

t
3

1
2

t
= 1
Nu t > 1 th

5
3

t
3

1
2

t
>
5
2

3
2
= 1.
Nu t < 1 th

5
3

t
3

1
2

t
<
5
2

3
2
= 1.
Nu t = 1 th

5
3

t
3

1
2

t
= 1.
Do t = 1, hay x = 2 (nhn).
Vy phng trnh cho c mt nghim duy nht x = 2 2
Nhn xt: Vi cc bi ton trn, c th thy c nhng bc bin i rt kinh in ca
phng php nh gi BT. Cn nhng bi ton sau y, cc bn cn phi c s kt hp
kho lo gia cc BT v vic xt tnh n iu ca hm s.
Bi 6. Gii phng trnh x.2
x
= x(3 x) + 2(2
x
1)
Gii
Phng trnh cho tng ng vi:
(x 2)(2
x
+x 1) = 0

x = 2 (nhn)
2
x
+ x 1 = 0
Xt phng trnh 2
x
+ x 1 = 0. t f(x) = 2
x
+x 1.
Ta c f

(x) = ln 2.2
x
+ 1 > 0 x R.
Nhn thy x = 0 tha phng trnh ny, do , phng trnh cho c 2 nghim l x = 2 v
x = 0 2
Qua bi ton ny, ta rt ra c mt iu: i vi vic gii phng trnh 2
x
+ x 1 = 0
nh trn, ta c th suy ra c iu kin l x > 1. Th nhng iu ny l khng nht thit
cn thit, v trong mt s trng hp, vic gii iu kin h qu l kh rc ri. Mt cch
n gin hn ta c th lm l tm ra cc nghim ca phng trnh cho, sau thay vo
li phng trnh ban u nhn nhng nghim tha mn. Nhng li trong nhiu trng hp
khc, nhng iu kin h qu li t ra rt hu hiu cho vic gii phng trnh nh chn min
nghim, s dng BT, xt du, tnh n iu, ...v.v...
Bi 7. Gii phng trnh 2x
2
6x + 2 = log
2
2x + 1
(x 1)
2
Gii
168
iu kin:

x 1 = 0
x >
1
2
.
Phng trnh c vit li thnh:
log
2

x +
1
2

+ 2

x +
1
2

= log
2
(x 1)
2
+ 2(x 1)
2
()
t f(x) = log
2
x + 2x vi x > 0. Ta c: f

(x) = 2 +
1
x. ln 2
> 0 x > 0.
Do , hm f ng bin trn (0; +).
Li t (), ta c f

x +
1
2

= f ((x 1)
2
), suy ra:
x +
1
2
= (x 1)
2

x =
3 +

7
2
x =
3

7
2
Vy phng trnh cho c tp nghim S =

3 +

7
2
;
3

7
2

2
Bi 8. Gii phng trnh (1 + x)(2 + 4
x
) = 3.4
x
Gii
D c x > 1. Xt hm s f(x) = (1 +x)(2 + 4
x
) 3.4
x
vi x (1; +).
Ta c: f

(x) = (x2)4
x
ln 4+4
x
+2, f

(x) = (x4)(ln 4)
2
+2.4
x
ln 4, f

(x) = (x2)4
x
(ln 4)
3
+
3.4
x
(ln 4)
2
.
f

(x) = 0 x = 2
3
ln 4
. t x
0
= 2
3
ln 4
.
Ta thy hm f

(x) nghch bin trn (1; x


0
), ng bin trn (x
0
; +) v f

(1) < 0, f

(x
0
) <
0, lim
x+
f

(x) = +, nn phng trnh f

(x) = 0 c ng 1 nghim. Do , phng trnh


f

(x) = 0 c ti a 2 nghim, suy ra phng trnh f(x) = 0 c khng qu 3 nghim.


Ta nhn thy c ba gi tr x tha mn phng trnh cho l x = 0, x =
1
2
, x = 1.
Vy phng trnh cho c tp nghim S =

0;
1
2
; 1

2
Bi 9. Gii phng trnh 12
x
+ 13
x
+ 14
x
= 2
x
+ 3
x
+ 4
x
+ 870
x
2400x
2
+ 1560x
Gii
Xt hm s f(x) = 12
x
+ 13
x
+ 14
x
(2
x
+ 3
x
+ 4
x
+ 870
x
2400x
2
+ 1560x).
Ta s chng minh o hm cp 4 ca hm s ny lun m hoc lun dng vi mi gi tr x
c th nhn c. Ta c:
f
(4)
(x) = 12
x
(ln 12)
4
+ 13
x
(ln 13)
4
+ 14
x
(ln 14)
4
2
x
(ln 2)
4
3
x
(ln 3)
4
4
x
(ln 4)
4
Ta xt cc trng hp sau:
Nu x < 0, p dng BT Bernoulli cho s x < 0, ta c 12
x
> x(12 1) +1 = 11x+1. Tng
t, ta cng c 13
x
> 12x + 1, 14
x
> 13x + 1. Suy ra:
12
x
+ 13
x
+ 14
x
> 36x + 3
169
Li c
2
x
+ 3
x
+ 4
x
+ 870
x
2400x
2
+ 1560x < 3 + 36x +x(870x
2
2400x + 1524) < 36x + 3
Do
12
x
+ 13
x
+ 14
x
> 2
x
+ 3
x
+ 4
x
+ 870
x
2400x
2
+ 1560x
iu ny mu thun vi bi cho.
Nu x 0, suy ra f
(4)
0, do phng trnh cho khng c qu 4 nghim.
Nhn thy cc gi tr x = 0, x = 1, x = 2, x = 3 tha mn phng trnh cho.
Vy phng trnh c tp nghim S = 0; 1; 2; 3 2
Bi 10. Gii phng trnh 9
x
(3
x
+ 2
x
) = 2
x
(8
x
+ 7
x
) + 5
x
(5
x
2
x
)
Gii
Ta c nhn xt rng, vi bi ton trn, ta khng thy cc c s c mi lin quan no n nhau,
cc hm s cha bin th ton l hm s m. Ta th bin i v dng phng trnh c li
f(a) = f(b). Ta c:
9
x
(3
x
+ 2
x
) = 2
x
(8
x
+ 7
x
) + 5
x
(5
x
2
x
)
10
x
+ 12
x
16
x
25
x
= 12
x
+ 14
x
18
x
27
x
Ta xt hm s f(t) = t
x
+ (t + 2)
x
(t + 6)
x
(t + 15)
x
vi t > 0. Phng trnh cho tng
ng vi
f(10) = f(12)
Ta thy rng, hm f lin tc v kh o hm trn [10; 12], nn theo nh l Largrange, tn ti
c (10; 12) sao cho f

(c) =
f(10) f(12)
10 12
= 0. Do , ta c:
x

c
x1
+ (c + 2)
x1
(c + 6)
x1
(c + 15)
x1

= 0

x = 0
c
x1
+ (c + 2)
x1
= (c + 6)
x1
+ (c + 15)
x1
Xt phng trnh c
x1
+ (c + 2)
x1
= (c + 6)
x1
+ (c + 15)
x1
. Ta c:
Nu x > 1 th x 1 > 0, suy ra c
x1
+ (c + 2)
x1
< (c + 6)
x1
+ (c + 15)
x1
.
Nu x < 1 th x 1 < 0, suy ra c
x1
+ (c + 2)
x1
> (c + 6)
x1
+ (c + 15)
x1
.
Nu x = 1 th c
x1
+ (c + 2)
x1
= (c + 6)
x1
+ (c + 15)
x1
= 2.
Th li, ta c phng trnh ny c tp nghim l S = 0; 1 2
Qua cc bi tp v d nu trn, cc bn s thy rng, ngi ra s thng hay bt u to
ra bi ton t mt hng ng thc, t mt phng trnh n gin, hay t trng hp xy ra
du bng ca mt bt ng no . H cng c cng ct giu biu thc ban u bao nhiu,
th mc kh ca bi ton cng theo m kh ln by nhiu. V cng vic ca chng ta l
tm ra con ng m ngi ra s dng. C th c bi ton, chng ta i theo mt hng
khc tc gi, nhng i vi mt s bi ton, dng nh l con ng tc gi xut pht cng l
con ng duy nht m ta c th s dng gii quyt c bi ton. nhun nhuyn hn,
cc bn hy luyn tp vi cc bi ton sau y:
Bi tp t luyn
170
Gii cc phng trnh sau:
1/ 2

3x
= x
2
+ 8x 14.
2/ 2
2x+1
+ 2
32x
=
8
log
3
(4x
2
4x + 4)
.
3/ (x + 3) log
2
3
(x + 2) + 4 (x + 2) log
3
(x + 2) = 16.
4/ e
x
+ (x
3
x) ln(x
2
+ 1) = e
3

x
.
PHNG PHP BIN I NG THC
Bn cht ca phng php bin i ng thc l c gng chuyn i phng trnh cho tr
thnh mt phng trnh tng ng, mt phng trnh h qu, hay mt h phng trnh, h
bt phng trnh m ta c th d dng gii quyt. Thng thng, ta s bin i bi v dng
hai v c cng c s hoc tr thnh mt hng ng thc quen thuc. Chng ta s lm r hn
nhng iu trn bng cch xt qua cc v d sau y.
Bi 1. Gii phng trnh 2
x
2
x+8
= 4
13x
Gii
tng: Ta c mt nhn xt kh th v: 4 l ly tha bc hai ca 2. Do , ta s bin i
c hai v v cng c s 2.
Li gii:
Phng trnh cho tng ng vi:
2
x
2
x+8
= 2
26x
x
2
x + 8 = 2 6x

x = 2
x = 3
Vy phng trnh cho c tp nghim l S = 2; 3 2
Bi 2. Gii phng trnh (x
2
x + 1)
x
2
1
= 1
Gii
Phng trnh tng ng vi:

x
2
x + 1 > 0
(x
2
x + 1 1)(x
2
1) = 0
(x
2
x)(x
2
1) = 0

x = 0
x = 1
Vy tp nghim ca phng trnh cho l S = 1; 0; 1 2
Nhn xt: i vi phng trnh c dng f(x)
g(x)
= 1, u tin, ta c iu kin l f(x) > 0.
Tip n, biu thc f(x)
g(x)
bng 1, th hoc f(x) = 1, hoc g(x) = 0, do , ta suy ra phng
trnh [f(x) 1]g(x) = 0. Vy, phng trnh cho s tng ng vi

f(x) > 0
[f(x) 1]g(x) = 0
.
Bi 3. Gii h phng trnh sau:

2
3x
= 5y
2
4y
4
x
+ 2
x+1
2
x
+ 2
= y
171
Gii
H tng ng vi:

2
3x
= 5y
2
4y
2
x
= y

2
x
= y
y
3
5y
2
+ 4y = 0

x = 0
y = 1

x = 2
y = 4
Vy h cho c tp nghim (x; y) = (0; 1); (2; 4)2
Bi 4. Gii phng trnh 2
x
+ 2
x1
+ 2
x2
= 3
x
3
x1
+ 3
x2
Gii
tng: Ta c ngay nhn xt l biu thc ly tha cha x ging nhau c hai v ca
phng trnh. Do , ta s c tng l rt ra phn t chung ca mi hai v.
Li gii:
Phng trnh cho tr thnh:
2
x

1 +
1
2
+
1
4

= 3
x

1 +
1
9

1
3

2
x
.
7
4
= 3
x
.
7
9

2
3

x
=

2
3

2
x = 2
Vy phng trnh cho c nghim duy nht x = 2 2
Bi 5. Gii phng trnh 5
x
.8
x1
x
= 500
Gii
iu kin xc nh: x = 0.
Phng trnh tng ng vi:
5
x
.2
3(x1)
x
= 5
3
.2
2
5
x3
= 2
3x
x
5
x3
= (2

1
x
)
x3
5
x3
= (
1
2
1
x
)
x3
(5.2
1
x
)
x3
= 1

x 3 = 0
5.2
1
x
= 1

x = 3
x = log
5
2
Vy phng trnh cho c tp nghim l S = 3; log
5
2 2
Bi 6. Gii h phng trnh sau:

log1
4
(y x) log
4
1
y
= 1
x
2
+ y
2
= 25 ()
Gii
iu kin xc nh: y > x; y > 0. Ta c bin i sau:
log1
4
(y x) log
4
1
y
= 1 log
4
(y x) log
4
1
y
= 1
172
log
4
y x
y
= 1 x =
3y
4
Th vo phng trnh (*) ta c:

3y
4

2
+y
2
= 25 y = 4
T iu kin bi, ta c y = 4, suy ra x = 3. Vy h phng trnh c nghim duy nht
(x; y) = (3; 4) 2
Bi 7. Gii phng trnh

2
x
.
3

4
x
.
3x

0.125 = 4
3

2
Gii
tng: Chng ta cn ch rng, nu trong bi ton c cn thc dng x

th cn phi c
iu kin ca x l

x 1
x N
.
Li gii:
KX:

x
1
3
3x N
. Nhn thy rng cc c s ca cc ly tha u l ly tha ca 2, do ta
s bin i bi ton v dng cc ly tha ca 2.
Phng trnh cho tng ng vi:

2
x
.2
2x
3
.2

1
2x
= 2
2
.2
1
3
2
x
2
+
x
3

1
2x
= 2
7
3

x
2
+
x
3

1
2x
=
7
3

x = 3
x =
1
5
Kt hp vi KX, ta c phng trnh ny c mt nghim duy nht x = 3 2
Bi 8. Gii phng trnh
1
3
log 3

3
(x + 1) +
1
503
log
81
(x 3)
2012
= 5 log
243
[4(x 2)]
Gii
tng: Nhn vo bi, ta thy ngay rng c s ca cc hm logarit u l ly tha ca
3. Do , ta s bin i chng v dng cc hm s logarit c s 3.
Li gii:
KX: x > 2, x = 3.
Phng trnh tng ng vi
log
3
(x + 1) + log
3
[x 3[ = log
3
(4x 8) (x + 1)[x 3[ = (4x 8)
Nu x > 3, ta c:
x
2
2x 3 = 4x 8

x = 1 (loi)
x = 5
Nu 2 < x < 3, ta c:
x
2
+ 2x + 3 = 4x 8

x = 1 +

12
x = 1

12 (loi)
173
Vy phng trnh cho c tp nghim S =

5; 1 +

12

2
V cho kinh nghim lm bi c cng c thm, cc bn hy th t gii cc bi ton t
luyn sau:
Bi tp t luyn.
Gii cc phng trnh:
1/ 2
x
.3
x1
.5
x2
= 12.
2/ (x
2
2x + 2)

4x
2
= 1.
3/ 2
x
2
+x
4.2
x
2
x
2
2x
+ 4 = 0
BI TP TNG HP
Sau y, chng ta s cng nhau th sc mnh vi nhng bi ton kh hn v hay hn. Cc bi
ton ny i hi s vn dng linh hot cc k nng bin i, t n ph, cng nh xt n
tnh n iu ca cc hm s ph cn dng n.
Bi 1. 3x
2
+ 1 + log
2006
4x
2
+ 2
x
6
+x
2
+ 1
= x
6
Gii
Vit li phng trnh:
log
2006
4x
2
+ 2
x
6
+x
2
+ 1
= x
6
3x
2
1
Lu rng (x
6
+x
2
+ 1) (4x
2
+ 2) = (x
6
3x
2
1).
t a = 4x
2
+ 2 2, b = x
6
+x
2
+ 1 1, phng trnh cho tr thnh:
log
2006
a +a = log
2006
b +b ()
Xt hm s f(x) = log
2006
x +x vi x > 0, ta c: f

(x) =
1
x ln 2006
+1 > 0 x > 0, li do hm
f lin tc trn (0; +), nn t (*) suy ra a = b. Ta cn phi gii phng trnh:
x
6
3x
2
1 = 0
t u = x
2
0. Phng trnh c vit li: u
3
3u 1 = 0.
t u = 2v v 0, ta c 4v
3
3v =
1
2
.
Xt g(v) = 4v
3
3v lin tc trn R. Ta c g

(v) = 0 v =
1
2
.
Ta c g(0) = 0, g

1
2

= 1, g(1) = 1, do , phng trnh ch c mt nghim duy nht


v
0
> 0, vi v
0
< 1. t v = cos t vi 0 < t <

2
, phng trnh tr thnh:
cos 3t =
1
2
v = cos

9
u = 2 cos

9
x =

2 cos

9
Vy phng trnh cho c tp nghim l S =

2 cos

9
;

2 cos

9

2
Bi 2. sin
2
x.2
cos 2x
+
1
2
sin
2
2x + cos 2x = 1
174
Gii
Phng trnh cho tng ng vi:
1 cos 2x
2
.2
cos 2x
+
1
2
(1 cos
2
2x) + cos 2x = 1
t t = cos 2x, ta c:
(1 t)2
t
+ 1 t
2
+ 2t = 2 (1 t)(2
t
+ t 1) = 0

t = 1
2
t
+t 1 = 0
Vi t = 1, suy ra x = k vi k Z.
Xt phng trnh f(t) = 2
t
+ t 1 = 0.
Ta c hm f l hm s tng trn R nn f(t) = 0 c nhiu nht mt nghim, li do f(0) = 0,
nn suy ra t = 0 l nghim ca phng trnh f(t) = 0, suy ra x =

4
+
k
2
vi k Z.
Vy phng trnh cho c tp nghim S = k[k Z

4
+
k
2
[k Z

2
Bi 3. Gii h phng trnh:

7
1+xy
+ 3
1+xy
+ 7
1x+y
= 10
1+xy
+ 10
1x+y
(1)
4x
2
+

2x + 3 = 8y + 1 (2)
(x, y R)
Gii
iu kin: x
3
2
, y >
1
8
.
Ta c b sau: Vi mi a > b > 1 a
x
+a
x
b
x
+b
x
, x R (3), du ng thc xy ra
khi x = 0. Tht vy, bt ng thc (3) tng ng vi:
(a
x
b
x
)

1
1
a
x
b
x

0 (4)
Nu x 0

a
x
b
x
(ab)
x
1
(a
x
b
x
)

1
1
a
x
b
x

0. Du ng thc xy ra khi x = 0.
Nu x 0

a
x
b
x
(ab)
x
1
a
x
b
x
)

1
1
a
x
b
x

0. Du ng tc xy ra khi x = 0.
p dng b trn vo bi ton, ta c:

10
t
+ 10
t
7
t
+ 7
t
10
t
+ 10
t
3
t
+ 3
t

7(10
t
+ 10
t
) 7
1+t
+ 7
1t
3(10
t
+ 10
t
) 3
1+t
+ 3
1t
(t = x y)
10
1+t
+ 10
1t
7
1+t
+ 7
1t
+ 3
1+t
+ 3
1t
10
1+xy
+ 10
1x+y
7
1+xy
+ 7
1x+y
+ 3
1+xy
+ 3
1x+y
(5)
T (1) v (5), ta c ng thc xy ra, do t = x y = 0 x = y. Thay vo phng trnh
(2) ta c:
4x
2
+

2x + 3 = 8x + 1
4x
2
6x +
9
4
= 2x + 3

2x + 3 +
1
4
175

2x
3
2

2
=

2x + 3
1
2

2x 1 =

2x + 3
2x 2 =

2x + 3

x =
3 +

17
4
x =
5

21
4
Vy HPT cho c hai nghim (x; y) =

3 +

17
4
;
3 +

17
4

; (x; y) =

21
4
;
5

21
4

2
Bi 4. Tm s nghim ca h phng trnh:

2x
2
6x.4
y
+ 4x = 3.4
y+1
(1)
1
(x + 3)
2
[y 2[ = 0 (2)
( ngh OLP 30-04 THPT chuyn Lng Th Vinh, ng Nai)
Gii
Trc tin, ta hy tm cc nghim p (nu c) ca h.
iu kin xc nh: x = 3. t t = 4
y
> 0, phng trnh (1) tr thnh:
2x
2
6tx + 4x 12t = 0 2(x + 2)(x 3t) = 0

x = 2
x = 3t
Khi x = 2, th vo phng trnh (2), ta c
1 [y 2[ = 0

y = 3
y = 1
T suy ra hai nghim ca h l (2; 3) v (2; 1).
V ta c c hai nghim p. Tip theo, ta s i tip theo con ng cn li tm s
nghim.
Vi x = 3t, th vo phng trnh (2), ta c:
1
(3t + 3)
3
[y 2[ = 0
Ta c t = 4
y
y = log
4
t.
Xt hm s f(t) =
1
(3t + 3)
3
[ log
4
t 2[ vi t > 0.
Ta s tm cch gii quyt du gi tr tuyt i ny, bng cch xt cc khong c th ca t:
Vi t 16, khi f(t) =
1
(3t + 3)
3
log
4
t + 2.
Ta c f

(t) =
9
(3t + 3)
4

1
t ln 4
< 0, t 16.
Do hm f nghch bin trn [16; +).
Mt khc f(16) > 0 v f(64) < 0, suy ra tn ti duy nht s t
1
(16; 64) l nghim ca
phng trnh f(t) = 0.
Do , nghim (3t
1
; log
4
t
1
) l nghim th ba ca h.
176
Vi 0 < t < 16, khi f(t) =
1
(3t + 3)
3
+ log
4
t 2.
Ta c f

(t) =
1
t ln 4

9
(3t + 3)
4
=
(3t + 3)
4
9t ln 4
(3t + 3)
4
.t. ln 4
.
t g(t) = (3t + 3)
4
9t ln 4 vi t [0; 16).
Ta c: g

(t) = 12(3t + 3)
3
9 ln 4 v g

(t) = 12.3
2
.(3t + 3)
2
> 0.
Do g

(t) ng bin trn [0; 16), hay vi mi t thuc (0; 16), ta c g

(t) > g

(0). M
g

(0) = 12.3
3
9 ln 4 > 0 suy ra g

(t) > 0, t (0; 16) .


T y, suy ra hm g(t) ng bin trn [0; 16), nn g(t) > g(0) = 81 > 0. Suy ra
f

(t) > 0, t (0; 16), hay hm f(t) ng bin trn (0; 16).
V f(1) < 0, f(16) > 0 nn phng trnh c nghim duy nht t
2
thuc khong (1; 16).
Do nghim (3t
2
; log
4
t
2
) l nghim th t ca h.
Vy h phng trnh cho c bn nghim 2
Bi tp t luyn
1/

4
x
y
+
y
x
= 32
log
3
(x y) = 1 log
3
(x + y)
. (HV CNBCVT 1999)
2/

x y = (log
2
y log
2
x)(2 + xy)
x
3
+y
3
= 16
. (H Ngoi thng 1999)
3/

4
x
2
16
+ 3

x +

x
2
+ 1 = 4
y
2
8y
+ 3

y 4 +

y
2
8y + 17
y(x
2
1) 4x
2
+ 3x 8 + ln(x
2
3x + 3) = 0
( ngh OLP 30-04 THPT chuyn Nguyn Bnh Khim, Qung Nam)
4/ log
2

x
2
3x + 2 + 1

1
3

x
2
+3x1
= 2.
( ngh OLP 30-04 THPT chuyn L Qu n, Khnh Ha)
5/ Xc nh s nghim ca h phng trnh (n x, y) sau:

x
2
+y
3
= 29
log
3
x. log
2
y = 1
CnucNo V: H PHNO TRNH
CC LOI H C BN
L thuyt
H phng trnh i xng loi I:
H phng trnh i xng loi I l h c dng

F(x; y) = 0
G(x; y) = 0
Trong F(x; y), G(x; y) l cc a thc i xng vi x, y.
Cch gii chung ca h l t S = x +y; P = xy (iu kin S
2
4P 0). Dng tnh i xng
ta a h v dng

F
1
(S; P) = 0
G
1
(S; P) = 0
Tm c S v P. T , theo nh l Viete o, x v y l nghim ca phng trnh
t
2
St + P = 0
H phng trnh i xng loi II:
H phng trnh i xng loi II l h c dng

F(x; y) = 0
F(y; x) = 0
Trong F(x; y) l mt a thc khng i xng.
Tr hai phng trnh v theo v c F(x; y) F(y; x) = 0 ()
Coi x l n s, y l tham s v t F(x; y) = f(x) + g(y) (g(y) c lp vi x)
th F(y; x) = f(y) + g(x). Khi
() G(x) = f(x) + g(y) f(y) g(x) = 0
Xt G(x), thay x = y th G(y) = f(y) + g(y) f(y) g(y) = 0
y l nghim ca phng trnh G(x) = 0
G(x) c cha nhn t (x y) theo nh l Bezout.
Nh vy ta c cch gii h i xng loi II l: Tr hai phng trnh v theo v c
G(x; y) = (x y).M(x; y) = 0
177
178
Sau gii h trong tng trng hp x = y v M(x; y) = 0.
Lu rng h i xng loi II cn c dng

f(x) = g(y)
f(y) = g(x)
v nu f(x), g(x) l hai hm s
n iu cng chiu th ta c th suy ra x = y m khng cn xt M(x; y) = 0 (v trng hp
ny s dn ti h v nghim). Nhng nu f(x), g(x) khng n iu cng chiu th trng hp
M(x; y) = 0 s cho nghim.
H phng trnh bc hai 2 n:
H phng trnh bc hai 2 n c dng tng qut

a
1
x
2
+b
1
xy +c
1
y
2
+ d
1
x + e
1
y +f
1
= 0
a
2
x
2
+b
2
xy +c
2
y
2
+ d
2
x + e
2
y +f
2
= 0
Trong a
i
, b
i
, c
i
, d
i
, e
i
, f
i
(i = 1, 2) l cc tham s v x, y l n s.
Ta cng xt mt s trng hp c bit ca h:
a) H cha mt phng trnh bc nht:
Ta tnh x theo y hoc ngc li t mt phng trnh, thay vo phng trnh cn li c mt
phng trnh bc 2 mt n.
b) H cha mt phng trnh thun nht bc 2:
Nu mt trong 2 phng trnh ca h khng cha hng t bc nht v s hng t do, chng
hn nu d
1
= e
1
= f
1
= 0 th h c th a v phng trnh bc hai bng cch t y = tx ri
thay vo phng trnh thun nht tm c t, t thay vo phng trnh th 2 tm
x, y tng ng.
Phng php ny cng c th p dng gii h gm 2 phng trnh bn ng cp bc hai:
()

a
1
x
2
+b
1
xy + c
1
y
2
+f
1
= 0
a
2
x
2
+b
2
xy + c
2
y
2
+f
2
= 0
V r rng t h ta c th to mt phng trnh thun nht bc 2 l
f
2
(a
1
x
2
+ b
1
xy + c
1
y
2
) f
1
(a
2
x
2
+b
2
xy +c
2
y
2
) = 0
Trong trng hp tng qut, php gii h bc 2 hai n s dn n gii mt phng trnh
bc cao ( 4). Nhng vi mt s h phng trnh, ta c th a v h (*) bng cch t
x = u +a; y = v +b trong u, v l n mi (phng php tnh tin nghim). Ta cn tm hng
s a, b hng t bc nht hai phng trnh b trit tiu, nh vy h thu c l h ng cp.
V d: Gii h phng trnh (I)

x
2
+ 3y
2
+ 4xy 18x 22y + 31 = 0 (1)
2x
2
+ 4y
2
+ 2xy + 6x 46y + 175 = 0 (2)
Gii
tng: t x = u +a; y = v + b ta c h phng trnh

u
2
+ 4uv + 3v
2
+u(2a + 4b 18) + v(6b + 4a 22) + 4ab +a
2
+ 3b
2
18a 22b + 31 = 0
2u
2
+ 4v
2
+ 2uv +u(4a + 2b + 6) + v(2a + 8b 46) + 2a
2
+ 4b
2
+ 2ab + 6a 16b + 175 = 0
179
h s ca u, v l 0 ta gii h phng trnh

2a + 4b 18 = 0
6b + 4a 22 = 0
4a + 2b + 6 = 0
2a + 8b 46 = 0

a = 5
b = 7
Li gii:
t x = u 5; y = v + 7 ta c h phng trnh
(II)

u
2
+ 3v
2
+ 4uv = 1 (3)
2u
2
+ 4v
2
+ 2uv = 1 (4)
H ny c th gii theo cch thng thng, nhng lu l tr 2 phng trnh v theo v ta c
ngay
u
2
+v
2
2uv = 0 u = v
Ta c h (II)

8u
2
= 1
u = v

u = v =
1
2

2
u = v =
1
2

2
Vy (I) c nghim (x; y) =

1
2

2
5;
1
2

2
+ 7

1
2

2
5;
1
2

2
+ 7

2
Vi nhng h c cha tham s, cch gii hon ton tng t:
V d: Tm m h c nghim:

x
2
+ 2xy = m (1)
x
2
+xy +y
2
= 1 (2)
Gii
Nu y = 0 ta c h

x
2
= m
x
2
= 1
ch c nghim khi m = 1.
Nu m = 1 th y = 0, t x = ty ta c h (I)

y
2
(t
2
+ 2t) = m (1)
y
2
(t
2
+t + 1) = 1 (2)
Chia (1) cho (2) v quy ng ta c t
2
+ 2t = m(t
2
+t + 1) (3).
H cho c nghim (x; y) (I) c nghim (t; y) (3) c nghim t (do t
2
+t +1 > 0 nn t
y lun tm c y tho (2)).
Vit li (3) di dng
(m1)t
2
+ (m2)t + m = 0
Ta c = 4 3m
2
nn (3) c nghim 0 [m[
2

3
Do m = 1 cng tho m
2

3
nn ta kt lun h c nghim [m[
2

3
2
180
H phng trnh 3 n bnh ng
H phng trnh 3 n bnh ng l h c cc phng trnh u bnh ng vi 3 n, ngha
l khi hon v 2 n tu th mi phng trnh u khng i.
Phng php c bn gii h l a v h phng trnh
()

x + y +z = a (1)
xy +yz + zx = b (2)
xyz = c (3)
Bng cch dng php th hoc nh l Viete o, ta a () v phng trnh mt n
x
3
ax
2
+bx c = 0
Gii phng trnh trn, tm nghim x
0
th vo (1) v (3) ta c

y + z = a x
0
yz =
c
x
0
Nh vy y, z l nghim ca phung trnh t
2
(a x
0
)t +
c
x
0
= 0.
V d: Gii h phng trnh

x +y +z = 1
x
2
+ y
2
+ z
2
= 9
x
3
+ y
3
+ z
3
= 1
Gii
T h phng trnh ta c

xy + yz +zx =
(x +y + z)
2
(x
2
+y
2
+z
2
)
2
3xyz = (x
3
+y
3
+z
3
) (x + y +z)(x
2
+y
2
+z
2
xy yz zx) = 12
Suy ra

xy + yz + zx = 4
xyz = 4
. Vy ta a h v dng

x +y + z = 1 (1)
xy + yz +zx = 4 (2)
xyz = 4 (3)
Theo nh l Viete o, x, y, z l nghim ca phng trnh
t
3
t
2
4t + 4 = 0

t = 1
t = 2
t = 2
T ta tm c nghim ca h l (x; y; z) = (1; 2; 2) v cc hon v.2
Nhn xt: nh l Viete o khng nm trong chng trnh ph thng, nn ta c th chng
181
minh li bng php th nh sau:
T (3) x, y, z = 0
Nhn hai v ca (2) cho x ta c
x
2
y +xyz +x
2
z = 4x x
2
(y + z) = 4x xyz = 4 4x
Nhn hai v ca (1) cho x
2
ta c
x
3
+ x
2
(y +z) = x
2
x
3
+ 4 4x = x
2
x 1; 2; 2
Sau y ta cng xem qua mt s bi tp tng hp:
Bi tp tng hp
Bi 1: Gii h phng trnh sau

x
2
y
2
2x + 2y = 3
y
2
2xy + 2x = 4
Gii
t x = u + 1 v y = v + 1.H phng trnh tng ng:

(u + 1)
2
(v + 1)
2
2(u + 1) + 2(v + 1) = 3
(v + 1)
2
2(u + 1)(v + 1) + 2(u + 1) = 4

u
2
v
2
= 3
v
2
2uv = 5

5u
2
5v
2
= 15
3v
2
6uv = 15

u
2
v
2
= 3
5u
2
+ 6uv 8v
2
= 0

u
2
v
2
= 3

u =
4
5
v
u = 2v
Vi u =
4
5
v. Ta c:
16
25
v
2
v
2
= 3 v
2
=
25
3

v =
5

3
; u =
4

3
v =
5

3
; u =
4

3
Vi u = 2v ta c 4v
2
v
2
= 3 v
2
= 1 (v nghim)
Kt lun: H c nghim (x; y) =

3
+ 1;
5

3
+ 1

3
+ 1;
5

3
+ 1

2
Nhn xt: Ngoi phng php tnh tin nghim, vi bi ny ta c th bin i ng thc
tm ra cch t n ph trn:
Xt h phng trnh tng ng:

(x 1)
2
(y 1)
2
= 3
(y
2
2y + 1) 2(xy y x + 1) = 5

(x 1)
2
(y 1)
2
= 3
(y 1)
2
2(x 1)(y 1) = 5
T ta t x = u + 1 v y = v + 1
Bi 2: Gii h phng trnh sau

x
2
2xy + 2y + 15 = 0
2x 2xy + y
2
+ 5 = 0
182
Gii
t x = u + 1 v y = v + 1. H phng trnh tng ng:

(u + 1)
2
2(u + 1)(v + 1) + 2(v + 1) + 15 = 0
2(u + 1) 2(u + 1)(v + 1) + (v + 1)
2
+ 5 = 0

u
2
2uv = 16
v
2
2uv = 6

3u
2
6uv = 48
8v
2
16uv = 48

3u
2
+ 10uv 8v
2
= 0
v
2
2uv = 6

u =
2
3
v
u = 4v
v
2
2uv = 6
Vi u =
2
3
v ta c:
v
2

4
3
v
2
= 6 v
2
= 18

v = 3

2
v = 3

2
Vi v = 3

2 u = 2

2
Vi v = 3

2 u = 2

2
Vi u = 4v.Ta c: v
2
+ 8v
2
= 6 9v
2
= 6 (v nghim)
Kt lun: H c nghim (x; y) l

2 + 1; 3

2 + 1

2 + 1; 3

2 + 1

2
Bi 3: Gii h phng trnh sau:

x
2
+y
2
= 8 x y
xy(xy +x +y + 1) = 12
Gii
tng: Nhn thy vai tr ca x v y l nh nhau nn ta s c gng phn tch ri t n
ph a h v dng i xng loi 1.
Li gii:
H phng trnh tng ng:

x
2
+x +y
2
+y = 8
xy(x + 1)(y + 1) = 12
(I)

(x
2
+x) + (y
2
+ y) = 8
(x
2
+x)(y
2
+ y) = 12
t x
2
+x = a v y
2
+ y = b. H (I) tr thnh

a + b = 8
ab = 12
Nh vy a, b l nghim ca phng trnh bc hai
X
2
8X + 12 = 0

X = 2
X = 6
Vi a = 2 ta c:
x
2
+x 2 = 0

x = 1
x = 2
Vi a = 6 ta c:
x
2
+x 6 = 0

x = 2
x = 3
183
Tng t gii vi b = 2 v b = 6
Kt lun: H c nghim (x; y) l (1; 2); (1; 3); (2; 2); (2; 3) v cc hon v. 2
Bi 4: Gii h phng trnh ()

x
3
+ 1 = 2(x
2
x +y)
y
3
+ 1 = 2(y
2
y +x)
Gii
Ta c ()

x
3
2x
2
+ 2x + 1 = 2y
y
3
2y
2
+ 2y + 1 = 2x
Nhn thy f(t) = t
3
2t
2
+ 2t + 1 c f

(t) = 3t
2
4t + 2 > 0 t nn f(t) ng bin trn R.
Nu x > y 2x > 2y f(y) > f(x) y > x (mu thun). Tng t vi x < y. Vy x = y.
Ta c h tng ng

x = y
x
3
2x
2
+ 1 = 0

x = y
x

1;
1

5
2

Vy h c nghim (x; y) = (1; 1), (


1 +

5
2
;
1 +

5
2
), (
1

5
2
;
1

5
2
) 2
Nhn xt: Ta cng c th gii bi trn bng bin i s cp. Nu tr hai phng trnh v
theo v ta s c
(x y)(x
2
+xy +y
2
2x 2y + 2) = 2(y x)

x = y
x
2
+xy + y
2
2(x + y) + 4 = 0
Xt trng hp x
2
+xy +y
2
2(x +y) + 4 = 0:
Thay y =
x
3
2x
2
+ 2x + 1
2
ta c phng trnh bc 6 theo x:
x
6
4x
5
+ 10x
4
14x
3
+ 16x
2
10x + 13 = 0
(x
6
4x
5
+ 4x
4
) + (6x
4
14x
3
+ 10x
2
) + (6x
2
10x + 13) = 0
Phng trnh trn v nghim do

x
6
4x
5
+ 4x
4
= x
4
(x 2)
2
0
6x
4
14x
3
+ 10x
2
> 0
6x
2
10x + 13 > 0
Nh vy so vi bin i s cp, cch dng o hm khin li gin n gin hn nhiu, nht l
trong nhng bi h phng trnh m - logarit.
Bi 5: Gii h phng trnh

2
x
2 = 3y 3
x
3
y
2 = 3x 2
y
Gii
Ta vit li h nh sau:

2
x
+ 3
x
= 3y + 2
2
y
+ 3
y
= 3x + 2
184
T suy ra x, y >
2
3
. Xt hm s f(t) = 2
t
+3
t
c f

(t) = 2
x
. ln 2+3
x
. ln 3 > 0 t (
2
3
; +)
Vy hm s f(t) ng bin trn (
2
3
; +). Suy ra x = y. Ta c h tng ng

x = y
2
x
+ 3
x
= 3x + 2 ()
Xt g(t) = 2
t
+3
t
3x2, ta c g

(t) = 2
t
. ln
2
2+3
t
. ln
2
3 > 0 nn g

(t) = 0 c ti a 1 nghim,
suy ra g(t) = 0 c ti a 2 nghim. Nh vy (*) ch c 2 nghim x = 1 v x = 0.
Kt lun: H c nghim (x; y) = (0; 0), (1; 1) 2
Bi tp t luyn
1) Gii h phng trnh

x
2
+ 2xy + 3y
2
2x 10y = 0
2x
2
+ 2xy +y
2
2y = 0
2) Gii h phng trnh

2x
2
+ 3xy +y
2
9x 6y = 6
x
2
+xy + 2y
2
3x 12y + 10 = 0
3) Tm m h c nghim:

3x
2
+ 2xy +y
2
= 11
x
2
+ 2xy + 3y
2
= 17 +m
H PHNG TRNH HON V
L thuyt
H phng trnh hon v l h c dng:

f(x
1
) = g(x
2
)
f(x
2
) = g(x
3
)
...
f(x
n1
) = g(x
n
)
f(x
n
) = g(x
1
)
Sau y l mt s nh l tng qut v h hon v:
nh l 1: Nu hai hm s f(x), g(x) cng tng trn tp A v (x
1
, x
2
, ..., x
n
) l nghim ca
h (trong x
i
A, i = 1, n) th x
1
= x
2
= .. = x
n
Chng minh: Khng gim tng qut gi s x
1
= max(x
1
, x
2
, ..., x
n
)
f(x
1
) f(x
2
) g(x
2
) g(x
3
) x
2
x
3
... x
n
x
1
x
1
x
2
x
3
... x
n
x
1
x
1
= x
2
= ... = x
n
2
185
nh l 2: Nu hm s f(x) gim v g(x) tng trn tp A v (x
1
, x
2
, ..., x
n
) l nghim ca
h (trong x
i
A, i = 1, n) th vi n l, x
1
= x
2
= .. = x
n
.
Chng minh: Khng gim tng qut gi s x
1
= max(x
1
, x
2
, ..., x
n
).
x
1
x
2
f(x
1
) f(x
2
) g(x
2
) g(x
3
) x
2
x
3
... x
n
x
1
f(x
n
) f(x
1
) x
1
x
2
x
1
= x
2
x
1
= x
2
= ... = x
n
2
nh l 3: Nu hm s f(x) gim, g(x) tng trn tp A v (x
1
, x
2
, ..., x
n
) l nghim ca h
(trong x
i
A, i = 1, n) th vi n chn, x
1
= x
3
= .. = x
n1
hoc x
2
= x
4
= ... = x
n
Chng minh: Khng gim tng qut gi s x
1
= max(x
1
; x
2
; ...; x
n
)
x
1
x
3
f(x
1
) f(x
3
) g(x
2
) g(x
4
) x
2
x
4
f(x
2
) f(x
4
) g(x
3
) g(x
5
) x
3
x
5
...
f(x
n2
) f(x
n
) g(x
n1
) g(x
1
) x
n1
x
1
f(x
n1
) f(x
1
) g(x
n
) g(x
2
) x
n
x
2
Vy ta c

x
1
x
3
... x
n1
x
1
x
2
x
4
... x
n
x
2

x
1
= x
3
= ... = x
n1
x
2
= x
4
= ... = x
n2
2
Trong gii ton, ta thng gp h hon v 3 n, trong a phn c th gii bng nguyn l
cc hn. Bi vit ny cng cp n mt s h c bit, c th gii bng cch bin i cc
phng trnh, dng bt ng thc hoc lng gic ho. Sau y l mt s bi tp v d.
Bi tp v d
Bi 1: Gii h phng trnh (I)

2x(y
2
+ 1) = y(y
2
+ 9)
2y(z
2
+ 1) = z(z
2
+ 9)
2z(x
2
+ 1) = x(x
2
+ 9)
.
Gii
H phng trnh tng ng

x =
y(y
2
+ 9)
2(y
2
+ 1)
y =
z(z
2
+ 9)
(z
2
+ 1)
z =
x(x
2
+ 9)
(x
2
+ 1)
Xt hm s f(t) =
t(t
2
+ 9)
2(t
2
+ 1)
(t R).
Ta c f

(t) =
t
4
4t
2
+ 9
2(t
2
+ 1)
2
> 0 (t R). Suy ra hm s f(t) ng bin trn R.
Khng mt tnh tng qut, gi s x = maxx; y; z.
x y f(x) f(y) z x z = x f(x) = f(y) x = y x = y = z.
Thay vo phng trnh th nht ca h ta c:
2x
3
+ 2x = x
3
+ 9x x
3
7x = 0

x =

7
x = 0
x =

7
186
Vy h phng trnh c ba nghim (x; y; z) l (

7;

7;

7); (0; 0; 0) v (

7;

7;

7). 2
Nhn xt: Bi 1 cho ta ci nhn s lc v nguyn l cc hn trong h hon v. Ta cng c
th tng qut bi trn nh sau:
Bi 1*:(Olympic 30-4-2009)
Gii v bin lun theo tham s a h phng trnh

2x(y
2
+a
2
) = y(y
2
+ 9a
2
)
2y(z
2
+a
2
) = z(z
2
+ 9a
2
)
2z(x
2
+a
2
) = x(x
2
+ 9a
2
)
Bi 2: Gii h phng trnh: (II)

y
3
6x
2
+ 12x 8 = 0
z
3
6y
2
+ 12y 8 = 0
x
3
6z
2
+ 12z 8 = 0
Gii
(II)

y
3
= 6x
2
12x + 8 (1)
z
3
= 6y
2
12y + 8 (2)
x
3
= 6z
2
12z + 8 (3)
Khng gim tng qut gi s x = maxx; y; z.
Ta c y
3
= 6(x 1)
2
+ 2 2 y > 1. Chng minh tng t c x, y, z > 1.
Xt f(t) = 6t
2
12t + 8; f

(t) = 12t 12 > 0 t > 1 f(t) ng bin trn (1; +)


Ta c x y f(x) f(y) y
3
z
3
y z f(y) f(z) z
3
x
3
x y z x x = y = z. Khi :
(II)

x = y = z
x
3
= 6x
2
12x + 8
x = y = z = 2
Vy (II) c nghim (x; y; z) = (2; 2; 2) 2
Nhn xt: bi ny ta thy f(t) khng n iu, khc vi bi 1. Do phi c thm nhn
xt x, y, z > 1. y chnh l mu cht ca bi ton.
Sau y ta s n vi mt cch nhn mi v nguyn l cc hn. Thay v nh gi x, y, z ring
l, ta s nh gi nhng biu thc hon v ca x, y, z l x +y, y + z, z +x:
Bi 3: (VMO 2006) Gii h phng trnh (III)

x
3
+ 3x
2
+ 2x 5 = y
y
3
+ 3y
2
+ 2y 5 = z
z
3
+ 3z
2
+ 2z 5 = x
Gii
Ta cng xem qua 2 cch gii ca bi ton.
Cch 1:
Cng 3 phng trnh ca h ta c:
x
3
+ 3x
2
+x 5 + y
3
+ 3y
2
+ y 5 + z
3
+ 3z
2
+z 5 = 0
(x 1)(x
2
+ 4x + 5) + (y 1)(y
2
+ 4y + 5) + (z 1)(z
2
+ 4z + 5) = 0
187
Nu x > 1 z
3
+ 3z
2
+ 2z 5 > 1 (z 1)(z
2
+ 4z + 6) > 0 z > 1.
Li t z > 1, lp lun tng t suy ra y > 1. Khi :
(x 1)(x
2
+ 4x + 5) + (y 1)(y
2
+ 4y + 5) + (z 1)(z
2
+ 4z + 5) > 0 (v l)
Tng t, nu x < 1 ta suy ra y < 1, z < 1 cng suy ra iu v l.
Vy x = y = z = 1 chnh l nghim ca h phng trnh. 2
Cch 2:
Ta c:
(III)

(x + 1)
3
= y + x + 6 (1)
(y + 1)
3
= z +y + 6 (2)
(z + 1)
3
= x +z + 6 (3)
Khng gim tng qut gi s x = maxx; y; z.
x +z y + z (z + 1)
3
(y + 1)
3
z y x +z x +y (z + 1)
3
(x + 1)
3
z x z = x x + y = y +z (x + 1)
3
= (y + 1)
3
x = y
Khi :
(III)

x = y = z
x
3
+ 3x
2
+ x 5 = 0
x = y = z = 1
Vy (III) c nghim (x; y; z) = (1; 1; 1) 2
Nhn xt: Cch 1 l phng php dng bt ng thc, s c cp sau. Xt cch 2,
ta thy vic nh gi x + y, y + z, z + x r rng c li th ca n. Ta vn c th so snh
x + y, y + z, z + x da vo mi quan h hon v gia x, y, z nhng v tri ca (1), (2), (3)
tr nn rt p. Ta cng xt mt bi tng t:
Bi 4: Gii h phng trnh (IV )

x = 3y
3
+ 2y
2
(1)
y = 3z
3
+ 2z
2
(2)
z = 3x
3
+ 2x
2
(3)
Gii
Ta c
(IV )

x +y = 3y
3
+ 2y
2
+y
y + z = 3z
3
+ 2z
2
+z
z + x = 3x
3
+ 2x
2
+ x
Khng gim tng qut gi s x = maxx; y; z.
Xt f(t) = 3t
3
+ 2t
2
+ t ta c f

(t) = 9t
2
+ 4t + 1 > 0 f(t) ng bin trn R
Li c x +y y + z f(y) f(z) y z x + y x + z f(y) f(x) y x
y = x f(y) = f(x) x +y = x +z y = z
188
Vy
(IV )

x = y = z
3x
3
+ 2x
2
x = 0

x = y = z
x 0; 1;
1
3

Vy (IV ) c nghim (x; y; z) = (0; 0; 0), (1; 1; 1),

1
3
;
1
3
;
1
3

2
Nhn xt: Cu hi t ra l: khi no ta phi xt (x + y; y + z; z + x)? Mu cht y
chnh l ta cn xy dng mt hm n iu trn min xc nh. Nu nguyn (IV ), ta xt
f(t) = 3t
3
+ 2t
2
th f

(t) = 9t
2
+ 4t c nghim t = 0 nn f(t) c th i chiu n iu. T
ta ngh ti vic bin i cc phng trnh.
c mt hm n iu, ta s cng hai v ca (1) cho ky, tng t vi (2) v (3):
(IV )

x +ky = 3y
3
+ 2y
2
+ ky
y + kz = 3z
3
+ 2z
2
+kz
z + kx = 3x
3
+ 2x
2
+kx
Khi ta xt f(t) = 3t
3
+ 2t
2
+kt c f

(t) = 9t
2
+ 4t +k. Ta cn f

(t) > 0 t, ngha l k >


4
9
Nhng s k cn phi m bo ta vn c th so snh VT ca (1), (2) v (3). D thy hng s
tt nht l k = 1. Khi ta c

9t
2
+ 4t + 1 > 0 t
x +y x +z y + z
T ta i n li gii nh trn.
Bi 5: Gii h phng trnh (V )

x = 2y
2
1 (1)
y = 2z
2
1 (2)
z = 2x
2
1 (3)
Gii
Khng gim tng qut gi s x = maxx; y; z.
(3) 2x
2
1 = z x
1
2
x 1
t x = cos t (t [0;
2
3
])
(3) z = 2 cos
2
t 1 = cos 2t
(2) y = 2 cos
2
2t 1 = cos 4t
(1) x = 2 cos
2
4t 1 = cos 8t
cos t = cos 8t t =
k2
7
t =
k2
9
(k Z)
t

0;
2
9
;
4
9
;
2
3
;
2
7
;
4
7

(do t

0;
2
3

)
Vy (V I) c nghim (x; y; z) = (1; 1; 1) ;

cos
2
9
; cos
4
9
; cos
8
9

1
2
;
1
2
;
1
2

cos
2
7
; cos
4
7
; cos
8
7

v cc hon v. 2
Nhn xt: Phng php lng gic ho cn s kho lo v hiu bit r v cng thc lng
gic. Nh bi 5, t x = 2y
2
1 lm ta nh n cng thc cos 2t = 2 cos
2
t 1, do ta mi
189
t x = cos t. Ta xt tip Bi 6:
Bi 6: Gii h phng trnh (V I)

x
2
= y + 2 (1)
y
2
= z + 2 (2)
z
2
= x + 2 (3)
Gii
Khng gim tng qut gi s x = maxx; y; z.
x
2
= y + 2 x + 2 1 x 2.
t x = 2 cos t (t

0;
2
3

).
(1) y = x
2
2 = 2(2 cos
2
t 1) = 2 cos 2t
(2) z = y
2
2 = 2(2 cos
2
2t 1) = 2 cos 4t
(3) x = z
2
2 = 2(2 cos
2
4t 1) = 2 cos 8t
cos t = cos 8t t =
k2
7
t =
k2
9
(k Z)
t

0;
2
9
;
4
9
;
2
3
;
2
7
;
4
7

do t

0;
2
3

Vy (V I) c nghim (x; y; z) = (2; 2; 2) ;

2 cos
2
9
; 2 cos
4
9
; 2 cos
8
9

; (1; 1; 1) ;

2 cos
2
7
; 2 cos
4
7
; 2 cos
8
7

v cc hon v. 2
Nhn xt: Bi 6 r rng cn s kho lo. Nu t x = sin t; cos t hay tan t th biu thc x
2
2
khng lin quan trc tip n cng thc lng gic no, nhng rng
(2 cos t)
2
2 = 2(2 cos t 1) = 2 cos 2t. V vy ta phi t x = 2 cos t.
Bi 7: Gii h phng trnh (V II)

x
4
y 6x
2
y + 4x
3
4x + y = 0
y
4
z 6y
2
z + 4y
3
4y + z = 0
z
4
x 6z
2
x + 4z
3
4z + x = 0
Gii
tng: u tin ta bin i
(V II)

y =
4x 4x
3
x
4
6x
2
+ 1
(1)
z =
4y 4y
3
y
4
6y
2
+ 1
(2)
x =
4z 4z
3
z
4
6z
2
+ 1
(3)
Ta th xt o hm ca f(t) =
4t 4t
3
t
4
6t
2
+ 1
. D thy
f

(t) =
4(t
2
+ 1)
3
(t
4
6t
2
+ 1)
2
> 0 t =

3 2

2
n y, nu cha nhn ra vn th ta s c nhn xt x = y = z
(V II) c nghim (x; y; z) = (0; 0; 0); (

3;

3;

3); (

3;

3;

3).
190
Thc ra y ta mc mt sai lm rt tinh vi l hm f(t) ch ng bin trn tng khong
xc nh ch khng ng bin trn min xc nh (v d f(2) =
24
7
> f(3) =
24
7
). V vy
bi 7 khng th dng hm n iu. Theo tng trn, ta ngh ti phng n 2: lng
gic ho.
Nhn vo phn thc, ta thng ngh ti cng thc v tan. rng f(tan a) = tan 4a do
ta t x = tan a.
Li gii:
KX: x, y, z =

3 2

2
t x = tan a (a [0; ) a =

2
)
T (1), (2), (3) ta tnh c: y = f(x) = tan 4a; z = f(y) = tan 16a; x = f(z) = tan 64a
tan a = tan 64a a =
k
63
(k = 0, 62)
Vy (V II) c nghim (x; y; z) = (tan a; tan 4a; tan 16a) vi a =
k
63
, k = 0; 62
Nhn xt: Nguyn l cc hn thng c dng trong cc bi h hon v nhng khng cn
tc dng vi nhng bi t lng gic. Nu xt hm n iu theo cch u tin th ta lm
mt 60 nghim ca h!
Bi 8: Gii h phng trnh: (V III)

x
3
3x = y (1)
y
3
3y = z (2)
z
3
3z = x (3)
(HSG Thi Bnh 2009)
Gii
T h phng trnh suy ra:

(x
3
3x)
3
3(x
3
3x)

3
3

(x
3
3x)
3
3(x
3
3x)

= x.
y l phng trnh a thc bc 27 nn c nhiu nht 27 nghim. Do h cho c nhiu
nht 27 nghim.
Xt x [2; 2]. Ta s chng minh h c 27 nghim trong trng hp ny, do khng cn xt
cc trng hp cn li.
t x = 2 cos t (t [0; ]). Khi ta c:

(1) y = 2 cos 3t
(2) z = 2 cos 9t
(3) x = 2 cos 27t
cos 27t = cos t

t = k

13
t = k

14
(k Z)
Vi t = k

13
(k Z), do t [0; ] nn k nhn cc gi tr 0;1;2;. . . ;12;13.
Vi t = k

14
(k Z), do t [0; ] nn k nhn cc gi tr 0;1;2;. . . ;12;13;14.
Nghim ca h ng vi t = 0 v t = c hai trng hp trng nhau nn h c 27 nghim
phn bit l

2 cos
k
13
; 2 cos
k3
13
; 2 cos
k9
13

vi k = 0, 13 v

2 cos
k
14
; 2 cos
k3
14
; 2 cos
k9
14

vi k = 1, 13 2
Nhn xt: Mu cht ca li gii chnh l nhn xt h c ti a 27 nghim, khi khng cn
xt cc trng hp cn li. Tht ra ta vn c th xt thm trng hp x, y, z (; 2) v
191
x, y, z (2; +), khi hm s f(t) = t
3
3t l hm n iu.
Tip theo ta cng xem qua mt s bi h hon v gii bng Bt ng thc:
Bi 9: Gii h phng trnh (IX)

x
2
+x 1 = y
y
2
+ y 1 = z
z
2
+ z 1 = x
Gii
Ta cng xem qua 2 cch gii cho bi ton:
Cch 1:
Khng mt tnh tng qut gi s: x = min x; y; z
x
2
1 = y x 0 [x[ 1
z
2
1 = x z 0 [z[ 1 1 y
2
+y 1 1 0 y
2
+y 2 y [2; 1] [0; 1].
* Nu x > 0 z x 1 z = x = 1 y = 1
* Nu x 0

x 1 z
2
+z 1 1 z [0; 1]
x 1 x(x + 1) 0 y 1
* Nu y = 1 x = y = z = 1
* Nu y > 1 : y [0; 1]
Ta c:

1 z
2
+z 1 = x
5
4
y
2
+ y 1 = z 0

0 y

5 1
2
1 z 0
[xyz[ < 1 ().
M xyz(x + 1)(y + 1)(z + 1) = (x
2
+x)(y
2
+y)(z
2
+z) = (x + 1)(y + 1)(z + 1)
Nu (x + 1)(y + 1)(z + 1) = 0 th x = y = z = 1
Nu (x + 1)(y + 1)(z + 1) = 0 th xyz = 1 (sai do (*))
Vy h c nghim (x; y; z) = (1; 1; 1), (1; 1; 1) 2
Cch 2:
Cng v theo v ta c: x
2
+ y
2
+z
2
= 3
H

x(x + 1) = y + 1
y(y + 1) = z + 1
z(z + 1) = x + 1
Nu x = 1 y = z = 1
Nu x, y, z = 1 th nhn v theo v, ta c: xyz = 1
Li c: 3 = x
2
+ y
2
+ z
2
3
3

x
2
y
2
z
2
= 3 x
2
= y
2
= z
2
= 1 x = y = z = 1 Vy h c
nghim (x; y; z) = (1; 1; 1), (1; 1; 1) 2
Bi 10: Gii h phng trnh (X)

x
3
2x 2 = y
y
3
2y 2 = z
z
3
2z 2 = x
Gii
192
H (X) tng ng vi

(x 2)[(x + 1)
2
+ 1] = y 2 (1)
(y 2)[(y + 1)
2
+ 1] = z 2 (2)
(x 2)[(z + 1)
2
+ 1] = x 2 (3)
rng

(1) [x 2[ [y 2[
(2) [y 2[ [z 2[
(3) [z 2[ [x 2[
[x 2[ = [y 2[ = [z 2[
Ta cng c x 2, y 2, z 2 cng du do x 2 = y 2 = z 2 x = y = z
Vy (X) tr thnh

x = y = z
x
3
3x 2 = 0

x = y = z = 1
x = y = z = 2
Kt lun: H phng trnh c nghim (x; y; z) = (1; 1; 1), (2; 2; 2) 2
Bi 11: Gii h phng trnh (XI)

4x
2
3x + 1 = y
4y
2
+y + 1 = 5z
4z
2
z + 1 = 3x
Gii
Vit li h phng trnh di dng
(XI)

(2x 1)
2
= y x
(2y 1)
2
= 5(z y)
(2z 1)
2
= 3(x z)

x y 0
y z 0
z x 0
x = y = z
Vy (X)

x = y = z
4x
2
4x + 1 = 0
x = y = z =
1
2
Kt lun: H c nghim duy nht x = y = z =
1
2
2
Nhn xt: Tng qut hn, ta c th gii h phng trnh

[f(x)]
2
= (y x)
p
.
[f(y)]
2
= (z y)
m
.
[f(z)]
2
= (x z)
n
.
vi m, n, p l cc s l v , , 0
Bi 12: Gii h phng trnh (XII)

x
1
=

3
9
cos(x
2
)
x
2
=

3
9
cos(x
3
)
x
3
=

3
9
cos(x
4
)
x
4
=

3
9
cos(x
1
)
( Olympic 30-4-2007)
Gii
193
T h phng trnh ta c [x
i
[

3
9
<
1
2
(i = 1, 4)

2
< x
i
<

2
0 < cos(x
i
) 1 0 < x
i
<
1
2
0 < x
i
<

2
Khng gim tng qut, gi s x
1
= maxx
1
; x
2
; x
3
; x
4

x
1
x
3

3
9
cos(x
2
)

3
9
cos(x
4
) x
2
x
4
x
2
x
4

3
9
cos(x
3
)

3
9
cos(x
1
) x
3
x
1
x
3
x
1
x
3
= x
1
Khi suy ra x
2
= x
4
.
Xt hm s f(x) = x +

3
9
cos(x) vi x

0;
1
2

Ta c: f

(x) = 1

3
9
sin(x) 1

3
9
> 0. Suy ra hm s f(x) ng bin trn

0;
1
2

.
Li c: x
1
+ x
4
= x
1
+ x
2
x
1
+

3
9
cos(x
1
) = x
2
+

3
9
cos(x
2
)
x
1
= x
2
(do hm s f(x) ng bin).
Do : x
1
= x
2
= x
3
= x
4
x
1
=

3
9
cos(x
1
) cos(x
1
) 3

3x
1
= 0
Xt hm s g(x) = cos(x) 3

3x vi x

0;
1
2

Ta c: g

(x) = sin(x) 3

3 < 0 x

0;
1
2

. Vy hm s g(x) nghch bin trn khong

0;
1
2

.
Do phng trnh g(x) = 0 c nhiu nht 1 nghim trn on ny.
Mt khc, g(
1
6
) = 0 nn phng trnh g(x) = 0 c nghim duy nht x =
1
6
.
Vy h phng trnh c nghim duy nht l

1
6
;
1
6
;
1
6

2.
Bi 13: Gii h phng trnh (XIII)

x = (y 1)
2
y = (z 1)
2
z = (t 1)
2
t = (x 1)
2
Gii
Gi s x = min x; y; z; t. T h phng trnh ta suy ra x, y, z, t > 0.
Trng hp 1: 0 < x 1
1 < x 1 0 0 (x 1)
2
< 1 0 < t 1 0 (t 1)
2
< 1
0 < z 1 0 (z 1)
2
< 1 0 < y 1.
Vy ta c: 0 < x, y, z, t 1.
Khi , t z x suy ra (t 1)
2
(y 1)
2
y t (z 1)
2
(x 1)
2
x z (y 1)
2
(t 1)
2
t y.
T suy ra: x = z, y = t.
194
T h phng trnh ta c:

x = (y 1)
2
y = (x 1)
2

x = y =
3 +

5
2
> 1 (loi)
x = y =
3

5
2

x = 0
y = 1

x = 1
y = 0
.
Trng hp 2: x > 1
x, y, z, t > 1. Khi ,t y x suy ra (z 1)
2
(y 1)
2
z y
(t 1)
2
(z 1)
2
t z (x 1)
2
(t 1)
2
x t. Suy ra: x = y = z = t.
T h phng trnh suy ra:
x = (x 1)
2

x =
3 +

5
2
x =
3

5
2
< 1
Vy h phng trnh c cc nghim l (x; y; z; t) =

3 +

5
2
;
3 +

5
2
;
3 +

5
2
;
3 +

5
2

5
2
;
3

5
2
;
3

5
2
;
3

5
2

;(0; 1; 0; 1); (1; 0; 1; 0) 2.


Bi 14: Gii h phng trnh (XIV )

x
2
1
+x
1
x
2
1 = 0
x
2
2
+x
2
x
3
1 = 0
x
2
3
+x
3
x
4
1 = 0
x
2
4
+x
4
x
1
1 = 0
. (Olympic 30-4-2011)
Gii
Khng gim tng qut gi s x
1
= minx
1
; x
2
; x
3
; x
4
.
Xt f(t) = t
2
+ t 1. H phng trnh tng tng

f(x
1
) = x
2
f(x
2
) = x
3
f(x
3
) = x
4
f(x
4
) = x
1
Hm s f(t) tng trn khong (
1
2
; +), gim trn khong (;
1
2
) v
f(t) f(
1
2
) =
5
4
t R nn x
i

5
4
i = 1, 4.
Nu x
1

1
2
:
Ta c:

x
4

5
4
f(x
4
)
1
2
>
11
6
= f(
5
4
)
x
4
>
1
2
.
195
Lp lun tng t ta cng c: x
2
, x
3
>
1
2
.
Do x
1
x
2
f(x
1
) f(x
2
) x
2
x
3
f(x
2
) f(x
3
) x
3
x
4
f(x
3
) f(x
4
) x
4
x
1
x
1
x
2
x
3
x
4
x
1
x
1
= x
2
= x
3
= x
4
= 1.
Nu x
1
<
1
2
:
Nu c k 1; 2; 3; 4 x
k

1
2
th theo trng hp trn ta c: x
i

1
2
i = 1, 4
x
1

1
2
( mu thun). Vy x
i
<
1
2
i = 1, 4.
Do x
1
x
3
f(x
1
) f(x
3
) x
2
x
4
f(x
2
) f(x
4
) x
3
x
1
x
3
= x
1
.
Lp lun tng t ta c x
2
= x
4
.
H phng trnh tr thnh:

x
1
= x
3
x
2
= x
4
f(x
1
) = x
2
f(x
2
) = x
3
x
1
+f(x
1
) = x
2
+f(x
2
) x
2
1
+ 2x
1
1 = x
2
2
+ 2x
2
1

x
1
= x
2
x
1
= x
2
2
x
1
= x
2
= 1
x
1
= x
2
= x
3
= x
4
= 1
Vy h phng trnh ban u c nghim l x
1
= x
2
= x
3
= x
4
= 1 2.
Bi 15: Gii h phng trnh (XV )

x
1
=
1
2

x
2
+
1
x
2

(1)
x
2
=
1
2

x
3
+
1
x
3

(2)
...
x
2012
=
1
2

x
1
+
1
x
1

(2012)
Gii
Kx: x
i
= 0 (i = 1; 2012)
(1) [x
1
[ =
1
2
[x
2
+
1
x
2
[ =
1
2
([x
2
[ +[
1
x
2
[) 1
Chng minh tng t c [x
i
[ 1 (i = 1; 2012)
Li c: x
1
.x
2
=
1
2
(x
2
2
+ 1) > 0 x
1
, x
2
cng du.
Chng minh tng t c x
1
, x
2
, ..., x
2012
cng du.
[
2012

i=1
x
i
[ =
2012

i=1
[x
i
[ [
2012

i=1
1
x
i
[ =
2012

i=1
[
1
x
i
[()
Cng (1), (2), ..., (2012) v theo v ta c:
2012

i=1
x
i
=
1
2
2012

i=1
(x
i
+
1
x
i
)
2012

i=1
x
i
=
2012

i=1
1
x
i
[
2012

i=1
x
i
[ = [
2012

i=1
1
x
i
[
2012

i=1
[x
i
[ =
2012

i=1
[
1
x
i
[() (do())
Li c: [x
i
[ 1
2012

i=1
[x
i
[ 2012
2012

i=1
[
1
x
i
[
196
Vy (**) tng ng
[x
1
[ = [x
2
[ = ...[x
2012
[ = 1

x
1
= x
2
= ... = x
2012
= 1
x
1
= x
2
= ... = x
2012
= 1
Th li ta thy (V III) c nghim (x
1
; x
2
; ...x
2012
) = (1; 1; ...; 1), (1; 1, ...; 1). 2
Nh vy, bng bin i kho lo, kt hp ng thc [
n

i=1
x
i
[ =
n

i=1
[x
i
[ x
1
, x
2
, ...x
n
cng
du, ta gii quyt bi ton m khng cn ti nguyn l cc hn.
Bi 16: Gii h phng trnh: (XV I)

x
2
1
= x
2
+ 1
x
2
2
= x
3
+ 1
.................
x
2
n1
= x
n
+ 1
x
2
n
= x
1
+ 1
(n N, n 2).
Gii
Khng mt tnh tng qut, gi s x
1
= maxx
1
, x
2
..., x
n
(*).
Th th x
2
n
= x
1
+ 1 = max x
1
+ 1, x
2
+ 1, ..., x
n
+ 1 = max x
2
1
, x
2
2
, ..., x
2
n
.
[x
n
[ = max[x
1
[ , [x
2
[ ..., [x
n
[ (**)
T (*) v (**) suy ra x
n
0.
Nu x
1
> 0 th x
2
n
= x
1
+ 1 > 1. V x
n
< 0 nn x
n
< 1( v l v x
2
n1
= x
n
+ 1 < 0).
Vy x
1
0 x
i
0 i = 1, n.
T (**) suy ra x
n
= minx
1
, x
2
..., x
n
.
V x
i
x
1
0 i nn [x
1
[ = min [x
i
[ hay x
1
2
= min x
i
2
.
Vy x
2
= x
2
1
1 = minx
2
i
1 = minx
i
= x
n
. T x
2
2
= x
3
+1 v x
2
n
= x
1
+1 suy ra x
1
= x
3
.
C tip tc nh vy ta c

x
1
= x
3
= ... = max x
i

x
2
= x
4
= ... = minx
i
=x
n
.
Vi n l:
Ta c x
1
= x
2
= ... = x
n
. T h ta c: x
2
1
x
1
1 = 0 x
1
=
1

5
2
.
Do h phng trnh c nghim l x
1
= x
2
= ... = x
n
=
1

5
2
.
Vi n chn:
T x
2
2
= x
3
+ 1, x
2
1
= x
2
+ 1 v x
1
= x
3
ta c (x
2
1
1)
2
= x
1
+ 1

x
1
= 0
x
1
= 1
x
1
=
1

5
2
Vy h phng trnh c cc nghim
(x
1
, x
2
, ..., x
n
) =

5
2
;
1

5
2
; . . . ;
1

5
2

, (1; 0; 1; 0. . . ; 1; 0), (0; 1; 0; 1; . . . ; 0; 1). 2


Bi 17: Gii h phng trnh: (XV III)

x
3
12y
2
+ 48y 64 = 0 (1)
y
3
12z
2
+ 48z 64 = 0 (2)
z
3
12x
2
+ 48x 64 = 0 (3)
197
Gii
Cng (1), (2), (3) v theo v ta c:
(x 4)
3
+ (y 4)
3
+ (z 4)
3
= 0()
Suy ra trong 3 s hng trn c t nht 1 s khng m, gi s l (z 4)
3
0 z 4
(2) y
3
16 = 12(z 2)
2
12.2
2
y 4
(1) x
3
16 = 12(y 2)
2
12.2
2
x 4
Vy x, y, z 4 V T() 0. Khi () x 4 = y 4 = z 4 = 0 x = y = z = 4
Th li ta thy (IX) c nghim (x; y; z) = (4; 4; 4) 2
Bi 18: Tm nghim dng ca h phng trnh (XV II)

30

x
1
+ 4

x
2
=
2009

x
2008
3
30

x
2
+ 4

x
3
=
2009

x
2008
4
......................................
30

x
2008
+ 4

x
1
=
2009

x
2008
2
Gii
Gi s (x
1
, x
2
, x
3
....., x
2008
) l mt nghim ca h phng trnh.
t a = max x
1
, x
2
, x
3
....., x
2008
, b = min x
1
, x
2
, x
3
....., x
2008
a b > 0.
Ta c:

34

a 30

x
1
+ 4

x
2
=
2009

x
2008
3
34

a 30

x
2
+ 4

x
3
=
2009

x
2008
4
......................................
34

a 30

x
2008
+ 4

x
1
=
2009

x
2008
2
34

a max

2009

x
2008
1
,
2009

x
2008
2
, ...,
2009

x
2008
2008

34

a
2009

a
2008
34
4018
a
2009
a
4016
a
2007
34
4018
a
2007

34
4018
Tng t ta c:

34

b 30

x
1
+ 4

x
2
=
2009

x
2008
3
34

b 30

x
2
+ 4

x
3
=
2009

x
2008
4
......................................
34

b 30

x
2008
+ 4

x
1
=
2009

x
2008
2
34

b min

2009

x
2008
1
,
2009

x
2008
2
, ...,
2009

x
2008
2008

34

b
2009

b
2008
34
4018
b
2009
b
4016
b
2007
34
4018
b
2007

34
4018
Vy a = b =
2007

34
4018
x
1
= x
2
= ...x
2008
=
2007

34
4018
.
Th li ta thy nghim ca h phng trnh l x
1
= x
2
= ...x
2008
=
2007

34
4018
2
Ta cng xem qua mt s bi h hon v lng gic:
198
Bi 19: Gii h phng trnh (XIX)

cos 2x + cos x = 2 cos y


cos 2y + cos y = 2 cos z
cos 2z + cos z = 2 cos x
Gii
Ta c:
(XIX)

(2 cos x 1)(cos x + 2) = 2 cos y 1


(2 cos y 1)(cos y + 2) = 2 cos z 1
(2 cos z 1)(cos z + 2) = 2 cos x 1

[2 cos x 1[ (cos x + 2) = [2 cos y 1[


[2 cos y 1[ (cos y + 2) = [2 cos z 1[
[2 cos z 1[ (cos z + 2) = [2 cos x 1[
[2 cos x1[ [2 cos y 1[ [2 cos z 1[ [2 cos x1[ (do cos x+2; cos y +2; cos z +2 1)
[2 cos x 1[ = [2 cos y 1[ = [2 cos z 1[
Khi (XIX) tr thnh:

[2 cos x 1[ (cos x + 2) = [2 cos x 1[


[2 cos y 1[ (cos y + 2) = [2 cos y 1[
[2 cos z 1[ (cos z + 2) = [2 cos z 1[

2 cos x 1 = 2 cos y 1 = 2 cos z 1 = 0


cos x + 2 = cos y + 2 = cos z + 2 = 1

x, y, z

3
+k[k Z

x, y, z 2k[k Z
2
Bi 20: Gii h phng trnh (XX)

2 cos 2x +
tan
2
x + tan
2
y
1 + tan
2
x
= 2
2 cos 2y +
tan
2
y + tan
2
z
1 + tan
2
= 2
2 cos 2z +
tan
2
z + tan
2
x
1 + tan
2
z
= 2
Gii
T ng thc cos 2 =
1 tan
2

1 + tan
2

, ta c
(XX)

tan
2
y tan
2
x
1 + tan
2
x
= 1 cos 2x
tan
2
z tan
2
y
1 + tan
2
y
= 1 cos 2y
tan
2
x tan
2
z
1 + tan
2
z
= 1 cos 2z
tan
2
x tan
2
y tan
2
z tan
2
x tan
2
x = tan
2
y = tan
2
z
1 cos 2x = 1 cos 2y = 1 cos 2z = 0

x = k
y = l
z = m
(k, l, m Z)
Th li ta thy (XX) c nghim (x; y; z) = (k; l; n) (k, l, m Z) 2
199
Bi 21: Gii h phng trnh (XXI)

e
x
e
xy
= y
e
y
e
yz
= z
e
z
e
zx
= x
Gii
Vi x = 0, ta c h

1 e
y
= y
e
y
e
yz
= z
e
z
e
z
= 0
Xt hm f(y) = 1 e
y
y (y R).
Ta c: f

(y) = e
y
1 = 0 y = 0
Do f(y) ng bin trn (; 0] v nghch bin trn [0; +) nn Maxf(y) = 0 v do phng
trnh f(y) = 0 c nghim duy nht y = 0 z = 0.
Vy (0;0;0) l mt nghim ca phng trnh.
Vi x = 0, h phng trnh tng ng vi h

e
x
=
ye
y
e
y
1
e
y
=
ze
z
e
z
1
e
z
=
xe
x
e
x
1
Xt hm g(t) =
te
t
e
t
1
(t = 0).
Ta c: g

(t) =
e
t
(e
t
t 1)
(e
t
1)
2
> 0 t = 0( v e
t
> t + 1 t = 0) lim
t0

g(t) = 1; lim
t0
+
g(t) = 1
D thy g(t) ng bin trn (; 0) v trn (0; +)
Suy ra: nu x > y g(x) > g(y) e
z
> e
x
z > x g(z) > g(x) e
y
> e
z
y > z
y > x(mu thun).
Tng t, nu x < y cng suy ra mu thun.
Vy x = y y = z v h phng trnh tr thnh

x = y = z = 0
e
x
1 = x
x, y, z .
Vy (0;0;0) l nghim duy nht ca phng trnh.
Sau y ta cng xem qua mt s bi h phng trnh cha tham s:
Bi 22: Cho a, b, c > 0. Gii h phng trnh: (XXII)

x
2
yz = a
y
2
xz = b
z
2
yx = c
Gii
Ta c: a
2
bc = (x
2
yz)
2
(y
2
xz)(z
2
yx) = x(x
3
+y
3
+z
3
3xyz)
b
2
ac = y(x
3
+y
3
+z
3
3xyz), c
2
ab = z(x
3
+y
3
+z
3
3xyz)
t k = x
3
+y
3
+z
3
3xyz, ta c: (a
2
bc)
2
(b
2
ac)(c
2
ab) = a(a
3
+ b
3
+c
3
3abc).
200
Suy ra k
2
(x
2
yz) = a(a
3
+b
3
+ c
3
3abc) k =

a
3
+b
3
+ c
3
3abc.
Vy h phng trnh c cc nghim l
x =
a
2
bc
k
, y =
b
2
ac
k
, z =
c
2
ab
k
vi k =

a
3
+b
3
+ c
3
3abc 2.
Bi 23: Gii v bin lun h phng trnh theo m: (XXIII)

mx = z +
1
z
my = x +
1
x
mz = y +
1
y
Gii
H phng trnh tng ng

mxz = z
2
+ 1
myx = x
2
+ 1
mzy = y
2
+ 1
x.y.z = 0
mxy > 0; myz > 0; mzx > 0 m
3
x
2
y
2
z
2
> 0 m > 0.
Ta thy x,y,z cng du v nu (x; y; z) l nghim ca h phng trnh th (x; y; z) cng
l nghim. Do ta ch cn xt x, y, z > 0. Gi s x = max x; y; z.
Nu x y z > 0 :
y
2
+ 1 z
2
+ 1 myz mxz y x y = x
y
2
+ 1 = x
2
+ 1 myx = myz x = z x = y = z.
Nu x z y > 0 :
x
2
+ 1 z
2
+ 1 myx mxz y z y = z
y
2
+ 1 = z
2
+ 1 myz = mxz x = y x = y = z.
C hai trng hp u suy ra x = y = z.
Do h phng trnh tr thnh

mx
2
= x
2
+ 1
x = y = z > 0

(m1)x
2
= 1
x = y = z > 0
* Vi 0 < m 1, h phng trnh v nghim.
* Vi m > 1, h phng trnh tng ng x = y = z =
1

m1
.
Tm li:
Vi m 1: h phng trnh v nghim.
Vi m > 1, h phng trnh c hai nghim l (
1

m1
;
1

m1
;
1

m1
) v
(
1

m1
;
1

m1
;
1

m1
) 2.
Bi 24: Gii h phng trnh (XXIV )

x
2
= y +a
y
2
= z +a
z
2
= x +a
(0 < a < 1)
Gii
201
Gi s x = max x; y; z.
T h phng trnh suy ra z
2
= max x
2
; y
2
; z
2
.
Nu z 0:
Ta c z
2
= max x
2
; y
2
; z
2
z = max x; y; z x = z x
2
= z
2
y + a = x +a x = y x = y = z.
T h phng trnh ta c: x
2
= x + a x
2
x a = 0 x =
1
2
+

1
4
+a. Suy ra:
x = y = z =
1
2
+

1
4
+a
Nu z < 0:
Gi s x 0, khi z
2
= x +a a z

a < a y
2
= z +a < 0 (v l).Do x < 0.
V x = max x; y; z, x < 0 nn y < 0. Khi ta c: z y x < 0 y
2
= z + a x
2
=
y + a x y.
Suy ra x = y = z = x =
1
2

1
4
+ a.
Vy h phng trnh c cc nghim l

1
2
+

1
4
+a;
1
2
+

1
4
+a;
1
2
+

1
4
+a

1
2

1
4
+a;
1
2

1
4
+ a;
1
2

1
4
+a

2.
Sau y l mt s h phng trnh hon v khng mu mc. Ta khng th dng nguyn l cc
hn, nhng vn c th dng lng gic ho, bt ng thc. Nhng cch gii ch yu vn l
bin i ng thc, da vo quan h gia cc phng trnh.
Bi 25: Gii h phng trnh (XXV )

x +
1
x

= 4

y +
1
y

= 5

z +
1
z

(1)
xy +yz + zx = 1 (2)
Gii
K: xyz = 0
Nu (x, y, z) l mt nghim ca h th (x, y, z) cng l mt nghim ca h v t (1) suy
ra x, y, z cng du nn ta ch cn xt x, y, z dng l .
t:

x = tan
y = tan
z = tan
, ; ;

0;

2

ta c:
(I)

tan +
1
tan

= 4

tan +
1
tan

= 5

tan +
1
tan

(3)
tan tan + tan tan + tan tan = 1 (4)
Li c:
(3) 3.
tan
2
+ 1
tan
= 4.
tan
2
+ 1
tan
= 5.
tan
2
+ 1
tan

3
sin 2
=
4
sin 2
=
5
sin 2
(5)
202
T (4) ta c:
(4) tan (tan + tan ) = 1 tan tan
tan =
1 tan tan
tan + tan
= cot ( +)
+ + =

2
(6)
T (5) v (6), suy ra 2, 2, 2 l cc gc trong mt tam gic vung, c cc cnh l 3, 4, 5 Do
:
2 =

2
=

4
tan = 1 = z
T ta c:

tan = y =
1
2
tan = x =
1
3
Vy h phng trnh cho c nghim l: (x; y; z) =

1
3
;
1
2
; 1

1
3
;
1
2
; 1

2
Bi 26: Gii h phng trnh (XXV I)

ab(a +b) = 6
bc(b +c) = 30
ac(a +c) = 12
Gii
R rng a, b, c khng th bng 0. t S = a +b +c, P = abc h phng trnh tr thnh

P
c
(S c) = 6
P
a
(S a) = 30
P
b
(S b) = 12

c =
PS
P + 6
a =
PS
P + 30
b =
PS
P + 12
Nh vy S = S

P
P + 6
+
P
P + 12
+
P
P + 30

Nu S = 0 th a + b = c, khi phng trnh u tin tr thnh abc = 6 v hai phng


trnh cn li l abc = 30 v abc = 12 iu ny khng th xy ra.
Do
P
P + 6
+
P
P + 12
+
P
P + 30
= 1
Bin i v thu gn ta c P
3
+ 24P
2
1080 = 0 (P 6)(P
2
+ 30P + 180) = 0
Vi P = 6 ta c a =
S
6
, b =
S
3
, c =
S
2
ab(a +b) =
S
3
36
= 6 S = 6
Vy (a, b, c) = (1, 2, 3).
Vi P
2
+ 30P + 180 = 0 P = 15 + 3

5; P = 15 3

5
203
Vi P = 15 + 3

5 ta c

a =
15 + 3

5
15 + 3

5
S =

5 3
2
S
b =
15 + 3

5
3 + 3

5
S =

5S
c =
15 + 3

5
9 + 3

5
S =

5 + 5
2
S
Kt hp ab(a +b) = 6 ta c

5S
3
= 6 S =
6

36
5
Vy (a, b, c) =

5
2
6

36
5
,

5
6

36
5
,
5 +

5
2
6

36
5

Tng t vi P = 153

5 ta tm c (a, b, c) =

3 +

5
2
.
6

36
5
,

5.
6

36
5
,
5

5
2
.
6

36
5

Kt lun: H c nghim
(a; b; c) =

5
2
.
6

36
5
,

5.
6

36
5
,
5 +

5
2
.
6

36
5

3 +

5
2
.
6

36
5
,

5.
6

36
5
,
5

5
2
.
6

36
5

2
Bi 27: Gii h phng trnh

2x
3
= 2y(x
2
+ 1) (z
2
+ 1) (1)
2y
4
= 3z(y
2
+ 1) 2(x
2
+ 1) (2)
2z
5
= 4x(z
2
+ 1) 3(y
2
+ 1) (3)
Gii
Xt 6 trng hp:
Nu y x z:
T (1) ta c:
0 = 2y(x
2
+ 1) (z
2
+ 1) 2x
3
0 2x(x
2
+ 1) (z
2
+ 1) 2x
3
0 2x(x
2
+ 1) (x
2
+ 1) 2x
3
(x 1)
2
0
T suy ra x = y = z = 1
Nu y z x :
2x
3
+z
2
+ 1 2x
3
+ z
2
+ 1 ()
T (1) ta c 2x
3
+z
2
+ 1 = 2y(x
2
+ 1) = 2yx
2
+ 2y
Li c:

2yx
2
+ 2y 2x
3
+ 2y
2yx
2
+ 2y 2x
3
+ 2z
2yx
2
+ 2y 2x
3
+ z
2
+ 1
2x
3
+z
2
+ 1 2x
3
+ z
2
+ 1
Kt hp () suy ra x = z, ngha l x = y = z = 1.
Nu z y x :
T (2) ta c
0 = 3z(y
2
+ 1) 2y
4
2(x
2
+ 1)
0 3y(y
2
+ 1) 2y
4
2(x
2
+ 1)
0 3z(y
2
+ 1) 2y
4
2(y
2
+ 1) = (2y
2
+ y + 2)(y 1)
2
204
T suy ra x = y = z = 1
Nu z x y:
T (2) ta c
0 = 8[3z(y
2
+ 1) 2y
4
2(x
2
+ 1)] 0 8[3x(y
2
+ 1) 2y
4
2(x
2
+ 1)]
0 (3y
2
+ 1 4x)
2
16(y x) [7(y + 1)
2
+ 8](y 1)
2
T suy ra y = 1 x = y = z = 1
Nu x y z:
T (3) ta c
0 = 4x(z
2
+ 1) 3(y
2
+ 1) 2z
5
0 4y(z
2
+ 1) 3(y
2
+ 1) 2z
5
0 (2z
3
+ 4z
2
+ 2z + 3)(z 1)
2
(z y)

2z
3
2

+
7
4

Do y = z x = y = z = 1
Nu x z y:
T (3) ta c
0 = 4x(z
2
+ 1) 3(y
2
+ 1) 2z
5
0 4z(z
2
+ 1) 3(y
2
+ 1) 2z
5
0 4x(z
2
+ 1) 3(z
2
+ 1) 2z
5
0 (2z
3
+ 4z
2
+ 2z + 3)(z 1)
2
T c x = y = z = 1.
Tm li h c nghim duy nht (x; y; z) = (1; 1; 1) 2.
Bi 28: Gii h phng trnh (XXV III)

x
2
+y
2
+xy = 37 (1)
x
2
+z
2
+xz = 28 (2)
y
2
+ z
2
+ yz = 19 (3)
Gii
Ta c:
(1) (2) y
2
z
2
+x (y z) = 9 (y z) (x +y +z) = 9 (4)
(2) (3) x
2
y
2
+z (x y) = 9 (x y) (x +y +z) = 9 (5)
Li c:
(4) (5) [(y z) (x y)] (x +y +z) = 0

x +y +z = 0
y z = x y
Nu : x +y +z = 0 z = (x +y)
Thay vo h ta c:

x
2
+y
2
+xy = 37
x
2
+y
2
+xy = 28
x
2
+y
2
+xy = 19
(v nghim)
205
Nu y z = x y = t :

x = y +t
z = y t
Thay vo (4) ta c:
t (y + y +t +y t) = 9 ty = 3 t =
3
y
(6)
Thay vo (3) ta c:
y
2
+ (y t)
2
+y (y t) = 19 3y
2
3ty +t
2
= 19 3y
2
+ t
2
= 28 (7)
Thay (6) vo (7) ta c:
3y
2
+
9
y
2
= 28 3y
4
28y
2
+ 9 = 0

y
2
= 9 y = 3 t = 1
y
2
=
1
3
y =

3
3
t = 3

3
Xt 4 trng hp (y; t) = (3; 1), (3; 1),

3
; 3

3
; 3

ta tm c nghim
ca h l
(x; y; z) = (4; 3; 2) , (4; 3; 2) ,

10

3
3
;

3
3
;
8

3
3

10

3
3
;

3
3
;
8

3
3

2
Bi tp t luyn
Bi 1:. Gii h phng trnh:

x = 3z
3
+ 2z
2
y = 3x
3
+ 2x
2
z = 3y
3
+ 2y
2
Bi 2: Gii h phng trnh:

2
x
2 = 3y 3
x
2
y
2 = 3x 3
y
( OLP 30-4-2008 T130)
Bi 3: Gii h phng trnh:

x
3
9(y
2
3y + 3) = 0
y
3
9(z
2
3z + 3) = 0
z
3
9(x
2
3x + 3) = 0
(OLP 30-4-2010)
Bi 4: Cho 100 s a
1
, a
2
, a
3
, ..., a
100
tha mn:

a
1
4a
2
+ 3a
3
0
a
2
4a
3
+ 3a
4
0
...........................
a
99
4a
100
+ 3a
1
0
a
100
4a
1
+ 3a
2
0
.
Tnh cc s a
2
, a
3
, ..., a
100
bit a
1
= 1
Bi 5: Gii h phng trnh:

3x y
x 3y
= x
2
3y z
y 3z
= y
2
3z x
z 3x
= z
2
(THTT 12-2010).
206
Bi 6: Gii h phng trnh:

y =
4x
2
4x
2
+ 1
z =
4y
2
4y
2
+ 1
x =
4z
2
4z
2
+ 1
PHNG PHP T N PH TRONG H PHNG
TRNH
Bi 1: Gii h phng trnh


2 (x y) (1 + 4xy) =

3 (1)
x
2
+y
2
= 1 (2)
Gii
t x = cos ; y = sin ta c phng trnh

2(sin cos )(1 + 2 sin 2) =

2.

2 sin( 45
0
).2

1
2
+ sin 2

3
4 sin( 45
0
)(sin 2 + sin 30
0
) =

3
8 sin( 45
0
). sin( + 15
0
) cos( 15
0
) =

3
4 cos( 15
0
)[cos 60
0
cos(2 30
0
)] =

3
2 cos( 15
0
) 4 cos( 15
0
). cos(2 30
0
) =

3
2 cos(3 45
0
) =

= 65
0
+k120
0
= 35
0
+ l120
0
(k, l Z)
Vy h cho c nghim (x; y) = (sin 65
0
; cos 65
0
), (sin 185
0
; cos 185
0
), (sin 305
0
; cos 305
0
),
(sin 85
0
; cos 85
0
), (sin 35
0
; cos 35
0
), (sin 205
0
; cos 205
0
). 2
Bi 2: Gii h phng trnh

(x y)(x
2
y
2
) = 3
(x +y)(x
2
+y
2
) = 15
Gii
Bin i h cho ta thu c

x
3
+ y
3
xy(x +y) = 3
x
3
+ y
3
+xy(x + y) = 15
t

u = x
3
+y
3
v = xy(x +y)
, h cho tr thnh

u +v = 15
u v = 3

u = 9
v = 6
207
Khi , ta c:

x
3
+ y
3
= 9
xy(x +y) = 6

x +y = 3
xy = 2

x = 1
y = 2

x = 2
y = 1
Vy h c nghim (x; y) = (1; 2), (2; 1). 2
Bi 3: Gii h phng trnh

x +
1
y
+

x +y 3 = 3
2x +y +
1
y
= 8
Gii
iu kin:

y = 0
x +
1
y
0
x +y 3 0
()
Bin i h cho tr thnh

x +
1
y
+

x +y 3 = 3
x +
1
y
+x +y 3 = 5
t

u =

x +
1
y
0
v =

x +y 3 0
Khi h cho tr thnh

u +v = 3
u
2
+ v
2
= 5

u + v = 3
uv = 2

u = 2
v = 1

u = 1
v = 2
tha mn ()
Nu

u = 2
v = 1

x +
1
y
=
1
4
x +y 3 = 1

x = 3
y = 1

x = 5
y = 1
(tha ())
Nu

u = 1
v = 2

x +
1
y
= 1
x +y 3 = 4

x = 4

10
y = 3 +

10

x = 4 +

10
y = 3

10
(tha ())
Kt lun: H c nghim (x; y) = (3; 1), (5; 1), (4

10; 3 +

10), (4 +

10; 3

10). 2
208
Bi 4: Gii h phng trnh

4
log
3
(xy)
= 2 + (xy)
log
3
2
x
2
+y
2
3x 3y = 12
Gii
iu kin: xy > 0
t: u = log
3
(xy) xy = 3
u
Khi , h cho tr thnh

xy = 3
x
2
+y
2
3(x + y) = 12

xy = 3
(x + y)
2
3(x +y) 18 = 0

x +y = 6
xy = 3

x +y = 3
xy = 3

x = 3 +

6
y = 3

x = 3

6
y = 3 +

6
Vy h c nghim (x; y) = (3 +

6; 3

6), (3

6; 3 +

6). 2
Bi 5: Gii h phng trnh

x
2
+y
2
+xy = 4y 1
x + y =
y
x
2
+ 1
+ 2
Gii
Nhn thy y = 0. H cho tng ng vi

x
2
+ 1
y
+x +y = 4
x +y =
y
x
2
+ 1
+ 2
t u =
x
2
+ 1
y
, v = x +y. H phng trnh c dng

u +v = 4
v =
1
u
+ 2
Gii h trn ta thu c u = 1, v = 3

x
2
+ 1
y
= 1
x +y = 3

x = 1
y = 2

x = 2
y = 5
Vy h c nghim (x; y) = (1; 2), (2; 5). 2
209
Bi 6: Gii h phng trnh

3
x
2
+y
2
1
+
2y
x
= 1
x
2
+y
2

2x
y
= 4
Gii
iu kin: xy = 0
t a = x
2
+ y
2
1, b =
x
y
vi ab = 0
H cho tr thnh

3
a
+
2
b
= 1
a 2b = 3

a = 1
b = 1

a = 9
b = 3
Nu

a = 1
b = 1

x = 1; y = 1
x = 1; y = 1
Nu

a = 9
b = 3

x = 3; y = 1
x = 3; y = 1
Vy h c nghim (x; y) = (1; 1), (1; 1), (3; 1), (3; 1). 2
Bi 7: Gii h phng trnh sau

x
2
+xy 3x +y = 0
x
4
+ 3x
2
y 5x
2
+y
2
= 0
Gii
Xt x = 0 y = 0. Vy (0; 0) l mt nghim ca h.
Xt x = 0, chia hai v ca phng trnh u cho x, hai v ca phng trnh th hai cho x
2
, ta
c h phng trnh sau

x +
y
x
+y = 3
x
2
+
y
2
x
2
+ 3y = 5

x +
y
x

+y = 3

x +
y
x

2
+ y = 5
t z = x +
y
x
, ta thu c h

z + y = 3
z
2
+y = 5
Gii h ny ta c

z = 2; y = 1
z = 1; y = 4
x = y = 1
Vy h c nghim (x; y) = (0; 0), (1; 1). 2
210
Bi 8: Gii h phng trnh

x
2
y + 2x
2
+ 3y 15 = 0
x
4
+y
2
2x
2
4y 5 = 0
Gii
H phng trnh cho tng ng vi h sau

(x
2
1)(y 2) + 4(x
2
1) + 4(y 2) = 5
(x
2
1)
2
+ (y 2)
2
= 10
t u = x
2
1, v = y 2
H tr thnh

u
2
+v
2
= 10
uv + 4(u +v) = 5

(u +v)
2
2uv = 10
uv + 4(u +v) = 5

u +v = 10
uv = 45

u +v = 2
uv = 3

u = 3
v = 1

u = 1
v = 3
Nu u = 3; v = 1 ta tm c (x; y) = (2; 1) v (x; y) = (2; 1)
Nu u = 1; v = 3 ta tm c(x; y) = (0; 5).
Vy h c nghim (x; y) = (2; 1), (2; 1), (0; 5). 2
Bi 9: Gii h phng trnh sau


2x 1 y

1 + 2

2x 1

= 8
y
2
+ y

2x 1 + 2x = 13
Gii
iu kin: x
1
2
. t t =

2x 1 vi t 0. H phng trnh tr thnh

t y (1 + 2t) = 8
y
2
+yt + t
2
= 12

t y 2ty = 8 (1)
(t y)
2
+ 3ty = 12 (2)
T (1) v (2), suy ra
2(t y)
2
+ 3 (t y) = 0

t y = 0
t y =
3
2
Vi t = y, ta c: t = y = 2. Khi :

2x 1 = 2 x =
5
2
y = 2.
Vi y = t +
3
2
, c 4t
2
+ 6t 13 = 0 t =
3 +

61
4
( do t 0). Khi :
t =
3 +

61
4

y =
3
2
+
3 +

61
4

2x 1 =
3 +

61
4

y =
3 +

61
4
x =
43 3

61
16
211
Vy h c nghim (x; y) =

5
2
; 2

43 3

61
16
;
3 +

61
4

. 2
Bi 10: Gii h phng trnh sau

y + 2 =
3
2
y + 2 (x 2)

x + 2 =
7
4
Gii
iu kin: x 2, y 2
t u =

x + 2, v =

y + 2 vi u, v 0 ()
H cho tr thnh

u
2
v =
7
2
(1)
v
2
+ 2 (u
2
4) u =
1
4
(2)
T (1) v (2), thu c:

u
2

7
2

2
+ 2u
3
8u =
1
4
u
4
+ 2u
3
7u
2
8u + 12 = 0
(u 1) (u 2)

u
2
+ 5u + 6

= 0 u = 1 u = 2
Vi u = 1 thay vo (1) c v =
5
2
, khng tha ().
Vi u = 2 thay vo (1) c v =
1
2
, tha ().
Vy h c nghim l (x; y) =

2;
7
4

. 2
Bi 11: Gii h phng trnh

2x
2
x(y 1) + y
2
= 3y
x
2
+ xy 3y
2
= x 2y
Gii
Xt y = 0 x = 0.
Xt y = 0. t t =
x
y
x = ty. H cho tr thnh

y
2
(2t
2
t + 1) = y(3 t) (1)
y
2
(t
2
+t 3) = y(t 2) (2)
T (2) suy ra

t 2 = 0
t
2
+ t 3 = 0
. Chia (1) cho (2) v theo v ta c
2t
2
t + 1
t
2
+t 3
=
3 t
t 2
3t
3
7t
2
3t + 7 = 0

t = 1
t = 1
t =
7
3
212
H cho c nghim (x; y) = (0; 0), (1; 1), (1; 1),

7
43
;
3
43

. 2
Bi 12: Gii h phng trnh
(I)

x
2
y
4
+ 2xy
2
y
4
+ 1 = 2

2 x

y
2
(1)

x y
2
+ x = 3 (2)
Gii
T (2) ta c: (2)

x y
2
= 3 x

x 3
x
2
+y
2
7x + 9 = 0
T (1) ta c:
(1) 2

(xy
2
+y
2
+ 1) (xy
2
y
2
+ 1) = 2

2 x

y
2

(xy
2
+y
2
+ 1) (xy
2
y
2
+ 1) = 2

y
2
+xy
2

()
t

u = xy
2
+ 1
v = y
2
. Phng trnh (*) tr thnh:

(u +v) (u v) = 2

v 0
u
2
v
2
= 4

2
(II)

v 0
3u
2
8

uv +

45 24

v
2
= 0 ()
Ta thy v = 0 khng l nghim ca h (II) nn
() 3

u
v

2
8

u
v
+

45 24

= 0

u
v
= 3
u
v
= 5
8

2
3

xy
2
+ 1
y
2
= 3
xy
2
+ 1
y
2
= 5
8

2
3

xy
2
+ 1
y
2
= 3 (do u (3

2)v)
xy
2
+ 1 = 3y
2
(x 3) y
2
+ 1 = 0 ( )
T (1) ta c: y
2
= x
2
+ 7x 9 thay vo (***) ta c
(x 3)

x
2
+ 7x 9

+ 1 = 0 x
3
+ 10x
2
30x + 28 = 0

x = 2
x = 4 +

2 (loi)
x = 4

2
Vi x = 2 y
2
= 1 y = 1
Vi x = 4

2 y
2
= 1 +

2 y =

1 +

2
Vy h phng trnh cho c nghim
(x; y) = (2; 1) , (2; 1) ;

2;

1 +

2;

1 +

. 2
213
PHNG PHP BIN I NG THC
Nhiu bi h phng trnh tuy nhn phc tp nhng c th gii bng nhng ng thc n
gin. Mu cht gii nhng bi ton dng ny l ta phi nhn ra quan h gia cc n s, t
lp nn nhng hng ng thc thch hp. Mt s ng thc c th khng quen thuc, nn
phng php ny i hi kinh nghim v s tinh .
Bi tp v d
Bi 1: Gii h phng trnh

x
2
+y
2
= 1 +xy

x
y + 1

2
+

y
x + 1

2
= 1
Gii
tng: H trn tuy l i xng loi I nhng bc kh cao, vic a v S = x+y v P = xy
c th gp nhiu kh khn. Nhng nu ta bit ng thc sau:
x
2
xy +y
2
= 1
x
y + 1
+
y
x + 1
= 1 th bi ton tr nn n gin.
Li gii: KX: x, y = 1
t
x
y + 1
= a,
y
x + 1
= b ta c h phng trnh mi:

a + b = 1
a
2
+b
2
= 1

a = 0
b = 1

a = 1
b = 0
T tm ra nghim ca h l (x; y) = (0; 1); (1; 0) 2
Bi 2: Gii h phng trnh

x
y
+
y
x

1
xy
=
1
x
2
+
1
y
2
1
x
x + 1
+
y
y + 1
=
x
2
xy +y
2
xy
Gii
tng: Cng nh bi 1, vic a v S = x + y v P = xy cn nhiu bin i phc tp v
rc ri. Thc ra bn cht bi ton l t ng thc sau:
xy = 1
1
x + 1
+
1
y + 1
= 1
Ta cng xem k hn v tng to ra bi ton:
u tin ta to ra 1 phng trnh cho tha mn xy = 1 chng hn nh
(xy 1)(x
2
+xy +y
2
) = 0 ()
thm phn tinh t cho bi ton ta bin i (*) thnh x
2
+ y
2
x
2
y
2
= xy(x
2
+y
2
1)
T lp c h phng trnh mi:

x
2
+y
2
x
2
y
2
= xy(x
2
+y
2
1)
1
x + 1
+
1
y + 1
= 1 + (x y)
2
214
Nhng mun che giu bn cht ca bi ton hn na ch cn thng qua php i bin n gin
(x; y) (
1
x
;
1
y
)
Cui cng li h thnh

x
y
+
y
x

1
xy
=
1
x
2
+
1
y
2
1
x
x + 1
+
y
y + 1
=
x
2
xy +y
2
xy
y cng chnh l li gii cho bi 2! 2
Bi 3: Gii h phng trnh

1
y
2 =
2
xy
2
xy (1)
1
(x + 1)
2
+
4
(y + 2)
2
= 1 (2)
Gii
tng: T (1) ta c: (xy 2)(
1
y
2
+ 1) = 0 xy = 2
Nhng nu t y th x =
2
y
vo (2) th s rt phc tp. Vy ta th tm mt quan h gia
1
x + 1
v
2
y + 2
(v hai biu thc ny xut hin (2)). Khi ta li n ng thc sau:
xy = 2
1
x + 1
+
2
y + 2
= 1
n y th mi chuyn tr nn n gin.
Li gii:
KX: x = 1; y = 2
H cho tng ng:

xy = 2
1
(x + 1)
2
+
4
(y + 2)
2
= 1

1
x + 1
+
2
y + 2
= 1
1
(x + 1)
2
+
4
(y + 2)
2
= 1
t a =
1
x + 1
; b =
2
y + 2
(a, b = 0) ta c

a + b = 1
a
2
+b
2
= 1

a = 0; b = 1
a = 1; b = 0
(loi)
Vy h v nghim. 2
Bi 4: Gii h phng trnh

v
2
+ 1
u
2

1 + y
2
v
2
+ v
2
u
2

u
2
1 + v
2

v
2
1 + u
2
+
1
v
2
u
2

= 9u
2
(1)
u
2
+ v
2
= 1 (2)
Gii
215
tng:
Ta c mnh sau: (
b c
a
+
c a
b
+
a b
c
)(
a
b c
+
b
c a
+
c
a b
) = 9 a +b +c = 0 ()
* Chng minh:
Xut pht t ng thc:
x y
z
+
y z
x
+
z x
y
=
(x y)(y z)(z x)
xyz
t x = b c, y = c a, z = a b x y = 3c, y z = 3a, z x = 3b
Th th
3a
b c
+
3b
c a
+
3c
a b
=
27abc
(b c)(c a)(a b)
ng thc trn tng ng vi iu phi chng minh.
p dng mnh trn ta c li gii sau:
Li gii:
p dng () vi a = u
2
, b = v
2
, c = 1 t (1) ta suy ra 9 = 9u
2
.
Vy ta c h tng ng:

9 = 9u
2
u
2
+v
2
= 1

u = 1
v = 0
Vy h c nghim (u; v) = (1; 0); (1; 0) 2
Bi 5: Gii h phng trnh

x + y +xy = 3 (1)
4
5y + 9
+
4
x + 6
+
1
1 + (x + 1)(y + 2)
=
x + 1
2
(2)
Gii
tng: T (1) ta n cch phn tch quen thuc l (x + 1)(y + 1) = 4.
(2) cng c x +1 v y +1 nn ta ngh ti t a = x +1; b = y +1. Tuy nhin mu cht ca
bi ton chnh l t c =
1
4
t (1) c abc = 1.
Khi ta li c mnh sau: abc = 1
1
1 + a + ab
+
1
1 + b +bc
+
1
1 + c +ca
= 1.
T ta i n li gii:
Li gii:
t x + 1 = a, y + 1 = b,
1
4
= c ta c h phng trnh

abc = 1
1
1 + a +ab
+
1
1 + b +bc
+
1
1 + c +ca
=
x + 1
2
Li c:
1
1 + a + ab
+
1
1 + b +bc
+
1
1 + c +ca
=
1
1 + a +ab
+
a
a +ab +abc
+
ab
ab +abc +a
2
bc
= 1
Thay vo phng trnh sau c 1 =
x + 1
2
x = 1 y = 1
Vy h c nghim duy nht l (x; y) = (1; 1) 2
Bi 6: Gii h phng trnh sau:

x
2
+y
2
= 2
(x +y)(1 + xy)
4
= 32
216
Gii
tng: R rng khng th gi nguyn (1 + xy)
4
. Mun vy ta ch ti ng thc
xy + 1 =
2xy + 2
2
=
(x + y)
2
2
. n y bi ton tr nn n gin.
Li gii:
H cho c vit li thnh:

x
2
+y
2
= 2
(x +y)(2 + 2xy)
4
= 2
9
Thay x
2
+ y
2
= 2 vo phng trnh th 2 ta c
(x +y)(x
2
+ y
2
+ 2xy)
4
= 2
9
(x + y)
9
= 2
9
x + y = 2
Vy ta c h

x
2
+ y
2
= 2
x +y = 2
x = y = 1
Vy nghim ca h l (x; y) = (1; 1) 2
Nhn xt: tng chnh ca bi ton chnh l dng php th a v ng thc ng
bc. Khi vic gii s d dng hn. Ta xt mt s v d tng t:
Bi 7: Gii h phng trnh

x
2
+y
2
= 2
(x +y)(4 x
2
y
2
2xy) = 2y
5
Gii
tng: Xt phng trnh th hai. x +y c bc nht, 4 x
2
y
2
2xy c bc 4 nhng
cc hng t cha ng bc. V vy ta ngh ti php th t phng trnh u to biu thc
thun nht.
Li gii: Thay phng trnh th nht vo phng trnh th 2 ta c:
(x + y)

(x
2
+y
2
)
2
xy(x
2
+y
2
) x
2
y
2

= 2y
5
(x + y)

x
4
+ y
4
+ x
2
y
2
xy(x
2
+y
2
)

= 2y
5
x
5
+ y
5
= 2y
5
x = y
Kt hp vi phng trnh u ta tm c nghim ca h l (x; y) = (1; 1), (1; 1) 2
Bi 8: Gii h phng trnh sau

3x
3
y
3
=
1
x +y
(1)
x
2
+y
2
= 1 (2)
Gii
tng: Nhn vo h ta d dng ngh n vic th s 1 t phng trnh (2) vo phng
trnh (1) c phng trnh thun nht.
Li gii:
KX: x +y = 0
Phng trnh (2) tng ng vi (x
2
+y
2
)
2
= 1
217
Th vo phng trnh (1) ta c:

3x
3
y
3

(x +y) =

x
2
+y
2

2
3x
4
y
4
+ 3x
3
y xy
3
= x
4
+y
4
+ 2x
2
y
2
2x
4
+ 3x
3
y 2x
2
y
2
xy
3
2y
4
= 0
(x y) (x + 2y)

2x
2
+xy +y
2

= 0

x = y
x = 2y
2x
2
+xy + y
2
= 0
Vi 2x
2
+xy +y
2
= 0 x = y = 0 (khng tho x
2
+y
2
= 1). Trng hp ny loi.
Vi x = y ta c:
2x
2
= 1 x
2
=
1
2

x = y =
1

2
x = y =
1

2
Vi x = 2y ta c:
5y
2
= 1 y
2
=
1
5

y =
1

5
; x =
2

5
y =
1

5
; x =
2

5
Kt lun: H c nghim (x; y) l

5
;
1

5
;
1

2
;
1

2
;
1

2
Bi 9: Gii h phng trnh sau

x
3
+ 4y y
3
16x = 0
y
2
= 5x
2
+ 4
Gii
tng: H phng trnh c th vit li nh sau:

x
3
y
3
= 4(4x y) (1)
y
2
5x
2
= 4 (2)
T ta ngh n vic th (2) vo (1) c phng trnh thun nht.
Li gii:
H phng trnh tng ng:

x
3
y
3
= 4 (4x y)
y
2
5x
2
= 4

x
3
y
3
=

y
2
5x
2

(4x y)
y
2
5x
2
= 4

21x
3
5x
2
y 4xy
2
= 0
y
2
5x
2
= 4

x (7x 4y) (3x + y) = 0


y
2
5x
2
= 4

x = 0
y =
7x
4
y = 3x
y
2
5x
2
= 4
Vi x = 0 ta c y = 2
Vi y =
7x
4

7x
4

2
5x
2
= 4
31
15
x
2
= 4 (V nghim)
218
Vi y = 3x (3x)
2
5x
2
= 4 x
2
= 1

x = 1; y = 3
x = 1; y = 3
Kt lun: H c nghim (x; y) l (0; 2) ; (0; 2) ; (1; 3) ; (1; 3) 2
Bi 10: Gii h phng trnh (I)

x
3
8x = y
3
+ 2y
x
2
3y
2
= 6
( H Tnh TST 07-08)
Gii
H phng trnh c th vit li nh sau:

x
3
y
3
= 2 (4x +y) (1)
x
2
3y
2
= 6 (2)
Ta ngh n cch ng bc (1) bng cch dng php th t (2). Nhng trc ta phi lm
xut hin s 6 (1) nn bin i:
(I)

x
3
y
3

= 6 (4x + y)
x
2
3y
2
= 6

x
3
y
3

x
2
3y
2

(4x +y)
x
2
3y
2
= 6

x
3
+x
2
y 12xy
2
= 0
x
2
3y
2
= 6

x (x 3y) (x + 4y) = 0
x
2
3y
2
= 6

x = 0
x = 3y
x = 4y
x
2
3y
2
= 6
Vi x = 0 y
2
= 2 (v nghim)
Vi x = 3y (3y)
2
3y
2
= 6 y
2
= 6

y = 1; x = 3
y = 1; x = 3
Vi x = 4y (4y)
2
3y
2
= 6 y
2
=
6
13

y =

6
13
; x = 4

6
13
y =

6
13
; x = 4

6
13
Kt lun: H c nghim (x; y) l (3; 1) ; (3; 1) ;

6
13
;

6
13

6
13
;

6
13

2
Bi 11: Gii h phng trnh

x
2
+y
2
+xy = 3
x
5
+y
5
+ 15xy(x +y) = 32
Gii
tng: Ta cn tm mi quan h gia cc hng t trong h. Mun vy ta ch ti hng
ng thc (x +y)
5
= x
5
+y
5
+ 5xy(x +y)(x
2
+xy + y
2
).
Li gii:
H cho c vit li nh sau:

x
2
+y
2
+xy = 3
x
5
+y
5
+ 5.3xy(x +y) = 32

x
2
+y
2
+xy = 3
x
5
+y
5
+ 5xy(x
2
+ xy +y
2
)(x +y) = 32
()
Cng vic tip theo tr nn n gin:
()

x
2
+ y
2
+ xy = 3
x +y = 2

x +y = 2
xy = 1
x = y = 1
219
Vy h c nghim (x; y) = (1; 1) 2
Bi 12: Gii h phng trnh

1
x

1
2y
= 2

y
4
x
4

1
x
+
1
2y
=

x
2
+ 3y
2

3x
2
+y
2

Gii
tng: Khai trin v phi ca hai phng trnh. gii bi ton ta ch cn ch cng
thc khai trin bc 5:

x
5
+ 5x
4
y + 10x
3
y
2
+ 10x
2
y
3
+ 5xy
4
+ y
5
= (x +y)
5
x
5
5x
4
y + 10x
3
y
2
10x
2
y
3
+ 5xy
4
y
5
= (x y)
5
Li gii:
H tng ng

1
x
+
1
2y
= 3(x
4
+ y
4
) + 10x
2
y
2
1
x

1
2y
= 2(y
4
x
4
)
Cng tr 2 phng trnh ta c:

2
x
= 5y
4
+x
4
+ 10x
2
y
2
1
y
= 5x
4
+y
4
+ 10x
2
y
2

2 = 5xy
4
+ x
5
+ 10x
3
y
2
1 = 5yx
4
+ y
5
+ 10x
2
y
3
Li cng tr 2 phng trnh ta c:

(x +y)
5
= 3
(x y)
5
= 1
Vy h phng trnh cho c 1 nghim l: (x; y) =

3 + 1
2
;
5

3 1
2

2
Sau y l mt s li gii bng cch ghp hng ng thc to ra nhng i lng khng m.
Bi 13: Gii h phng trnh

x
2
4

3x 2 + 10 = 2y
y
2
6

4y 3 + 11 = x
Gii
H cho tng ng vi:

(x 2)
2
+ (

3x 2 2)
2
+ x + 4 = 2y
(y 3)
2
+ (

4y 3 3)
2
+ 2y = x + 4
Cng 2 phng trnh trn v theo v ta thu c:
(x 2)
2
+ (y 3)
2
+ (

3x 2 2)
2
+ (

4y 3 3)
2
= 0
T tm c nghim ca h l (x; y) = (2; 3) 2
Bi 14: Gii h phng trnh

1
x
+
1
y
+
1
z
= 2 (1)
2
xy

1
z
2
= 4 (2)
220
Gii
Ta c:
(1)

1
x
+
1
y
+
1
z

2
= 4

1
x
+
1
y
+
1
z

2
=
2
xy

1
z
2

1
x
2
+
1
y
2
+
1
z
2
+
2
xy
+
2
yz
+
2
zx
=
2
xy

1
z
2

1
x
2
+
2
xz
+
1
z
2

1
y
2
+
2
yz
+
1
z
2

= 0

1
x
+
1
z

2
+

1
y
+
1
z

2
= 0

1
x
=
1
z
1
y
=
1
z
x = y = z
Th vo h ta tm c nghim (x; y; z) =

1
2
;
1
2
;
1
2

2
Bi 15: Gii h phng trnh

(2 x) (1 2x) (2 +y) (1 + 2y) = 4

10z + 1 (1)
x
2
+y
2
+z
2
+ 2xz + 2yz +x
2
y
2
+ 1 = 0 (2)
Gii
Ta c:
(2) (x + y +z)
2
+(xy 1)
2
= 0

x +y +z = 0
xy 1 = 0

z = (x +y)
y =
1
x

z =

x +
1
x

y =
1
x
Thay vo (1), ta c:
(2 x) (1 2x)

2 +
1
x

1 +
2
x

= 4

1 10

x +
1
x

(2 x) (1 2x)

2x + 1
x

x + 2
x

= 4

1 10

x +
1
x

(4 x
2
) (1 4x
2
)
x
2
= 4

1 10

x +
1
x

x
2
+
1
x
2

17 = 4

1 10

x +
1
x

(3)
t: t = x +
1
x

[t[ 2
x
2
+
1
x
2
= t
2
2
.
T ta c:
(3) 4

t
2
2

17 = 4

1 10t
4t
2
25 = 4

1 10t

4t
2
25

2
16 (1 10t) = 0

4t
2
20t + 29

(2t + 3) (2t + 7) = 0
t =
7
2
(do [t[ 2) x +
1
x
=
7
2
2x
2
+ 7x + 2 = 0 x =
7

33
4
221
Vi x =
7 +

33
4
y =
7

33
4
z =
7
2
Vi x =
7

33
4
y =
7 +

33
4
z =
7
2
Vy h phng trnh cho c 2 nghim l
(x; y; z) =

7 +

33
4
;
7

33
4
;
7
2

33
4
;
7 +

33
4
;
7
2

2.
Bi tp t luyn
Gii h phng trnh:
1)

xy x = 2
1
(x + 1)
4
+
16
(y + 1)
4
= 1
2)

x
3
+ 5xy
2
3y
3
= 2x y
x
2
+ 2xy = 1
3)

x
5
+y
5
x
3
+y
3
=
31
7
x
2
+ xy +y
2
= 3
4)

x
3
+y
3
xy
2
= 1
4x
4
+y
4
4x y = 0
5)

x
2
+y
2
= 4
8x
2
+x
5
= 8y
2
+y
5
222
PHNG PHP DNG N IU HM S
L thuyt
nh l 1: Nu hm s y=f(x) lun ng bin (hoc lun ngch bin) th s nghim ca
phng trnh : f(x) = k khng nhiu hn mt v f(x) = f(y) x = y.
nh l 2: Nu hm s y=f(x) lun ng bin (hoc lun nghch bin) v hm s y=g(x)
lun ngch bin (hoc lun ng bin) trn D th s nghim trn D ca phng trnh: f(x)=g(x)
khng nhiu hn mt.
nh l 3: Cho hm s y=f(x) c o hm n cp n v pt f
(k)
(x) = 0 c m nghim, khi
pt f
(k1)
(x) = 0 c nhiu nht l m+1 nghim.
Nh vy, bn cht ca vn chnh l vic phn tch
g(x) = h(y) f(p(x)) = f(q(y))
Sau ta s xt tnh ng bin , nghch bin ca hm c trng f(x). Trong tin cho
vic nh gi th ta thng bin i hm c trng f(x) thnh mt hm a thc .V c th
lm c iu th chng ta cn s dng , kt hp kho lo 2 k thut t n ph v h s
bt nh.
Ch : Hm s c trng f(x) khng c tnh duy nht , tc l nu t mt phng trnh no
ta rt ra c 1 hm s c trng th chc chn cn c th tm c hm s th 2, v vn
ca chng ta l lm sao c th tm c 1 hm s n gin v mt tng , cng nh n
gin trong vic nh gi ( xt tnh n iu ) ca n trong bi ton.
Bi tp v d
Bi 1: Gii h phng trnh:
(I)

tan x tan y = y x (1)

y + 1 1 =

y + 8 (2)
( Olympic 30-4 nm 2005)
Gii
tng: Ta thy bi ton dng ny rt c trng cho phng php c nu v cch ra
ny cng thng hay c s dng trong cc thi C H, thi HSG. iu quan trng l
cn chng minh c x = y t h trn.
Li gii:
K:

1 y

y + 8 x
T phng trnh ban u ta c : tan x +x = tan y +y
Nh vy hm c trng ca phng trnh l f(t) = tant +t.
Li c f

(t) =
1
cos
2
t
+ 1 > 0 nn f(t) l mt hm ng bin trn tp xc nh, suy ra x = y.
Th vo (2) ta c:
223

y + 1 1 =

y + 8

y + 1 =

y + 8 + 1 y + 8 = 4y 4

y + 8
3y 8 = 4

y + 8

y
8
3
9y
2
48y + 64 = 16y + 128
y = 8
Vy x = y = 8 l nghim duy nht ca (I) 2.
Nhn xt: bi ton trn bn cht ca vn c th hin rt r rng ngay phng
trnh u tin. Nn c th nhn xt rng y l mt bi ton d trong vic nhn ra tng
ca phng php.
Bi 2: Gii h phng trnh (II)

(x +

1 + x
2
)(y +

1 + y
2
) = 1 (1)
x

6x 2xy + 1 = 4xy + 6x + 1 (2)


Gii
tng: Nhn qua ta thy (2) c v d khai thc hn thng qua vic bnh phng
tm quan h gia x, y. Nhng tht ra mu cht ca bi ton li l (1). Ch cn rng
1
y +

y
2
+ 1
= y +

y
2
+ 1 th ta gii quyt xong.
Li gii:
KX: 6x 2xy + 1 0 ()
Ta c: (1) x +

x
2
+ 1 = y +

y
2
+ 1 f(x) = f(y) vi f(t) = t +

t
2
+ 1
Li c: f

(t) =

t
2
+ 1 + t

t
2
+ 1
>
[t[ t

t
2
+ 1
0 do f(t) ng bin trn R.
Vy f(x) = f(y) x = y. Th vo (2) ta c:
x

6x + 2x
2
+ 1 = 4x
2
+6x+1

2x
2
+ 6x + 1
x
2

2
=
25x
2
4

2x
2
+ 6x + 1 = 3x

2x
2
+ 6x + 1 = 2x
Nu

2x
2
+ 6x + 1 = 3x

2x
2
+ 6x + 1 = 9x
2
x 0
x = 1 y = 1
Nu

2x
2
+ 6x + 1 = 2x

2x
2
+ 6x + 1 = 4x
2
x 0
x =
3

11
2
y =
3 +

11
2
Th li () ta thy (II) c nghim (x; y) = (1; 1); (
3

11
2
;
3 +

11
2
) 2
Bi 3: Gii h phng trnh (III)

3x 1 + 4 (2x + 1) =

y 1 + 3y (1)
(x + y) (2x y) + 4 = 6x 3y (2)
Gii
tng: Ta thy (1) c dng n iu, nhng khng th tm c hm s c trng. Nh
vy r rng ta phi thay i x hoc y. Chuyn qua phn tch (2) th thy rng
(2) (x +y +1)(2x y +4) = 0. Vy ta tnh c y theo x, t vic xy dng hm c
trng cho (1) tr nn d dng v ch cn mt n.
Li gii:
224
KX: x
1
3
, y 1 ().
Ta c:
(2) (x + y)(2x y) + 4 = 4(x +y) (2x y) (x +y + 1)(2x y + 4) = 0
y = 2x + 4 ( do () x +y + 1
7
3
> 0). Thay vo (1) ta c:

3x 1 + 2x 8 =

2x + 3 2 (3x 1) +

3x 1 = 2 (2x + 3) +

2x + 3
Xt hm s f (t) = 2t
2
+t vi t 0. Phng trnh (12) c th vit li thnh
f

3x 1

= f

2x + 3

Ta c f

(t) = 4t + 1 > 0 t > 0 nn hm s f ng bin trn (0; +). Do

3x 1 =

2x + 3 x = 4 y = 12.
Vy (III) c nghim (x, y) = (4, 12) 2
Bi 4: Gii h phng trnh (IV )

x
3
+ x 2 = y
3
+ 3y
2
+ 4y (1)
x
5
+ y
3
+ 1 = 0 (2)
Gii
tng: Rt t nhin ta nhn vo tng PT nh gi vi mc ch tm mi quan h gia
hai bin.
T (1) ta thy rng 2 v l 2 a thc c lp ca 2 bin x ,y v cng bc. Nh vy vic p
dng phng php s dng tnh n iu c c hi thnh cng rt cao. V y cng l lc
chng ta dng ti k thut h s bt nh.
u tin, ta chn mt a thc bt k lm chun (1). D thy nn chn a thc bn v tri
v nhn n n gin hn. Vi tng ta c hm s c trng f(t) = t
3
+t 2 , nh vy
vic ca chng ta cn lm l phn tch :
y
3
+ 3y
2
+ 4y = g
3
(y) + g(y) 2
R rng g(y) c dng g(y) = y + b t y ta khai trin v c b = 1
Nh vy ta c phng trnh x
3
+ x 2 = (y + 1)
3
+ (y + 1) 2 ti y th tng gii bi
ton c hon thin.
Li gii:
T (1) ta c
x
3
+ x 2 = (y + 1)
3
+ (y + 1) 2 ()
Xt hm s c trng: f(t) = t
3
+ t 2 c f

(t) = 3t
2
+ 1 > 0, t R
Suy ra f(t) ng bin trn R , Kt hp vi (*) ta c x = y + 1
Th vo (2) suy ra:
x
5
+ (x 1)
3
+ 1 = 0 x
5
+x
3
3x
2
+ 3x = 0
x(x
4
+x
2
3x + 3) = 0

x = 0
x
4
+x
2
3x + 3 = 0 (v nghim)
225
Vy (IV ) c nghim duy nht (x; y) = (0, 1). 2
Bi 5: Gii h phng trnh (V )

(4x
2
+ 1)x + (y 3)

5 2y = 0 (1)
4x
2
+ y
2
+ 2

3 4x = 7 (2)
(H khi A
2010)
Gii
tng: Kh c th khai thc g t (2), do ta s bt u phn tch t (1). Phng trnh
ny c x, y tch bit nn kh nng dng n iu l cao. Vy ta bin i phng trnh v dng
g(x) = h(y) (4x
2
+ 1)x = (3 y)

5 2y
t

5 2y = a y =
5 a
2
2
v (3 y)

5 2y = (3
5 a
2
2
)a =
1 + a
2
2
.a =
a
3
2
+
a
2
Ta hi vng rng f(t) =
t
3
2
+
t
2
chnh l hm c trng m ta cn tm. Vy cn phi phn tch
(4x
2
+ 1).x = 4x
3
+ x =
p(x)
3
2
+
p(x)
2
. R rng p(x) c dng mx + n , dng h s bt nh ta
thu c m = 2; n = 0 p(x) = 2x. Nh vy hm s c trng chnh l f(t) =
t
3
2
+
t
2
Li gii: K: x
3
4
; y
5
2
.
Ta c
(1) (4x
2
+ 1)x = (3 y)

5 2y
(2x)
3
2
+
2x
2
=

(5 2y)
3
2
+

5 2y
2
Xt hm s c trng : f(t) =
t
3
2
+
t
2
c f

(t) =
3t
2
2
+
1
2
> 0, t R
Suy ra hm f(t) l hm ng bn trn R , hay : 2x =

5 2y

x 0
4x
2
= 5 2y
Ti y ta th vo (2) ta c :
4x
2
+ (
5
2
2x
2
)
2
+ 2

3 4x 7 = 0
Xt hm s g(x) = 4x
2
+(
5
2
2x
2
)
2
+2

3 4x7 c g

(t) > 0, x
3
4
suy ra g(x) ng bin
trn TX , li c g(
1
2
) = 0 x =
1
2
y = 2
Vy V c nghim (x; y) = (
1
2
; 2) 2.
Bi 6: Gii h phng trnh (V I)

x
5
+xy
4
= y
10
+ y
6
(1)

4x + 5 +

y
2
+ 8 = 6(2)
Gii
tng: Nhn vo h trn ta thy rng khng th dng php th. Phng trnh 1 c 2 n
khng c lp vi nn ta ngh ti vic th nhm li v phn tch nhn t, li c x y
2
l mt
nhn t chung, tuy vy vic phn tch ra l tng i phc tp khi phi dng hng ng thc
226
lin quan ti A
5
B
5
.
Ta cng xt cch khc n gin hn. Vi d on x = y
2
l mi quan h duy nht ca x ,y ta
ngh ti dng n iu. Hy th nh gi:
(1) c hng t xy
4
m trong vic xt hm c trng ta li ch c th xt hm mt n nh vy
mt iu rt t nhin l ta tm cch bin i sao cho (1) tr thnh PT c 2 n tch bit. V
chia 2 v cho y
5
l cch duy nht gip chng ta gii quyt c tt c vn nu trn. Ta
cng i n li gii chi tit:
Li gii:
K: x
5
4
D thy y = 0 khng tho h. Vi y= 0 chia c 2 v ca PT(1) cho y
5
ta c:
(1)
x
5
y
5
+
x
y
= y
5
+ y ()
Xt hm s c trng f(t) = t
5
+ t c f

(t) = 5t
4
+ 1 > 0, t R
Suy ra f(t) ng bin trn R , kt hp vi (*) ta c:
x
y
= y x = y
2
Th vo PT(2) ta c

4x + 5 +

x + 8 = 6 ()
Ta c x =
5
4
khng tho (**). t V T() = g(x).
Ta c g

(x) =
2

4x + 5
+
1
2

x + 8
> 0 x >
5
4
do g(x) ng bin trn (
5
4
; +)
Li c g(1) = 6 nn (**) c nghim duy nht x = 1. Suy ra y = 1.
Vy (V I) c nghim (x; y) = (1 : 1); (1; 1) 2
Nhn xt: Vi nhng phng trnh c bc cao th vic h bc v t n ph rt hu ch. Ta
cng xem qua mt s bi tng t:
Bi 7: Gii h phng trnh (V II)

x
3
(4y
2
+ 1) + 2 (x
2
+ 1)

x = 6 (1)
x
2
y

2 + 2

4y
2
+ 1

= x +

x
2
+ 1 (2)
Gii
KX: x 0.
Nu x = 0, t phng trnh th hai ca h ta c 0 = 6 (sai). Vy x > 0, chia c hai v ca (2)
cho x
2
ta thu c
2y

1 +

4y
2
+ 1

=
1
x

1 +

1
x
2
+ 1

(3)
Xt hm s f (t) = t

1 +

1 + t
2

vi t R, phng trnh (3) c th vit li thnh


f(2y) = f(
1
x
)
Ta c f

(t) = 1 +
2t
2
+ 1

t
2
+ 1
> 0 t R, do 2y =
1
x
.Thay vo (1) ta c
x
3
+ x + 2

x
2
+ 1

x 6 = 0 x
3
+x 6 = 2

x
2
+ 1

x (4)
Xt cc hm s g (x) = x
3
+ x 6, h(x) = 2 (x
2
+ 1)

x vi x (0; +).
D thy g(x), h(x) n iu ngc chiu trn (0; +) v g(1) = h(1) nn (4) c nghim duy
227
nht x = 1 y =
1
2
.
Vy (V II) c nghim (x, y) =

1,
1
2

2.
Bi 8: Gii h phng trnh (V III)

x
2
+ 1 3x
2
y + 2

4y
2
+ 1 + 1

= 8x
2
y
3
x
2
y x + 2 = 0
Gii
Vi x = 0 hoc y = 0 th h v nghim.
Vi x = 0 v y = 0.Phng trnh (1) tng ng vi

x
2
+ 1 4x
2
y + x

4y
2
+ 1 1
.4y
2
= 8x
2
y
3

x
2
+ 1 4x
2
y +x

4y
2
+ 1 1
= 2x
2
y

x
2
+ 1 4x
2
y +x = 2x
2
y

4y
2
+ 1 2x
2
y

x
2
+ 1 + x = 2x
2
y

4y
2
+ 1 + 1

1
x

1
x
2
+ 1 + 1

= 2y

(2y)
2
+ 1 + 1

(3)
Xt hm s: f(t) = t

t
2
+ 1 + 1

c f

(t) =

t
2
+ 1+
t

t
2
+ 1
+1 =
t
2
+t + 1 +

t
2
+ 1

t
2
+ 1
> 0
Nn hm s f(t) = t

t
2
+ 1 + 1

ng bin trn R
T phng trnh (3) ta c
f

1
x

= f (2y)
1
x
= 2y 2xy = 1
Th vo phng trnh (2) ta c:
2x
2
y 2x + 4 = 0 x 2x + 4 = 0

x = 4
y =
1
8
Kt lun: H c nghim (x; y) =

4;
1
8

2
Bi 9: Gii h phng trnh (IX)

x
3
3x
2
+ 2 =

y
3
+ 3y
2
(1)
3

x 2 =

y
2
+ 8y (2)
Gii
tng: Chng ta li bt u tm ti t ci n gin ti phc tp
T (1) rng ta c dng g(x) = h(y) nh mong mun nh vy tng dng tnh n
iu xt hm c trng xut hin. V s tt hn nu g(x), h(y) l hm a thc. Vy ta
th bnh phng loi b cn thc:
(1) (x
3
3x
2
+ 2)
2
= y
3
+ 3y
2
Cng vic tip theo l tm hm c trng. D thy h(y) = y
3
+ 3y
2
l la chn tt v y l
hm s n gin v ng bin trn [0; +].
228
Ta s c gng phn tch (x
3
3x
2
+ 2)
2
= q
3
(x) + 3q
2
(x) . ng nht h s s tm c
q(x) = x
2
2x 2. Suy ra x
2
2x 2 = y (ch iu kin c nghim l
x
3
3x
2
+ 2 = (x 1)(x
2
2x 2) 0 x
2
2x 2 0 do x 2).
Nhng cu hi t ra l, vic khai trin v ng nht h s vi (x
3
3x
2
+ 2)
2
kh phc tp.
Li ch rng hm s c trng khng phi l duy nht. Liu c mt hm s no n gin
hn? Vy 1 iu t nhin l ta s i tm cch t n ph : 1 hm cha cn no khng
phi lu tha.
rng (1) x
3
3x
2
+ 2 =

y
3
+ 3y
2
x
3
3x
2
+ 2 = y

y + 3
Nh vy ta s t a =

y + 3 y = a
2
3 , y

y + 3 = (a
2
3)a = a
3
3a
Phn tch hon ton tng t nh cc v d trc , ta c (x 1)
3
3(x 1) = (

y + 3)
3

y + 3 v hm c trng f(t) = t
3
3t l hm ng bin trn [1; +] .Nh vy tng
r rng.
Li gii:
KX: x 2; y 0
Ta c:
(1) x
3
3x
2
+ 2 = y

y + 3 (x 1)
3
3(x 1) = (

y + 3)
3
3

y + 3
Xt hm c trng f(t) = t
3
3t c f

(t) = 3t
2
3 0, t 1 ( v

y + 3

3; x 1 1 )
Suy ra hm s ng bin trn [1; +] , hay x 1 =

y + 3 y = x
2
2x 2
Th vo (2):
(2) 9(x2) = y
2
+8y 9(x2) = (x
2
2x 2)
2
+8(x
2
2x2) x
4
4x
3
+8x
2
17x+6 = 0
(x 3)(x
3
x
2
+ 5x 2) = 0

x = 3
x
3
x
2
+ 5x 2 = 0
Xt Q(x) = x
3
x
2
+5x 2 c Q

(x) = 3x
2
2x +5 > 0, x suy ra y l mt hm ng bin
trn R
Li c x 2 Q(x) Q(2) = 13 > 0 suy ra phng trnh Q(x) = 0 v nghim.
Vy (IX) c nghim (x; y) = (3; 1) 2.
Nhn xt: Nhng bi ton trn cho thy c nhiu cch a hai v ca mt phng
trnh v hm c trng. Tuy nhin mt s bi ton kh hn s i hi phi bin i cc phng
trnh ca h tm ra hm c trng.
Bi 10: Gii h phng trnh (X)

x
3
(2x + 3y) = 1
x(y
3
2) = 1
Gii
Vi x = 0 d thy khng phi l nghim ca h trn.
Vi x = 0 ta c
(X)

x
3
(2 + 3y) = 1
x
3
(y
3
2) = 3x
2
x
3
(2 + 3y +y
3
2) = 3x
2
+ 1 y
3
+ 3y =
1
x
3
+
3
x
Ti y hm s c trng l r: f(t) = t
3
+ 3t
229
C f

(t) = 3t
2
+ 3 > 0, t R suy ra f(t) l hm s ng bin trn R y =
1
x
ti y ta ch
vic th vo mt PT bt k trong 2 PT th bi ton s c hon tt :
(X)

y =
1
x
x
3
(2 +
3
x
) = 1

y =
1
x
2x
3
+ 3x
2
1 = 0

x = 1, y = 1
x =
1
2
, y = 2
Th li ta thy (X) c nghim (x; y) = (1; 1); (
1
2
; 2) 2.
Sau y ta cng xem qua mt s bi h phng trnh logarit:
Bi 11: Gii h phng trnh
(XI)

log
3
(2x + 1) log
3
(x y) + 1 =

4x
2
+ 4x + 2

(x y)
2
+ 1 + (x y)
2
4x(x + 1) (1)
log
3
(2y 2) + 4x
2

4x
2
+ 1 = 1

2 (2)
Gii
tng: Phng trnh (1) nhn phc tp hn nhng kh nng nhm nhn t li cao, nn
ta s phn tch (1). D thy biu thc trong log l khng th thay i, do ta s ly 2x + 1
v x y lm chun. Vic xy dng hm c trng bng h s bt nh ging nh nhng bi
trn, nn y ta s khng nhc li.
Li gii:
Ta c:
(1)

(2x + 1)
2
+ 1 (2x + 1)
2
log
3
(2x + 1) =

(x y)
2
+ 1 (x y)
2
log
3
(x y) ()
Xt f(t) =

t
2
+ 1 t
2
log
3
t (t > 0) ta c f

(t) =
t

t
2
+ 1

2t +
1
t

2
2

2 < 0
nn f(t) nghch bin trn (0; +).
Do () 2x + 1 = x y x = y 1
Khi (2) log
3
(2x) + 4x
2

4x
2
+ 1 = 1

2.
Xt g(x) = log
3
(2x) + 4x
2

4x
2
+ 1 (x > 0) ta c g

(x) = 4x(2
1

4x
2
+ 1
) +
1
x
> 0 nn
g(x) ng bin. Li c g

1
2

= 1

2 nn (2) c nghim duy nht x =


1
2
y =
3
2
Vy (IX) c nghim (x; y) = (
1
2
;
3
2
) 2.
Bi 12: Gii h phng trnh

e
y
2
x
2
=
x
2
+ 1
y
2
+ 1
3log
2
(x + 2y + 6) = 2log
2
(x +y + 2) + 1
Gii
KX:

x + 2y + 6 > 0
x + y + 2 > 0
Xt hm s: f(t) = e
t
(t + 1), t [0, +). Ta c f

(t) = e
t
(t + 1) + e
t
= e
t
(t + 2) > 0 nn y
l hm ng bin.
Do :
e
y
2
x
2
=
x
2
+ 1
y
2
+ 1
e
x
2
(x
2
+ 1) = e
y
2
(y
2
+ 1) f(x
2
) = f(y
2
) x
2
= y
2
x = y
230
Phng trnh th hai ca h tng ng vi:
3log
2
(x + 2y + 6) = 2log
2
(x +y + 2) + 1
log
2

(x + 2y + 6)
3

= log
2

2(x +y + 2)
2

(x + 2y + 6)
3
= 2(x +y + 2)
2
()
Xt hai trng hp:
Nu x = y th thay vo (*), ta c: (3x + 6)
3
= 2(2x + 2)
2
.
Theo KX ta c x > 1 .Li c
(3x + 6)
3
2(2x + 4)
2
= (x + 2)
2
(27x + 46) > 0 (3x + 6)
3
> 2(2x + 4)
2
Do : (3x + 6)
3
> 2(2x + 4)
2
> 2(2x + 2)
2
nn (*) v nghim.
Nu x = y, thay vo (*), ta c:
(x + 6)
3
= 2(2)
2
(6 x)
3
= 8 6 x = 2 x = 4 y = 4
Th li ta thy h cho c nghim duy nht l (x, y) = (4, 4) 2
Nhn xt: Trong bi ny nn ch cc nh gi trong trng hp x = y, bi v khi
phng trnh bc ba thu c phi gii theo cng thc tng qut, iu thng b trnh cc
k thi HSG. Do , vic tm mt nh gi thch hp chng minh nghim khng tha bi
l mt cch rt hay.
Bi tp t luyn
Gii cc h phng trnh sau:
Bi 1)

(8x 3)

2x 1 y 4y
3
= 0
4x
2
8x + 2y
3
+y
2
2y + 3 = 0
Bi 2)

x
3
(3y + 55) = 64
xy(y
2
+ 3y + 3) = 12 + 51x
Bi 3)

2x
3
4x
2
+ 4x 1 = 2x
3
(2 y)

3 2y

x + 2 =
3

14 x

3 2y + 1
Bi 4)


x +y + 1 +
3

x + y = 5

x
2
+xy + 4 +

y
2
+xy + 4 = 12
Bi 5)

x
3
y
3
2 = 3x 3y
2
x
2
+

1 x
2
3

2y y
2
+ 2 = 0
231
PHNG PHP H S BT NH
H s bt nh l ngun gc cho nhiu li gii p. Bi vit ny s cp n nhng bin i
trong h phng trnh hu t da vo h s bt nh. Mc tiu ca chng ta l sau khi bin i
thu c mt phng trnh c th phn tch thnh nhn t hoc c chnh phng
(nu l bc 2). Ta cng xem qua mt s v d:
Bi tp v d
Bi 1: Gii h phng trnh (I)

x
3
y
3
= 35 (1)
2x
2
+ 3y
2
= 4x 9y (2)
Gii
tng: Khng th dng php th gii h trn. V th ta hi vng c th t hai phng
trnh ca h a v dng (x + a)
3
= (y + b)
3
( rng x, y c lp vi nhau). Mun vy ta
nhn (2) cho mt s . Cng vic ca ta l tm a, b, (lu phng trnh (1) c bc 3 (cao
nht) nn ta mc nh h s nh c, cc s a, b, s c chn ph hp)
Ly (1) + .(2) ta c:
x
3
y
3
35 + (2x
2
+ 3y
2
4x + 9y) = 0
x
3
+ 2x
2
4x y
3
+ 3y
2
+ 9y 35 = 0 ()
Ta cn tm a, b, tho:
V T() = (x +a)
3
(y +b)
3

a
3
b
3
= 35
3a = 2
3a
2
= 4

= 3
a = 2
b = 3
Do () tr thnh: (x 2)
3
(y + 3)
3
= 0.
Li gii: Ta c h phng trnh:

x
3
y
3
= 35
x 2 = y + 3

x = y + 5
(y + 5)
3
y
3
= 35

x = 2; y = 3
x = 3; y = 2
Vy (I) c nghim (x; y) = (2; 3); (3; 2) 2
Nhn xt: Bi ton trn cho ta mt ci nhn tng quan v h s bt nh trong h phng
trnh hu t. y l mt bi c bn v x, y ng c lp. Ta cng xem qua mt bi tng t:
Bi 2: (VMO 2010) Gii h phng trnh

x
4
y
4
= 240 (1)
x
3
2y
3
= 3(x
2
4y
2
) 4(x 8y) (2)
Gii
tng: Nh bi trn, do x, y tch bit nn ta hi vng t hai phng trnh ca h a v
dng (x +)
4
= (y + )
4
. Mun vy ta nhn phng trnh th hai cho mt s k. Cng vic
232
ca ta l tm cc s , , k.
H phng trnh cho tng ng:

x
4
+ a = y
4
+ a + 240
x
3
3x
2
+ 4x = 2y
3
12y
2
+ 32y
Suy ra:
x
4
+a +k(x
3
3x
2
+ 4x) = y
4
+a + 240 + k(2y
3
12y
2
+ 32) ()
Cn chn k sao cho :
(x + )
4
= (y +)
4
x
4
+ 4x
3
+ 6
2
x
2
+ 4
3
x +
4
= y
4
+ 4y
3
+ 6
2
y
2
+ 4
3
y +
4
()
T (*) v (**) ng nht h s ta c:

a =
4
k = 4
3k = 6
2
4k = 4
3
a + 240 =
4
2k = 4
12k = 6
2
32k = 4
3

k = 8
= 2
a = 16
= 4
Li gii:
H cho tng ng:

x
4
+ 16 = y
4
+ 256
x
3
3x
2
+ 4x = 2y
3
12y
2
+ 32y
Ly phng trnh th hai nhn cho (-8) cng vi phng trnh th nht ta c:
x
4
+ 16 8(x
3
3x
2
+ 4x) = y
4
+ 256 8(2y
3
12y
2
+ 32y)
(x 2)
4
= (y 4)
4

x 2 = y 4
x 2 = 4 y

x = y 2
x = 6 y
Nu x = y 2 thay vo (1) ta c:
8y
3
24y
2
+ 32y + 224 = 0 (y + 2)(8y
2
40y + 112) = 0 y = 2 x = 4
Nu x = 6 y thay vo (1) ta c:
y
3
9y
2
+ 36y 44 = 0 (y 2)(y
2
7y + 22) = 0 y = 2 x = 4
Vy h phng trnh cho c nghim (x; y) = (4; 2), (4; 2) 2
Nhn xt: bi ny hoc cc bi tng t vi s m ln hn hay nh hn nu khng cha
cc hng t c dng x
m
.y
n
ta u c th s dng phng php trn. Tuy nhin ch cn qua
mt php i bin h s khng cn p na. Chng ta cng xt bi ton sau:
Bi 3: Gii h phng trnh (II)

x
2
+ y
2
=
1
5
(1)
4x
2
+ 4x
57
25
= y(3x + 1) (2)
233
Gii
tng: rng 2 phng trnh ca h u c bc hai v xut hin c hng t xy nn
vic dng HSB nh bi 1 s gp nhiu kh khn. Mt hng i thng dng ca ta vi h
loi ny l a v phng trnh bc hai theo (ax +by). lm iu , ta nhn (1) cho , (2)
cho v cng li:
(1). + (2).

x
2
+y
2

1
5

4x
2
+ 3xy + 3x +y
57
25

= 0

1 +
4

x
2
+
3

xy +y
2

+(3x +y)

5

57
25
= 0
xut hin hng t ax +by chnh l (3x +y). Do ta hi vng c

1 +
4

x
2
+
3

xy +y
2
= k(3x +y)
2
rng h s ca y
2
l 1 nn k = 1. Khai trin v ng nht h s ta c

1 +
4

= 9
3

= 6

= 1
= 2
Vy ta tm c , .
Li gii: Ly (1) + (2).2 ta c
(3x +y)
2
+ 2(3x +y)
119
25
= 0

3x +y
7
5

3x +y +
17
5

= 0

3x + y =
7
5
3x + y =
17
5
Nu 3x +y =
7
5
:
Ta c h phng trnh

x
2
+y
2
=
1
5
y = 3x +
7
5

x =
2
5
; y =
1
5
x =
11
25
; y =
2
25
Nu 3x + 7 =
17
5
:
Ta c h phng trnh

x
2
+y
2
=
1
5
y = 3x
17
5

y = 3x
17
5
10x
2
+
102x
5
+
284
25
= 0
(v nghim)
Vy (II) c nghim (x; y) =

2
5
;
1
5

11
25
;
2
25

2.
Nhn xt: Nhng h phng trnh cha hng t x
2
, xy, y
2
phn ln c th a v phng
trnh bc hai theo ax + by. Bi h trn cn c 2 cch gii khc, l dng php th hoc t n
ph tng - hiu. Ta cng xem qua mt bi tng t:
Bi 4: Gii h phng trnh

x
2
+ 2xy + 2y
2
+ 3x = 0 (1)
xy +y
2
+ 3y + 1 = 0 (2)
Gii
234
tng:
Nh bi trn ta s bin i a v phng trnh bc hai theo mx + ny. lm iu ta
nhn phng trnh th nht vi v phng trnh th cho ri cng li:
(1). + (2). (x
2
+ 2xy + 2y
2
+ 3x) + (xy +y
2
+ 3y + 1) = 0

x
2
+

+ 2

xy +

+ 2

y
2

+ 3

x +

+ = 0
Ta cn chn v sao cho:
x
2
+

+ 2

xy +

+ 2

y
2
=

x +

2
x
2
+

+ 2

xy +

+ 2

y
2
= x
2
+ 2

xy +

2

2
y
2
ng nht h s ta c:

+ 2 =
2

+ 2 =

2

= 2 = 2
cho n gin chn = 1 v = 2.
Li gii:
Ly phng trnh th nht cng vi phng trnh th hai nhn (2) ta c:
x
2
+ 4xy + 4y
2
+ 3x + 6y + 2 = 0
(x + 2y)
2
+ 3(x + 2y) + 2 = 0 (x + 2y + 1)(x + 2y + 2) = 0

x + 2y + 1 = 0
x + 2y + 2 = 0
Nu x + 2y + 1 = 0 x = 2y 1 thay vo phng trnh th hai ca h ta c:
y
2
+ 2y + 1 = 0

y = 1 +

2 x = 3 2

2
y = 1

2 x = 3 + 2

2
Nu x + 2y + 2 = 0 x = 2y 2 thay vo phng trnh th hai ca h ta c:
y
2
+y + 1 = 0

y =
1 +

5
2
x = 3 +

5
y =
1

5
2
x = 3

5
Vy h phng trnh cho c 4 nghim (x, y) l (3 2

2; 1 +

2); (3 + 2

2; 1

2);

3 +

5;
1 +

5
2

5;
1

5
2

2
Nhn xt: Nhng h phng trnh c cha cc phn t x
2
, xy, y
2
phn ln c th a v
phng trnh bc hai theo mx +ny, t c th tnh x theo y hoc y theo x ri th ngc lai
vo mt trong hai phng trnh ca h s ra c nghim.
Bi 5: Gii h phng trnh

x
4
4x
2
+y
2
6y + 9 = 0 (1)
x
2
y + x
2
+ 2y 22 = 0 (2)
235
Gii
tng: H phng trnh trn c bc kh cao (bc 4) nhng c th gim bc bng cch t
t = x
2
. Vy cch t nhin nht chnh l ta a v phng trnh bc 2. m bo chnh
phng ta s dng h s bt nh nh sau:
Ta c:
(1) + (2).a x
4
+x
2
(4 + ay +a) + y
2
6y + 9 + 22ay 22a = 0
Coi y l phng trnh bc 2 theo x
2
ta c:
= y
2
(a
2
4) + y(16a + 2a
2
+ 24) + a
2
+ 80a 20
l mt bnh phng th trc ht h s ca y
2
phi l s chnh phng, ngha l ta gii
PT nghim nguyn a
2
4 = k
2
Tm c cc nghim ca PT ny, ln lt th li. D thy a = 2 th

a
2
4 = 0
16a + 2a
2
+ 24 = 0
a
2
+ 80a 20 = 144
Vy ta chn a = 2.
Li gii:
Xt (1) + (2).2 ta c:
(1) +(2).2 x
4
4x
2
+y
2
6y +9 +2(x
2
y +x
2
+2y 22) = 0 (x
2
+y +5)(x
2
+y 7) = 0
Nu y = x
2
5, thay vo (1) ta c phng trnh
x
4
4x
2
+ (x
2
+ 5)
2
+ 6(x
2
+ 5) + 9 = 0 x
4
+ 6x
2
= 32(v nghim)
Nu y = x
2
+ 7, thay vo (1) ta c phng trnh
x
4
4x
2
+ (x
2
7)
2
+ 6(x
2
7) + 9 = 0 x
4
6x
2
+ 8 = 0 x 2;

2
T ta tm c nghim ca h l (x; y) = (2; 3), (2; 3), (

2; 5), (

2; 5) 2
Bi 6: Gii h phng trnh

x
3
+ 3xy
2
= 49 (1)
x
2
8xy +y
2
= 8y 17x (2)
Gii
tng: Do phng trnh th nht ca h c bc cao nht nn ta s nguyn, nhn phng
trnh th hai cho ri cng vi phng trnh th nht ta c :
x
3
+ 3xy
2
+ 49 + (x
2
8xy + y
2
+ 17x 8y) = 0 (1)
Mt khc ta c th nhm c nghim ca h l (x; y) = (1, 4), do ta mong mun (1)
phn tch c thnh:
(x + 1)(ax
2
+bx +cy
2
+dy + 49) = 0 (d thy h s ca xy trong ngoc bng 0)
ax
3
+ bx
2
+cxy
2
+dxy + 49x +ax
2
+ bx +cy
2
+ dy + 49 = 0
236
ax
3
+ (a + b)x
2
+ cxy
2
+dxy +cy
2
+ (b + 49)x + dy + 49 = 0 (2)
T (1) v (2) ng nht h s ta c:

a = 1
a +b =
c = 3
d = 8
c =
b + 49 = 17
d = 8

a = 1
c = = 3
b = 2
d = 24
Li gii:
Nhn phng trnh th hai ca h cho 3 ri cng vi phng trnh th nht c :
x
3
+ 3xy
2
+ 49 + 3x
2
24xy + 3y
2
24y + 51x = 0
(x + 1)(x
2
+ 2x + 3y
2
24y + 49) = 0
(x + 1)((x + 1)
2
+ 3(y 4)
2
) = 0

x + 1 = 0
x + 1 = 0, y 4 = 0

x = 1
x = 1, y = 4
Nu x = 1 thay vo phng trnh th nht ca h c y
2
= 16 y = 4
Nu x = 1, y = 4 thay vo h thy tha.
Vy h phng trnh cho c nghim (x, y) l (1, 4), (1, 4) 2
Ta cng xem qua mt cch gii khc cho bi ton:
tng: rng h c bc ba nhng l bc ba theo x, v ch c bc hai theo y. Vy ta
s ly (1) cng vi (2).a v a v mt phng trnh bc hai theo y:
y
2
(3x +a) + y(8ax 8a) + x
3
+ ax
2
+ 17ax + 49 = 0
Ta cn chn a l mt bnh phng ca mt biu thc theo x. C th ta c:
= 3x
4
+x
3
(4a) + x
2
(15a
2
51a) + x(15a
2
147a) + 16a
2
49a
Nu l bnh phng th n s phi c dng 3[f(x)]
2
. Mun vy trc ht 16a
2
49a phi
l
(3) ln ca mt s chnh phng. n gin ta chn a Z trc, ngha l cn phi gii
phng trnh nghim nguyn
16a
2
49a = 3b
2
D thy c 16a
2
49a 0 a(16a 49) 0 a 0; 1; 2; 3
Ln lt th ta thy a = 3 th = 3(x + 1)
4
Li rng 0 x = 1. Vy ta c ngay nghim x = 1 (trng hp tng qut th ta
phi tnh y theo x).
Li gii: Xt (1) + 3.(2) ta c phng trnh
y
2
(3x + 3) 24y(x + 1) + x
3
+ 3x
2
+ 51x + 49 = 0 ()
237
() c = 3x
4
12x
3
18x
2
12x
3
3 = 3(x + 1)
4
0 x = 1 Vy ta c h phng
trnh

x = 1
x
3
+ 3xy
2
= 49

x = 1
y
2
= 16

x = 1
y = 4
Do h c nghim (x; y) = (1; 4), (1; 4) 2
Nhn xt: Hai cch trn u xt (1) + (2).a v u ra a = 3 tuy tng khc nhau.
Cch 1 rt hiu qu trong nhng bi c th tm c nghim. Cch 2 mang tnh tng qut ,
v tng gii phng trnh nghim nguyn kh th v. Bi ny cn mt cch gii khc bng
phng php t n ph tng - hiu.
Sau y ta s khai thc su v cch 1. Lu rng vi nhng bi cng on c nhiu nghim
th cng d dng h s bt nh.
Bi 7: Gii h phng trnh

x
4
+ 2(3y + 1)x
2
+ (5y
2
+ 4y + 11)x y
2
+ 10y + 2 = 0 (1)
y
3
+ (x 2)y +x
2
+x + 2 = 0 (2)
Gii
tng: gii bi trn ta s tin hnh 4 bc:
Bc 1: tm nghim ca h. Nu bit c nghim th tng ca ta s r rng hn
nhiu. y ln lt th x = 2, 1, 0, 1, 2, 3, ... ta tm c 2 nghim ca h l (x; y) =
(1; 1), (2; 2) 2
Tuy nhin lu rng bc ca hai phng trnh l kh cao (4 v 3). Nu phn tch nh bi 5
th ta vn phi gii quyt mt h phng trnh hai n bc 3 v 2 (sau khi gim bc). V vy
y, do bit 2 nghim, ta s tin hnh bc 2:
Bc 2: tm quan h tuyn tnh gia 2 nghim ny: D thy l y = x.
Bc 3: thay vo h v phn tch thnh nhn t: Ta thay x bi y hoc y bi x (ty
trng hp xem cch no c li), vi bi ny ta thay y = x vo hai phng trnh ca h v
thu c

x
4
+ 2(3x + 1)x
2
+ (5x
2
4x + 11)x x
2
10x + 2 = 0
x
3
(x 2)x +x
2
+ x + 2 = 0

(x + 1)
2
(x 1)(x 2) = 0
(x + 1)
2
(x 2) = 0
Vic phn tch trn l khng kh v ta bit trc nghim.
Bc 4: La chn biu thc thch hp:
Nh th, so vi phng trnh th nht va nhn c th phng trnh th hai thiu i mt
biu thc l x 1 , nhng ch rng biu thc ny cng tng ng vi y 1. Ta s chn
mt trong hai biu thc ny nhn vo.
R rng nu chn y 1 th vic nhn s to ra mt a thc c cha bin y ng bc vi a
thc phng trnh th nht ban u. Vy ta s nhn phng trnh sau cho y 1.
Li gii:
Nhn phng trnh th hai cho y + 1, ri ly phng trnh th nht, tr phng trnh va
nhn c, ta c:
(x +y)(x y + 2)(x
2
2x +y
2
+ 3y + 5) = 0]
238
- Vi x = y, ta a phng trnh th hai ca h v (y + 2)(y 1)
2
= 0
- Vi x = y 2, ta a phng trnh th hai ca h v (y 1)
2
(y + 4) = 0
D thy khng xy ra x
2
2x +y
2
+ 3y + 5 = 0
Th li ta thy h c nghim (x, y) = (1, 1), (2, 2), (6, 4) 2
Nhn xt: Vn dng linh hot 4 bc trn c th cho ta li gii rt p v ngn gn. Ta
cng xem qua mt bi tng t:
Bi 8: Gii h phng trnh

(x y)
2
+x +y = y
2
(1)
x
4
4x
2
y + 3x
2
= y
2
(2)
Gii
tng: u tin ta thy (1) x
2
2xy +x +y = 0.
C th tm c 3 nghim nguyn ca h ny l (x; y) = (0, 0), (2; 2), (1; 2). T 2 nghim u
ta thy x = y. Thay vo h ta c

x
2
2x = 0
[x(x 2)]
2
= 0
Nh vy ta s phi nhn x(x 2) hoc y(y 2) vo (1). rng nu chn x(x 2) ta s lm
mt bc 4. Vy khu chun b hon tt.
Li gii:
Xt x = 0 v x = 2 ta thy h c nghim (x; y) = (0; 0), (2; 2).
Vi x / 0; 2 xt (1).[x(x 2)] + (2) ta c phng trnh
x(x 2)(x
2
2xy + x + y) + (x
4
4x
2
y + 3x
2
+y
2
) = 0 (x y)(2x
3
x
2
+x y) = 0
Nu x = y thay vo (1) ta c 2x x
2
= 0. Trng hp ny loi do x / 0; 2
Nu 2x
3
x
2
+x y = 0 ta c h phng trnh
()

2x
3
x
2
+x y = 0
x
2
2xy +x +y = 0
Cng hai phng trnh ta c
2x
3
2xy + 2x = 0 2x(x
2
+ 1 2y) = 0 x
2
+ 1 = 2y (do x = 0)
Thay vo (1) ta c phng trnh
2x
3
+x
2
+ 1 = 0 x = 1
T tm c (x; y) = (1; 2).
Vy h cho c nghim (x; y) = (1; 2), (0; 0), (2; 2) 2
M RNG:
tng: Nh cp, nu bit cng nhiu nghim th ta c li gii cng p. Sau y l
mt cch phn tch khi ta bit c 3 nghim. Ta s ln lt lp 3 quan h tuyn tnh gia x v
239
y. Trong bi ny, l

y = x
y = 2x
y = 2
Do (2) c bc cao hn nn ta xt
(1). + (2) x
4
+ x
2
(4y + + 3) + x( 2y) + y +y
2
= 0 (3)
Ta s chn quan h no d s dng nht, chnh l y = 2. Nh vy (3) c nghim y = 2,
ngha l
x
4
+x
2
( 5) + x(3) + 2 + 4 = 0 =
x
4
+ 5x
2
4
x
2
3x + 2
= (x + 1)(x + 2)
Khi (3) tr thnh 2x
3
+x
2
+x +y = 0
Ta tip tc khai thc mt trong hai quan h cn li, rng nh phn tch trn, vi
= x(x 2) ta thu c 2x
3
x
2
+x y = 0
Vy ta i n li gii ngn gn nh sau:
Li gii:
Th vi x M = 0; 2; 1; 2 ta tm c nghim (x; y) = (0; 0), (2; 2), (1; 2). Xt x / M ta
c
(1). + (2) y
2
+ y( 2x 4x
2
) + x +x
2
( + 3) + x
4
= 0 ()
Chn = x(x 2) ta c 2x
3
x
2
+x y = 0
Chn = (x + 1)(x + 2) ta c 2x
3
+x
2
+ x + y = 0
Cng li ta c 4x
3
+ 2x = 0 (sai do x / M). Vy trng hp ny loi.
Tm li h c nghim (x; y) = (0; 0), (2; 2), (1; 2) 2
Bi tp t luyn
Gii cc h phng trnh sau:
1)

x
3
+ y
3
= 9 (1)
x
2
+ 2y
2
= x + 4y (2)
2)

x
3
y
3
+ 3y
2
+ 2x 5y + 3 = 0(1)
x
2
+ 2y = 1(2)
3)

x
2
+ y
2
= 1
21x + 3y + 96x
2
+ 28xy = 117
4)

6x
2
y + 2y
3
+ 35 = 0
5x
2
+ 5y
2
+ 2xy + 5x + 13y = 0
5)

x
4
y
4
= 1215
2x
3
4y
3
= 9(x
2
4y
2
) 18(x 8y)
240
K THUT T N PH TNG - HIU
L thuyt
Trong vic gii h phng trnh th t n ph l cch hiu qu v rt a dng. Phng php
t n ph tng - hiu l mt cch t d nhn bit, tuy khng phi ti u nhng cng c th
a v mt h phng trnh gii bng phng php th.
Ni dung phng php chnh l, t mt h phng trnh theo 2 n x, y ta t
()

a = mx + ny
b = ux +vy
hoc ()

x = ma +nb
y = ua + vb
(a, b l n mi) .
Hai php t trn thc ra tng ng, v ta c th tnh x, y theo a, b v ngc li. Lu rng:
Php t () c dng vi nhng h m ta c th nhm cc hng t mt cch thch hp
to ra phng trnh n gin hn theo mx +ny v ux +vy.
Php t () c dng vi nhng h phng trnh m ta hi vng c th kh mt hay mt
s hng t no sau khi khai trin. (lu rng h phng trnh hu t m 2 n tch bit th
c th gii quyt bng h s bt nh).
Khi gp mt bi ton ta nn th () trc, v php t ny cho h n gin hn.
Mt trng hp c bit hn l php t

a = mx +ny
b = nx my
dng cho h phng trnh c mt
phng trnh i xng, c th a mt s bi v h i xng loi I.
Phng php ny c th gii quyt nhiu h phng trnh hu t, c bit l h bc hai. V t
mt phng trnh bc hai 2 n m
1
x
2
+ m
2
xy + m
3
y
2
+ m
4
x + m
5
y + m
6
= 0 ta c th a v
dng n
1
ab +n
2
a +n
2
b +n
3
= 0, t tnh c a theo b.
Sau y l cc v d c th:
Bi tp v d
Bi 1: Gii h phng trnh

x
2
+y
2
= xy +x +y
x
2
y
2
= 3
Gii
tng: D thy nu t a = x + y, b = x y th t phng trnh sau c ab = 3 nn mc
tiu dng php th thnh cng. Vic cn li l a phng trnh u v bin a, b (d thy
x
2
xy + y
2
=
1
4
[3(x +y)
2
+ (x y)
2
] )
Li gii:
t a = x +y; b = x y ta c h phng trnh

3a
2
+b
2
= 4b
ab = 3

a =
3
b
27
b
2
+b
2
= 4b

b = 3
a = 1
T tm c (x; y) = (2; 1) 2
Bi 2: Gii h phng trnh sau

x
2
+y
2
+x = 3
x
2
4y
2
+
2xy
x + y 1
= 1
241
Gii
KX: x + y = 1
Phng trnh (2) tng ng:
(x + y 1)

x
2
4y
2

+x +y + 2xy + 1 = 0

x
3
4y
3
4xy
2
+x
2
y

x
2
+ 4y
2
+ x + y + 2xy 1 = 0
(x + 2y)

x
2
xy 2y
2

xy +x + 2y
2
+ 2y

x
2
xy 2y
2
+y + 1

= 0
(x + 2y)

x
2
xy 2y
2

+ (x + 2y) (y + 1)

x
2
xy 2y
2
+y + 1

= 0
(x + 2y 1)

x
2
xy 2y
2
+y + 1

= 0

x + 2y = 1
x
2
xy 2y
2
+ y + 1 = 0
Nu x + 2y = 1 thay vo phng trnh (1) ta c

x = 1 2y
5y
2
6y 1 = 0

x = 1 2y

y =
3 +

14
5
y =
3

14
5

y =
3 +

14
5
; x =
1 2

14
5
y =
3

14
5
; x =
1 + 2

14
5
Nu x
2
xy 2y
2
+y + 1 = 0 ta c h phng trnh:
()

x
2
+y
2
+x = 3
x
2
xy 2y
2
+y + 1 = 0
Nh vy mu cht ca bi ton li l h phng trnh ()
tng: s dng php th, ta phi a mt phng trnh v dng p.ab + q.a + r.b = m
(n a, b). Nh vy cc hng t bc 2 trong phng trnh phi c a v tch ca a v b (
a, b l bc nht). Ngha l ta phi phn tch x
2
xy 2y
2
thnh nhn t.
D thy x
2
xy 2y
2
= (x +y)(x 2y), vy ta t

a = x +y
b = x 2y
Li gii:
t a = x +y; b = x 2y

x =
2a + b
3
y =
a b
3
Ta c HPT:

(2a +b)
2
+ (a b)
2
+ 3(2a +b) = 27 (1)
3ab +a b + 3 = 0 (2)
T (2) c a =
b 3
3b + 1
. Th vo (1) c phng trnh bc 4 theo b:
54b
4
+ 135b
3
648b
2
729b = 0 b

9
2
; 1; 0; 3

242
Nh vy h tng ng

b =
9
2
; a =
3
5
b = 1; a = 2
b = 0; a = 3
b = 3; a = 0
T tm c nghim
(x; y) =

11
10
;
17
10

, (1; 1), (2; 1), (1; 1),

1 2

14
5
;
3 +

14
5

1 + 2

14
5
;
3

14
5

2
Nhn xt: Nh vy t mt h khng th dng php th, qua php t n ph a v h
gii bng php th. D phng trnh bc 4 c h s cao nhng tng gii l r rng.
H () cng c th gii bng h s bt nh. C th, ly phng trnh u nhn vi 2 v
cng vi phng trnh sau ta thu c
3x
2
x(y 2) + y 5 = 0 (x 1)(3x y + 5) = 0
Bi 3: Gii h phng trnh

x
2
+y
2
=
2
5
(1)
4x
2
+ 3x
57
25
= y(3x + 1) (2)
Gii
tng: Xt cc hng t bc 2 c hai phng trnh, ta thy khng th phn tch thnh
nhn t. Tuy nhin lu rng ta c th thay i h s ca x
2
v y
2
(2) bng php th (1).
Nh vy ta hng ti cch t (), nhng y kh khn hn v ta khng bit biu thc cn
phn tch l g. Tip tc khai thc bi ton:
Ta s th (1) vo (2) bin i biu thc bc 2. Ngha l ta xt:
(1). + (2) x
2
(4 + ) + 3xy +y
2
+ 3x + y =
57
25
+

5
Mc ch ca ta l phn tch f(x) = x
2
(4 + ) + 3xy + y
2
thnh nhn t. Mun vy th
ca f(x) phi l mt bnh phng. Ta xt
= 9y
2
4(4 + )y
2
= y
2
(9 16 4
2
)
Nh vy ta phi tm 9 164
2
l s chnh phng, ni cch khc l gii phng trnh
nghim nguyn
9 16 4
2
= k
2
T tm c 4; 2; 0 (tht ra ch cn tm nghim nguyn ca 9 164
2
0 ri
th li). D thy ch nhn = 2. Khi ta c phn tch 2x
2
+3xy 2y
2
= (2x y)(x +2y).
Vy t a = 2x y; b = x + 2y.
Li gii:
Ly (2) 2.(1) ta c h tng ng

x
2
+y
2
=
2
5
2x
2
+ 3xy 2y
2
+ 3x + y =
47
25
243
t a = x + 2y, b = 2x y ta c h i xng :

a
2
+b
2
= 1
a +b +ab
47
25
= 0

a =
3
5
; b =
4
5
a =
4
5
; b =
3
5
Vy ta tm c (x; y) =

2
5
;
1
5
); (
11
25
;
2
25

2
Nhn xt: Bi ton trn cng c th gii bng h s bt nh. C th ta xt
(1).25 + (2).50 25(3x +y)
2
+ 50(3x + y) 119 = 0 3x + y

7
5
;
17
5

T d dng lm tip.
Bi 4: Gii h phng trnh

x
3
+ 3xy
2
= 49
x
2
8xy +y
2
= 8y 17x
Gii
tng: Th xt hng t bc 2 trc, ta thy khng th phn tch x
2
8xy+y
2
thnh nhn t
v cng khng c cch no bin i biu thc ny. n hng t bc 3, d x
3
+3xy
2
= x(x
2
+3y
2
)
nhng khng d a v bc 3 theo a, b vi php t (). Vy ta chuyn qua php t ().
Trc tin ta th cch t x = ua+vb; y = vaub v mc ch ca ta chnh l kh ab, a
2
b, ab
2
sau khi a v h bc 3 theo a, b.
Gi ta s tm u, v:
t

x = ua +vb
y = va ub
thay vo h ta c:

a
3
(u
3
+ 3uv
2
) + b
3
(v
3
+ 3u
2
v) + a
2
b(3v
3
3u
2
v) + ab
2
(3u
3
3uv
2
) = 49
a
2
(u
2
8uv +v
2
) + b
2
(v
2
+ 8uv +u
2
) + 8ab(u
2
v
2
) = 9ua 25vb
Ta s ng nht h s sao cho h s ca a
2
b, ab
2
v ab bng 0 :

3v
3
3u
2
v = 0
3u
3
3uv
2
= 0
u
2
v
2
= 0

u = v
u = v
Do u = v hay u = v u c cng cch t nn ta chn u = v = 1 (tht ra c th ly u, v tu
, v v tri ca phng trnh u l thun nht).
Li gii:
t x = a +b, y = a b ta c h:

4a
3
+ 4b
3
= 49
6a
2
+ 10b
2
= 9a 25b

8a
3
+ 8b
3
= 98
6a
2
9a = 10b
2
+ 25b
Ti y c th gii tip bng h s bt nh. C th, ly phng trnh th nht cng phng
trnh th hai nhn (-6) ta c
(2a 3)
3
= (2b 5)
3
2a 3 = 2b 5 a = b 1
244
T tm c (a; b) =

5
2
;
3
2

3
2
;
5
2

Nh vy h c nghim (x; y) = (1; 4), (1; 4) 2


Nhn xt: T bi trn c th rt ra mt kinh nghim nh l nu trong mt phng trnh c
mx
3
v 3mxy
2
, hoc my
3
v 3mx
2
y th ta c th t x = a + b; y = a b.Sau y ta xt mt
bi tng t:
Bi 5: Gii h phng trnh

6x
2
y + 2y
3
+ 35 = 0 (1)
5x
2
+ 5y
2
+ 2xy + 5x + 13y = 0 (2)
Gii
tng: Theo nhn xt bi 4, do phng trnh u c 2y
3
+6x
2
y nn ta t x = a+b; y = ab.
Li gii:
t x = a +b; y = a b ta c (1) tr thnh:
6(a +b)
2
(a b) + 2(a b)
3
+ 35 = 0 6

a
2
+ 2ab + b
2

(a b) + 2

a
3
3a
2
b + 3ab
2
b
3

+ 35 = 0
6

a
3
a
2
b + 2a
2
b 2ab
2
+ ab
2
b
3

+ 2a
3
6a
2
b + 6ab
2
2b
3
+ 35 = 0 a
3
b
3
+
35
8
= 0
Tng t, (2) tr thnh: 6a
2
+ 9a + 4b
2
4b = 0
Vy ta c h phng trnh

a
3
b
3
=
35
8
6a
2
+ 9a + 4b
2
4b = 0
Gii h ny bng h s bt nh, ta tm c (a; b) =

1;
3
2

3
2
; 1

Vy ta c (x; y) =

1
2
;
5
2

1
2
;
5
2

2
Nhn xt: Nu bit nghim ca h, ta cng c th trnh by ngn gn hn bng h s bt
nh nh sau:
(1) + 3.(2) (6y + 15)x
2
+ 3(2y + 5)x + 2y
3
+ 15y
2
+ 39y + 35 = 0 (2y + 5)

x +
1
2

2
+

y +
5
2

= 0
Bi 6: Gii h phng trnh

x
4
2x = y
4
y
(x
2
y
2
)
3
= 3
( kim tra i d tuyn trng THPT Chuyn HSP H Ni)
Gii
t x + y = a, x y = b, 3 = c
3
.
T phng trnh th hai ca h, ta c: (ab)
3
= c
3
ab = c.
Ta c: x =
a +b
2
, y =
a b
2
. Suy ra:
x
4
y
4
= (x y)(x +y)(x
2
+ y
2
) = ab

a + b
2

2
+

a b
2

=
ab
2
(a
2
+b
2
)
245
Li c na:
2x y = (a +b)
(a b)
2
=
a + 3b
2
=
a +c
3
b
2
Do , phng trnh th nht ca h cho tng ng vi:
ab
2
(a
2
+b
2
) =
a + c
3
b
2
c(a
2
+b
2
) = a +c
3
b
Ta c h mi l:

c(a
2
+ b
2
) = a + c
3
b
ab = c
c

a
2
+
c
2
a
2

= a +
c
4
a
ca
4
+c
3
= a
3
+ ac
4
(ca 1)(a
3
c
3
) = 0 a =
1
c
a = c
Suy ra h ny c hai nghim l: (a, b) = (c, 1);

1
c
, c
2

.
Xt hai trng hp:
Nu a = c, b = 1 th x =
c + 1
2
=
3

3 + 1
2
, y =
3

3 1
2
.
Nu a =
1
c
, b = c
2
th x =
1
2

1
c
+c
2

=
1 + c
3
2c
=
2
3

3
, y =
1
2

1
c
c
2

=
1 c
3
2c
=
1
3

3
Vy h cho c hai nghim l: (x, y) =

3 + 1
2
,
3

3 1
2

2
3

3
,
1
3

2.
Bi tp t luyn
Gii h phng trnh:
1)

x
2
y
2
= y
x
2
+xy +x = 6
2)

4(x
2
+ y
2
) + 4xy +
3
(x +y)
2
= 7
2x +
1
x + y
= 3
3)

3(x
2
+ y
2
) +
1
(x y)
2
= 2(10 xy)
2x +
1
x y
= 5
4)

x
2
+ 2x + 6 = y + 1
x
2
+xy +y
2
= 7
5)

x
2
y
2
= 4x + 6y 1
x
4
+y
4
5x
2
5y
2
= 2x
2
y
2
10xy 1
6)

x
2
+y
2
= 1
21x + 3y + 48x
2
48y
2
+ 28xy = 69
246
PHNG PHP DNG BT NG THC
V tng, dng bt ng thc trong phng trnh v h phng trnh l tng t. Nhng
trong nhiu bi h, vic nh gi cc n s phc tp hn nhiu. Ta cng xem qua mt s bi
tp:
Bi tp v d
Bi 1: Gii h phng trnh sau:

x
6
+ y
8
+ z
10
= 1 (1)
x
2007
+y
2009
+ z
2011
= 1 (2)
Gii
T (1) ta c: 1 x, y, z 1
T (1) v (2) ta c:
x
2007
+y
2009
+z
2011
= x
6
+ y
8
+ z
10
x
6

1 x
2001

+y
8

1 y
2001

+z
10

1 z
2001

= 0 (3)
T 1 x, y, z 1 ta thy:
x
6

1 x
2001

, y
8

1 y
2001

, z
10

1 z
2001

0
Do :
(3)

x = y = z = 1
x = y = z = 0
Kt hp vi (1 )suy ra h c cc nghim l: (x; y; z) = (1; 0; 0) , (0; 1; 0) , (0; 1; 0) , (0; 0; 1) 2
Bi 2: Gii h phng trnh

3(a + b) = 2 [ab + 1[
9(a
3
+b
3
) = [a
3
b
3
+ 1[
Gii
Ta c:

a
3
b
3
+ 1

(ab + 1)(a
2
b
2
ab + 1)

= [(ab + 1)[

(a
2
b
2
ab + 1)

= [(ab + 1)[

(ab + 1)
2
3ab

[(ab + 1)[

(ab + 1)
2
3ab

=
3
2
(a + b)

9
4
(a +b)
2
3ab

=
9
8
(a +b)(3a
2
+ 2ab + 3b
2
)
Li c: 5(a +b)(a b)
2
0 9(a
3
+b
3
)
9
8
(a +b)(3a
2
+2ab +3b
2
) 9(a
3
+b
3
) [a
3
b
3
+ 1[
ng thc xy ra nn a = b.
Vy h c nghim duy nht a = b =
3 +

5
2
2.
Bi 3: Gii h phng trnh

a +b =
3

24
(

a +

b)

a + 3b
+
1

b + 3a

= 2
Gii
247
p dng bt ng thc AM-GM ta c:

a + 3b

1
2

a
a + b
+
a +b
a + 3b

a + 3b

1
2

1
2
+
2b
a + 3b

a +

a + 3b

1
2

a
a +b
+
3
2

Chng minh tng t ta cng c:

a +

b + 3a

1
2

b
a + b
+
3
2

Cng lai ta c
(

a +

b)

a + 3b
+
1

b + 3a

2
ng thc xy ra nn a = b.
Vy h c nghim duy nht a = b =
3

24
2
2.
Bi 4: Gii h phng trnh:

x
4
+y
4
= 2
x
3
2x
2
+ 2x = y
2
Gii
Vit li h cho di dng:

x
4
+ y
4
= 2
x
3
2x
2
+ 2x 1 = y
2
1

x
4
+ y
4
= 2
(x 1)(x
2
x + 1) = y
2
1
Xt cc kh nng sau:
Nu x > 1 (x 1)(x
2
x + 1) > 0 y
2
> 1 y
4
> 1 x
4
+ y
4
> 2
Khi y h cho v nghm.
Nu 0 < x < 1 y
2
< 1 y
4
< 1
Trong trng hp ny h cng v nghim.
Nu x < 0 th x
3
2x
2
+ 2x < 0 y
2
< 0 : v l.
Ti x = 0, ta c h

y
4
= 2
y
2
= 0
(v nghim)
Ti x = 1, h tr thnh:

y
4
= 1
y
2
= 1
y = 1
Vy h cho c nghim (x; y) = (1; 1), (1; 1) 2
Bi 5: Gii h phng trnh

x +
4

32 x y
2
= 3
4

x +

32 x + 6y = 24
Gii
iu kin 0 x 32.
Cng hai phng trnh v theo v:
(

x +

32 x) + (
4

x +
4

32 x) = y
2
6y + 21 (1)
248
Li c:

y
2
6y + 21 = (y 3)
2
+ 12 12 (2)

x +

32 x

2(x + 32 x) = 8
4

x +
4

32 x

2(

x +

32 x)

2.8 = 4
T suy ra (

x +

32 x) + (
4

x +
4

32 x) 12 (3)
Kt hp (1),(2) v (3) ta c (

x+

32 x)+(
4

x+
4

32 x) = y
2
6y+21 = 12

x = 16
y = 3
Vy h cho c nghim x = 16, y = 3 2.
Bi 6: Gii h phng trnh

x +
2xy
3

x
2
2x + 9
= x
2
+ y (1)
y +
2xy
3

y
2
2y + 9
= y
2
+x (2)
Gii
tng: y l h phng trnh i xng loi II, nhng nu lm theo cch thng thng
th s rt kh khn do c s xut hin ca cn bc ba. rng khi ta cng 2 phng trnh
li hng t (x + y) mi v s c n gin, khi VT xut hin 2xy, cn VP xut hin
x
2
+ y
2
, v do y l h phng trnh i xng loi II nn s c nghim x = y. T ta ngh
ti vic nh gi 2xy v x
2
+ y
2
.
Li gii:
Vi x = 0 y = 0. Xt x, y = 0 :
Cng (1) v (2) v theo v:
x +y + 2xy(
1
3

x
2
2x + 9
+
1
3

y
2
2y + 9
) = x
2
+y
2
+x +y
2xy(
1
3

x
2
2x + 9
+
1
3

y
2
2y + 9
) = x
2
+y
2
(3)
Suy ra 2xy > 0. Mt khc ta c:

1
3

x
2
2x + 9
=
1
3

(x 1)
2
+ 8

1
3

8
=
1
2
1
3

y
2
2y + 9
=
1
3

(y 1)
2
+ 8

1
3

8
=
1
2

1
3

x
2
2x + 9
+
1
3

y
2
2y + 9
1 2xy(
1
3

x
2
2x + 9
+
1
3

y
2
2y + 9
) 2xy x
2
+y
2
(4)
T (3) v (4) suy ra x = y = 1. Th li cc nghim (0; 0), (1; 1)u tha.
Vy h phng trnh c nghim (x; y) l (0;0), (1; 1). 2
Bi 7: Gii h phng trnh


3 + 2x
2
y x
4
y
2
+x
4
(1 2x
2
) = y
4
1 +

1 + (x y)
2
= x
3
(x
3
x + 2y
2
)
249
Gii
Nhn thy cc biu thc trong cn rt phc tp. Nhng phng trnh sau gi cho ta
a v cc bnh phng, t hi vng c th nh gi bt ng thc mt cch hp l.
Li gii:
Nhn thy

3 + 2x
2
y x
4
y
2
=

4 (1 x
2
y)
2
2 v 1 +

1 + (x y)
2
2
y
4
+ 2x
6
x
4
x
6
x
4
+ 2x
3
y
2
(x
3
y
2
)
2
0
Suy ra x
3
= y
2

1 x
2
y = 0
x y = 0
x
3
= y
2
x = y = 1
Th li ta thy h c nghim (x; y) = (1; 1) 2
Bi 8: Gii h phng trnh

y
3
x
3
= 7
x
3
y
2
+x = 2
Gii
H phng trnh tng ng

y
3
8 = x
3
1
(y
3
7) y
2
+x = 2

y
3
8 = x
3
1
y
3
y
2
4 = 1 x

y
3
8 = x
3
1 (1)
(y 2)(y
2
+y + 2) = 1 x (2)
Ta c y
2
+y + 2 = (y +
1
2
)
2
+
7
4
> 0, y R.
Xt cc trng hp:
y > 2 : Ta c

(1) x > 1
(2) x < 1
H v nghim
y < 2 : Ta c

(1) x < 1
(2) x > 1
H v nghim
y = 2 : x = 1
Vy h phng trnh c nghim (1;2) 2.
Nhn xt: Mu cht ca li gii trn chnh l on c nghim (x; y) = (1; 2), t c
nhng bin i ng thc hp l chng minh y l nghim duy nht. Ta cng xem qua mt
bi tng t:
Bi 9: Gii h phng trnh

y = x
3
+ 3x + 4
x = 2y
3
6y 2
()
Gii
tng: d dng nhm c nghim ca h phng trnh l x = y = 2.
Ta hi vng y l nghim duy nht ca h, v th ta s c gng a h phng trnh ban u
v dng:

y 2 = (x 2).f(x)
x 2 = (y 2).g(y)
Ri t hai phng trnh trn da vo du ca f(x) v g(y) ta s nh gi gia x v y, v
250
lm c iu ny ta s chia trng hp xt.
Li gii:
Vit li h di dng

x
3
3x 2 = 2 y
2y
3
6y 4 = x 2

(x 2)(x + 1)
2
= 2 y (1)
2(y 2)(y + 1)
2
= x 2 (2)
Nu x > 2 :

(1) y < 2
(2) y > 2
V nghim
Nu x < 2:

(1) y > 2
(2) y < 2
V nghim
Suy ra x = 2 y = 2.
Th li ta thy x = y = 2 tha h.
Vy h phng trnh c nghim (2;2) 2.
Bi 10: Gii h phng trnh

x
5
+y
5
+z
5
= 3
x
6
+y
6
+z
6
= 3
Gii
tng: t hai phng trnh trn ta suy ra x
6
+y
6
+z
6
= x
5
+y
5
+z
5
.
Ta ngh ti vic nh gi gia x
6
+ y
6
+ z
6
v x
5
+ y
5
+ z
5
, c th l ta s i chng minh
x
6
+y
6
+z
6
x
5
+y
5
+z
5
.
D nhin chng minh c bt ng thc ny ta phi ngh ti vic s dng cng c mnh
l bt ng thc Holder.
Li gii:
T gi thit ta suy ra x
6
+y
6
+z
6
= x
5
+ y
5
+z
5
Ap dng bt ng thc Holder cho 6 b ba s dng c :
(x
6
+y
6
+z
6
)
5
.(1 + 1 + 1) ([x[
5
+[y[
5
+[z[
5
)
6
(x
5
+y
5
+z
5
)
6
ng thc xy ra

x
6
= y
6
= z
6
x, y, z 0
Kt hp vi gi thit suy ra x = y = z = 1.
Th li ta thy h phng trnh cho c nghim (1; 1; 1) 2
Bi 11: Gii h phng trnh

x
2
y
2
2x +y
2
= 0 (1)
7x
2
14x + 3y
3
+ 10 = 0 (2)
Gii
T (1) suy ra:
y
2
=
2x
x
2
+ 1
1

x 0
1 y 1
y
3
1 ()
251
T (2) suy ra 7(x 1)
2
+ 3(y
3
+ 1) = 0
M 7(x 1)
2
0; 3(y
3
+ 1) 0 (do (*)) 7(x 1)
2
+ 3(y
3
+ 1) 0
ng thc xy ra

(x 1)
2
= 0
y
3
+ 1 = 0

x = 1
y = 1
Th li ta thy h c nghim (x; y) = (1; 1) 2
Bi 12: Gii h phng trnh ()

x
2
+y
2
+z
2
+ 2xy xz yz = 3
x
2
+y
2
+yz zx 2xy = 1
Gii
tng: Ta thy rng phng trnh th nht c th phn tch thnh mt bnh phng
ca mt tng theo x, y, z v cn d mt lng theo z
2
, phng trnh th hai cng c th phn
tch thnh bnh phng ca mt tng c cha x, y, z nhng li thiu mt lng theo z
2
.
T ta ngh ti vic nh gi z
2
t mi phng trnh.
Li gii:
Vit li h (*) di dng

(x
2
+y
2
+ 2xy) z(x +y) + z
2
= 3
(x
2
+y
2
2xy) z(x y) = 1

(x +y)
2
z(x +y) +
z
2
4
+
3z
2
4
= 3
(x y)
2
z(x y) +
z
2
4

z
2
4
= 1

(x +y
z
2
)
2
= 3(1
z
2
4
)
(x y
z
2
)
2
=
z
2
4
1
()

1
z
2
4
0
z
2
4
1 0

z
2
4
z
2
4
z
2
= 4

z = 2
z = 2
z = 2, ()

(x +y 1)
2
= 0
(x y 1)
2
= 0

x + y 1 = 0
x y 1 = 0

x = 1
y = 0
z = 2, ()

(x +y + 1)
2
= 0
(x y + 1)
2
= 0

x +y + 1 = 0
x y + 1 = 0

x = 1
y = 0
Th li ta thy h c nghim (x; y; z) = (1; 0; 2), (1; 0; 2) 2.
Bi 13: Gii h phng trnh

2
x
+ 4
y
= 32
xy = 8
Gii
tng: Ta thy phng trnh th nht ca h c dng tng, phng trnh th hai c dng
tch nn ngh ti vic s dng bt ng thc AM-GM nh gi.
Li gii:
Do xy = 8 > 0 x, y cng du
252
Nu x < 0, y < 0 V T < 2 < V P phng trnh v nghim.
Nu x > 0, y > 0:
p dng bt ng thc AM-GM c:
2
x
+ 4
y
= 2
x
+ 2
2y
2

2
x
2
2y
= 2

2
x+2y
2

2
2

2xy
= 2

2
2

2.8
= 32
ng thc xy ra

2
x
= 2
2y
x = 2y
xy = 8

x = 4
y = 2
Th li ta thy nghim (4;2) tha h.
Vy h phng trnh c nghim (4;2) 2
Bi 14: Gii h phng trnh

2x
2
x
2
+ 1
= y
2y
2
y
2
+ 1
= z
2z
2
z
2
+ 1
= x
(Thi v ch ton Bungari 1977)
Gii
Ta thy x = y = z = 0 l mt nghim ca h.
Nu

x = 0
y = 0
z = 0
th x, y, z > 0. Khi , nhn 3 v ca h phng trnh ta c
8x
2
y
2
z
2
(z
2
+ 1)(y
2
+ 1)(z
2
+ 1)
= xyz (x
2
+ 1)(y
2
+ 1)(z
2
+ 1) = 8xyz
Mt khc p dng bt ng thc AM-GM c:
(x
2
+ 1)(y
2
+ 1)(z
2
+ 1) 2

x
2
.2

y
2
.2

z
2
= 8 [xyz[ = 8xyz (do xyz > 0)
ng thc xy ra

x, y, z > 0
x
2
= y
2
= z
2
= 1
x = y = z = 1
Th li ta thy x = y = z = 1 tha h.
Vy h phng trnh c nghim (0;0;0), (1;1;1) 2.
Nhn xt:
1) Bi h trn cng c th gii bng hm n iu. C th xt hm f(t) =
2t
2
t
2
+ 1
, ta c
f

(t) =
4t
(t
2
+ 1)
2
nn f(t) ng bin trn (0; +).
D thy x, y, z 0 do khng gim tng qut gi s x = maxx; y; z th
f(x) f(y) y z f(y) f(z) z x x = y = z.
Ta cng xem qua bi ton tng qut hn:
253
Bi 15: Gii h phng trnh sau:

2x
2
x
2
+ 1
= y
3y
3
y
4
+y
2
+ 1
= z
4z
4
z
6
+z
4
+z
2
+ 1
= x
Gii
R rng nu mt trong s cc n x, y, z bng 0 th cc x = y = z = 0.
Nh vy x = y = z = 0 l nghim ca h cho.
Nu x, y, z = 0 x, y, z > 0
Ta c:
x
2
+ 1 2x
2x
x
2
+ 1
1 y =
2x
2
x
2
+ 1
x (1)
y
4
+ y
2
+ 1 3y
3y
2
y
4
+ y
2
+ 1
1 z =
3y
3
y
4
+y
2
+ 1
y (2)
z
6
+z
4
+z
2
+ 1 4z
3

4z
3
z
6
+ z
4
+z
2
+ 1
1 x =
4z
4
z
6
+ z
4
+ z
2
+ 1
z (3)
T (1),(2),(3) ta suy ra x = y = z = 1.
Vy h cho c nghim x = y = z = 0 hoc x = y = z = 1 2.
Bi 16: Gii h phng trnh

x
4
+ y
2
= 9
x
2
+ y
2
+ xy 3x 4y + 4 = 0
Gii
Gi s h phng trnh c nghim (x, y) Vit li phng trnh th hai theo x:
x
2
+x(y 3) + (y 2)
2
= 0
phng trnh c nghim x th

x
= (y 3)
2
4(y 2)
2
0 3y
2
+ 10y 7 0 1 y
7
3
(1)
Vit li phng trnh th hai theo y:
y
2
+ y(x 4) + x
2
3x + 4 = 0
phng trnh c nghim y th:

y
= (x 4)
2
4(x
2
3x + 4) 0 3x
2
+ 4x 0 0 x
4
3
(2)
T (1) v (2) x
4
+y
2

4
3

4
+

7
3

2
=
697
81
< 9 (mu thun)
Vy h phng trnh cho v nghim 2
Bi 17: Gii h phng trnh:

x +

y +

z =

2010
1
3x + 2y
+
1
3y + 2z
+
1
3z + 2x
=
1
x + 2y + 2z
+
1
y + 2z + 2x
+
1
z + 2x + 2y
254
Gii
tng: Do s n v phng trnh khng bng nhau nn ta s dng bt ng thc. Kh c
th so snh

x +

y +

z vi

2010, nn ta s khai thc phng trnh sau.


Li gii:
KX: x, y, z khng m v khng ng thi bng 0.
Theo BT Cauchy-Shwarz ta c:
1
3x + 2y
+
1
x + 2y + 2z
=
2(2x + 2y +z)
(3x + 2y)(x + 2y + 2z)

2(2x + 2y +z)
(2x + 2y +z)
2

1
3x + 2y
+
1
x + 2y + 2z

2
2x + 2y + z
Tng t ta cng c:

1
3y + 2x
+
1
y + 2x + 2z

2
x + 2y + 2z
1
3z + 2x
+
1
z + 2x + 2y

2
y + 2x + 2z
Suy ra:
1
3x + 2y
+
1
3y + 2x
+
1
3z + 2x

1
2x + 2y +z
+
1
x + 2y + 2z
+
1
y + 2x + 2z
Du = xy ra x = y = z.
Thay vo h ta c: 3

x =

2010 x =
670
3
Vy h phng trnh c nghim duy nht (x; y; z) = (
670
3
,
670
3
,
670
3
). 2
Bi 18: Gii h phng trnh:

x +a +

y +a +

z +a = 3

a
2
+ 1
a

a x +

a y +

a z = 3

a
2
1
a
Vi a > 1 v a l hng s.
Gii
t A =

x +a +

y +b +

z +a, B =

a x +

a y +

a z
p dng BT Cauchy-Schwarz:

A
2
3(3a +x +y + z)
B
2
3(3a x y z)
Cng v theo v ta c: A
2
+B
2
18a ()
Mt khc theo gi thit ta li c:
A
2
+B
2
= 9

a
2
+ 1
a
+
a
2
1
a

= 18a
Vy du = trong (*) xy ra

a +x =

a +y =

a + z =

a
2
+ 1
a

a x =

a y =

a z =

a
2
1
a
x = y = z =
1
a
255
Vy h phng trnh c nghim duy nht (x, y, z)=

1
a
,
1
a
,
1
a

. 2
Bi 19: Gii h phng trnh

1 + x
1
+

1 + x
2
+... +

1 + x
2010
= 2010

2011
2010

1 x
1
+

1 x
2
+ ... +

1 x
2010
= 2010

2011
2010
Gii
iu kin: -1 x
i
1 (i= 1, 2, 3, ..., 2010). Ta c
2010
2
.
2011
2010
= (

1 + x
1
+

1 + x
2
+... +

1 + x
2010
)
2
2010(2010 + x
1
+x
2
+... +x
2010
)
Suy ra x
1
+ x
2
+... +x
2010
1 (1)
Li c
2010
2
.
2011
2010
= (

1 x
1
+

1 x
2
+... +

1 x
2010
)
2
2010(2010 x
1
x
2
... x
2010
)
Do x
1
+ x
2
+... +x
2010
1 (2)
T (1) v (2) suy ra x
1
+x
2
+ ... + x
2010
= 1
Do h phng trnh cho tr thnh

1 + x
1
= 1 +x
2
= ... = 1 +x
2010
1 + x
1
= 1 +x
2
= ... = 1 +x
2010
x
1
+x
2
+ ... + x
2010
= 1
x
1
= x
2
= ... = x
2010
=
1
2010
Vy h phng trnh c nghim x
i
=
1
2010
(i = 1, 2010) 2
Bi 20: Gii h phng trnh:

x
+
1

y
+
1

z
= 3

3
x +y + z = 1
xy + yz +zx =
7
27
+ 2xyz()
Gii
tng: Bi ny c th gii bng cch bnh thng vn cho ta kt qu,nhng phng
trnh (*) chnh l BT IMO 1984. Nh th h c lin quan n BT. Phng php BT s
cho ta li gii p v gn hn trong bi ton ny.
Ta c:
() xy +yz +zx 2xyz =
7
27
Li c:
xyz (x +y z)(y + z x)(z +x y) = (1 2x)(1 2y)(1 2z)
9xyz 4(xy + yz +zx) 1 2xyz
8(xy + yz +zx) 2
9
Ta phi chng minh xy + yz + zx
1
3
=
(x + y +z)
2
3
(ng)
Du = xy ra x = y = z =
1
3
256
Th li vo h phng trnh thy tha mn.
Vy h phng trnh c nghim duy nht (x, y, z)=(
1
3
,
1
3
,
1
3
). 2
Bi 21: Gii h phng trnh

xy(x + y) = x
2
xy +y
2
1
x
3
+
1
y
3
= 16
Gii
iu kin x, y = 0.
t a =
1
x
v b =
1
y
T gi thit ta suy ra
1
ab

1
a
+
1
b

=
1
a
2

1
ab
+
1
b
2
a + b = a
2
ab +b
2

1
x
3
+
1
y
3
= a
3
+ b
3
= (a + b)(a
2
ab +b
2
) = (a + b)
2
Ta c bt ng thc sau: a
2
ab +b
2

(a + b)
2
4
a + b
(a + b)
2
4
0 a +b 4

1
x
3
+
1
y
3
= (a + b)
2
16
ng thc xy ra nn a = b x = y =
1
2
.
Vy h c nghim duy nht x = y =
1
2
2.
Bi 22: Gii h phng trnh

2009

i=1
x
i
= 2009
2009

i=1
x
8
i
=
2009

i=1
x
6
i
Gii
Gi s (x
1
, x
2
, . . . , x
2009
) l mt nghim ca h.
Khng gim tng qut gi s x
2
1
x
2
2
... x
2
2009
.
p dng bt ng thc BCS ta c
2009
2009

i=1
x
2
i

2009

i=1
x
i

2009

i=1
x
2
i
2009 (1)
p dng bt ng thc Chebysev cho cc b s (x
2
1
, x
2
2
, . . . , x
2
2009
) v (x
6
1
, x
6
2
, . . . , x
6
2009
) c
sp th t ta c

2009

i=1
x
2
i

2009

i=1
x
6
i

2009
2009

i=1
x
8
i
(2)
T (1) v (2) ta suy ra
2009

i=1
x
8
i

2009

i=1
x
6
i
(3)
ng thc xy ra x
1
= x
2
= ... = x
2009
= 1
Th li ta thy x
i
= 1 (i = 1, 2009) l nghim duy nht ca h phng trnh. 2
257
Bi tp t luyn
Bi 1: Gii h phng trnh

x
2
y
2
2x +y
2
= 0
2x
2
4x + 3 + y
3
= 0
Bi 2: Gii h phng trnh

xy
3
= 9
x + 3y = 6
Bi 3: Gii h phng trnh

x +

y +

z = 3
(1 + x)(1 + y)(1 + z) = (1 +
3

xyz)
3
Bi 4: Gii h phng trnh

3(x
2
+ y
2
+ z
2
) = 1
x
2
y
2
+y
2
z
2
+z
2
x
2
= xyz(x +y +z)
3
Bi 5: Gii h phng trnh

x
+
1

y
+
1

z
= 3

3
x +y +z = 1
Bi 6: Gii h phng trnh

y
7
+ 1 = (x + 1)(x
2
+ 1)(x
4
+ 1)
x
7
+ 1 = (y + 1)(y
2
+ 1)(y
4
+ 1)
Bi 7: Gii h phng trnh

30
y
x
2
+ 4y = 2007
30
z
y
2
+ 4z = 2007
30
x
z
2
+ 4x = 2007
Bi 8: Gii h phng trnh

2x
2
x
2
+ 1
= y
3y
3
y
4
+y
2
+ 1
= z
4z
4
z
6
+ z
4
+ z
2
+ 1
= t
5t
5
t
8
+t
6
+ t
4
+t
2
+ 1
= x
Bi 9: Gii h phng trnh

x
3
+ y = 3x + 4
2y
3
+z = 6y + 6
3z
3
+x = 9z + 8
Bi 10: Gii h phng trnh

x
2
+ y
2
= 1
125y
5
125y
3
+ 6

15 = 0
258
TNG HP CC BI H PHNG TRNH
H phng trnh hu t
Bi 1: Gii h phng trnh

x
2
4xy +x + 2y = 0 (1)
x
4
8x
2
y + 3x
2
+ 4y
2
= 0 (2)
Gii
Nhn thy phng trnh (1) c hng t x
2
+2y v phng trnh (2) li c hng t x
4
+4y
2
,Nn
ta ngh n vic phn tch c 2 phng trnh u c hng t x
2
+ 2y
H phng trnh tng ng

x
2
+ 2y +x 4xy = 0
x
4
+ 4x
2
y + 4y
2
+ 3x
2
12x
2
y = 0

(x
2
+ 2y) + x(1 4y) = 0
(x
2
+ 2y)
2
+ 3x
2
(1 4y) = 0
t x
2
+ 2y = a v 1 4y = b ta c HPT:

a +xb = 0
a
2
+ 3x
2
b = 0
x
2
b
2
+ 3x
2
b = 0 x
2
b(b + 3) = 0

x = 0
b = 0
b = 3
Vi x = 0 y = 0
Vi b = 0 y =
1
4
x
2
x +x +
1
2
= 0 (V nghim)
Vi b = 3 y = 1 x
2
4x + x + 2 = 0 x
2
3x + 2 = 0

x = 2
x = 1
Kt lun: H c nghim (x; y) l (0; 0); (2; 1); (1; 1) 2
Bi 2: Gii h phng trnh sau

81x
3
y
2
81x
2
y
2
+ 33xy
2
29y
2
= 4 (1)
25y
3
+ 9x
2
y
3
6xy
3
4y
2
= 24 (2)
Gii
Nhn thy VT(1) c nhn t y
2
chung. V th chia c hai v phng trnh (1) cho y
2
(y = 0)
ri bin i phng trnh (1) v dng f(t) = 0.T ta s bin i phng trnh (2) v dng
f(t

) = 0.
Nhn thy x = 0 hoc y = 0 khng tho h. Xt xy = 0.
H tng ng vi

81x
3
81x
2
+ 33x 29 =
4
y
2
24
y
3
+
4
y
= 25 + 9x
2
6x

3(3x 1)
3
+ 2(3x 1) = 24 +

2
y

2
3

2
y

3
+ 2.
2
y
= 24 + (3x 1)
2
t 3x 1 = a v
2
y
= b, ta c h i xng loi 2:

3a
3
+ 2a = 24 +b
2
3b
3
+ 2b = 24 +a
2
Tr v theo v ca h ta c
3(a b)(a
2
+ ab + b
2
) + 2(a b) + (a b)(a + b) = 0
(a b)(3a
2
+ 3ab + 3b
2
+ 2 + a + b) = 0
a = b
259
Thay vo mt trong hai phng trnh ca h ta c
3a
3
a
2
+ 2a 24 = 0 (a 2)(3a
2
+ 5a + 8) = 0 a = 2

3x 1 = 2
2
y
= 2

x = 1
y = 1
Kt lun: H c nghim (x; y) = (1; 1) 2
Bi 3: Gii h phng trnh

x
3
+ 3xy
2
= 6xy 3x 49 (1)
x
2
8xy +y
2
= 10y 25x 9 (2)
Gii
Xt (1) + (2).3 ta c
x
3
+ 3xy
2
6xy + 3x + 49 + 3x
2
24xy + 3y
2
30y + 75x + 27 = 0
(x + 1)(x
2
+ 2x + 3y
2
30y + 76) = 0
(x + 1)[(x + 1)
2
+ 3(y 5)
2
] = 0
x = 1; y = 5
Vy h c nghim duy nht (x; y) = (1; 5). 2
Nhn xt: Bi ton trn cng c th gii bi phng php n ph tng - hiu. C th t
x = a +b; y = a b.
Bi 4: Gii h phng trnh sau:

9y
3
(3x
3
1) = 125
45x
2
y + 75x = 6y
2
Gii
Vi x = 0 hoc y = 0 h v nghim
Vi x = 0 v y = 0 ta c h tng ng vi:

27x
3
+
125
y
3
= 9
45.
x
2
y
+ 75.
x
y
2
= 6

27x
3
+
125
y
3
= 9
3.x.
5
y

3x +
5
y

= 6
t a = 3x v b =
5
y
( b = 0).H tr thnh:

a
3
+b
3
= 9
ab(a +b) = 6

(a +b)
3
3ab(a +b) = 9
ab(a +b) = 6

a +b = 3
ab = 2

a = 1
b = 2

a = 2
b = 1
T tm c nghim ca h l (x; y) l

1
3
;
5
2

2
3
; 5

2
Bi 5: Gii h phng trnh sau

x
2
+y
2
+ 2x = 3
2

x
3
+y
3

+ 6x
2
= 3

x
2
+y
2

+ 5
260
Gii
H phng trnh tng ng:

(x + 1)
2
+ y
2
= 4
2x
3
+ 6x
2
+ 2y
3
= 3 (3 2x) + 5

(x + 1)
2
+y
2
= 4
2x
3
+ 6x
2
+ 6x + 2 + 2y
3
= 16

(x + 1)
2
+ y
2
= 4
(x + 1)
3
+ y
3
= 8
t a = x + 1 v b = y, H tr thnh:
()

a
2
+b
2
= 4
a
3
+b
3
= 8
n y ta s lp phng trnh thun nht t h trn:
()

a
2
+b
2

3
=

a
3
+ b
3

2
3a
2
b
2

a
2
+ b
2

2a
3
b
3
= 0
a
2
b
2

3a
2
+ 3b
2
2ab

= 0

a = 0
b = 0
Nu a = 0 : x = 1 y = 2
Nu b = y = 0 : x = 1
Vy h phng trnh c nghim (x; y) = (1; 2), (1; 0) 2
Bi 6: Gii h phng trnh

2y(x
2
y
2
) = 3x
x(x
2
+ y
2
) = 10y
Gii
Nu x = 0 hoc y = 0 th h c nghim (x; y) = (0; 0). Xt xy = 0, chia hai phng trnh v
theo v ta c:
2y(x
2
y
2
)
x(x
2
+y
2
)
=
3x
10y
20y
2
(x
2
y
2
) = 3x
2
(x
2
+ y
2
)
3x
4
17x
2
y
2
+ 20y
4
= 0 (x
2
4y
2
)(3x
2
5y
2
) = 0

x
2
= 4y
2
x
2
=
5y
2
3
Nu x
2
= 4y
2
ta c h

2y.3y
2
= 3x
x.5y
2
= 10y

2y
3
= x
xy = 2

x = 2; y = 1
x = 2; y = 1
Nu x
2
=
5y
2
3
ta c h

2y.
2y
2
3
= 3x
x.
8y
2
3
= 10y

4y
3
= 9x
4xy = 15

x =
15
2
4

135
; y =
4

135
2
x =
15
2
4

135
; y =

4

135
2
261
Vy h phng trnh c nghim (x; y) = (0; 0), (2; 1), (2; 1),

15
2
4

135
;
4

135
2

15
2
4

135
;

4

135
2

2
Bi 7: Gii h phng trnh

z
2
+ 2xyz = 1 (1)
3x
2
y
2
+ 3xy
2
= 1 +x
3
y
4
(2)
z +zy
4
+ 4y
3
= 4y + 6y
2
z (3)
Gii
V z = 0 khng l nghim ca h phng trnh nn (1) xy =
1 z
2
2z
t z = tan () vi

2
,

2

` 0 (a) cos = 0, sin = 0


Ta c: xy =
1 z
2
2z
=
1 tan
2

2 tan
= cot 2
Thay vo (2) ta c :
3cot
2
2 + 3y cot 2 = 1 +ycot
3
2 y =
3cot
2
2 1
cot
3
2 3 cot 2
=
1
cot 6
= tan 6
Ta suy ra: x = cot 2. cot 6
Thay vo (3) ta c :
z =
4 tan 6 4tan
3
6
1 6tan
2
6 + tan
4
6
= tan 24 ()
T ()v ()ta c:
tan 24 = tan 24 = +k, k Z =
k
23
, k Z
Vi

2
,

2

` 0ta thu c:
=

23
,
2
23
,
3
23
,
4
23
,
5
23
,
6
23
,
7
23
,
8
23
,
9
23
,
10
23
,
11
23
Vy h phng trnh c cc nghim l:
(x; y; z) = (cot 2. cot 6; tan 6; tan ) vi
=

23
,
2
23
,
3
23
,
4
23
,
5
23
,
6
23
,
7
23
,
8
23
,
9
23
,
10
23
,
11
23
2
Bi 8: Gii h phng trnh

x(y
2
+ 1)
x
2
+ y
2
=
3
5
y(x
2
1)
x
2
+ y
2
=
4
5
Gii
T h suy ra xy = 0 v x = 1. Bnh phng hai phng trnh v cng li ta c

x(y
2
+ 1)
x
2
+y
2

2
+

y(x
2
1)
x
2
+y
2

2
= 1
x
2
(y
2
+ 1)
2
+ y
2
(x
2
1)
2
= (x
2
+ y
2
)
2
(x
2
y
2
+ 1 x
2
y
2
)(x
2
+y
2
) = 0
(x
2
1)(y
2
1) = 0 y
2
1 = 0 y = 1
262
Vi y = 1 ta c h

2x
x
2
+ 1
=
3
5
x
2
1
x
2
+ 1
=
4
5
x = 3
Vi y = 1 ta c

2x
x
2
+ 1
=
3
5
x
2
1
x
2
+ 1
=
4
5
x =
1
3
Th li ta thy h c nghim (x; y) = (3; 1), (
1
3
; 1) 2
Bi 9: Gii h phng trnh

x +y +xy = z
2
2003
+ 2z
2
2002
x
4
+ y
4
= 2z
2
2004
(x + y)
z1
= (z + 2004)
xy
(I)
Gii
T h ta c:
2z
2
2004
= x
4
+ y
4
2x
2
y
2
xy z
2
2003
(1)
Li c
(x + y)
4
4

x
2
+y
2

2
4.2

x
4
+y
4

= 16z
2
2004
x +y 2z
2
2002
(2)
(1) v (2) cho ta:
x + y +xy z
2
2003
+ 2z
2
2002
Du =

xy ra khi v ch khi x = y = z
2
2002
. Khi ta c:
(I)

x = y = z
2
2002
(2x)
z1
= (z + 2004)
xy

x = y = z = 1

x = y =
1
2
z =
1
2
2002

2
Vy h c 3 nghim: (x; y; z) = (1; 1; 1) ,

1
2
;
1
2
;
1
2
2002

2
Bi 10: Gii h phng trnh

x +y +z + t = 15 (1)
x
2
+ y
2
+ z
2
+t
2
= 65 (2)
x
3
+ y
3
+ z
3
+t
3
= 315 (3)
xt = yz (4)
(I)
Gii
Ta c
(2) (x +t)
2
+ (y +z)
2
2xt 2yz = 65
(x +y +z + t)
2
2(x + t)(y +z) 4xt = 65(do(4))
(x + y +z + t)
2
2(x + t) [15 (x + t)] 4xt = 65(do(1))
15
2
2(x + t) [15 (x +t)] 4xt = 65
(x +t)
2
15(x +t) 2xt = 80(5)
263
Li c:
(3) (x +t)
3
+ (y +z)
3
3xt(x +t) 3yz(y +z) = 315
(x + t)
3
+ (y +z)
3
3xt(x +y + z +t) = 315(do(4))
(x + y +z +t)
3
3(x +t)(y +z)(x + y +z +t) 45xt = 315(do (1))
15
3
45(x + t) [15 (x + t)] 45xt = 315
(x + t)
2
15(x + t) xt = 68(6)
Ta c: (6) (5) xt = 12 (5)
Thay vo (5) ta c:
(x +t)
2
15(x +t) + 56 = 0

x +t = 8
x +t = 7
Kt hp (5) ta tm c (x; t) = (2; 6), (3; 4). T nghim ca h l
(x; y; z; t) = (6; 4; 3; 2), (6; 3; 4; 2), (2; 4; 3; 6), (2; 3; 4; 6), (4; 6; 2; 3), (4; 2; 6; 3), (3; 6; 2; 4), (3; 2; 6; 4) 2
Bi 11: Gii h phng trnh:

x
3
+y
3
+x
2
(y +z) = xyz + 14 (1)
y
3
+ z
3
+y
2
(x +z) = xyz 21 (2)
z
3
+ x
3
+z
2
(x + y) = xyz + 7 (3)
Gii
(1) + (2) + (3) x
3
+y
3
+z
3
+

x
2
+y
2
+z
2

(x +y +z) = 3xyz
(x +y + z)
3
3 (x +y +z) (xy + yz + zx) +

x
2
+ y
2
+ z
2

(x + y +z) = 0
(x + y +z)

x
2
+ y
2
+z
2
(xy +yz +zx) + x
2
+ y
2
+ z
2

= 0

x
2
+y
2
+z
2
(xy + yz +zx) + x
2
+y
2
+z
2
= 0 ()
x + y +z = 0 ()
T () ta c:

x
2
+y
2
+z
2
(xy + yz +zx) 0
x
2
+y
2
+z
2
0
V T
(5)
0
Du = xy ra x = y = z = 0
T (**) ta c: z = (x +y)
Thay vo (1) v (3) ta c h phng trnh sau:

y
3
+ xy (x +y) = 14
x
3
+xy (x + y) = 7
(I)
* Xt x = 0 (I)

y
3
= 14
0 = 7
(v nghim)
Xt x = 0. t: y = kx ta c:
(I)

x
3

k
3
+k
2
+k

= 14 (4)
x
3

k
2
+k + 1

= 7 (5)

k
3
+k
2
+k
k
2
+k + 1
= 2 k
3
k
2
k2 = 0 k = 2 y = 2x
264
Thay vo (5) ta c: x = 1 y = 2 z = 3
Vy h phng trnh c 1 nghim l: (x; y; z) = (1; 2; 3) 2
Bi 12: Gii h phng trnh sau:

x
3
+x(y z)
2
= 2
y
3
+y(z x)
2
= 30
z
3
+z(x y)
2
= 16
Gii
Ta a h v dng:

x(x
2
+ y
2
+z
2
2yz) = 2 (1)
y(x
2
+y
2
+z
2
2xz) = 30 (2)
z(x
2
+y
2
+z
2
2xy) = 16 (3)
Ly (1) + (2) 2(3) ta c:
(x +y 2z)

x
2
+y
2
+z
2

= 0

x +y 2z = 0 y = 2z x
x
2
+y
2
+z
2
= 0 x = y = z = 0 (l)
Thay y = 2z x vo phng trnh (1) v (3) ta c:
x(2x
2
+ z
2
2xz) = 2 (4)
z(4x
2
+ 5z
2
4xz) = 16(5)
t z = kx ta tm c k = 2
Vy h phng trnh cho c 1 nghim l: (x, y, z) = (1, 3, 2) 2
Bi 13: Gii h phng trnh

x
2
+y +x
3
y +xy
2
+xy =
5
4
(1)
x
4
+y
2
+xy(1 + 2x) =
5
4
(2)
(H khi A - 2006)
Gii
H phng trnh tng ng:

x
2
+y +xy + xy(x
2
+y) =
5
4
(x
2
+y)
2
+xy =
5
4
t u = x
2
+ y; v = xy ta c h

u +v + uv =
5
4
u
2
+ v =
5
4

v =
5
4
u
2
u
3
+u
2
+
u
4
= 0

u = 0, v =
5
4
u =
1
2
, v =
3
2
265
Vi u = 0, v =
5
4
ta tm c (x; y) =

5
4
;
3

25
16

Vi u =
1
2
, v =
3
2
ta tm c (x; y) =

1;
3
2

Vy h c nghim (x; y) =

5
4
;
3

25
16

1;
3
2

2
Bi 14: Gii h phng trnh

y
2
+z
2
x
2
=
4yz
x
z
2
+x
2
y
2
=
4zx
y
x
2
+ y
2
z
2
=
4xy
z
Gii
XK: x, y, z = 0
Nhn hai v ca cc phng trnh ln lt vi x
2
, y
2
, z
2
ta c:

x
2
y
2
+ x
2
z
2
x
4
= 4xyz
y
2
z
2
+x
2
y
2
y
4
= 6xyz
x
2
z
2
+ y
2
z
2
z
4
= 8xyz
Li c:
(1) 2 (2) + (3) 2y
4
y
2
z
2
x
2
y
2
x
4
+ 2x
2
z
2
z
4
= 0
y
2
(2y
2
x
2
z
2
) (x
2
z
2
)
2
= 0
y
2

2y
2

y
2
+
6xz
y

(x
2
z
2
)
2
= 0
y
4
6xyz (x
2
z
2
)
2
= 0
(y
2
x
2
+z
2
)(y
2
+x
2
z
2
) 6xyz = 0

4yz
x

8xy
z
6xyz = 0
32y
2
6xyz = 0 16y 3xz = 0
Thay xz =
16
3
vo (2) v y =
3xz
16
vo (1) v (3) ta c:

x
2
+y
2
+
1
4
z
2
= 0
x
2
y
2
+z
2
= 32
1
2
x
2
+y
2
z
2
= 0
y l h phng trnh tuyn tnh theo (x
2
; y
2
; z
2
), ta tm c (x
2
, y
2
, z
2
) =

64,
288
5
,
128
5

T h ban u ta thy vi nhng gi tr (x


2
; y
2
; z
2
) trn, cc hng t
yz
x
,
zx
y
,
xy
z
u dng.
Gi s x < 0, t (1) ta c yz < 0. Tng t vi y < 0 v z < 0.
Vy nghim ca h l (x, y, z) =

8, 12

2
5
, 8

2
5

8, 12

2
5
, 8

2
5

,
266

8, 12

2
5
, 8

2
5

8, 12

2
5
, 8

2
5

v cc hon v 2
Bi 15: Gii h phng trnh

2z (x + y) + 1 = x
2
y
2
(1)
y
2
+z
2
= 1 + 2xy + 2zx 2yz (2)
y

3x
2
1

= 2x

x
2
+ 1

(3)
Gii
V x =
1

3
khng tha phng trnh (3)nn:
(3) y =
2x (x
2
+ 1)
3x
2
1
x +y =
3x
3
x 2x (x
2
+ 1)
3x
2
1
x +y =
x
3
3x
3x
2
1
t x = tan ,

2
;

2

6
;

6

cos = 0, cos 3 = 0
Ta c:
tan +y =
tan
3
3 tan
3tan
2
1
y = tan 3 tan
(1) z =
x
2
y
2
1
2 (x +y)
(do x = ykhng tha phng trnh (1) tan 3 = 0)
z =
(2 tan tan 3) . tan 3 1
2 tan 3
=
2 tan . tan 3 tan
2
3 1
2 tan 3
z = tan
tan 3 + cot 3
2
= tan
1
2

sin 3
cos 3
+
cos 3
sin 3

z = tan
1
sin 6
(2) x
2
+y
2
+z
2
2xy 2zx + 2yz = 1 +x
2
(y + z x)
2
= 1 +x
2

tan 3 tan + tan


1
sin 6
tan

2
= 1 + tan
2

sin 3
cos 3

1
2 sin 3. cos 3
tan

2
=
1
cos
2

2sin
2
3 1
2 sin 3. cos 3
tan

2
=
1
cos
2

cos 6
sin 6
+ tan

2
=
1
cos
2

cos 6. cos + sin 6. sin


sin 6. cos

2
=
1
cos
2

cos 5
sin 6. cos

2
=
1
cos
2

cos 5 = sin 6 cos 5 = cos

2
6

cos 5 = cos

2
6

cos 5 = cos

2
+ 6

5 =

2
6

+k2
5 =

2
+ 6

+ k2

=

22
+
k2
11
, =

2
k2
=

22
+
k2
11
, =

2
k2
(k Z)
Vi:

2
;

2

6
;

6

=

22
;
3
22
;
5
22
;
7
22
;
9
22
Vy h phng trnh cho c nghim l:
267
(x; y; z) =

tan ; tan 3 tan ; tan


1
sin 6

, =

22
;
3
22
;
5
22
;
7
22
;
9
22
2
Bi 16: Gii h phng trnh

(x + 2)
2
+ (y + 3)
2
= (y + 3) (x + z 2)
x
2
+ 5x + 9z 7y 15 = 3yz
8x
2
+ 18y
2
+ 18xy + 18yz = 84x 72y 24z 176
(I)
Gii
t:a = x + 2; b = y + 3 ta c:
(I)

a
2
+ab +b
2
+ bz 4b = 0 (1)
a
2
+a 7b + 3bz = 0 (2)
8a
2
2a + 18

b
2
+ab +bz 4b

30z + 94 = 0 (3)
Li c:
(1) b
2
+ ab + bz 4b = a
2
()
Thay ()vo (3) ta c:
8a
2
2a 18a
2
30z + 94 = 0
10a
2
+ 2a + 30z 94 = 0
z =
5a
2
+a 47
15
Thay z =
5a
2
+ a 47
15
vo (2)ta c:
a
2
+ a 7b b

5a
2
+a 47
5

= 0

5a
2
+a 12
15

b = a
2
+ a
b =
5 (a
2
+a)
5a
2
+a 12
(V a =
1

241
10
khng l nghim ca phng trnh)
Nhn 2 v ca phng trnh (1)vi 3 ri tr cho phng trnh (2)v theo v, ta c:
2a
2
a + 3ab + 3b
2
5b = 0 (4)
Thay b =
5 (a
2
+a)
5a
2
+ a 12
vo (4) ta c:
2a
2
a +
15a (a
2
+ a)
5a
2
+a 12
+ 3

5 (a
2
+a)
5a
2
+a 12

25 (a
2
+ a)
5a
2
+a 12
= 0

2a
2
a

5a
2
+ a 12

2
+

15a

a
2
+ a

25

a
2
+a

5a
2
+a 12

+ 75

a
2
+a

2
= 0
50a
6
+ 70a
5
208a
4
94a
3
+ 482a
2
+ 156a = 0
25a
6
+ 35a
5
104a
4
47a
3
+ 241a
2
+ 78a = 0
a (a + 2)

25a
4
15a
3
74a
2
+ 101a + 39

= 0
a (a + 2)

5a
2
14a + 13

5a
2
+ 11a + 3

= 0
a = 0 a = 2 a =
11

61
10
268
TH1: a = 0 :

b = 0
z =
47
15

x = 2
y = 3
TH2: a = 2 :

b =
5
3
z =
29
15

x = 4
y =
4
3
TH3: a =
11 +

61
10
:

b =
17 2

61
15
z =
39 +

61
15

x =
31 +

61
10
y =
28 + 2

61
15
TH4: a =
11

61
10
:

b =
17 + 2

61
15
z =
39

61
15

x =
31 +

61
10
y =
28 + 2

61
15
Vy h phng trnh cho c 4 nghim l:
(x; y; z) =

2; 3;
47
15

4;
4
3
;
29
15

31 +

61
10
;
28 + 2

61
15
;
39 +

61
15

31 +

61
10
;
28 + 2

61
15
;
39

61
15

2
Bi 17: Cho cc tham s dng a, b, c. Tm nghim dng ca h

x +y + z = a +b + c (1)
4xyz a
2
x b
2
y c
2
z = abc (2)
Gii
Ta c:
(2)
a
2
yz
+
b
2
xz
+
c
2
xy
+
abc
xyz
= 4 (3)
t: x
1
=
a

yz
; y
1
=
b

xz
; z
1
=
c

xy
ta c:
(3) x
2
1
+y
2
1
+z
2
1
+x
1
.y
1
.z
1
= 4 (4)
D thy: 0 < x
1
, y
1
, z
1
< 2 nn tn ti cc gi tr u, vtha: 0 < u, v <

2
v x
1
= 2 sin u; y
1
=
2 sin v
. Khi :
(4) z
2
1
+ 4z
1
. sin u. sin v + 4sin
2
u + 4sin
2
v 4 = 0
C

= (2 sin u. sin v)
2

4sin
2
u + 4sin
2
v 4

= 4

1 sin
2
u

1 sin
2
v

= 4cos
2
u.cos
2
v > 0
Vy
(4)

z
1
= 2 sin u. sin v 2 cos u. cos v < 0
z
1
= 2 sin u. sin v + 2 cos u. cos v > 0
269
Do : a = 2

yz. sin u; b = 2

zx. sin v; c = 2

xy (cos u. cos v sin u. sin v)


Thay vo (1) ta c:
x + y +z = 2

yz. sin u + 2

zx. sin v + 2

xy (cos u. cos v sin u. sin v)

x cos v

y cos u

2
+

x sin v +

y sin u

2
= 0

x cos v

y cos u =

x sin v +

y sin u

2 = 0
Ta tnh c:

z =

x sin v +

y sin u =
b

x
2

zx
+
a

y
2

yz
=
a + b
2

z
z =
a + b
2
Tng t, ta cng c: y =
c + a
2
; x =
b +c
2
Vy h phng trnh cho c 1 nghim l: (x; y; z) =

b +c
2
;
c +a
2
;
a +b
2

2
Bi 18: Gii h phng trnh

xz = x + 8 (1)
4y
2
= 7xz 3x 28 (2)
x
2
+ 4z
2
= 140 4y
2
(3)
Gii
Ta c (1) x = xz 8
Thay vo (2) ta c: 4y
2
= 7xz 3 (xz 8) 28 y
2
= xz 1 (y
2
0 xz 1)
Thay vo (3) ta c:
x
2
+ 4z
2
= 140 4 (xz 1) (x + 2z)
2
= 144

x + 2z = 12
x + 2z = 12

x = 2z + 12 (4)
x = 2z 12 (5)
Thay (4) vo (1) ta c:
z (2z + 12) = 2z + 12 + 8 z
2
7z + 10 = 0

z = 2 x = 8 y
2
= 15 y =

15
z = 5 x = 2 y
2
= 9 y = 3
Thay (5) vo (1) ta c:
z (2z 12) = 2z 12 + 8 z
2
+ 5z 2 = 0

z =
5 +

33
2
x = 7

33 ()
z =
5

33
2
x = 7 +

33 y
2
=

33 y =
4

33
Loi (*) do xz < 0. Vy h phng trnh c 6 nghim l:
(x; y; z) =

8;

15; 2

, (2; 3; 5) ,

7 +

33;
4

33;
5

33
2

2
Bi 19: Gii h phng trnh

x +y +z = 0(1)
x
2
+y
2
+z
2
= 10(2)
x
7
+y
7
+z
7
= 350(3)
270
Gii
T (1), (2) ta c xy +yz +zx = 5 (3)
Li c: (1) x
3
+ y
3
+z
3
= 3xyz(4)
(2) (4)

x
2
+y
2
+z
2

x
3
+y
3
+ z
3

= 30xyz
x
5
+ y
5
+z
5
+x
2
y
2
(x +y) + y
2
z
2
(y +z) + x
2
z
2
(x +z) = 30xyz
x
5
+y
5
+z
5
x
2
y
2
z xy
2
z
2
x
2
yz
2
= 30xyz
x
5
+y
5
+z
5
xyz(xy +yz + zx) = 30xyz
x
5
+y
5
+z
5
+ 5xyz = 30xyz x
5
+y
5
+z
5
= 25xyz
(2) (5)

x
2
+y
2
+z
2

x
5
+ y
5
+ z
5

= 250xyz
x
7
+y
7
+z
7
+x
2
y
2
(x
3
+y
3
) + y
2
z
2
(y
3
+ z
3
) + z
2
x
2
(z
3
+x
3
) = 250xyz
x
7
+y
7
+z
7
+x
2
y
2
(3xyz z
3
) + y
2
z
2
(3xyz x
3
) + z
2
x
2
(3xyz y
3
) = 250xyz
x
7
+y
7
+z
7
+ 3xyz(x
2
y
2
+y
2
z
2
+z
2
x
2
) x
2
y
2
z
2
(x +y + z) = 250xyz
x
7
+y
7
+z
7
+ 3xyz

(xy +yz + xz)


2
2xyz(x +y +z)

= 250xyz
x
7
+y
7
+z
7
+ 45xyz = 250xyz
x
7
+y
7
+z
7
= 135xyz(6)
Thay (6) vo (3) ta c: 135xyz = 350 xyz = 2. Vy ta c:

x +y + z = 0
xy + xz +yz = 5
xyz = 2
Suy ra x, y, zl nghim ca phng trnh t
3
5t 2 = 0 t = 1 +

2, t = 2, t = 1

2
Vy h c nghim l

1 +

2; 2; 1

v cc hon v 2.
Bi 20: Gii h phng trnh ()

30x 4y +
1
xy
= 2012
4y 2012z +
1
zx
= 30
2012y 30z +
1
yz
= 4
Gii
tng: R rng cc s 30, 4, 2012 c chn ngu nhin do ta c th t 30 = a; 4 =
b; 2012 = c thy r hn tng bi ton. Khi , nh ni trn, ta cn tnh a, b, c
theo x, yz. D thy h thu c l mt h tuyn tnh 3 n vi x, y, z l tham s. C th dng
cng thc Cramer tnh nghim, nhng do trong chng trnh ph thng khng c cng thc
Cramer cho h 3 n nn ta s a v h 2 n.
Li gii:
t 30 = a; 4 = b; 2012 = c ta c HPT:
(II

ax by +
1
xy
= c (1)
by cz +
1
zx
= a (2)
cy az +
1
yz
= b (3)

c = ax by +
1
xy
bz (ax by +
1
xy
).x +
1
zx
= a (4)
(ax by +
1
xy
).y az +
1
yz
= b (5)
271
Xt (4), (5) l 1 h 2 n theo a, b. Dng cng thc Cramer ta tnh c:

D =

x
2
+ 1 xy z
xy z y
2
1

= (1 + x
2
+y
2
+z
2
) = 0
D
a
=

1
xy

1
y
xy z
1
yz

1
x
y
2
1

=
(1 + x
2
+y
2
+z
2
)
xz
D
b
=

x
2
+ 1
1
xy

1
y
xy z
1
xy

1
x

=
(x
2
+y
2
+z
2
+ 1)
yz

a =
D
a
D
=
1
xz
b =
D
b
D
=
1
yz
c =
1
xy
Tr li HPT u ta c

xz =
1
30
yz =
1
4
xy =
1
2012

x =
1

15090
y =

15
4024
z =

503
30
Vy () c nghim (x; y; z) =

15090
;

15
4024
;

503
30

2
Bi 21: Gii h phng trnh ()

x
4
+ 4x
2
+ y
2
4y = 2
x
2
y + 2x
2
+ 6y = 23
Gii
H tng ng

t 4y = 2 x
4
4x
2
y(x
2
+ 6) = 23 2x
2
vi t = y
2
Ta tnh c

D = x
2
+ 6
D
t
= x
6
10x
4
30x
2
+ 104
D
y
= 23 2x
2
Do t = y
2
nn
D
t
D
=

D
y
D

2
(x
2
+ 6)(x
6
10x
4
30x
2
+ 104) = (23 2x
2
)
2
(1 x)(1 + x)(1 + x
2
)(x
4
+ 16x
2
+ 95) = 0 x = 1
T ta tm c nghim ca h l (x; y) = (1; 3), (1; 3) 2
Bi 22: Gii h phng trnh

x
4
+x
3
y + 9y = y
3
x +x
2
y
2
+ 9x
x(y
3
x
3
) = 7
Gii
Ta c:
x
4
+ x
3
y + 9y = y
3
x + x
2
y
2
+ 9x (x
4
xy
3
) + (x
3
y x
2
y
2
) 9(x y) = 0
(x y)

x(x
2
+xy + y
2
) + x
2
y 9

= 0 (x y)

x(x + y)
2
9

= 0
272
T phng trnh th hai ca h, ta thy x = y nn t bin i trn, suy ra:
x(x +y)
2
9 = 0 x(x +y)
2
= 9 ()
Ta c: x(y
3
x
3
) = 7 y
3
x
3
=
7
x
y =
3

x
3
+
7
x
.
Thay vo (*), ta c:
x(x +
3

x
3
+
7
x
)
2
= 9
Ta s chng minh rng v tri l mt hm ng bin theo bin x. Tht vy:
x(x +
3

x
3
+
7
x
)
2
= x

x
2
+ 2x
3

x
3
+
7
x
+
3

x
3
+
7
x

= x
3
+ 2x
2
.
3

x
3
+
7
x
+x
3

x
3
+
7
x

2
= x
3
+ 2x
3

x
6
+ 7x
2
+
3

x(x
4
+ 7)
2
T (*) suy ra x > 0 v trong biu thc trn, cc s m ca bin x u dng nn y l hm
ng bin; suy ra n c khng qu mt nghim.
Thay trc tip x = 1 vo biu thc, ta thy tha.
Vy h cho c ng mt nghim l: (x, y) = (1, 2) 2.
Nhn xt: im c bit ca bi ny l x l c h phng trnh mi sau khi bin i, nu
nh ta dng cch i s trc tip, phn tch ra c mt nghim x = 1 th phng trnh bc
cao cn li kh m gii c. Cch lp lun theo tnh n iu ca hm s th ny va trnh
c iu va lm cho li gii nh nhng hn.
Bi 23 : Gii h phng trnh

x y + z w = 2
x
2
y
2
+z
2
w
2
= 6
x
3
y
3
+z
3
w
3
= 20
x
4
y
4
+z
4
w
4
= 66
(China TST 2006)
Gii
t

x +z = a
xz = b
y +w = c
yw = d
ta c HPT:

a c = 2 (1)
a
2
2b c
2
+ 2d = 6 (2)
a
3
3ab c
3
+ 3cd = 20 (3)
a
4
4a
2
b + 2b
2
c
4
+ 4c
2
d 2d
2
= 66 (4)
T (1) v (2) ta c

c = a 2
d = 2a + b + 5
T (3) v (4) c

5a 2b 14 = 0
5a
2
2ab 10a b 13 = 0

5a 2b = 14
4a b = 13

a = 4
b = 3

c = 2
d = 0
273
Th ngc li ta tm c nghim (x; y; w; z) ca h l (1; 0; 3; 2) v cc hon v. 2
Bi 24: Gii h phng trnh

(2x
2
3x + 4)(2y
2
3y + 4) = 18
x
2
+y
2
+xy 7x 6y + 14 = 0
Gii
Xt x
2
+ y
2
+xy 7x 6y + 14 = 0 () l phng trnh bc hai theo bin x, vit li l:
x
2
+x(y 7) + y
2
6y + 14 = 0
Phng trnh ny c nghim khi:

y
= (y 7)
2
4(y
2
6y + 14) 0 3y
2
+ 10y 7 0 1 y
7
3
Hon ton tng t, xem (*) l phng trnh bc hai theo bin y, vit li l:
y
2
y(x 6) + (x
2
7x + 14) = 0
Phng trnh ny c nghim khi:

x
= (x 6)
2
4(x
2
7x + 14) 0 3x
2
+ 16x 20 0 2 x
10
3
Ta xt hm s: f(t) = 2t
2
3t + 4(t R) f

(t) = 4t 3 = 0 t =
3
4
< 1.
Suy ra, trn [1, +), hm s ny ng bin. Li c f(x) f(2) = 6, f(y) f(1) = 3
f(x).f(y) 3.6 = 18.
T phng trnh th nht ca h th ta thy ng thc phi xy ra, tc l x = 2, y = 1.
Thay hai gi tr ny vo (*), ta thy khng tha.
Vy h phng trnh cho v nghim. 2
Nhn xt: tng gii ca bi ny khng kh v cng kh quen thuc khi ch cn tm
min xc nh ca bin thng qua vic tnh Delta ca mt phng trnh bc hai; tuy trong
li gii trn c kho st hm s nhng thc ra cc kt qu c th chng minh bng bt
ng thc i s thun ty nn cng c gii chnh ca bi ny l i s. V do vic hai
biu thc ca x v y phng trnh u ca h ging nhau c th dn n nh gi sai hng
m dng gii tch, xt hm s khai thc phng trnh u tin trong khi iu khng
em li kt qu g. Cc h s c chn ra s rt p chnh l u im ni bt ca bi ton ny.
Bi 25: Gii h phng trnh

x
4
+ y
4
+ xy = 2xy
2
+ 7 (1)
x
2
y + 4xy +xy
3
+ 11(x y
2
) = 28 (2)
Gii
Xt (1).4 (2) x
2
(y + 4) x(y
3
+ 8y
2
+ 11) + 4y
4
+ 11y
2
= 0 ().
Nu y = 4 ta c phng trnh 27x + 1200 = 0 x = 16. Th li thy (x; y) = (16 4)
274
khng tho h.
Vy y = 4. Khi , coi (*) l phng trnh bc 2 theo x, ta tm c nghim
x
1,2
=
y
3
+ 8y
2
+ 11

(y
3
+ 8y
2
+ 11)
2
4(y + 4)(4y
4
+ 11y
2
)
2(y + 4)
x
1,2
=
y
3
+ 8y
2
+ 11 (y
3
11)
2(y + 4)
x
1
= y
2
; x
2
=
4y
2
+ 11
y + 4
Vi x = y
2
thay vo (1) ta c y
3
= 7 y =
3

7 = x =
3

49. Vi x =
4y
2
+ 11
y + 4
thay vo
phng trnh u v rt gn ta c
y
6
+4y
4
6y
3
+4y
2
12y+9 = 0 (y
3
+2y3)
2
= 0 (y1)(y
2
+y+3) = 0 = y = 1 = x = 3
Vy h c nghim (x; y) = (
3

49;
3

7), (3; 1) 2
Bi 26: Gii h phng trnh

x
2
+ 3y
2
= 4
x
4
+ 9y
4
= 10
Gii
T h phng trnh ta c
(x
2
+ 3y
2
)
2
(x
4
+ 9y
4
) = 6x
2
y
2
= 6 = x
2
y
2
= 1
Nu xy = 1 ta c:
x
2
+ 2

3xy + 3y
2
= 4 + 2

3 = x + y

3 =

4 + 2

3 = (

3 + 1)
Li c:
y =
1
x
= x
2
(

3 + 1)x +

3 = 0 x = 1, x =

3; y =
1
x
Nu xy = 1 ta c
x
2
+ 2

3xy + 3y
2
= 4 2

3 = x +y

3 =

4 2

3 = (

3 1)
Li c
y =
1
x
= x
2
(

3 1)x

3 = 0 x = 1, x =

3; y =
1
x
Vy h phng trnh c nghim (x; y) = (1, 1), (1, 1) and

3,
1

3,
1

2
Bi 27: Gii h phng trnh
() (x + y)(z + 1) = (x +z)(y + 1) = (y +z)(x + 1) = 2a(a + 1)
(2012 Philippine Math Olympiad National Finals Oral Round)
Gii
275
t

x +yz = u
y +zx = v
z +xy = w
ta c h phng trnh
u + v = v +w = w + u = 2a(a + 1)
H ny c nghim u = v = w = a(a + 1) do h (*) tng ng
x +yz = y +zx = z + xy = a(a + 1)

(x y) = z(x y)
(y z) = x(y z)
x + zx = a(a + 1)

x = y z = 1
y = z x = 1
x + yz = a(a + 1)
Tm li ta c s sau:

x = y
y = z = x = y = z a, (a + 1)
` x = 1 = x = y = 1 ; z = a
2
+a 1
z = 1
y = z = y = z = 1 ; x = a
2
+a 1
` x = 1 = x = z = 1 ; y = a
2
+a 1

Vy h c nghim (x; y; z) = (a; a; a), (a1; a1; a1), (1; 1; a


2
+a1) v cc hon v. 2
Bi 28: Cho h phng trnh

x +y +z = a (1)
x
2
+y
2
+z
2
= b
2
(2)
xy = z
2
(3)
a) Gii h trn vi n (x; y).
b) Cc s a, b phi tho iu kin g cc nghim x, y, z ca h dng v khc nhau?
Gii
Cu a) Bnh phng 2 v ca (1): a
2
= x
2
+y
2
+z
2
+ 2xy + 2yz + 2zx
x +y = a z, kt hp (2) v (3) ta c:
a
2
= b
2
+ 2z
2
+ 2(a z).z
hay
a
2
= b
2
+ 2az
276
T :
z =
a
2
b
2
2a
T y tm c x, y bi cc h thc: x +y = a z =
a
2
+b
2
2a
; xy = z
2
=
(a
2
b
2
)
2
4a
2

x =
a
2
+ b
2
4a

10a
2
b
2
3a
4
3b
4
4a
y =
a
2
+b
2
4a

10a
2
b
2
3a
4
3b
4
4a
Cu b) x, y > 0 th x + y =
a
2
+b
2
2a
> 0 a > 0
z > 0
a
2
b
2
2a
> 0 a > [b[
x = y cn c 10a
2
b
2
3a
4
3b
4
> 0 1 >
[b[
a
>
1

3
a > [b[ >
a

3
()
(*) chnh l iu kin cn c. 2
Bi 29: Gii h phng trnh
()

uv
3
+vy
3
= 14
ux
2
+vy
2
= 5
ux + vy = 2
u + v = 1
Gii
t a
k
= ux
k
+ vy
k
. rng
a
k+1
= (x +y)a
k
xya
k1
t A = x +y; B = xy. Ln lt xt k = 2, 3 t h (*) ta c:

5 = 2A B
14 = 5A 2B

A = 4
B = 3
Do x, y l nghim ca phng trnh t
2
4t + 3 = 0.Nh vy (x; y) = (1; 3), (3; 1).
Th vo h (*) ta c

u +v = 1
u + 3v = 2
u = v =
1
2
Vy h (*) c nghim (x; y; u; v) =

1; 3;
1
2
;
1
2

3; 1;
1
2
;
1
2

2
Bi 30: Gii h phng trnh
a
2
+bc ab = b
2
+ca bc = c
2
+ab ca
Gii
277
Vi c = 0 ta tm c nghim (a; b; c) = (0; 0; 0)
Vi c = 0, nu (a; b; c) l nghim ca h th

a
c
;
b
c
; 1

cng l nghim (do cc phng trnh l


thun nht). Vy khng gim tng qut gi s c = 1.
Khi ta c h phng trnh
a
2
+b ab = b
2
+a b = 1 +ab a
T a
2
+ b ab = 1 +ab a, ta ca
2
+ a 1 = b(2a 1) b =
a
2
+a 1
2a 1
(d thy a =
1
12
)
Thay vo phng trnh a
2
+b ab = b
2
+a b ta c (a 1)(a
3
6a
2
+ 5a 1) = 0
Nu a = 1 ta c nghim (a; b; c) = (1; 1; 1).
Trng hp cn li, a l nghim ca x
3
6x
2
+5x 1 = 0. C th gii phng trnh ny bng
cch t t =
2

3
y + 2 (xem cch gii tng qut Phng trnh bc 3). T ta tm c
nghim
2 +
2

3
cos
t + 2k
3
vi t = arccos
3

3
2

7
v k = 1, 2
Nh vy h phng trnh c nghim (c, c, c) vi c R v

2 +
2

3
cos
t + 2k
3

, c f

2 +
2

3
cos
t + 2k
3

, c

(trong c R, t = arccos
3

3
2

7
, k = 0, 2 v f(x) =
x
2
+ x 1
2x 1
) 2
H phng trnh v t
Bi 1: Gii h phng trnh

x +

x
2
2x + 2 = 3
y1
+ 1
y +

y
2
2y + 2 = 3
x1
+ 1
Gii
Do nhm c nghim x = y = 1 nn ta s gii bng hm s v chng minh n l nghim duy
nht ca h.
t

x 1 = a
y 1 = b
, khi h phng trnh tng ng

a +

a
2
+ 1 = 3
b
b +

b
2
+ 1 = 3
a
Nhn thy y l h i xng theo 2 n a v b, tr v theo v ta c
a +

a
2
+ 1 + 3
a
= b +

b
2
+ 1 + 3
b
Ta xt hm s c trng f(t) = t +

t
2
+ 1 + 3
t
c f

(t) =

t
2
+ 1 + t

t
2
+ 1
+ 3
t
ln 3
V

t
2
+ 1 >

t
2
t nn f(t) > 0t Vy hm s ng bin trn R nn a = b
Thay vo phng trnh ta c a +

a
2
+ 1 = 3
a
ln(

a
2
+ 1 + a) a ln 3 = 0
Xt hm s G(a) = ln(

a
2
+ 1 + a) a ln 3
Ta c G

(a) =
1

a
2
+ 1
ln 3 < 1 ln 3 < 0a R
278
Nn hm s nghch bin v ta nhn thy a = 0 l nghim nn phng trnh c duy nht.
Vy h phng trnh cho c nghim l (x; y) = (1; 1) 2
Bi 2: Gii h phng trnh

x +y

xy = 3

x + 1 +

y + 1 = 4
( thi H khi A - 2006)
Gii
KX: x 1, g 1, xy 0
t t =

xy 0. T phng trnh u ta c x +y = 3 +t ()
Bnh phng hai v ca phng trnh sau ta c
x +y + 2 + 2

xy +x +y + 1 = 16 ()
Thay (*) vo (**) ta c
3 + t + 2 + 2

t
2
+ 3 + t + 1 = 16 2

t
2
t + 4 = 11 t

0 t 11
4(t
2
+t + 4) = (11 t)
2

0 t 11
3t
2
+ 26t 105 = 0
t = 3
T tm c nghim ca h l (x; y) = (3; 3) 2
Bi 3: Gii h phng trnh

x +

x
2
+y + 3 = 2
2

x + 4 + 3

y + 8 = 13
Gii
K: x 0; x
2
+ y + 3 0; y + 8 0
T phng trnh (2) s dng BT Cauchy-Schwarz ta c:
13
2
=

x + 4 + 3

y + 8

2
13 (x + 4 + y + 8) x +y 1 ()
T phng trnh (1), bnh phng hai v ta c:
x +x
2
+y + 3 + 2

x (x
2
+y + 3) = 4 x +y = 1 x
2
2

x (x
2
+y + 3) x +y 1 ()
T (*) v (**) suy ra x +y = 1
T du bng xy ra cc BT nn

x = 0

x + 4
2
=

y + 8
3

x = 0
y = 1
Th li ta thy h c nghim duy nht (x; y) = (0; 1) 2
279
Bi 4: Gii h phng trnh sau vi z 0:
()

(3 x)
2003
= y + 2
log
3
1
2z y
+ log1
3
(y + 2) = log 1

9 + 4y
log
2

x
2
+ z
2

= 2 + log
2
x
Gii
K: x > 0; 2z > y; y > 2
Ta c:
()

(3 x)
2003
= y + 2
log
3
(2z y) log
3
(y + 2) = log
3
(9 + 4y)
log
2

x
2
+z
2

= log
2
4x

(3 x)
2003
= y + 2
(2z y) . (y + 2) = 9 + 4y
x
2
+ z
2
= 4x

(3 x)
2003
= y + 2
y
2
+ 9 + z
2
+ 6y 2yz 6z = z
2
2z
x
2
4x + 4 = 4 z
2

(3 x)
2003
= y + 2 (1)
(y + 3 z)
2
= z
2
2z (2)
(x 2)
2
= 4 z
2
(3)
Nu (x
0
, y
0
, z
0
) l nghim ca h ta c:
(x
0
2)
2
= 4 z
0
2
4 z
0
2
0 2 z
0
2 (4)
(y
0
+ 3 z
0
)
2
= z
0
2
2z
0
z
0
2
2z
0
0 z
0
0 z
0
2 (5)
Kt hp vi iu kin bi ton l z
0
0vi (4) v (5) ta c: z
0
= 0 z
0
= 2
Vi z
0
= 0 t (2) v (3) ta c

x
0
= 0
y
0
= 3

x
0
= 4
y
0
= 3
khng tha iu kin bi ton
Vi z
0
= 2 t (2) v (3) ta c

x
0
= 2
y
0
= 1
Th li ta thy h phng trnh c nghim duy nht l: (x; y; z) = (2; 1; 2) 2
Bi 5: Gii h phng trnh

x
+
y
x
=
2

x
y
+ 2
y(

x
2
+ 1 1) =

3x
2
+ 3
Gii
iu kin xc nh: x > 0, y = 0.
Phng trnh th nht ca h tng ng vi:
1

x
+
y
x
=
2

x
y
+ 2 y

x +y
2
= 2x

x + 2xy y
2
+y(

x 2x) 2x

x = 0
280
Xem y l phng trnh bc hai theo bin y, ta c:

x
= (

x 2x)
2
+ 8x

x = x + 4x

x + 4x
2
= (

x + 2x)
2
> 0
Do , phng trnh ny c hai nghim l:
y
1
=
(2x

x) (

x + 2x)
2
=

x, y
2
=
(2x

x) + (

x + 2x)
2
= 2x,
Xt hai trng hp:
Nu y =

x, thay vo phng trnh th hai ca h, ta c:

x(

x
2
+ 1 1) =

3x
2
+ 3
D thy:

x(

x
2
+ 1 1) < 0 <

3x
2
+ 3 nn phng trnh ny v nghim.
Nu y = 2x, thay vo phng trnh th hai ca h, ta c:
2x(

x
2
+ 1 1) =

3x
2
+ 3

x
2
+ 1.(2x

3) = 2x

x
2
+ 1 =
2x
2x

3
()
(d thy x =

3
2
khng tha mn ng thc nn ch xt x =

3
2
v php bin i trn l ph
hp).
Xt hai hm s: f(x) =

x
2
+ 1, x > 0 v g(x) =
2x
2x

3
, x > 0.
Ta c: f

(x) =
x

x
2
+ 1
> 0 nn l hm ng bin, g

(x) =
2

3
(2x

3)
2
< 0 nn l hm nghch
bin. Suy ra phng trnh (*) c khng qu mt nghim.
Nhm thy x =

3 tha mn (*) nn y cng chnh l nghim duy nht ca (*).


Vy h cho c nghim duy nht l (x, y) = (

3, 2

3) 2.
Nhn xt: Quan h ca x v y c che giu ngay trong phng trnh u tin, nu nhn
thy iu th cc bc tip theo s rt d nhn bit.
Bi 6: Gii h phng trnh sau

3 + 2x
2
y x
4
y
2
+x
4

1 2x
2

= y
2
1 +

1 + (x y)
2
= x
3

x
3
x + 2y
2

Gii
H phng trnh tng ng:

4 (1 x
2
y)
2
= 2x
6
x
4
+y
2

1 + (x y)
2
= 1 x
6
+ x
4
2x
3
y
2
Cng v theo v hai phng trnh ca h trn ta c:

4 (1 x
2
y)
2

1 + (x y)
2
= x
6
2x
3
y +y
2
+ 1

4 (1 x
2
y)
2
=

1 + (x y)
2
+

x
3
y

2
+ 1 ()
Ta thy:

V T() 2
V P() 2
V P() = V T() = 2

1 x
2
y = 0
x y = 0
x
3
y = 0
x = y = 1
281
Th x = y = 1 vo h thy tha mn.Vy h cho c nghim duy nht (x; y) = (1; 1) 2
Bi 7: Gii h phng trnh

2x
2
+ 4y
2
xy
= 4

2
y

3
x

(x + y) 1 (1)

(x + 1)
2
+ xy + 3x + 2y + 5 2x

x (y + 3) =

x +

y + 3 (2)
Gii
KX:

x > 0
y 3; y = 0

2
y

3
x

(x + y) 0
(x + 1)
2
+xy + 3x + 2y + 5 2x

x (y + 3) 0
Bin i phng trnh (1):
(1) 2
x
y
+ 4
y
x
= 4

2
x
y
3
y
x
1 1 (3)
t
x
y
= t (t = 0) , phng trnh (3) tr thnh:
2t +
4
t
+ 1 = 4

2t
3
t
1
Li c:
(4) 4t
2
+
16
t
2
+ 1 + 16 +
8
t
+ 4t = 32t
48
t
16
4t
2
+
16
t
2
28t +
56
t
+ 33 = 0 4

t
2
+
4
t
2

28

t
2
t

+ 33 = 0 (5)
t t
2
t
= u, phng trnh (4) tr thnh:
4

u
2
+ 4

28u + 33 = 0 4u
2
28u + 49 = 0 u =
7
2
t
2
t
=
7
2
2t
2
7t 4 = 0

t = 4
t =
1
2
Ly hai gi tr ca t va tm c thay vo phng trnh (4) th t = 4 tha mn.Suy ra x = 4y
Bin i phng trnh (2) kt hp vi h qu bin i ca phng trnh (1) l x = 4y ta c:
(2) (x + 1)
2
+xy + 3x + 2y + 5 2x

x (y + 3) = x + y + 2

x (y + 3)
x
2
+ xy + 3x 2x

x (y + 3) + x +y + 3

x (y + 3) = 0
(x + 1)

x +y + 3 2

x (y + 3)

= 0
(x + 1)

y + 3

2
= 0

x = y + 3
x = 4y

x = 4
y = 1
(chn)
282
Vy h c nghim duy nht (x; y) = (4; 1) 2
Bi 8: (THTT T8/415) Gii h phng trnh

log
2
x = 2
y+2
(1)
4

1 + x +xy

4 + y
2
= 0 (2)
Gii
K: x > 0. T (2) suy ra y < 0 v nu y 0 V T(2) > 0.
Ta bin i phng trnh (2):
(2) 4

1 + x = xy

4 + y
2
> 0 16(1 + x) = x
2
y
2
(4 + y
2
)
4x
2
y
2
16x +x
2
y
4
16 = 0
(xy
2
4)(4x + xy
4
+ 4) = 0
xy
2
4 = 0 x =
4
y
2
(3)
Thay (3) vo (1) ta c log
2
4
y
2
= 2
y+2
4.2
y
+ 2 log
2
(y) 2 = 0 (5) (do y < 0)
Nhn thy y = 1 l nghim ca (5). Xt f(y) = 4.2
y
+ 2 log
2
(y) 2 c f

(y) = 4.2
y
. ln 2 +
2
y ln 2
> 0 y < 0
Do hm s f(y) ng bin trn (; 0). Vy (5) c nghim duy nht y = 1. Th li ta
thy h c nghim (x; y) = (4; 1) 2.
Bi 9: Gii h phng trnh

1 x
2
+
1

1 y
2
=
35
12
x

1 x
2

1 y
2
=
7
12
Gii
t

1 x
1 + x
= a v

1 y
1 + y
= b.
Cng hai phng trnh v theo v ta c

1 + x
1 x
+

1 y
1 + y
=
7
2

1
a
+b =
7
2
(1)
Tr hai phng trnh v theo v ta c:

1 x
1 + x
+

1 + y
1 y
=
7
3
a +
1
b
=
7
3
(2)
T ta c h phng trnh

1
a
+ b =
7
2
a +
1
b
=
7
3

2 + 2ab = 7b
3ab + 3 = 7b

3a
2
7a + 2 = 0
b =
7
3

1
a

a = 2, b = 3
a =
1
3
, b =
1
2
T tm c nghim ca h l (x; y) = (
3
5
,
4
5
), (
4
5
,
3
5
) 2
Bi 10: Gii h phng trnh

2x +

2y = 4 (1)

2x + 5 +

2y + 5 = 6 (2)
283
Gii
KX: x, y 0
Cng hai phng trnh ta c

2x + 5 +

2x +

2y + 5 +

2y = 10
Tr (2) cho (1) ta c

2x + 5

2x+

2y + 5

2y = 2
5

2x + 5 +

2x
+
2

2y + 5 +

2y
=
2 t a =

2x + 5 +

2x, b =

2y + 5 +

2y ta c h phng trnh:

a + b = 10
5
a
+
5
b
= 2
a = b = 5
Ta cn gii phng trnh

2x + 5 +

2x = 5 x = 2 (chn). Tng t c y = 2
Vy h c nghim (x; y) = (2; 2) 2
Bi 11: Gii h phng trnh

x y 1 = 1 (1)
y
2
+x + 2y

x y
2
x = 0 (2)
Gii
KX: x 0; x y 1 0
T (1) ta c:

x =

x y 1 + 1 x = x y 1 + 2

x y 1 + 1 y = 2

x y 1
y
2
= 4(x y 1) (y + 2)
2
= 4x y + 2 = 2

x
T (2) ta c:
(y +

x)
2
= xy
2
y +

x = y

x
Ta c h mi l

y + 2 = 2

x
y +

x = y

y = 1; x =
1
4
y = 2; x = 4
Th li ta thy h c nghim (x; y) = (
1
4
; 1), (2; 4) 2
Bi 12: Tm a h sau c nghim thc x, y, z:

x 1 +

y 1 +

z 1 = a 1

x + 1 +

y + 1 +

z + 1 = a + 1
Gii
KX: x 1, y 1, z 1
H cho tng ng

x 1 +

x + 1

y 1 +

y + 1

z 1 +

z + 1

= 2a

x + 1

x 1

y + 1

y 1

z + 1

z 1

= 2
284
t u =

x 1 +

x + 1; v =

y 1 +

y + 1; s =

z 1 +

z + 1
Do x 1, y 1, z 1 nn u

2, v

2, s

2 . Ngc li nu u

2, v

2, s

2, ta c

x + 1

x 1 =
2

x + 1 +

x 1
=
2
u

x + 1 =
1
2

u +
2
u

x =
1
4

u
2
+
4
u
2

1 x 1
Tng t vi y, z. Vy ta cn tm a h sau c nghim u

2, v

2, s

2 :
()

u +v +s = 2a
1
u
+
1
v
+
1
s
= 1
(1)
iu kin cn: Gi s h (*) c nghim. Theo BT Cauchy-Schwarz ta c
2a = (u +v +s)

1
u
+
1
v
+
1
s

9 a
9
2
iu kin : Gi s a
9
2
. Ta chng minh h (*) c nghim.
Ly s = 3 ( tho s

2) . Khi (*) tr thnh

u + v = 2a 3
u.v =
3 (2a 3)
2
u, v l 2 nghim ca phng trnh t
2
2 (2a 3) t +
3 (2a 3)
2
u, v =
2a 3

(2a 3) (2a 9)
2
t h = 2a 9 0

h + 6 2

2
> (h + 3)
2
> h(h + 6).
Tc l (2a 3) 2

2 >

(2a 3) (2a 9) u >

2, v >

2.
Nh vy h phng trnh c nghim u

2, v

2, s

2.
Tm li a
9
2
l gi tr cn tm 2
Bi 13: Gii h phng trnh

1
4
+
2

x +

y
x +y

= 2
4

1
4

2

x +

y
x + y

= 1
Gii
KX: x, y 0
Nu x = 0 hoc y = 0 th h v nghim. Xt x, y > 0.
t u =

x; v =

y ta c h phng trnh

1
4
+
2u + v
u
2
+v
2

= 2

1
4

2u +v
u
2
+ v
2

= 1

u
=
1
4
+
2u +v
u
2
+v
2
1

v
=
1
4

2u +v
u
2
+ v
2

u
+
1

v
=
1
2
2

u

1

v
=
4u + 2v
u
2
+ v
2
Nhn hai phng trnh ta c
4
u

1
v
=
2u + v
u
2
+v
2
(4v u)(u
2
+v
2
) = (2u +v).uv (2v u)(u
2
+ 2v
2
) = 0 u = 2v
285
T tm c nghim ca h l (x; y) = (64(17 + 12

2); 16(17 + 12

2)) 2
Bi 14: Gii h phng trnh

x +
5x + 7

5y
x
2
+ y
2
= 7
y +
7

5x 5y
x
2
+y
2
= 0
Gii
x = 0 : y =

5
y = 0 : v nghim
x, y = 0 :
Nhn (1) cho x v (2) cho y ta c:

x
2
+
5x
2
+ 7

5xy
x
2
+ y
2
= 7x
y
2
+
7

5xy 5y
2
x
2
+y
2
= 0
Tr 2 phng trnh v theo v ta c x
2
y
2
+ 5 = 7x ()
Nhn (1) cho y v (2) cho x ta c:

xy +
5xy + 7

5y
2
x
2
+y
2
= 7y
xy +
7

5x
2
5xy
x
2
+y
2
= 0
Cng 2 phng trnh v theo v c: 2xy + 7

5 = 7y ()
T (*) v (**) ta c HPT:

x
2
y
2
+ 5 = 7x (3)
2xy + 7

5 = 7y (4)
Ta c: (4) y(7 2x) = 7

7 2x = 0
y =
7

5
7 2x
Th vo (3) ta c phng trnh
x
2

245
(7 2x)
2
+ 5 = 7x
x(4x
3
28x
2
+ 69x 140)
(2x 7)
2
= 7x x = 7 (do x = 0) y =

5
Kt lun: H c nghim (x; y) = (0;

5), (7;

5) 2
Bi 15: Gii h phng trnh

x +

y =

4x y (1)

x
2
16 = 2 +

y 3x (2)
Gii
Xt (1) ta c
(1) 2x 2

x
2
y = 4x y y 2x = 2

x
2
y

y = 0 (loi)
y = 4x 4
Thay y = 4x 4 vo (2) ta c

x
2
16 = 2 +

x 4

x
2
16 3 =

x 4 1

x
2
25

x
2
16 + 3
=
x 5

x 4 + 1
(x 5)(
x 5

x
2
16 + 3

x 4 + 1
) = 0 ()
286
Do
x + 5

x
2
16 + 3

x + 5

x
2
+ 3
> 1
1

x 4 + 1
nn () x = 5 y = 16
Th li ta thy h c nghim (x; y) = (5; 16) 2
Bi 16: Gii h phng trnh

x
4
+ 2x
3
5x
2
+ y
2
6x 11 = 0 (1)
x
2
+x =
3

y
2
7 6

y
2
7
(2)
Gii
KX: y
2
7 > 0
Xt (1) ta c:
(1) x
2
(x
2
+x) +x(x
2
+x) 6(x
2
+x) +y
2
7 4 = 0 (x
2
+x 6)(x
2
+x) +(y
2
7) = 4
t a = x
2
+x; b =

y
2
7 ta c h phng trnh

a(a 6) + b
2
= 4
a =
3b 6
b

a = 0; b = 2
a = 1; b = 3
a = 5; b = 3 (loi)
a = 6; b = 2 (loi)
T tm c nghim ca h l (x; y) = (1;

11), (0;

11),

5
2
; 4

5 1
2
; 4

2
Bi 17: Gii h phng trnh

x + y +

x + 3 =
y 3
x
(1)

x + y +

x = x + 3 (2)
Gii
Ta c iu kin x 0, y 3
Nu y = 3: thay vo (2) ta c

x + 3 +

x = x + 3 (v nghim)
Nu y > 3:
Xt (1) ta c:
(1)

x +y +

x + 3 =
x + y (x + 3)
x

(x +y (x + 3)

x +y

x + 3
=
x +y (x + 3)
x
(y 3)(
1

x + y

x + 3

1
x
) = 0

x +y

x + 3

1
x
= 0

x +y

x + 3 = x
T y ta c h phng trnh

x + y

x + 3 = x

x + y +

x = x + 3
287
Tr hai phng trnh ta c

x +

x + 3 = 3 x = 1 y = 8
Vy h c nghim (x; y) = (1; 8) 2
Bi 18: Gii h phng trnh

(x y)
4
= 13x 4,

x +y +

3x y =

2
Gii
iu kin: x +y 0 v 3x y 0
t:

3x y = a

x +y = b
(a, b 0)
a
2
b
2
2
= x y H phng trnh tr thnh:

(a
2
b
2
)
4
16
=
13(a
2
+b
2
)
4
4(1)
a +b =

2
Phng trnh (1) tng ng:
(a b)
4
.(a +b)
4
16
=
13(a
2
+b
2
)
4
4
Th a + b =

2 vo phng trnh trn


(a b)
4
= 13(a
2
+ b
2
) 16(2)
Vi tng a (2) v mt phng trnh ng cp, ta s vit li (2) thnh:
(a b)
4
=
13
2
(a
2
+b
2
).2 4.4
T a +b =

2 (a +b)
2
= 2 v (a +b)
4
= 4. Th vo phng trnh trn:
(a b)
4
=
13
2
.(a
2
+b
2
)(a +b)
2
4(a +b)
4
3a
4
+ 2a
3
b 34(ab)
2
+ 2ab
3
+ 3b
4
= 0(3)
Nu b = 0 3a
4
= 0 a = 0 m a +b =

2 b = 0 Chia 2 v ca (3) cho b


4
:
(3) 3

a
b

4
+ 2

a
b

3
34

a
b

2
+ 2
a
b
+ 3 = 0
t t =
a
b
(t 0). Phng trnh trn tr thnh:
3t
4
+ 2t
3
34t
2
+ 2t + 3 = 0
(t 3)(3t 1)(t
2
+ 4t + 1) = 0

t = 3
t =
1
3
(t
2
+ 4t + 1 > 0t 0)
Nu t = 3
a
b
= 3 kt hp vi a +b =

3x y =
9
8
x +y =
1
8

x =
5
16
y =
3
16
288
Nu t =
1
3

a
b
=
1
3
kt hp vi a +b =

3x y =
1
8
x +y =
9
8

x =
5
16
y =
13
16
Vy h c nghim (x; y) =

5
16
;
3
16

5
16
;
3
16

. 2
Bi 19: Gii h phng trnh

2x + 2
4

6 x y
2
= 2

2 (1)
4

2x + 2

6 x + 2

2y = 8 +

2 (2)
Gii
KX: x [0; 6]
Cng v theo v hai phng trnh ca h ta c

2x + 2

6 x +
4

2x + 2
4

6 x =

2
+ 6 + 3

2 (3)
Li c: V T(3) = 1.

2x +

2.

12 2x + 1.
4

2x +

2.
4

24 4x
p dng BT Cauchy-Schwarz cho V T(3) ta c
V T(3)

3. (2x + 12 2x) +

2x + 2

6 x

6 +

3.6 = 6 + 3

2 ()
D dng nhn thy V P(3) 6 + 3

2 () T (*), (**) v (3) suy ra


V T(3) = V P(3) = 6 + 3

2x =

6 x
4

2x =
4

6 x
y =

x = 2
y =

2
Th li ta thy h c nghim duy nht (x; y) = (0; 6) 2
Bi 20: Gii h phng trnh

x
2
x y.
3

x y = y
2(x
2
+y
2
) = 11 + 3

2x 1
Gii
Ta thy 2x 1 0 x
1
2
> 0.
Trong phng trnh th nht, x y v y cng du. Nu y x th y > 0 v x y 0, mu
thun.
Do , x > y > 0.
t
3

x y = a > 0 x y = a
3
.
Ta c (x
2
x y)a
2
= y
2
. Thay y = x a
3
, ta c:
(x
2
2x + a
3
)a
2
= (x a
3
)
2
ng thc ny tng ng vi:
a
2
x
2
2a
2
x + a
5
= x
2
2a
3
x +a
6
289
R rng a = 1 th ng thc ng nn khi phn tch thnh nhn t, ta c:
(a 1)(x
2
+ 2a
2
x +ax a
5
) = 0
D thy rng x
2
+2a
2
x+axa
5
khng th xy ra v a
5
maxa
3
, a
6
= maxxy, (xy)
2
.
Nu maxx y, (x y)
2
= (x y)
2
th d thy a
5
(x y)
2
< x
2
< x
2
+ 2a
2
x +ax.
Nu maxxy, (xy)
2
= xy th d thy a
5
xy, suy ra 0 < xy 1 v t PT th hai,
vi x
2
+y
2

11
2
. Nu x 1 th x
2
+y
2
2, mu thun. Suy ra x > 1 x
2
> x > 1 > x y.
Trong c hai trng hp, ta u c x
2
+ 2a
2
x + ax a
5
> 0.
Vy a = 1 y = x 1, thay vo phng trnh th hai, ta c:
2(2x
2
2x + 1) = 11 + 3

2x 1 (2x 1)
2
10 = 3

2x 1
t

2x 1 = b 0 th ta c phng trnh sau:


b
4
10 = 3b (b 2)(b
3
+ 2b
2
+ 4b + 5) = 0
D thy PT ny c nghim khng m duy nht l b = 2, tng ng vi nghim ca h cho
l (x, y) =

5
2
,
3
2

2
Bi 21: Gii h phng trnh

x
3
3x
2
+ 2 =

y
3
+ 3y
2
3

x 2 =

y
3
+ 8y
Gii
iu kin: x 2, y
2
(y + 3) 0, y(y + 8) 0 y 0.
Phng trnh th nht vit li l:
x
3
3x
2
+ 2 = y

y + 3
t x 1 = a 1,

y + 3 = b

3, ta c:
a
3
3a = b
3
3b (a b)(a
2
+b
2
+ab 3) = 0
D thy a
2
+b
2
+ ab 1 + 3 +

3 > 3 nn ta c a = b hay
a =

y + 3 y = a
2
3
Bnh phng hai v ca PT th hai, ta c: 9(x2) = y
2
+8y hay 9(a1) = (a
2
3)(a
2
+5)
on nghim l a = 2 v phn tch thnh nhn t, ta c:
(a 2)(a
3
+ 2a
2
+ 6a + 3) = 0
Suy ra phng trnh trn c nghim duy nht l a = 2 tng ng vi nghim ca h l
(x, y) = (3, 1).
Th li ta thy tha.Vy h c nghim duy nht l (x, y) = (3, 1) 2
Bi 22: Gii h phng trnh (I)

x + 3 = y
3
6

y + 2 = z
3
25

z + 1 = x
3
+ 1
290
Gii
t a =

x + 3, b =

y + 2, c =

z + 1 (a, b, c 0) ta c:
(I)

a =

b
2
2

3
6
b =

c
2
1

3
25
c =

a
2
3

3
+ 1

a b =

b
2
2

3
b 6 = f(b)
b c =

c
2
1

3
c 25 = g(c)
c a =

a
2
3

3
a + 1 = h(a)
Li c:

a 0
b 0

b
2
2

3
6 > 1
3

c
2
1

3
25 > 2
3

b >

3
c >

3
Suy ra:

a
2
3

3
+ 1 >

a >

3
a
2
3 >
3

3 1 >
3

1
2
()
Ta c:

(b) = 3

b
2
2

2
.2b 1 > 3.1.2

3 1 > 0 b >

3
g

(c) = 3

c
2
1

2
.2c 1 > 3.2
2
.2

3 1 > 0 c >

3
h

(a) = 3

a
2
3

2
.2a 1 > 3.

1
2
2
3
.2

3 1 > 3.
1
2
.2

3 1 > 0 a()
Suy ra: f(b), g(c), h(a) l hm ng bin v f(2) = g(2) = h(2) = 0.
Nu a > 2 :
h(a) > h(2) = 0 c > a > 2 g(c) > g(2) = 0 b > c > 2
f(b) > f(2) = 0 a > b > 2 a > b > c > a
Vy trng hp ny loi. L lun tng t vi a < 2.
Vy ta c:
a = 2 c = a +h(a) = 2 b = c + g(c) = 2
a = b = c = 2

x + 3 = 2

y + 2 = 2

z + 1 = 2

x = 1
y = 2
z = 3
Th li ta thy h phng trnh c 2 nghim l: (x; y; z) = (1; 2; 3) 2
Nhn xt: Nu c th on nghim ca h, ta cng c li gii p nh sau:
Ta c:
(I)

x + 3 2 = y
3
8

y + 2 2 = z
3
27

z + 1 2 = x
3
1

x 1

x + 3 + 2
= (y 2)(y
2
+ 2y + 4)
y 2

y + 2 + 2
= (z 3)(z
2
+ 3z + 9)
z 3

z + 1 + 2
= (x 1)(x
2
+ x + 1)

x 1 = (y 2)(y
2
+ 2y + 4)(

x + 3 + 2) (1)
y 2 = (z 3)(z
2
+ 3z + 9)(

y + 2 + 2) (2)
z 3 = (x 1)(x
2
+x + 1)(

z + 1 + 2) (3)
291
Nu x = 1 : y = 2, z = 3
T h trn ta c:

(1) [x 1[ > [y 2[
(2) [y 2[ > [z 3[
(3) [z 3[ > [x 1[
(mu thun)
Vy x = 1 y = 2, z = 3
Bi 23: Gii h phng trnh

2x + 2y

xy = 3

3x + 1 +

3y + 1 = 4
Gii
Nnhn thy phng trnh th nht ca h c h s bc nht ca x; y t l vi nhau nn ta
thm bt a v ng h s.
T phng trnh th nht ta c 2x + 2y

xy = 3
3
2
(2x + 2y)
3
2

xy =
9
2
t

3x + 1 = a

3y + 1 = b

x =
a
2
1
3
y =
b
2
1
3
H phng trnh cho tng ng

(3x + 1) + (3y + 1)
3
2

xy =
13
2

3x + 1 +

3y + 1 = 4
Thay a; b vo h ta c

a
2
+b
2

1
2

(a
2
1)(b
2
1) =
13
2
a + b = 4
t

S = a +b
P = ab
v th (2) vo (1) ta c

P
2
+ 2P 15 = 19 4P (vi P
19
4
)
P
2
154P + 376 = 0 P = 4 P =
94
15
(loi) S = P = 4
Vy a, b l nghim ca phng trnh X
2
4X + 4 = 0 a = b = 2

3x + 1 = 2

3y + 1 = 2
Gii h ny ta thu c (x; y) = (1; 1) 2
Nhn xt: Ngoi cch gii nh trn ta vn c th lm nh sau: Nhn phng trnh th nht
cho 2, nhn phng trnh th hai cho 4, v tr v theo v ta s a c v dng tng cc bnh
phng khng m A
2
+B
2
+C
2
= 0.
Bi 24: Gii h phng trnh

3y
2
+ 2y(x + 1) + 1 = 4y

x
2
+ 2y + 1
y(y x) = 3 3y
Gii
292
KX: x
2
+ 2y + 1 0
T h ta c
4y
2
4y

x
2
+ 2y + 1 + x
2
+ 2y + 1 = y
2
2xy +x
2
(2y

x
2
+ 2y + 1)
2
= (x y)
2
Nu 2y

x
2
+ 2y + 1 = x y 3y x =

x
2
+ 2y + 1 :
Ta c h

3y x 0
9y
2
6xy 2y = 1 (1)
y
2
xy + 3y = 3 (2)
Xt (1) (2).6 ta c 3y
2
20y + 17 = 0 y =
17
3
y = 1
Vi y = 1 th x = 1 (chn). Vi y =
17
3
th x =
415
51
(chn)
Nu 2y

x
2
+ 2y + 1 = y x y +x =

x
2
+ 2y + 1 :
Ta c h

x +y 0
y
2
+ 2xy 2y = 1
y
2
xy + 3y = 3
Gii tng t, ta c h trn v nghim.
Vy h phng trnh u c nghim (x; y) = (1; 1),

415
51
;
17
3

. 2
Bi 25: Gii h phng trnh

3
3x2y
5.6
x
+ 4.2
3x2y
= 0 (1)

x y =

y + (

2y

x)(

2y +

x)
2
(2)
Gii
iu kin x y 0
T phng trnh (2) ta c

x y

y = (

2y

x)(

2y +

x)
2

x 2y

x y +

y
=
2y x

2y +

2y +

2
(x 2y)

x y +

y
+

2y +

= 0
Vi x = 2y thay vo (1) ta c 3
4y
5.6
2y
+ 4.2
4y
= 0 9
2y
5.6
2y
+ 4.4
2y
= 0
Chia 2 v ca phng trnh cho 9
2y
v t a =

2
3

2y
(0 < a < 1) ta c:
4a
2
5a + 1 = 0

a =
1
4
a = 1 (loi)
Vy ta c

2
3

2y
=
1
4
y =
1
2
log3
2
4 x = log3
2
4
Vy h cho c nghim (x; y) =

log3
2
4;
1
2
log3
2
4

2
293
Bi 26: Gii h phng trnh

x
11
+ xy
10
= y
22
+y
12
(1)
7y
4
+ 13x + 8 = 2y
4 3

x (3x
2
+ 3y
2
1) (2)
( thi chn i tuyn TP HCM 2009-2010)
Gii
Xt y = 0, thay vo h tm c x =
8
13
Xt x = 0 t (1) suy ra y = 0 nhng (x; y) = 0 khng tho (2).
Xt xy = 0. T phng trnh (1) ta c

x
y

11
+
x
y
= y
11
+y
Xt hm s f(t) = t
11
+ t ta c f

(t) = 11t
10
+ 1 > 0 t R f(t) ng bin trn R. T
ta c:
(1) f

x
y

= f(y) x = y
2
> 0
Khi (2) tr thnh:
7x
2
+ 13x + 8 = 2x
2 3

x(3x
2
+ 3x 1) 7t + 13t
2
+ 8t
3
= 2
3

3 + 3t t
2
vi t =
1
x
> 0 (3)
Phng trnh ny c th gii bng phng php dng hm s n iu (xem chng Cc
phng php gii phng trnh). Li gii c th nh sau:
(3) (2t + 1)
3
+ 2(2t + 1) = 3 + 3t t
2
+ 2
3

3 + 3t t
2
g(2t + 1) = g(
3

3 + 3t t
2
) vi g(t) = t
3
+ 2t
Do g

(t) = 3t
2
+ 2 > 0 t > 0 nn ta suy ra
2t + 1 =
3

3 + 3t t
2
(2t + 1)
3
= 3 + 3t t
2

t = 1 (loi)
t =
5

89
16
(loi)
t =

89 5
16
(chni)
Suy ra x =
16

89 5
y =

16

89 5
Vy h c nghim (x; y) =

8
13
; 0

16

89 5
;

16

89 5

2
Bi 27: Gii h phng trnh

(1 + 4
xy
)5
1x+y
= 1 + 3
xy+2
x
2
3y

y
1
x
= 1 2y
Gii
294
K:y x 0 hoc y x 0
t x y = u. Phng trnh th nht tr thnh:
(1 + 4
u
)5
1u
= 1 + 9.3
u
(1)
Nu u > 0 th V T < 5 + 5 = 10 cn V P > 1 + 9 = 10 (VN)
Nu u < 0 th V T > 5 + 5 = 10 cn V P < 1 + 9 = 10 (VN)
Nu u = 0 th x = y
Thay vo phng trnh sau ta c:
x
2
3x

x
1
x
= 1 2x (K: x 1 hoc 1 x < 0)
3x
2
3

x(x
2
1) = 1 2x ()
t a =

x
2
1(a > 0), b =

x th (*) tr thnh:
a
2
3ab + 2b
2
= 0

a = b
a = 2b

x
2
1 =

x

x
2
1 = 2

x =
1

5
2
x = 2

5
Vy h c nghim x = y =
1

5
2
v x = y = 2

5 2
Bi 28: Gii h phng trnh

x
2
+ 91 =

y 2 + y
2
(1)

y
2
+ 91 =

x 2 + x
2
(2)
Gii
K: x, y 2
Ly (1) tr (2) ta c:

x
2
+ 91

y
2
+ 91 =

y 2 + y
2

x 2 x
2

x
2
y
2

x
2
+ 91 +

y
2
+ 91
=
y x

x 2 +

y 2
+ (y
2
x
2
)
(x y)(
x +y

x
2
+ 91 +

y
2
+ 91
+
1

x 2 +

y 2
+x +y) = 0
x = y
Thay vo h ban u ta c:

x
2
+ 91 =

x 2 + x
2

x
2
+ 91 10 =

x 2 1 + (x
2
9)

x
2
9

x
2
+ 91 + 10
=
x 3

x 2 + 1
+ (x 3)(x + 3)

x = 3
x + 3

x
2
+ 91 + 10
=
1

x 2 + 1
+ (x + 3)
295
D thy
x + 3

x
2
+ 91 + 10
< x + 3
x + 3

x
2
+ 91 + 10
< x + 3 +
1

x 2 + 1
Do x = 3 (tho x 2). Vy h c nghim x = y = 3 2
Bi 29: Gii h phng trnh

x
4
y
4
(x
2
+y
2
) + 1 = 1 xy (1)

1 x
2
+

1 y
2
=

2 (2)
Gii
KX:

1 x, y 1
x
4
y
4
(x
2
+y
2
) + 1 0
t a = x
2
+ y
2
; b = xy

a 2b
1 b 1
T (1) ta c:

b
4
a + 1 = 1 b b
4
a + 1 = b
2
2b + 1 a = b
4
b
2
+ 2b
Li c a 2b b
4
b
2
+ 2b 2b b
2
(b
2
1) 0 () Do b [1; 1] b
2
(b
2
1) 0 nn
() b
2
(b
2
1) = 0

a = 2b
b 0; 1
T a = 2b x = y. Vy ta c:
Nu b = 0 x = y = 0 (tho (2)).
Nu b = 1 x = y = 1 (khng tha (2)).
Nu b = 1 x
2
= 1 (v nghim).
Kt lun: H c nghim (x; y) = (0; 0) 2
Bi 30: Gii h phng trnh

2xy
x +y
+

x
2
+y
2
2
=
2

xy + x + y
2
(1)
2012
x+y1
3x +y + 1 =

(2x 1)
2
+x y + 1 (2)
Gii
iu kin: xy 0
Phng trnh (1) vit li:
2xy
x +y

x + y
2
+

x
2
+ y
2
2

xy = 0
4xy (x +y)
2
2(x +y)
+
x
2
+y
2
2
xy

x
2
+y
2
2
+

xy
= 0

(x y)
2
2(x +y)
+
(x y)
2
2(

x
2
+y
2
2
+

xy)
= 0

x = y
1
x +y
=
1

x
2
+y
2
2
+

xy
(3)
296
Xt (3) ta c:
(3) x +y =

x
2
+y
2
2
+

xy 2(x +y) =

2(x
2
+y
2
) + 2

xy
(

y)
2
=

2(x
2
+y
2
) (x +y) (

y)
2
=
2(x
2
+y
2
) (x + y)
2

2(x
2
+y
2
) + x + y
(

y)
2
=
(x y)
2

2(x
2
+y
2
) + x +y
(

y)
2
=
(

y)
2
.(

x +

y)
2

2(x
2
+y
2
) + x + y

x = y
1 =
(

x +

y)
2

2(x
2
+ y
2
) + x +y
(4)
Xt (4) ta c:
(4)

2(x
2
+y
2
) + x + y = (

x +

y)
2

2(x
2
+y
2
) = 2

xy (5)
T (3) x +y > 0 m xy 0 x, y 0
p dng bt ng thc Cauchy cho 2 s dng x,y ta chng minh c (5) xy ra khi v ch
khi x = y. Hp tt c cc trng hp, kt lun (1) x = y
Th y = x vo phng trnh (2), ta c: 2012
2x1
2x + 1 =

(2x 1)
2
+ 1.
t 2x 1 = t. Phng trnh trn tr thnh:
2012
t
= t +

t
2
+ 1 ln(t +

t
2
+ 1) t ln 2012 = 0 (6)
Xt f(t) = ln(t+

t
2
+ 1)t ln 2012. Ta c f

(t) =
1 +
t

t
2
+ 1
t +

t
2
+ 1
ln 2012 =
1

t
2
+ 1
ln 2012 <
0 vi mi t R f(t) nghch bin trn R
M f(0) = 0 (6) c duy nht mt nghim t = 0
Ta c t = 0 x =
t + 1
2
=
1
2
y = x =
1
2
Vy h c duy nht 1 nghim (x; y) =

1
2
;
1
2

2
CnucNo VI: SANO TO PHNO TRNH - H
PHNO TRNH
xy dng mt s phng trnh c gii bng cch a
v h phng trnh
V d 1. Xt h i xng loi hai

x = 2 3y
2
y = 2 3x
2
x = 2 3

2 3x
2

2
.
Ta c bi ton sau
Bi 1 (THTT, s 250, thng 04/1998). Gii phng trnh
x + 3

2 3x
2

2
= 2.
Gii
t y = 2 3x
2
. Ta c h

x + 3y
2
= 2
y = 2 3x
2

x = 2 3y
2
(1)
y = 2 3x
2
(2)
Ly (1) tr (2) ta c
x y = 3(x
2
y
2
)

x y = 0
3(x + y) = 1

y = x
y =
1 3x
3
.
Vi y = x, thay vo (1) ta c
3x
2
+x 2 = 0 x

1,
2
3

.
Vi y =
1 3x
3
, thay vo (2) ta c
1 3x
3
= 2 3x
2
9x
2
3x 5 = 0 x =
1

21
6
.
Kt lun: Phng trnh cho tp nghim S =

1;
2
3
;
1

21
6

2
Nhn xt: T li gii trn ta thy rng nu khai trin (2 3x
2
)
2
th s a phng trnh
cho v phng trnh a thc bc bn, sau bin i thnh
(x + 1)(3x 2)(9x
2
3x 5) = 0.
297
298
Vy nu khi xy dng bi ton, ta c lm cho phng trnh khng c nghim hu t th
phng php khai trin a v phng trnh bc cao, sau phn tch a v phng trnh
tch s gp nhiu kh khn.
V d 2. Xt mt phng trnh bc hai c c hai nghim l s v t
5x
2
2x 1 = 0 2x = 5x
2
1.
Do xt

2y = 5x
2
1
2x = 5y
2
1
2x = 5

5x
2
1
2

2
1. Ta c bi ton sau.
Bi 2. Gii phng trnh
8x 5

5x
2
1

2
= 4.
Gii
t 2y = 5x
2
1. Khi

2y = 5x
2
1
8x 5.4y
2
= 4

2y = 5x
2
1 (1)
2x = 5y
2
1. (2)
Ly (1) tr (2) theo v ta c
2(y x) = 5(x
2
y
2
)

y x = 0
2 = 5(x + y)

y = x
y =
5x + 2
5
.
Vi y = x, thay vo (1) ta c
5x
2
2x 1 = 0 x =
1

6
5
.
Vi y =
5x + 2
5
, thay vo (1) ta c

10x + 4
5
= 5x
2
1 25x
2
+ 10x 1 = 0 x =
5

50
25
.
Phng trnh cho c tp nghim S =

6
5
,
1

2
5

2
Nhn xt: Php t 2y = 5x
2
1 c tm ra nh sau: Ta t ay +b = 5x
2
1, vi a, b tm
sau. Khi thu c h

ay +b = 5x
2
1
8x 5 (ay + b)
2
= 4

ay +b + 1 = 5x
2
8x + 4 5b
2
= 5a
2
y
2
+ 10aby.
h trn l h i xng loi II th

a
8
=
5
5a
2
=
b + 1
4 5b
2
10ab = 0

b = 0
a = 2.
Vy ta c php t 2y = 5x
2
1. Sau y l mt bi tng t:
Bi 3. Gii phng trnh 5(5x
2
17)
2
343x 833 = 0
Gii
299
tng: t ay + b = 5x
2
17 (a = 0)

ay +b = 5x
2
17
5(ay + b)
2
343x 833 = 0 ()
T (*) c
5(ay)
2
+ 10aby + b
2
343x 833 = 0
x =
5(ay)
2
+ 10aby +b
2
833
343
ax +b =
5a
3
.y
2
+ 10a
2
.by + b
2
.a 833a
343
+b ()
Ta hi vng c ax + b = 5y
2
17, kt hp vi (**) suy ra
5y
2
17 =
5a
3
.y
2
+ 10a
2
.by +b
2
.a 833a
343
+b
343.5y
2
5831 = 5a
3
.y
2
+ 10a
2
.by +b
2
.a 833a + 343b
ng nht h s ta c

343 = a
3
a
2
b = 0
833a + 343b = 5831

a = 7
b = 0
Vy ta c li gii sau:
Li gii:
t 7y = 5x
2
+ 17 ta c h phng trnh

7y = 5x
2
17
245y
2
343x 833 = 0

7y = 5x
2
17 (1)
7x = 5y
2
17 (2)
Ly (1) tr (2) ta c
7(y x) = 5(x +y)(x y)

x = y
5x + 5y = 7
Nu x = y, thay vo (1) c
5x
2
7x 17 = 0 x =
7

389
10
Nu 5x + 5y = 7, kt hp (2) ta c
7 + 5x +
5(5x
2
17)
7
= 0 25x
2
+ 35x 36 = 0 x =
35 5

193
50
Kt lun: Phng trnh c tp nghim S =

389
10
;
35 5

193
50

2
V d 3. Xt mt phng trnh bc ba
4x
3
3x =

3
2
8x
3
6x =

3 6x = 8x
3

3
Do ta xt

6y = 8x
3

3
6x = 8y
3

3
6x = 8

8x
3

3
6

3
300
1296x + 216

3 = 8

8x
3

3
162x + 27

3 =

8x
3

3
.
Ta c bi ton sau
Bi 4. Gii phng trnh 162x + 27

3 =

8x
3

3
.
Gii
t 6y = 8x
3

3. Ta c h

6y = 8x
3

3
162x + 27

3 = 216y
3

6y = 8x
3

3 (1)
6x = 8y
3

3 (2)
Ly (1) tr (2) theo v ta c
6(y x) = 8(x
3
y
3
) (x y)

x
2
+ xy +y
2

+ 6

= 0. (3)
V x
2
+xy +y
2
0 nn 8 (x
2
+xy +y
2
) + 6 > 0. Do t (3) ta c x = y.
Thay vo (1) ta c
6x = 8x
3

3 4x
3
3x =

3
2
4x
3
3x = cos
5
6
(4)
S dng cng thc cos = 4 cos
3

3
3 cos

3
, ta c
cos
5
6
= 4 cos
3
5
18
3 cos
5
18
,
cos
17
6
= 4 cos
3
17
18
3 cos
17
18
,
cos
7
6
= 4 cos
3
7
18
3 cos
7
18
.
Vy x = cos
5
18
, x = cos
17
18
, x = cos
7
18
l tt c cc nghim ca phng trnh (4) v cng l
tt c cc nghim ca phng trnh cho 2
Nhn xt: Php t 6y = 8x
3

3 c tm ra nh sau :
Ta t ay + b = 8x
3

3. Khi t phng trnh cho c h

ay +b = 8x
3

3
162x + 27

3 = a
3
y
3
+ 3a
2
by
2
+ 3ab
2
y +b
3
.
Cn chn a v b sao cho

a
162
=
8
a
3
=
b +

3
27

3 b
3
3a
2
b = 3ab
2
= 0

b = 0
a = 6.
Vy ta c php t 6y = 8x
3

3.
301
V d 4. Ta s xy dng mt phng trnh v t c t nht mt nghim theo mun. Xt
x = 3. Khi
2x 5 = 1 (2x 5)
3
= 1
do x=3
= x 2.
Ta mong mun c mt phng trnh cha (ax + b)
3
v cha
3

cx +d, hn na phng trnh


ny c gii bng cch a v h gn i xng loi hai (ngha l khi tr theo v hai phng
trnh ca h ta c tha s (x y)). Vy ta xt h

(2y 5)
3
= x 2
(2x 5)
3
= x + 2y 2.
Nu c php t 2y 5 =
3

x 2, th sau khi thay vo phng trnh


(2x 5)
3
= x + 2y 2
ta c
8x
3
60x
2
+ 150x 125 = x +
3

x 2 + 5 2.
Ta c bi ton sau
Bi 5. Gii phng trnh
3

x 2 = 8x
3
60x
2
+ 151x 128.
Gii
Cch 1:
Tp xc nh R. Phng trnh vit li
3

x 2 = (2x 5)
3
+x 3. (1)
t 2y 5 =
3

x 2. Kt hp vi (1) ta c h

(2y 5)
3
= x 2 (2)
(2x 5)
3
= x + 2y 2 (3)
Ly (3) tr (2) theo v ta c
2 (x y)

(2x 5)
2
+ (2x 5) (2y 5) + (2y 5)
2

= 2(y x)

x y = 0 (4)
(2x 5)
2
+ (2x 5) (2y 5) + (2y 5)
2
+ 1 = 0. (5) (v nghim)
Ta c (4) y = x. Thay vo (2) ta c
(2x 5)
3
= x 2 8x
3
60x
2
+ 149x 123 = 0
(x 3)(8x
2
36x + 41) = 0 x = 3.
Vy phng trnh c nghim duy nht x = 3 2
Nhn xt: T nghim duy nht trn, ta ngh n cch dng n iu hm s nh sau:
302
Cch 2.
Tp xc nh R. t y =
3

x 2. Ta c h

8x
3
60x
2
+ 151x 128 = y
x = y
3
+ 2
Cng v theo v hai phng trnh ca h ta c
8x
3
60x
2
+ 152x 128 = y
3
+y + 2
8x
3
60x
2
+ 150x 125 + 2x 5 = y
3
+y
(2x 5)
3
+ (2x 5) = y
3
+y. (*)
Xt hm s f(t) = t
3
+ t. V f

(t) = 3t
2
+ 1 > 0, t R nn hm f ng bin trn R. Do
() vit li
f(2x 5) = f(y) 2x 5 = y.
Bi vy
(2x 5) =
3

x 2 (2x 5)
3
= x 2
8x
3
60x
2
+ 149x 123 = 0
(x 3)(8x
2
36x + 41) = 0 x = 3.
Phng trnh c nghim duy nht x = 3 2
V d 5. Xt mt phng trnh bc ba no , chng hn xt 4x
3
+3x = 2. Phng trnh ny
tng ng
8x
3
+ 6x = 4 8x
3
= 4 6x 2x =
3

4 6x.
Ta lng ghp phng trnh cui vo mt hm n iu nh sau
(2x
3
) + 2x =
3

4 6x + 4 6x 8x
3
+ 8x 4 =
3

4 6x.
Ta c bi ton sau:
Bi 6. Gii phng trnh
8x
3
+ 8x 4 =
3

4 6x.
Gii
Tp xc nh ca phng trnh l R.
Cch 1:
Phng trnh cho tng ng
(2x)
3
+ 2x =
3

4 6x + 4 6x. (1)
Xt hm s f(t) = t
3
+t, t R. V f

(t) = 3t
2
+1 > 0, t R nn hm s f(t) ng bin trn
R. M (1) vit li f

4 6x

= f(2x) nn n tng ng
3

4 6x = 2x 8x
3
+ 6x = 4 4x
3
+ 3x = 2. (2)
303
V hm s g(x) = 4x
3
+ 3x c g

(x) = 12x
2
+ 3 > 0, x R nn PT (2) c khng qu mt
nghim. Xt
2 =
1
2

(
3
)
2
4
3
1
3
= 2

5.
Do , nu t =
3

2 +

5 th 2 =
1
2

. Ta c
1
2

= 3

1
2

+ 4

1
2

3
.
Vy x =
1
2

=
1
2

2 +

5 +
3

l nghim duy nht ca (2) v cng l


nghim duy nht ca phng trnh cho 2
Cch 2:
Phng trnh vit li thnh
(2x)
3
=
3

6x + 4 8x + 4.
t 2y =
3

4 6x. Ta c h

8y
3
= 4 6x
8x
3
+ 8x 4 = 2y

8y
3
= 6x + 4 (a)
8x
3
= 2y + 4 8x. (b)
Ly (b) tr (a) theo v ta c
8(x
3
y
3
) = 2(y x) (x y)[4(x
2
+ xy +y
2
) + 1] = 0 y = x.
Thay y = x vo (a) ta c
8x
3
= 6x + 4 4x
3
+ 3x = 2.
n y lm ging cch 1.
Bi 7. Gii phng trnh
3

6x + 1 = 8x
3
4x 1
( ngh Olympic 30-4-2006)
Gii
Tp xc nh ca phng trnh l R. t
3

6x + 1 = 2y. Ta c h

8x
3
4x 1 = 2y
6x + 1 = 8y
3

8x
3
= 4x + 2y + 1 (1)
8y
3
= 6x + 1. (2)
Ly (1) tr (2) theo v ta c
8(x
3
y
3
) = 2(y x) (x y)[4(x
2
+ xy +y
2
) + 1] = 0 y = x.
Thay y = x vo (2) ta c
8x
3
6x = 1 4x
3
3x = cos

3
. (3)
304
S dng cng thc cos = 4 cos
3

3
3 cos

3
, ta c
cos

3
= 4 cos
3

9
3 cos

9
,
cos
7
3
= 4 cos
3
7
9
3 cos
7
9
,
cos
5
3
= 4 cos
3
5
9
3 cos
5
9
.
Vy x = cos

9
, x = cos
5
9
, x = cos
7
9
l tt c cc nghim ca phng trnh (3) v cng l
tt c cc nghim ca phng trnh cho 2
Nhn xt: Ta cn c th gii cch khc nh sau : Phng trnh vit li
6x + 1 +
3

6x + 1 = (2x)
3
+ 2x. (3)
Xt hm s f(t) = t
3
+t, t R. V f

(t) = 3t
2
+1 > 0, t R nn hm s f(t) ng bin trn
R. M (2) c th vit li thnh f

6x + 1

= f(2x) nn tng ng
3

6x + 1 = 2x 8x
3
6x = 1 4x
3
3x =
1
2
.
V d 6. Xt mt tam thc bc hai lun nhn gi tr dng : x
2
+ 2. Khi


x
2
+ 2

dx =
x
3
3
+ 2x +C.
Ch cn chn C = 0 ta c mt a thc bc ba ng bin l h(x) =
x
3
3
+2x. Ta c h(3) = 15.
Vy ta thu c mt hm s a thc bc ba ng bin g(x) v tho mn g(3) = 0 l
g(x) =
x
3
3
+ 2x 15.
Ta s tm mt a thc bc ba ng bin k(x) sao cho k(x) = x g(x) = 0, mun vy ta xt
x
3
3
+x 15 = x
x
3
3
+ ( 1)x 15 = 0.
Do chn sao cho 1 = 2 = 3, khi k(x) =
x
3
3
+3x15 v k(x) = y tng ng
vi
x
3
3
+ 3x 15 = y x
3
+ 9x 45 = 3y.
T phng trnh cui ny thay x bi y ta thu c h i xng loi hai

x
3
+ 9x 45 = 3y
y
3
+ 9y 45 = 3x.
T h trn, s dng php th ta thu c phng trnh

x
3
+ 9x 45
3

3
+ 9

x
3
+ 9x 45
3

45 = 3x

x
3
+ 9x 45

3
+ 81

x
3
+ 9x 45

= 1215 + 81x.
Vy ta thu c bi ton sau.
305
Bi 8. Gii phng trnh

x
3
+ 9x 45

3
+ 81

x
3
+ 9x 45

= 1215 + 81x (1)


Gii
Tp xc nh R. t x
3
+ 9x 45 = 3y. Kt hp vi (1) ta c h

x
3
+ 9x 45 = 3y (2)
y
3
+ 9y 45 = 3x. (3)
Ly (2) tr (3) theo v ta c
x
3
y
3
+ 9x 9y = 3y 3x x
3
y
3
+ 12(x y) = 0
(x y)(x
2
+xy + y
2
+ 12) = 0 x = y.
Thay vo (2) ta c
x
3
+ 9x 45 = 3x (x 3)

x
2
+ 3x + 15

= 0 x = 3.
Phng trnh cho c nghim duy nht x = 3. 2
Nhn xt: Php t x
3
+ 9x 45 = 3y c tm ra nh sau: Ta t x
3
+ 9x 45 = ay. Khi


x
3
+ 9x 45 = ay
a
3
y
3
+ 81ay = 1215 + 81x

x
3
+ 9x 45 = ay
a
3
y
3
+ 81ay 1215 = 81x.
Vy cn chn a tho mn iu kin
a
3
1
=
81a
9
=
1215
45
=
81
a
a = 3.
Do t x
3
+ 6x 45 = 3y, ta s thu c mt h i xng loi hai.
V d 7. Chn mt phng trnh ch c hai nghim l 0 v 1 l 11
x
= 10x + 1. T phng
trnh ny ta thit lp mt h i xng loi hai, sau li quay v phng trnh nh sau :

11
x
= 10y + 1
11
y
= 10x + 1

y = log
11
(10x + 1)
11
x
= 10y + 1

11
x
1
10
= log
11
(10x + 1)
11
x
= 10 log
11
(10x + 1) + 1 11
x
= 2 log
11
(10x + 1)
5
+ 1.
Ta c bi ton sau.
Bi 9. Gii phng trnh 11
x
= 2 log
11
(10x + 1)
5
+ 1.
Gii
iu kin x >
1
10
. t y = log
11
(10x + 1), khi 11
y
= 10x + 1. Kt hp vi phng trnh
cho ta c h

11
x
= 10y + 1 (1)
11
y
= 10x + 1 (2)
Ly (1) tr (2) theo v ta c
11
x
11
y
= 10y 10x 11
x
+ 10x = 11
y
+ 10y. (3)
306
Xt hm s f(t) = 11
t
+ 10t. Ta c f

(t) = 11
t
ln 11 + 10 > 0, t R. Vy hm s f ng bin
trn R. M (3) chnh l f(x) = f(y) nn x = y. Thay vo (1) ta c
11
x
= 10x + 1 11
x
10x 1 = 0. (4)
Xt hm s g(x) = 11
x
10x 1 trn khong

1
10
; +

. Ta c
g

(x) = 11
x
ln 11 10, g

(x) = 11
x
(ln 11)
2
> 0.
Vy hm s g c th lun lm trn khong

1
10
; +

, suy ra th ca hm g v trc
honh c vi nhau khng qu hai im chung, suy ra (4) c khng qu 2 nghim. M g(1) = 0,
g(0) = 0 nn x = 0 v x = 1 l tt c cc nghim ca (4).
Vy nghim ca phng trnh cho l x = 0 v x = 1. 2
V d 8. Ta s s dng phng php lp sng tc phng trnh t h phng trnh i
xng loi hai. Xut pht t

4x =

30 + u
4u =

x + 30,
s dng php th ta c phng trnh 4x =

30 +
1
4

x + 30. T phng trnh ny ta li


thu c h i xng loi hai

4u =

30 +
1
4

x + 30
4x =

30 +
1
4

u + 30.
T h ny, tip tc s dng php th ta thu c phng trnh
4x =

30 +
1
4

30 +
1
4

30 +
1
4

x + 30.
Ta c bi ton sau.
Bi 10 ( ngh Olympic 30/04/2010). Gii phng trnh
4x =

30 +
1
4

30 +
1
4

30 +
1
4

x + 30.
Gii
x l nghim th x > 0. t u =
1
4

30 +
1
4

x + 30, t phng trnh cho ta c h

4u =

30 +
1
4

x + 30
4x =

30 +
1
4

u + 30.
(1)
307
Gi s x u. Khi
4u =

30 +
1
4

x + 30

30 +
1
4

u + 30 = 4x u x x = u.
Vy t h (1) ta c x = u v
4x =

30 +
1
4

x + 30. (2)
t v =
1
4

x + 30, t (2) ta c h

4x =

30 + v
4v =

x + 30.
(3)
Gi s x v. Khi
4v =

x + 30

v + 30 = 4x 4v 4x v x v = x.
Vy v = x v 4x =

x + 30

x 0
16x
2
= x + 30
x =
1 +

1921
32
.
Phng trnh cho c nghim duy nht x =
1 +

1921
32
. 2
s dng cng thc lng gic sng tc cc phng
trnh a thc bc cao
Trong mc ny ta s dng mt s cng thc lng gic sng tc ra cc phng trnh a
thc bc cao. Vic gii cc phng trnh a thc bc cao l rt phc tp, trong nhiu trng
hp l khng th. Tuy nhin s dng tnh cht phng trnh a thc bc n (n = 1, 2, . . . ) c
khng qu n nghim, v mt s nh hng trong qu trnh sng tc ton, ta c c li
gii rt ngn gn v n tng cho cc phng trnh dng ny.
V d 9. T cng thc
cos 6 = 32 cos
6
48 cos
4
+ 18 cos
2
1
Ly cos = x ta c
cos 6 = 32x
6
48x
4
+ 18x
2
1
Chn 6 =

3
ta c
32x
6
48x
4
+ 18x
2
1 =
1
2
Ta c bi ton sau.
Bi 11 ( ngh OLYMPIC 30/04/2009). Gii phng trnh
64x
6
96x
4
+ 36x
2
3 = 0.
Gii
308
Ta c
cos 6 = 2 cos
2
3 1 = 2

4 cos
3
3 cos

2
1
= 32 cos
6
48 cos
4
+ 18 cos
2
1. (1)
Phng trnh cho tng ng
32x
6
48x
4
+ 18x
2
1 =
1
2
32x
6
48x
4
+ 18x
2
1 = cos

3
. (2)
T (1) suy ra (2) c 6 nghim l x = cos


3.6
+
k2
6

, k = 1, 5. 2
Nhn xt: Vic s dng cng thc biu din cos n theo cos , sin n theo sin s gip ta
gii c nhng phng trnh dng ny.
V d 10. T cos 5 = 16 cos
5
20 cos
3
+ 5 cos , t cos =
x
2

3
ta c
cos 5 =
16x
5
288

20x
3
24

3
+
5x
2

3
=
x
5
18

5x
3
6

3
+
5x
2

3
=
x
5
15x
3
+ 45x
18

3
.
Chn 5 =

6
c

3
2
=
x
5
15x
3
+ 45x
18

3
x
5
15x
3
+ 45x 27 = 0. Ta c bi ton sau.
Bi 12 ( ngh Olympic 30/04/2011). Gii phng trnh
x
5
15x
3
+ 45x 27 = 0.
Gii
Tp xc nh R. t x = 2

3t, thay vo phng trnh cho ta c


288

3t
5
360

3t
3
+ 90

3t 27 = 0
2

16t
5
20t
3
+ 5t

3 16t
5
20t
3
+ 5t = cos

6
. (1)
Mt khc ta c
cos 5 + cos = 2 cos 3cos 2
cos 5 = 2

4 cos
3
3 cos

2 cos
2
1

cos
cos 5 = 2

8 cos
5
10 cos
3
+ 3 cos

cos
cos 5 = 16 cos
5
20 cos
3
+ 5 cos . (2)
T (2) suy ra (1) c 5 nghim l t = cos


6.5
+
k2
5

, k = 0, 4 Vy phng trnh cho c 5


nghim l x = 2

3 cos

30
+
k2
5

, k = 0, 4. 2
Nhn xt: Trong li gii trn, php t x = 2

3t tm ra nh sau : Do cng thc


cos 5 = 16 cos
5
20 cos
3
+ 5 cos ,
nn t x = at, vi a s tm sau. Thay x = at vo phng trnh cho ta c
a
5
t
5
15a
3
t
3
+ 45at 27 = 0.
Ta tm a tho iu kin
a
5
16
=
15a
3
20
=
45a
5

a
4
16
=
3a
2
4
= 9 a = 2

3. Vy ta c php t
x = 2

3t.
309
V d 11. T sin 5 = 16 sin
5
20 sin
3
+ 5 sin , ly sin = 2x ta c sin 5 = 512x
5

160x
3
+ 10x. Chn 5 =

3
, ta c

3
2
= 512x
5
160x
3
+ 10x 1024x
5
320x
3
+ 20x

3 = 0.
Ta c bi ton sau.
Bi 13. Gii phng trnh 1024x
5
320x
3
+ 20x

3 = 0.
Gii
t x =
t
2
, thay vo phng trnh cho ta c
32t
5
40t + 10 =

3 16t
5
20t
3
+ 5t = sin

3
. (1)
Ta c
sin 5 + sin = 2 sin 3cos 2
sin 5 = 2

3 sin 4 sin
3


1 2 sin
2

sin
sin 5 = 2

8 sin
5
10 sin
3
+ 3 sin

sin
sin 5 = 16 sin
5
20 sin
3
+ 5 sin . (2)
T (2) suy ra (1) c 5 nghim l t = sin


3.5
+
k2
5

, k = 0, 1, 2, 3, 4. Phng trnh cho


c 5 nghim l x =
1
2
sin

15
+
k2
5

, k = 0, 1, 2, 3, 4. 2
V d 12. Ta xt phng trnh x
3
3x =

x + 2. D thy x = 2 l nghim ca phng trnh


ny. Ta khng i theo con ng lng gic m s bin i i s ngay t phng trnh ban
u, c th l bnh phng hai v ta c
x
6
6x
4
+ 9x
2
x 2 = 0
(x 2)

x
5
+ 2x
4
2x
3
4x
2
+x + 1

= 0.
Ta ch xt x
5
+ 2x
4
2x
3
4x
2
+ x + 1 = 0. Li t thm n ph che lp vn k hn:
thay x bi 2x, ta c 32x
5
+ 32x
4
16x
3
16x
2
+ 2x + 1 = 0. Ta c bi ton sau.
Bi 14 ( ngh Olympic 30/04/2011). Gii h phng trnh

x
2
+ 4y
2
= 1
16x
5
20x
3
+ 5x + 512y
5
160y
3
+ 10y +

2 = 0.
Gii
t t = 2y, thay vo h ta c

x
2
+t
2
= 1 (1)
16(x
5
+t
5
) 20(x
3
+ t
3
) + 5(x + t) =

2. (2)
310
t

x = sin
t = cos
vi [0; 2]. Thay vo (2) ta c

16 sin
5
20 sin
3
+ 5 sin

16 cos
5
20 cos
3
+ 5 cos

2
sin 5 + cos 5 =

2 sin

5 +

4

= 1 =
3
20
+
k2
5
.
V [0; 2] nn

4
,
13
20
,
21
20
,
29
20
,
37
20

, t y suy ra x, y. 2
Nhn xt: Li gii trn tn dng mt s trng lp th v ca hm sin v cos :
sin 5 = 16 sin
5
20 sin
3
+ 5 sin ; cos 5 = 16 cos
5
20 cos
3
+ 5 cos .
s dng cc hm lng gic hyperbolic
S dng cc ng nht thc i s c xut s t cc hm lng gic hypeblic ta c th sng
tc c mt s phng trnh a thc bc cao c cch gii c th.
V d 13. Ta c
1
2

a
5

1
a
5

= 16m
5
+ 20m
3
+ 5m, trong m =
1
2

a
1
a

.
t m =
x
2

2
, khi
1
2

a
5

1
a
5

=
16x
5
128

2
+
20x
3
16

2
+
5x
2

2
=
x
5
8

2
+
10x
3
8

2
+
20x
8

2
.
Ly
1
2

a
5

1
a
5

=
18
8

2
, ta c bi ton sau.
Bi 15. Gii phng trnh x
5
+ 10x
3
+ 20x 18 = 0.
Gii
Ta thy rng
x =

a
1
a

2a
2
xa

2 = 0 a =
x

x
2
+ 8
2

2
.
Do ta c quyn t x =

a
1
a

. Khi
x
5
= 4

a
5
5a
3
+ 10a
10
a
+
5
a
3

1
a
5

10x
3
= 20

a
3
3a +
3
a

1
a
3

, 20x = 20

a
1
a

.
Thay vo phng trnh cho ta c
4

a
5

1
a
5

18 = 0 4

2(a
5
)
2
18a
5
4

2 = 0

a
5
=
9 +

113
4

2
a
5
=
9

113
4

2
.
311
Phng trnh c nghim duy nht x =

9 +

113
4

2
9 +

113

. 2
Nhn xt: Trong li gii trn, php t x =

a
1
a

c tm ra nh sau :
Do cng thc
1
2

a
5

1
a
5

= 16m
5
+20m
3
+5m, trong m =
1
2

a
1
a

nn ta t x = pm,
thay vo phng trnh cho ta c
p
5
m
5
+ 10p
3
m
3
+ 20pm18 = 0
Ta tm p tho mn iu kin
p
5
16
=
10p
3
20
=
20p
5

p
4
16
=
p
2
2
= 4 p = 2

2.
Vy ta c php t x =

a
1
a

.
V d 14. T ng nht thc
1
2

a
5
+
1
a
5

= 16m
5
20m
3
+ 5m, trong m =
1
2

a +
1
a

.
Ly m = x ta c
1
2

a
5
+
1
a
5

= 16x
5
20x
3
+ 5x
Ly
1
2

a
5
+
1
a
5

= 7 ta c phng trnh
16x
5
20x
3
+ 5x + 7 = 0
T phng trnh ny ta c phng trnh
(x 1)(16x
5
20x
3
+ 5x + 7) = 0
Vy ta c bi ton sau.
Bi 16 ( ngh OLYMPIC 30/04/2008). Gii phng trnh
16x
6
16x
5
20x
4
+ 20x
3
+ 5x
2
+ 2x 7 = 0. (1)
Gii
Ta c
(1)

x = 1
16x
5
20x
3
+ 5x + 7 = 0

x = 1
16x
5
20x
3
+ 5x = 7 (2)
Tip theo ta gii phng trnh (2).
Nu [x[ 1 th t x = cos t, vi t [0; ]. Thay vo (2) ta c
6 cos
5
t 20 cos
3
t + 5 cos t = 7 cos 5t = 7 (v nghim).
Nu [x[ > 1 th xt phng trnh
x =
1
2

a +
1
a

a
2
2xa + 1 = 0

a = x +

x
2
1
a = x

x
2
1.
312
Vy nu t a = x +

x
2
1 th x =
1
2

a +
1
a

. Ta c
16x
5
= 16

1
2

a +
1
a

5
=
1
2

a
5
+ 5a
3
+ 10a +
10
a
+
5
a
3
+
1
a
5

20x
3
= 20

1
2

a +
1
a

3
=
5
2

a
3
+ 3a +
3
a
+
1
a
3

; 5x =
5
2

a +
1
a

.
Suy ra 16x
5
20x
3
+ 5x =
1
2

a
5
+
1
a
5

. Thay vo (2) c
1
2

a
5
+
1
a
5

= 7

a
5

2
+ 14a
5
+ 1 = 0

a
5
= 7

48
a
5
= 7 +

48

a =
5

48
a =
5

7 +

48.
Do (7

48)(7 +

48) = 1 nn (2) c nghim duy nht


x =
1
2

48 +
5

7 +

48

.
Nghim ca (1) l x = 1, x =
1
2

48 +
5

7 +

48

. 2
Nhn xt: Khi gii phng trnh, hy nhm nhng nghim p ca phng trnh bt c
lc no c th.
sng tc mt s phng trnh ng cp i vi hai biu
thc
Ta bit rng nu mt phng trnh ng cp bc k i vi hai biu thc P(x) v Q(x) th c
gii bng cch chia c hai v cho [P(x)]
k
(hoc [Q(x)]
k
), sau t t =
P(x)
Q(x)
(hoc t =
Q(x)
P(x)
),
a v phng trnh a thc bc k theo t. Vn dng iu ny ta c mt phng php n gin
to ra nhiu phng trnh th v.
V d 15. Xt mt phng trnh bc hai
7t
2
+ 13t 2 = 0.
Ly t =
x 1
x
2
+ x + 1
ta c
7.

x 1
x
2
+ x + 1

2
+ 13.
x 1
x
2
+x + 1
2 = 0.
Quy ng b mu ta c bi ton sau
Bi 17 ( ngh OLYMPIC 30/04/2009). Gii phng trnh
2(x
2
+x + 1)
2
7(x 1)
2
= 13(x
3
1).
Gii
313
Tp xc nh R. Do x
2
+ x + 1 > 0 nn chia c hai v phng trnh cho (x
2
+ x + 1)
2
> 0 ta
c
2 7.

x 1
x
2
+x + 1

2
= 13.
x 1
x
2
+ x + 1
.
t t =
x 1
x
2
+x + 1
. Khi
2 7t
2
= 13t 7t
2
+ 13t 2 = 0

t = 2
t =
1
7
.
Khi t = 2 ta c
x 1
x
2
+x + 1
= 2 2x
2
+ 3x + 1 = 0

x = 1
x =
1
2
.
Khi t =
1
7
ta c
x 1
x
2
+x + 1
=
1
7
x
2
6x + 8 = 0

x = 2
x = 4.
Phng trnh cho c bn nghim x = 1, x =
1
2
, x = 2, x = 4. 2
Nhn xt: Phng trnh ny c nhiu hn mt nghim, v cc nghim ca phng trnh ny
u l s nguyn v s hu t, do ta c th gii nhanh chng bng cch khai trin a v
phng trnh bc bn, sau nhm nghim, a v phng trnh tch.
V d 16. Xt mt phng trnh bc hai c nghim
2t
2
7t + 3 = 0.
Ly t =

x
2
+x + 1
x 1
ta c
2
x
2
+x + 1
x 1
7

x
2
+x + 1
x 1
+ 3 = 0.
Quy ng b mu ta c
2(x
2
+x + 1) + 3(x 1) = 7

(x 1)(x
2
+ x + 1).
Ta c bi ton sau
Bi 18 ( ngh OLYPIC 30/04/2007). Gii phng trnh
2x
2
+ 5x 1 = 7

x
3
1. (1)
Gii
iu kin x 1.
(1) 3(x 1) + 2(x
2
+ x + 1) = 7

(x 1)(x
2
+x + 1). (2)
314
V x = 1 khng phi l nghim nn chia c hai v ca (2) cho x 1 > 0 ta c
3 + 2
x
2
+x + 1
x 1
= 7

x
2
+x + 1
x 1
. (3)
t t =

x
2
+x + 1
x 1
x
2
+ (1 t
2
)x + 1 + t
2
= 0. iu kin ca t l

t 0

x
= t
4
6t
2
3 0
t

3 + 2

3.
Phng trnh (3) tr thnh 2t
2
7t + 3 = 0 t

3,
1
2

.
Kt hp vi iu kin ca t ta c t = 3. Vy

x
2
+ x + 1
x 1
= 3 9x 9 = x
2
+ x + 1 x
2
8x + 10 = 0 x = 4

6.
Kt hp vi iu kin ta c x = 4

6 l tt c cc nghim ca phng trnh (1). 2


Nhn xt: Gi Q(x) = x 1, P(x) = x
2
+ x + 1. Mu cht ca li gii l phn tch v tri
ca PT (1) thnh
V T = 2P(x) + 3Q(x).
Tinh ta s thy 2 l h s ca x
2
trong v tri ca (1). Cng t suy ra 3. Tuy nhin d
dng tm c cc s 2 v 3 bng phng php h s bt nh
2x
2
+ 5x 1 = p(x
2
+ x + 1) + q(x 1)
2x
2
+ 5x 1 = px
2
+ (p + q)x +p q.
ng nht h s ta c

p = 2
p +q = 5
p q = 1

p = 2
q = 3.
V d 17. Xt x = 2. Khi
(x
2
+ 2x + 2) = 10, x + 1 = 3,
3(x
2
+ 2x + 2) 8(x + 1) = 6,
(x + 1)(x
2
+ 2x + 2) = 30,
(x + 1)(x
2
+ 2x + 2) = x
3
+ 3x
2
+ 4x + 2.
Vy vi x = 2 th
3(x
2
+ 2x + 2) 8(x + 1) =

30.
6

30
=
6

30

x
3
+ 3x
2
+ 4x + 2
Ta c bi ton sau
Bi 19. Gii phng trnh
3x
2
2x 2 =
6

30

x
3
+ 3x
2
+ 4x + 2
315
Gii
iu kin
x
3
+ 3x
2
+ 4x + 2 0 (x + 1)(x
2
+ 2x + 2) x 1.
Phng trnh cho vit li
3(x
2
+ 2x + 2) 8(x + 1) =
6

30

(x + 1)(x
2
+ 2x + 2). (1)
D thy x = 1 khng l nghim ca (1).
Tip theo xt x = 1. Chia c hai v ca (1) cho x + 1 > 0 ta c
3.
x
2
+ 2x + 2
x + 1
8 =
6

30

x
2
+ 2x + 2
x + 1
. (2)
t t =

x
2
+ 2x + 2
x + 1
> 0. Khi
3t
2
8 =
6

30
t 3t
2

30
t 8 = 0 3

30t
2
6t 8

30 = 0. (3)
Nhn xt rng t l nghim dng ca phng trnh (3), hay

10
3
. Vy

x
2
+ 2x + 2
x + 1
=

10
3
3x
2
+ 6x + 6 = 10x + 10

x = 2
x =
2
3
.
Kt hp vi iu kin ta thy x = 2 l nghim duy nht ca phng trnh cho. 2
Bi 20. Gii phng trnh
x
2
3x + 1 =

3
3

x
4
+ x
2
+ 1. (1)
Gii
Tp xc nh R. V
x
4
+x
2
+ 1 = (x
2
+ 1)
2
x
2
= (x
2
+x + 1)(x
2
x + 1)
nn
(1) 2(x
2
x + 1) (x
2
+x + 1) =

3
3

(x
2
+ x + 1)(x
2
x + 1)
2
x
2
x + 1
x
2
+x + 1
1 =

3
3

x
2
x + 1
x
2
+x + 1
.
t t =

x
2
x + 1
x
2
+ x + 1
> 0. Khi
2t
2
1 +

3
3
t = 0 2

3t
2
+t

3 = 0

t =
1

3
t =
3
2

3
(loi)
316
Vy

x
2
x + 1
x
2
+ x + 1
=
1

3
2x
2
4x + 2 = 0 x = 1.
Phng trnh cho c nghim duy nht x = 1. 2
Bi 21. Gii phng trnh 2(x
2
3x + 2) = 3

x
3
+ 8.
Gii
iu kin : x > 3. Phng trnh tng ng
2(x
2
2x + 4) 2(x + 2) = 3

(x + 2)(x
2
2x + 4)
2 2.
x + 2
x
2
2x + 4
= 3

x + 2
x
2
2x + 4
.
t t =

x + 2
x
2
2x + 4
0. Khi
2 2t
2
= 3t 2t
2
+ 3t 2 = 0

t =
1
2
t = 2 (loi).
Vy

x + 2
x
2
2x + 4
=
1
2
x
2
6x 4 = 0

x = 3

13
x = 3 +

13.
Phng trnh cho c hai nghim x = 3

13 v x = 3 +

13. 2
Bi 22 ( ngh OLYMPIC 30/04/2009). Gii phng trnh

x
2
+x 6 + 3

x 1

3x
2
6x + 19 = 0.
Gii
iu kin

x
2
+x 6 0
x 1 0
3x
2
6x + 19 0
x 2. Phng trnh tng ng

x
2
+ x 6 + 3

x 1 =

3x
2
6x + 19
x
2
+x 6 + 6

(x
2
+ x 6)(x 1) + 9x 9 = 3x
2
6x + 19
3

(x 2)(x + 3)(x 1) = x
2
8x + 17
3

(x
2
+ 2x 3)(x 2) = (x
2
+ 2x 3) 10(x 2) (1)
3

x
2
+ 2x 3
x 2
=
x
2
+ 2x 3
x 2
10. (2)
(Do x = 2 khng l nghim ca (2)). t t =

x
2
+ 2x 3
x 2
0. Thay vo (2) ta c
3t = t
2
10 t
2
3t 10 = 0

t = 2 (loi)
t = 5.
317
Vy

x
2
+ 2x 3
x 2
= 5 x
2
23x + 47 = 0 x =
23

341
2
.
Kt hp vi iu kin ta thy phng trnh cho c hai nghim x =
23

341
2
. 2
Bi 23. Gii phng trnh
x
4
+ 2x
3
+ 2x
2
2x + 1 =

x
3
+x

1 x
2
x
. (1)
Gii
Do x
4
+2x
3
+2x
2
2x+1 = x
2
(x+1)
2
+(1 x)
2
> 0, x R nn nu x l nghim ca (1) th

x > 0
1 x
2
x
> 0
0 < x < 1.
Vi iu kin th
(1) x
2
(x + 1)
2
+ (1 x)
2
= (x
2
+ 1)

(1 x)[x(1 + x)]. (2)


t u = x(1 +x), v = 1 x (iu kin u > 0, v > 0). Khi u +v = x
2
+1. Vy (2) tr thnh
u
2
+v
2
= (u + v)

uv

u
v

2
+ 1 =

u
v

v
+

. (3)
t t =

v
, thay vo (3) ta c
t
4
+ 1 = t
3
+t (t 1)(t
3
1) = 0 t = 1.
Vy

v
= 1 u = v. Do
x(1 + x) = 1 x x
2
+ 2x 1 = 0

x = 1 +

2
x = 1

2 (loi)
Phng trnh cho c nghim duy nht x = 1 +

2. 2
Bi 24. Gii phng trnh

5x
2
+ 14x + 9

x
2
x 20 = 5

x + 1. (1)
Gii
iu kin

5x
2
+ 14x + 9 0
x
2
x 20 0
x + 1 0

(x + 1)(5x + 9) 0
(x + 4)(x 5) 0
x + 1 0
x 5.
Ta c
(1)

5x
2
+ 14x + 9 =

x
2
x 20 + 5

x + 1
318
2x
2
5x + 2 = 5

(x + 4)(x 5)(x + 1)
2(x
2
4x 5) + 3(x + 4) = 5

(x
2
4x 5)(x + 4). (2)
Vi iu kin x 5, chia c hai v ca (2) cho x + 4 > 0 ta c
2.
x
2
4x 5
x + 4
+ 3 = 5

x
2
4x 5
x + 4
(3)
t t =

x
2
4x 5
x + 4
0, thay vo (3) ta c
2t
2
5t + 3 = 0

t = 1
t =
3
2
.
Khi t = 1, ta c

x
2
4x 5
x + 4
= 1 x
2
5x 9 = 0 x =
5

6
2
.
Khi t =
3
2
, ta c

x
2
4x 5
x + 4
=
3
2
, ngha l
4(x
2
5x 5) = 9x + 36 4x
2
35x 56 = 0

x = 8
x =
7
4
.
Kt hp vi iu kin ta c cc nghim ca phng trnh cho l x = 8, x =
5 +

61
2
. 2
xy dng phng trnh t cc ng thc
Xut pht t mt ng thc no , chng ta c th xy dng ln cc phng trnh v t.
Chng hn t hng ng thc
(a +b +c)
3
= a
3
+b
3
+c
3
+ 3(a +b)(b +c)(c + a)
ta c
(a +b +c)
3
= a
3
+b
3
+ c
3
(a +b)(b + c)(c +a) = 0.
Bng cch chn a, b, c sao cho (a +b +c)
3
= a
3
+ b
3
+ c
3
ta x to ra c phng trnh v t
cha cn bc ba.
V d 18. Cho
a =
3

7x + 1, b =
3

x
2
x 8, c =
3

x
2
8x 1
th a
3
+b
3
+c
3
= 8. Ta c bi ton sau
Bi 25 ( ngh OLYMPIC 30/04/1999). Gii phng trnh
3

7x + 1
3

x
2
x 8 +
3

x
2
8x 1 = 2.
319
Gii
Tp xc nh R. t
a =
3

7x + 1, b =
3

x
2
x 8, c =
3

x
2
8x 1.
Khi

a
3
+b
3
+c
3
= 8 (1)
a +b +c = 2 (2)
Mt khc ta c hng ng thc
(a +b +c)
3
= a
3
+b
3
+ c
3
+ 3(a +b)(b +c)(c + a). (3)
Thay (1), (2) vo (3) ta c
(a +b)(b +c)(c + a) = 0

a = b
b = c
c = a.
Vy

7x + 1 =
3

x
2
x 8
3

x
2
x 8 =
3

x
2
8x 1
3

x
2
8x 1 =
3

7x + 1

7x + 1 = x
2
x 8
x
2
x 8 = x
2
8x 1
x
2
8x 1 = 7x 1

x
2
8x 9 = 0
7x = 7
x
2
x = 0

x = 1
x = 9
x = 1
x = 0.
Thay cc gi tr 1, 0, 1, 9 vo phng trnh cho thy tho mn. Vy phng trnh c tp
nghim S = 1; 0; 1; 9. 2
V d 19. Cho
a =
3

3x
2
x + 2001, b =
3

3x
2
7x + 2002, c =
3

6x 2003
th a
3
+b
3
+ c
3
= 2002. Ta c bi ton sau
Bi 26. Gii phng trnh
3

3x
2
x + 2001
3

3x
2
7x + 2002
3

6x 2003 =
3

2002.
Gii
t
a =
3

3x
2
x + 2001, b =
3

3x
2
7x + 2002, c =
3

6x 2003.
Khi
(a +b +c)
3
= a
3
+b
3
+c
3
(a +b)(b +c)(c + a) = 0.
320
Vic gii phng trnh cho c quy v gii

3x
2
x + 2001 =
3

3x
2
7x + 2002
3

3x
2
7x + 2002 =
3

6x 2003
3

6x 2003 =
3

3x
2
x + 2001.

x =
1
6
x =
1

13
6
Vy phng trnh c tp nghim S =

1
6
;
1

13
6

. 2
V d 20. Cho
a =
3

1945x + 1975, b =
3

60x + 15, c =
3

15 x
th a
3
+b
3
+c
3
= 2004x + 2005. Ta c bi ton sau
Bi 27. Gii phng trnh
3

1945x + 1975 +
3

60x + 15 +
3

15 x
3

2004x + 2005 = 0.
Gii
Tp xc nh R. t
a =
3

1945x + 1975, b =
3

60x + 15, c =
3

15 x.
Khi a
3
+ b
3
+c
3
= 2004x + 2005. Thay vo PT cho ta c
a + b + c
3

a
3
+b
3
+c
3
= 0 a +b +c =
3

a
3
+ b
3
+c
3
(a + b + c)
3
= a
3
+ b
3
+c
3
(1)
Mt khc ta c hng ng thc
(a + b + c)
3
= a
3
+b
3
+c
3
+ 3(a + b)(b + c)(c +a). (2)
T (1) v (2) suy ra
(a +b)(b + c)(c +a) = 0

a = b
b = c
c = a.
Vy

1945x + 1975 = 60x 15


60x + 15 = x 15
15 x = (1945x + 1975)

x =
1990
2005
x =
30
59
x =
1990
1944
.
Vy phng trnh c ba nghim x =
1990
2005
, x =
30
59
, x =
1990
1944
. 2
V d 21. Cho a =
3

3x + 1, b =
3

5 x, c =
3

2x 9 th a
3
+b
3
+c
3
= 4x 3. Ta c bi
ton sau
Bi 28. Gii phng trnh
3

3x + 1 +
3

5 x +
3

2x 9
3

4x 3 = 0.
321
V d 22. T hng ng thc
a
3
+b
3
ab(a +b) = (a +b)(a b)
2
,
ly a =
3

x + 1, b =
3

x + 2. Khi
a
3
+b
3
ab(a + b)
=x + 1 x 2 +
3

x
2
+ 3x + 2

x + 1
3

x + 2

=1 +
3

x
2
+ 3x + 2

x + 1
3

x + 2

.
Bng cch cho a
3
+b
3
ab(a +b) = 0 ta c bi ton sau:
Bi 29. Gii phng trnh
3

x
2
+ 3x + 2

x + 1
3

x + 2

= 1.
Gii
Tp xc nh R. Phng trnh vit li
(x + 1) + (x 2) +
3

x
2
+ 3x + 2

x + 1
3

x + 2

= 0. (*)
t a =
3

x + 1, b =
3

x + 2. Thay vo (*) ta c
a
3
+b
3
ab(a +b) = 0 (a +b)(a b)
2
= 0

a = b
a = b.
Vy

x + 1 =
3

x + 2
3

x + 1 =
3

x + 2

x + 1 = x 2
0x = 1 (v)
x =
3
2
.
Th li ta thy x =
3
2
l nghim duy nht ca phng trnh. 2
xy dng phng trnh t cc h i xng loi II
Xt h phng trnh

(x +)
2
= ay + b (1)
(y + )
2
= ax + b (2)
T (2) suy ra

y + =

ax +b
y + =

ax +b

y =

ax + b

y =

ax +b

.
Thay vo (1) ta c

(x +)
2
=
a

ax +b

+b ()
(x + )
2
=
a

ax +b

+b.
322
n y bng cch chn , , a, b ta s xy dng c cc phng trnh v t. Cch gii cc
phng trnh dng ny l t y + =

ax + b (hoc

ax +b) a v h i xng loi


II trn bit cch gii. By gi ta s i xy dng mt s phng trnh dng ny
V d 23. Cho = 3, = 2, a = 3, b = 8 thay vo () ta c
(3x + 2)
2
=

3x + 8 + 6.
Ta c bi ton sau
Bi 30. Gii phng trnh
9x
2
+ 12x 2 =

3x + 8.
Gii
iu kin x
8
3
. Phng trnh vit li
(3x + 2)
2
6 =

3x + 8. (1)
t 3y + 2 =

3x + 8, suy ra (3y + 2)
2
= 3x + 8. Kt hp vi (1) ta c h

(3x + 2)
2
= 3y + 8 (2)
(3y + 2)
2
= 3x + 8. (3)
x, y tho mn (1) v (2) th x
8
3
v y
8
3
. Ly (2) tr (3) ta c
3(x y)(3x + 3y + 4) = 3(y x) (x y)(3x + 3y + 5) = 0

x y = 0
3x + 3y + 5 = 0

y = x
3y = (3x + 5).
Vi y = x, thay vo (2) ta c
(3x + 2)
2
= 3x + 8 9x
2
+ 9x 4 = 0

x =
1
3
(nhn)
x =
4
3
(loi).
Vi y = (3x + 5), thay vo (2) ta c
(3x + 2)
2
= 3x + 3 9x
2
+ 15x + 1 = 0

x =
5 +

21
6
(loi)
x =
5

21
6
(nhn).
Cc nghim ca phng trnh cho l x =
1
3
v x =
5

21
6
. 2
V d 24. Cho = 1, = 1, a =
1
2
, b =
3
2
thay vo () ta c
(x + 1)
2
=

x
2
+
3
2
2

1
2
+
3
2
2 (x + 1)
2
=

x
2
+
3
2
+ 2.
Ta c bi ton sau
323
Bi 31. Gii phng trnh 2x
2
+ 4x =

x + 3
2
.
V d 25. Cho = 2, = 1, a = 8000, b = 1 thay vo () ta c
(2x 1)
2
= 4000

8000x + 1 + 4001
Ta c bi ton sau
Bi 32. Gii phng trnh
x
2
x 1000

8000x + 1 = 1000
Sau y l mt cch xy dng h khc:
Nu xt h

(x +)
3
= ay +b
(y +)
3
= ax +b.
T phng trnh di ta c
y + =
3

ax +b y =
3

ax + b

.
Thay vo phng trnh trn ca h :
(x + )
3
=
a
3

ax +b

+ b.
V d 26. Chn = 1, = 1, a = 3, b = 5, ta c
(x + 1)
3
= 3
3

3x + 5 + 2.
Ta c bi ton sau
Bi 33 ( ngh OLYMPIC 30/04/2009). Gii phng trnh
x
3
+ 3x
2
3
3

3x + 5 = 1 3x.
Gii
Tp xc nh R. Phng trnh cho tng ng
(x + 1)
3
= 3
3

3x + 5 + 2. (1)
t y + 1 =
3

3x + 5. Ta c h

(x + 1)
3
= 3y + 5 (1)
(y + 1)
3
= 3x + 5 (2)
Ly (1) tr (2) theo v ta c
(x + 1)
3
(y + 1)
3
= 3(x y)
(x y)[(x + 1)
2
+ (x + 1)(y + 1) + (y + 1)
2
+ 3] = 0
324
x = y

do (x + 1)
2
+ (x + 1)(y + 1) + (y + 1)
2
0

.
Thay vo (1) ta c
(x + 1)
3
= 3x + 5 x
3
+ 3x
2
4 = 0

x = 1
x = 2.
Phng trnh cho c hai nghim l x = 1 v x = 2. 2
V d 27. Cho = 2, = 0, a = 4004, b = 2001 ta c
(2x)
3
= 2002
3

4004x 2001 2001.


Ta c bi ton sau
Bi 34. Gii phng trnh

8x
3
+ 2001
2002

3
= 4004x 2001.
Xy dng phng trnh v t da vo tnh n iu ca hm s.
Da vo kt qu Nu hm s y = f(x) n iu th f(x) = f(y) x = y ta c th xy dng
c nhiu phng trnh, h phng trnh.
V d 28. Xt hm s f(t) = t
3
+ t ng bin trn R. Cho f(x + 1) = f(
3

7x
2
+ 9x 4) ta
c
2(x + 1)
3
+ (x + 1)
2
+ 1 = 2(3x 1)

3x 1 + 3x 1 + 1.
Ta c bi ton sau
Bi 35. Gii phng trnh
x
3
4x
2
5x + 6 =
3

7x
2
+ 9x 4.
Gii
Tp xc nh R. t y =
3

7x
2
+ 9x 4. Ta c h

x
3
4x
2
5x + 6 = y (1)
7x
2
+ 9x 4 = y
3
(2)
Cng (1) v (2) theo v ta c
x
3
+ 3x
2
+ 4x + 2 = y
3
+ y (x + 1)
3
+ (x + 1) = y
3
+y. (3)
Xt hm s f(t) = t
3
+t. V f

(t) = 3t
2
+1 > 0, t R nn hm s f ng bin trn R. Do
(3) vit li
f(x + 1) = f(y) x + 1 = y.
Bi vy
3

7x
2
+ 9x 4 = x + 1 7x
2
+ 9x 4 = (x + 1)
3
325
x
3
4x
2
6x + 5 = 0 (x 5)(x
2
+x 1)

x = 5
x =
1

5
2
.
Kt lun: Phng trnh c tp nghim S = 5;
1

5
2
. 2
V d 29. Xt hm s f(t) = t
3
+ 2t ng bin trn R. Cho
f

x
3
+ 9x
2
19x + 11

= f(x 1).
ta c
x
3
+ 9x
2
19x + 11 + 2
3

x
3
+ 9x
2
19x + 11 = (x 1)
3
+ 2(x 1).
Khai trin v rt gn ta c bi ton sau
Bi 36 ( ngh OLYMPIC 30/04/2009). Gii phng trnh
x
3
6x
2
+ 12x 7 =
3

x
3
+ 9x
2
19x + 11.
Gii
t y =
3

x
3
+ 9x
2
19x + 11. Ta c h

y
3
= x
3
+ 9x
2
19x + 11
y = x
3
6x
2
+ 12x 7

y
3
= x
3
+ 9x
2
19x + 11
2y = 2x
3
12x
2
+ 24x 14
Cng hai phng trnh vi nhau ta c
y
3
+ 2y = x
3
3x
2
+ 5x 3 y
3
+ 2y = (x 1)
3
+ 2(x 1). (*)
Xt hm s f(t) = t
3
+ 2t. Vi mi t
1
= t
2
, ta c
f(t
1
) f(t
2
)
t
1
t
2
= t
2
1
+t
1
t
2
+ t
2
2
+ 2 =

t
1
+
t
2
2

2
+
3t
2
2
2
+ 2 > 0.
Vy hm s f(t) ng bin trn R. Do
() f(y) = f(x 1) y = x 1

x
3
+ 9x
2
19x + 11 = x 1
x
3
+ 9x
2
19x + 11 = x
3
3x
2
+ 3x 1
x
3
6x
2
+ 11x 6 = 0 x 1, 2, 3 .
Phng trnh cho c ba nghim x = 1, x = 2, x = 3. 2
V d 30. Xt hm s f(t) = 2t
3
+t
2
+ 1 n iu trn [0; +). Cho
f(x + 1) = f

3x 1

ta c
2(x + 1)
3
+ (x + 1)
2
+ 1 = 2(3x 1)

3x 1 + 3x 1 + 1.
Ta c bi ton sau
326
Bi 37. Gii phng trnh
2x
3
+ 7x
2
+ 5x + 4 = 2(3x 1)

3x 1.
Gii
iu kin x
1
3
. t y =

3x 1, y 0. Ta c h

2x
3
+ 7x
2
+ 5x + 4 = 2y
3
3x 1 = y
2
.
Cng theo v hai phng trnh trn ta c
2x
3
+ 7x
2
+ 8x + 3 = 2y
3
+y
2
2(x + 1)
3
+ (x + 1)
2
= 2y
3
+y
2
. (*)
Xt hm s f(t) = 2t
3
+t
2
. V f

(t) = 6t
2
+2t 0, t 0 nn hm s f ng bin trn [0; +).
Do
() f(x + 1) = f(y) x + 1 = y x + 1 =

3x 1

x + 1 0
3x + 1 = x
2
+ 2x + 1

x 1

x = 0
x = 1

x = 0
x = 1.
Kt hp vi iu kin ta c x = 1 l nghim duy nht ca phng trnh cho. 2
V d 31. Xt hm s f(t) = t
3
+t n iu trn R. Nu cho f(2x) = f(
3

6x + 1) th c
8x
3
+ 2x = 6x + 1 +
3

6x + 1
3

6x + 1 = 8x
3
4x 1.
Ta c bi ton sau
Bi 38. Gii phng trnh
3

6x + 1 = 8x
3
4x 1.
V d 32. Xt hm s f(t) = t

1 +

t
2
+ 2

. Ta c
f

(t) = 1 +

t
2
+ 2 +
t
2

t
2
+ 2
> 0, t R.
Vy hm s f(t) = t

1 +

t
2
+ 2

ng bin trn R. Cho f(2x + 3) = f(3x) ta c


(2x + 3)

1 +

4x
2
+ 12x + 11

= 3x

1 +

9x
2
+ 2

.
Ta c bi ton
Bi 39. Gii phng trnh
(2x + 3)

4x
2
+ 12x + 11 + 3x

1 +

9x
2
+ 2

= 5x 3.
327
V d 33. Xt hm s ng bin trn khong (0; +) l
f(t) = log
2
t 2t + t
2
, t > 0.
Cho f

x + 2

= f

2 +
1
x

ta c
log
2

x + 2 2

x + 2 + x + 2 = log
2

2 +
1
x

2 +
1
x

2 +
1
x

1
2
log
2
(x + 2) + x + 3 = log
2
2x + 1
x
+

1 +
1
x

2
+ 2

x + 2.
Ta c bi ton sau
Bi 40. Gii phng trnh
1
2
log
2
(x + 2) + x + 3 = log
2
2x + 1
x
+

1 +
1
x

2
+ 2

x + 2.
Gii
iu kin

x (2; +)
x

;
1
2

(0; +)
x

2;
1
2

(0; +) .
Khi phng trnh vit li
log
2

x + 2 2

x + 2 + x + 2 = log
2

2 +
1
x

2 +
1
x

2 +
1
x

2
. (1)
Xt hm s f(t) = log
2
t 2t +t
2
, t > 0. Ta c
f

(t) =
1
t. ln 2
+ 2t 2 2

1
t. ln 2
.2t 2 = 2

2
ln 2
2 > 0
Vy hm s f(t) ng bin trn khong (0; +), do
(1) f

x + 2

= f

2 +
1
x

x + 2 = 2 +
1
x
. (2)
Vi iu kin x

2;
1
2

(0; +), bnh phng hai v phng trnh (2) ta c


x + 2 = 4 +
4
x
+
1
x
2
x
3
2x
2
4x 1 = 0

x = 1
x =
3

13
2
.
Kt hp vi iu kin, ta thy PT cho c hai nghim x = 1 v x =
3 +

13
2
. 2
328
Xy dng phng trnh v t da vo cc phng trnh lng gic.
T mt phng trnh lng gic n gin no , kt hp vi cc php bin i lng gic th
s tm ra cc phng trnh v t hay. T mt phng trnh lng gic n gin no , kt hp
vi cc php bin i lng gic th s tm ra cc phng trnh v t hay.
V d 34. T phng trnh cos 3t = cos
t
2
, vi t [0; ], ta thy phng trnh ny tng
ng vi 8 cos
3
t 6 cos t =

2(1 + cos t). t x = 2 cos t ta c bi ton sau.


Bi 41 ( ngh OLYMPIC 30/04/2006). Gii phng trnh
x
3
3x =

x + 2. (1)
Gii
iu kin x 2. Nu x > 2 th
x
3
3x = x + x(x
2
4) > x >

2x =

x +x >

x + 2.
Vy x > 2 khng tho mn (1), do gii phng trnh (1), ch cn xt 2 x 2. Khi
t x = 2 cos t, iu kin t [0; ]. Thay vo (1) ta c
8 cos
3
t 6 cos t =

2 (1 + cos t) 4 cos
3
t 3 cos t = cos
t
2
cos 3t = cos
t
2

3t =
t
2
+ k2
3t =
t
2
+k2

t =
k4
5
t =
k4
7
(k Z) .
Do t [0; ] nn ch ly cc nghim t = 0, t =
4
5
, t =
4
7
. Phng trnh cho c ba nghim
x = 2, x = 2 cos
4
5
, x = 2 cos
4
7
. 2
V d 35. T phng trnh cos 3t = sin t, vi t [0; ], ta thy phng trnh ny tng ng
vi 4 cos
3
t 3 cos t =

1 cos
2
t. t x = cos t ta c bi ton sau:
Bi 42 ( ngh Olympic 30/04/2003-ton 10). Gii phng trnh
4x
3
3x =

1 x
2
.
Nu thay x bi x 1 ta c bi ton kh hn:
Bi 43. Gii phng trnh 4x
3
12x
2
+ 9x 1 =

2x x
2
.
V d 36. T phng trnh sin 3t = cos t,vi t [0; ], ta thy phng trnh ny tng ng
vi
3 sin t 4 sin
3
t = cos x sin t

3 4 sin
2
t

= cos t

1 cos
2
t

4 cos
2
t 1

= cos t

1 cos
2
t =
cos t
4 cos
2
t 1
.
Ly x = cos t ta c bi ton sau.
329
Bi 44. Gii phng trnh

1 x
2
=
x
4x
2
1
.
Gii
T iu kin [x[ 1, x =
1
2
v x =
1
2
, ta t
x = cos t, t [0; ] , x =

3
, x =
2
3
.
Thay vo phng trnh cho ta c

1 cos
2
t =
cos t
4 cos
2
t 1

1 cos
2
t

4 cos
2
t 1

= cos t
sin t

4 4 sin
2
t 1

= cos t sin t

3 4 sin
2
t

= cos t
3 sin t 4 sin
3
t = cos x sin 3t = cos t sin 3t = sin

2
t

3t =

2
t +k2
3t =

2
t

+k2

t =

8
+
k
2
t =

4
+k
(k Z).2
Trn on [0; ], ta nhn c cc nghim t
1
=

8
, t
2
=
5
8
, t
3
=

4
. Nghim ca phng trnh
cho l cos

8
, cos
5
8
, cos

4
. 2
V d 37. Ta c sin 5 = 16 sin
5
20 sin
3
+5 sin . Xt sin 5t = cos t,vi t [0; ], ta thy
phng trnh ny tng ng vi
16 sin
5
t 20 sin
3
t + 5 sin t = cos t
sin t

16 sin
4
t 20 sin
2
t + 5

= cos t
sin t

16

1 cos
2
t

2
20

1 cos
2
t

+ 5

= cos t
sin t

16 cos
4
t 12 cos
2
t + 1

= cos t

1 cos
2
t =
cos t
16 cos
4
t 12 cos
2
t + 1
.
Ly x = cos t ta c bi ton sau.
Bi 45. Gii phng trnh

1 x
2
=
x
16x
4
12x
2
+ 1
.
Gii
T iu kin [x[ 1 v 16x
4
12x
2
+ 1 = 0, ta t
x = cos t, t [0; ] , 16 cos
4
t 12 cos
2
t + 1 = 0.
Thay vo phng trnh cho ta c

1 cos
2
t =
cos t
16 cos
4
t 12 cos
2
t + 1
sin t

16 cos
4
t 12 cos
2
t + 1

= cos t
330
sin t

16

1 sin
2
t

2
12

1 sin
2
t

+ 1

= cos t
sin t

16 sin
4
t 20 sin
2
t + 5

= cos t
16 sin
5
t 20 sin
3
t + 5 sin t = cos t sin 5t = cos t
sin 5t = sin

2
t

t =

12
+
k
3
t =

8
+
k
2
(k Z).2
V d 38. Xt sin 5t = cos 3t,vi t [0; ], phng trnh ny tng ng vi
16 sin
5
t 20 sin
3
t + 5 sin t = cos 3t
sin t

16 sin
4
t 20 sin
2
t + 5

= 4 cos
3
t 3 cos t
sin t

16

1 cos
2
t

2
20

1 cos
2
t

+ 5

= 4 cos
3
t 3 cos t
sin t

16 cos
4
t 12 cos
2
t + 1

= 4 cos
3
t 3 cos t

1 cos
2
t

16 cos
4
t 12 cos
2
t + 1

= 4 cos
3
t 3 cos t.
Ly x = cos t ta c bi ton sau.
Bi 46. Gii phng trnh

1 x
2
(16x
4
12x
2
+ 1) = 4x
3
3x.
Gii
T iu kin [x[ 1, ta t x = cos t, t [0; ]. Thay vo phng trnh cho ta c

1 cos
2
t

16 cos
4
t 12 cos
2
t + 1

= 4 cos
3
t 3 cos t
sin t

16

1 sin
2
t

2
12

1 sin
2
t

+ 1

= cos 3t
sin t

16 sin
4
t 20 sin
2
t + 5

= cos 3t
16 sin
5
t 20 sin
3
t + 5 sin t = cos 3t sin 5t = cos 3t
sin 5t = sin

2
3t

t =

16
+
k
4
t =

4
+ k
(k Z).2
V d 39. T sin

3t

4

= sin

t +

4

, vi t [0; ], ta c
sin 3t cos 3t = sin t + cos t
3 sin t 4 sin
3
t 4 cos
3
t + 3 cos t = sin t + cos t
2 cos t + 3 sin t 4 sin
3
t = 4 cos
3
t + sin t
2 cos t + sin t

3 4 sin
2
t

= 4 cos
3
t + sin t
2 cos t + sin t

4 cos
2
t 1

= 4 cos
3
t + sin t
2 cos t +

1 cos
2
t

4 cos
2
t 1

= 4 cos
3
t +

1 cos
2
t.
Ly x = cos t ta c bi ton sau.
Bi 47. Gii phng trnh 2x + (4x
2
1)

1 x
2
= 4x
3
+

1 x
2
.
Gii
331
T iu kin [x[ 1, ta t x = cos t, t [0; ]. Thay vo phng trnh cho ta c
2 cos t +

4 cos
2
t 1

1 cos
2
t = 4 cos
3
t +

1 cos
2
t
2 cos t +

4 cos
2
t 1

sin t = 4 cos
3
t + sin t
sin t

4 cos
2
t 2

= 4 cos
3
t 2 cos t
sin t

2 cos
2
t 1

= cos t

2 cos
2
t 1

2 cos
2
t 1

(sin t cos t) = 0

cos 2t = 0
sin t = cos t

2t =

2
+ k
tan t = 1

t =

4
+
k
2
t =

4
+k
t =

4
+
k
2
.
Bi 48. Gii bt phng trnh

1 + x +

1 x 2
x
2
4
. (1)
Gii
iu kin 1 x 1. t x = cos , [0; ]. Thay vo (1) ta c

1 + cos +

1 cos 2
cos
2

4
2 cos

2


4

2 sin
2

2


4

cos
2

2


4

2 cos

2


4

1 cos
2

2


4

cos
2

2


4

cos
4

2


4

cos
2

2


4

2 cos

2


4

+ 2 0

cos

2


4

cos
2

2


4

+ 2 cos

2


4

+ 2

0. (2)
V (2) lun ng vi mi [0; ] nn nghim ca (1) l 1 x 1.
S dng cn bc n ca s phc sng to v gii h phng trnh.
Cho s phc z = r (cos +i sin ) , r > 0. Khi cc cn bc n ca z l
z
k
=
n

cos
+ 2k
n
+i sin
+ 2k
n

, k = 0, 1, 2, . . . , n 1.
Cc cn bc hai ca s phc z = r (cos +i sin ) , r > 0 l
z
0
=

cos

2
+i sin

2

, z
1
=

cos

2
+i sin

2

.
Cc cn bc ba ca s phc z = r (cos + i sin ) , r > 0 l
z
0
=
3

cos

3
+ i sin

3

,
z
1
=
3

cos
+ 2
3
+i sin
+ 2
3

,
z
2
=
3

cos
+ 4
3
+i sin
+ 4
3

.
332
Mt phng trnh nghim phc f(z) = 0, vi z = x +iy, ta bin i thnh
h(x, y) + ig(x, y) = 0

h(x, y) = 0
g(x, y) = 0.
Ngha l mt phng trnh nghim phc, bng cch tch phn thc v phn o lun c th
a v h phng trnh.
1. Sng tc cc h phng trnh bng cch lu tha ba mt s phc cho trc
tm cn bc ba ca s phc 1 + i, ta tm s phc z = x +iy, x R, y R sao cho
(x +iy)
3
= 1 +i x
3
+ 3x
2
yi + 3xy
2
i
2
+ y
3
i
3
= 1 +i
x
3
3xy
2
+ (3x
2
y y
3
)i = 1 +i

x
3
3xy
2
= 1
3x
2
y y
3
= 1.
Gii h ny ta tm c x v y, t c z. Tuy nhin, c th tm z bng cch khai cn bc ba
ca 1 + i nh sau : Ta c
1 + i =

cos

4
+i sin

4

.
Vy cc cn bc ba ca 1 + i l
z
0
=
3

cos

12
+i sin

12

=
6

cos

12
+i sin

12

,
z
1
=
3

cos

4
+ 2
3
+ i sin

4
+ 2
3

=
6

cos
3
4
+ i sin
3
4

,
z
2
=
3

cos

4
+ 4
3
+ i sin

4
+ 4
3

=
6

cos
17
12
+ i sin
17
12

.
T y, ngc li ta tm c nghim ca h

x
3
3xy
2
= 1
3x
2
y y
3
= 1
l

x =
6

2 cos

12
y =
6

2 sin

12
;

x =
6

2 cos
3
4
y =
6

2 sin
3
4
;

x =
6

2 cos
17
12
y =
6

2 sin
17
12
.
V d 40. Xt s phc z = 5

cos

3
+i sin

3

= 5

1
2
+

3
2
i

. Gi s x+yi l s phc tho


mn iu kin
(x +yi)
3
= 5

1
2
+

3
2
i

x
3
+ 3x
2
yi + 3xy
2
i
2
+y
3
i
3
=
5
2
+
5

3
2
i
2x
3
6xy
2
+ (6x
2
y 2y
3
)i = 5 + 5

3i

2x
3
6xy
2
= 5
6x
2
y 2y
3
= 5

3.
Ta c bi ton sau
333
Bi 49. Gii h phng trnh

2x
3
6xy
2
= 5 (1)
6x
2
y 2y
3
= 5

3 (2).
Gii
Nhn hai v ca (2) vi i ri cng vi (1) ta c
2x
3
6xy
2
+ (6x
2
y 2y
3
)i = 5 + 5

3i
x
3
3xy
2
+ (3x
2
y y
3
)i =
5
2
+
5

3
2
i
x
3
+ 3x
2
yi + 3xy
2
i
2
+y
3
i
3
=
5
2
+
5

3
2
i
(x +yi)
3
= 5

1
2
+

3
2
i

.
Vy x +yi l mt cn bc ba ca s phc
z = 5

1
2
+

3
2
i

= 5

cos

3
+i sin

3

.
M z c cc cn bc ba l
z
0
=
3

cos

9
+i sin

9

,
z
1
=
3

cos

3
+ 2
3
+ i sin

3
+ 2
3

=
3

cos
7
9
+i sin
7
9

,
z
2
=
3

cos

3
+ 4
3
+ i sin

3
+ 4
3

=
3

cos
13
9
+i sin
13
9

.
Vy cc nghim ca h phng trnh l

x =
3

5 cos

9
y =
3

5 sin

9
;

x =
3

5 cos
7
9
y =
3

5 sin
7
9
;

x =
3

5 cos
13
9
y =
3

5 sin
13
9
.
2
2. Sng tc cc h phng trnh t hai s phc cho trc.
V d 41. Xt hai s phc z
1
v z
2
nh sau

z
1
= 7

5i
z
2
=

5i

z
1
+z
2
= 7
z
1
z
2
= 7

5i + 5
Vy z
1
v z
2
l nghim ca phng trnh
z
2
7z + 5 + 7

5i = 0 z 7 +
5 + 7

5i
z
= 0
z +
5
z
+
7

5i
z
= 7 z +
5z
zz
+
7

5iz
zz
= 7
334
Gi s z = x +yi vi x, y R. Khi phng trnh trn vit li
x +yi +
5(x yi)
x
2
+y
2
+
7

5(xi + y)
x
2
+ y
2
= 7
x +yi +
5x + 7

5y
x
2
+y
2
+
7

5x 5y
x
2
+ y
2
i = 7
x +
5x + 7

5y
x
2
+y
2
7 +

y +
7

5x 5y
x
2
+ y
2

i = 0

x +
5x + 7

5y
x
2
+y
2
7 = 0
y +
7

5x 5y
x
2
+ y
2
= 0.
Ta c bi ton sau
Bi 50. Gii h phng trnh

x +
5x + 7

5y
x
2
+y
2
= 7
y +
7

5x 5y
x
2
+y
2
= 0.
Ta xt tip mt bi tng t:
Bi 51. Gii h phng trnh

x +
3x y
x
2
+y
2
= 3
y
x + 3y
x
2
+ y
2
= 0.
Gii
iu kin x
2
+y
2
= 0. Xt s phc z = x +iy. Khi
iz = ix y, x
2
+ y
2
= [z[
2
= z.z
H phng trnh cho vit li

x +
3x y
x
2
+ y
2
= 3 (1)
yi
x + 3y
x
2
+ y
2
i = 0. (2)
Ly (1) tr (2) theo v ta c
x yi +
3x y
x
2
+y
2
+
x + 3y
x
2
+ y
2
i = 3 x yi +
3(x + yi)
x
2
+y
2

y xi
x
2
+y
2
= 3.
Hay
z + 3
z
z.z
+
iz
z.z
= 3 z + 3
1
z
+
i
z
= 3 (z)
2
3z + 3 + i = 0.
Ta c = 9 4(3 + i) = 3 4i. Xt s phc a + bi tho mn iu kin
3 4i = (a + bi)
2

a
2
b
2
= 3
2ab = 4

a
2
b
2
= 3
b =
2
a
.
335
Khi
a
2

4
a
2
= 3 a
4
+ 3a
2
4 = 0 a
2
= 1 a = 1.
Vy (a; b) = (1; 2) ; (a; b) = (1; 2) . Do = 3 4i c hai cn bc hai l (1 2i). Suy
ra

z =
3 + (1 2i)
2
= 2 i
z =
3 (1 2i)
2
= 1 +i.
Ngha l

x yi = 2 i
x yi = 1 +i

(x; y) = (2; 1)
(x; y) = (1; 1).
Vy h cho c hai nghim l

x = 2
y = 1
v

x = 1
y = 1.
. 2
V d 42. Xt hai s phc

z
1
=

2 i
z
2
= 2

2 + 2i

z
1
+z
2
= 3

2 + i
z
1
z
2
= 6.
Khi z
1
v z
2
l hai nghim ca phng trnh
z
2

2 + i

z + 6 = 0 z +
6
z
= 3

2 + i z +
6z
z.z
= 3

2 + i.
Gi s z = u +vi, khi phng trnh vit li
u + vi +
6(u vi)
u
2
+ v
2
= 3

2 + i
u +
6u
u
2
+v
2
+

v
6v
u
2
+v
2

= 3

2 + i

u +
6u
u
2
+v
2
= 3

2
v
6v
u
2
+v
2
= 1

1 +
6
u
2
+v
2

= 3

2
v

1
6
u
2
+v
2

= 1.
Ly u =

x, v =

y, ta c

x
3

1 +
6
x +y

1
6
x +y

= 1.
Ta c bi ton sau
Bi 52. Gii h phng trnh

x
3

1 +
6
x +y

1
6
x +y

= 1.
Mt bi tng t tng xut hin trong k thi VMO 1996:
336
Bi 53. Gii h phng trnh

3x

1 +
1
x +y

= 2

7y

1
1
x +y

= 4

2.
Gii
iu kin x > 0 v y > 0. t u =

x, v =

y. H phng trnh tr thnh

1 +
1
u
2
+v
2

=
2

3
v

1
1
u
2
+v
2

=
4

7
V u
2
+v
2
l bnh phng ca mun ca s phc z = u +iv nn ta bin i h phng trnh
thnh

u +
u
u
2
+v
2
=
2

3
v
v
u
2
+ v
2
=
4

u +
u
u
2
+v
2
=
2

3
(1)
iv
iv
u
2
+v
2
=
4

2i

7
(2)
Cng (1) v (2) theo v ta c
u +iv +
u iv
u
2
+v
2
=
2

3
+
4

2i

7
. (3)
V
u iv
u
2
+v
2
=
z
[z[
2
=
z
z.z
=
1
z
nn (3) vit li
z +
1
z
=
2

3
+
4

2i

7
z
2
2

3
+
2

2i

z + 1 = 0. (4)
Ta c

3
+
2

2i

2
1 =
1
3

8
7
+
4

2i

21
1 =
38
21
+
4

2i

21
.
Xt s phc a + bi tho mn iu kin

38
21
+
4

2i

21
= (a +bi)
2

a
2
b
2
=
38
21
2ab =
4

21
Khi
a
2

8
21a
2
=
38
21
21a
4
+ 38a
2
8 = 0 a
2
=
19 + 23
21
=
4
21
.
Vy (a; b) =

21
;

; (a; b) =

21
;

. Do
z = u +iv =

21

i.
337
V u > 0 v v > 0 nn

u =
1

3
+
2

21
v =
2

7
+

2.
Do

x =

3
+
2

21

2
=
11
21
+
4
3

7
y =

7
+

2
=
22
7
+
8

7
.
H phng trnh cho c nghim duy nht l

x =
11
21
+
4
3

7
y =
22
7
+
8

7
.
. 2
Bi 54 (VMO 2006). Gii h phng trnh

1
12
y + 3x

x = 2

1 +
12
y + 3x

y = 6.
Gii
iu kin x 0, y 0, y + 3x = 0. t u =

3x 0, v =

y 0. Thay vo h ta c

1
12
u
2
+v
2

3
= 2

1 +
12
u
2
+v
2

v = 6

u
12u
u
2
+v
2
= 2

3 (1)
v +
12v
u
2
+v
2
= 6. (2)
Nhn PT (2) vi i, sau cng vi PT (1) ta c
u +vi
12(u vi)
u
2
+ v
2
= 2

3 + 6i. (3)
Xt s phc z = u +vi, vi u 0, v 0. Khi (3) vit li
z
12z
z.z
= 2

3 + 6i z
12
z
= 2(

3 + 3i)
z
2
2(

3 + 3i)z 12 = 0. (4)
Ta c

= (

3 + 3i)
2
+ 12 = 6 + 6

3i + 12 = 6 + 6

3i
= 12

1
2
+

3
2
i

= 12

cos

3
+i sin

3

.
Vy

c hai cn bc hai l

12

cos

6
+i sin

6

12

3
2
+
1
2
i

3 +

3i

.
338
T (4) ta c

z =

3 + 3i + 3 +

3i =

3 + 3 +

3 +

i
z =

3 + 3i 3

3i =

3 3 +

i.
Do u 0 v v 0 nn

u =

3 + 3
v =

3 + 3

x =

3 + 3

2
3
y =

3 + 3

x = 4 + 2

3
y = 3

4 + 2

.
H cho c nghim duy nht

x = 4 + 2

3
y = 3

4 + 2

.
. 2
S dng bt ng thc lng gic trong tam gic
Trong thi vo i hc Nng nghip 1, nm 1995 c bi ton sau:
Bi 55. Gii phng trnh
sin
2
x + sin
2
y + sin
2
(x +y) =
9
4
. (1)
Phng trnh ny khin ta lin tng n mt bt ng thc c bn trong tam gic : Vi mi
tam gic ABC ta c
sin
2
A + sin
2
B + sin
2
C
9
4
, (2)
du bng xy ra khi v ch khi A = B = C =

3
.
Trong v tri ca bt ng thc (2), ly A = x, B = y, C = (x + y), ta thu c v tri
ca phng trnh (1). Li gii ca phng trnh (1) cng thu c da trn c s php chng
minh bt ng thc lng gic (2).
Gii
Ta c
(1) 1
cos 2x + cos 2y
2
+ sin
2
(x + y) =
9
4
1 cos(x +y) cos(x y) + 1 cos
2
(x +y) =
9
4
cos
2
(x + y) + cos(x y) cos(x + y) +
1
4
= 0. (3)
Phng trnh (3) l phng trnh bc hai i vi cos(x +y), ta c
= cos
2
(x y) 1 0.
V cos
2
(x y) 1 nn phng trnh c nghim th cos
2
(x y) = 1. Do

cos(x y) = 1
cos(x + y) =
cos(x y)
2
=
1
2

cos(x y) = 1
cos(x + y) =
cos(x y)
2
=
1
2

x y = k2
x + y =
2
3
+l2
(4)

x y =

2
+k2
x + y =

3
+l2.
(5)
339
Xt (4) ta c:
(4)

x y = k2; x + y =
2
3
+l2
x y = k2; x + y =
2
3
+l2

x =

3
+ (k +l); y =

3
+ (l k)
x =

3
+ (k +l); y =

3
+ (l k)
Xt (5) ta c:
(5)

x y =

2
+k2; x +y =

3
+l2
x y =

2
+k2; x +y =

3
+ l2

x =

12
+ (k + l); y =
5
12
+ (l k)
x =
5
12
+ (k + l); y =

12
+ (l k)
Theo trn ta suy ra mt hng sng tc mt lot cc phng trnh lng gic hai n kh
th v l : T mt bt ng thc c bn trong tam gic, trong bt ng thc cha cc gc
A, B, C ca mt tam gic, ta ln lt ly
A = f(x, y), B = g(x, y), C = [f(x, y) + g(x, y)],
v thay du bt ng thc bi du ng thc, ta s c mt phng trnh lng gic hai n
x v y tng ng. Cn vic gii phng trnh th thng c tin hnh tng t nh hai cch
gii trnh by trn.
Ch rng cch lm nh trn khng phi lc no cng thnh cng, c th ta xy dng c
nhng phng trnh lng gic hai n nhng ta li khng gii c nhng phng trnh m ta
va xy dng nn. Tuy nhin y l mt phng php sng tc bi ton rt ng quan tm,
n cho ta nhiu phng trnh th v c bi ln li gii.
V d 43. Xt bt ng thc
cos A + cos B + cos C
3
2
, ABC,
du bng xy ra khi v ch khi A = B = C =

3
. Ly A = x, B = y, C = (x + y), ta c
bi ton sau
Bi 56. Gii phng trnh
cos x + cos y cos(x +y) =
3
2
.
Gii
Phng trnh cho tng ng
2 cos
x +y
2
cos
x y
2

2 cos
2
x +y
2
1

=
3
2
2 cos
2
x + y
2
2 cos
x +y
2
cos
x y
2
+
1
2
= 0
cos
2
x + y
2
cos
x + y
2
cos
x y
2
+
1
4
= 0

cos
x + y
2

1
2
cos
x y
2

2
+
1
4
sin
2
x y
2
= 0
340

cos
x +y
2
=
1
2
cos
x y
2
sin
x y
2
= 0

cos
x +y
2
=
1
2
cos
x y
2
cos
x y
2
= 1

cos
x y
2
= 1
cos
x + y
2
=
1
2

cos
x y
2
= 1
cos
x + y
2
=
1
2
.
V d 44. Cng xt bt ng thc
cos A + cos B + cos C
3
2
, ABC,
du bng xy ra khi v ch khi A = B = C =

3
. Ly
A = x
y
2
, B =
3y
2
, C =

x
y
2

+
3y
2

= (x +y),
ta c bi ton sau
Bi 57. Gii phng trnh
cos

x
y
2

+ cos
3y
2
cos(x +y) =
3
2
.
Gii
Phng trnh cho tng ng
2 cos
x +y
2
cos
x 2y
2

2 cos
2
x + y
2
1

=
3
2
2 cos
2
x +y
2
2 cos
x + y
2
cos
x 2y
2
+
1
2
= 0
cos
2
x +y
2
cos
x +y
2
cos
x 2y
2
+
1
4
= 0

cos
x +y
2

1
2
cos
x 2y
2

2
+
1
4
sin
2
x 2y
2
= 0

cos
x +y
2
=
1
2
cos
x 2y
2
sin
x 2y
2
= 0

cos
x +y
2
=
1
2
cos
x 2y
2
cos
x 2y
2
= 1

cos
x 2y
2
= 1
cos
x +y
2
=
1
2
hoc

cos
x 2y
2
= 1
cos
x +y
2
=
1
2
.
Cng vic tip theo tr nn n gin.
V d 45. Xt bt ng thc
sin
A
2
+ sin
B
2
+ sin
C
2

3
2
, ABC,
du bng xy ra khi v ch khi A = B = C =

3
. Ly A = 2x, B = 2y, C = (2x + 2y), ta
c bi ton sau
341
Bi 58. Gii phng trnh
sin x + sin y + cos(x +y) =
3
2
.
Gii
Phng trnh cho tng ng
2 sin
x +y
2
cos
x y
2
+ 1 2 sin
2
x +y
2
=
3
2
2 sin
2
x + y
2
2 sin
x + y
2
cos
x y
2
+
1
2
= 0
sin
2
x + y
2
sin
x +y
2
cos
x y
2
+
1
4
= 0

sin
x + y
2

1
2
cos
x y
2

2
+
1
4
sin
2
x y
2
= 0

sin
x + y
2
=
1
2
cos
x y
2
sin
x y
2
= 0

sin
x +y
2
=
1
2
cos
x y
2
cos
x y
2
= 1

cos
x y
2
= 1
sin
x + y
2
=
1
2
hoc

cos
x y
2
= 1
sin
x +y
2
=
1
2
.
V d 46. Xt bt ng thc
cos
2
A
2
+ cos
2
B
2
+ cos
2
C
2

9
4
, ABC,
du bng xy ra khi v ch khi A = B = C =

3
. Ly A = 2x, B = 2y, C = (2x + 2y), ta
c bi ton sau
Bi 59. Gii phng trnh
cos
2
x + cos
2
y + sin
2
(x +y) =
9
4
.
Gii
Phng trnh cho tng ng
1 +
cos 2x + cos 2y
2
+ sin
2
(x + y) =
9
4
1 + cos(x +y) cos(x y) + 1 cos
2
(x + y) =
9
4
cos
2
(x +y) cos(x y) cos(x +y) +
1
4
= 0

cos(x + y)
1
2
cos(x y)

2
+
1
4
sin
2
(x y) = 0

cos(x +y)
1
2
cos(x y) = 0
sin(x y) = 0
342

cos(x + y) =
1
2
cos(x y) = 0
cos(x y) = 1

cos(x y) = 1
cos(x + y) =
1
2

cos(x y) = 1
cos(x + y) =
1
2
.
V d 47. Quan st cc bin i trn ta thy c th xy dng nn mt s phng trnh c li
gii tng t. Chng hn bi ton sau
Bi 60. Gii phng trnh
cos
2
x + cos
2
y + cos
2
(x y) =
3
4
Gii
Phng trnh cho tng ng
1 +
cos 2x + cos 2y
2
+ cos
2
(x y) =
3
4
1 + cos(x +y) cos(x y) + cos
2
(x y) =
3
4
cos
2
(x +y) + cos(x y) cos(x +y) +
1
4
= 0

cos(x +y) +
1
2
cos(x y)

2
+
1
4
sin
2
(x y) = 0

cos(x +y) +
1
2
cos(x y) = 0
sin(x y) = 0

cos(x +y) =
1
2
cos(x y)
cos(x y) = 1

cos(x y) = 1
cos(x +y) =
1
2

cos(x y) = 1
cos(x +y) =
1
2
.
V d 48. Xt bt ng thc
sin
A
2
sin
B
2
sin
C
2

1
8
, ABC,
du bng xy ra khi v ch khi A = B = C =

3
. Ly
A = 2x y, B = 3y, C = [(2x y) + 3y] = 2(x +y),
ta c bi ton sau
Bi 61.
sin
y 2x
2
sin
3y
2
cos(x +y) =
1
8
.
Gii
343
Phng trnh tng ng
4 [cos(x +y) cos(2y x)] cos(x +y) = 1
4 cos
2
(x +y) 4 cos(x + y) cos(2y x) + 1 = 0
[2 cos(x +y) cos(2y x)]
2
+ sin
2
(2y x) = 0

2 cos(x +y) = cos(2y x)


sin(2y x) = 0

2 cos(x +y) = cos(2y x)


cos(2y x) = 1

cos(2y x) = 1
cos(x +y) =
1
2
hoc

cos(2y x) = 1
cos(x + y) =
1
2
.
V d 49. Xt bt ng thc
cos Acos Bcos C
1
8
, ABC,
du bng xy ra khi v ch khi A = B = C =

3
. Ly
A = x, B = y, C = (x + y),
ta c bi ton sau
Bi 62. Gii phng trnh
cos x. cos y. cos(x +y) =
1
8
.
Gii
Phng trnh cho tng ng
4 cos(x + y) [cos(x +y) + cos(x y)] = 1
4 cos
2
(x +y) + 4 cos(x +y) cos(x y) + 1 = 0
[2 cos(x +y) + cos(x y)]
2
+ sin
2
(x y) = 0

2 cos(x +y) = cos(x y)


sin(x y) = 0

2 cos(x +y) = cos(x y)


cos(x y) = 1

cos(x y) = 1
cos(x +y) =
1
2
hoc

cos(x y) = 1
cos(x +y) =
1
2
.
V d 50. Xt bt ng thc
cos A + cos B + cos C
3
2
, ABC,
du bng xy ra khi v ch khi A = B = C =

3
. Ly
A = x, B = y, C = (x + y),
ta c phng trnh
cos x + cos y cos(x +y) =
3
2
. (1)
344
Ly
A = 2x y, B = 2y x, C = [(2x y) + (2y x)] = (x + y)
ta c phng trnh
cos(2x y) + cos(2y x) cos(x +y) =
3
2
. (2)
Cng (1) v (2) ta c bi ton sau
Bi 63. Gii phng trnh
cos x + cos y + cos(2x y) + cos(2y x) = 3 + 2 cos(x +y).
Gii
Phng trnh cho tng ng
[cos x + cos(2y x)] + [cos y + cos(2x y)] = 3 + 2 cos(x +y)
2 cos y cos(x y) + 2 cos x cos(x y) = 3 + 2 cos(x +y)
2 cos(x y) (cos x + cos y) = 3 + 2 cos(x + y)
4 cos(x y) cos
x +y
2
cos
x y
2
= 3 + 4 cos
2
x +y
2
2
4 cos
2
x + y
2
4 cos(x y) cos
x +y
2
cos
x y
2
+ 1 = 0

2 cos
x + y
2
cos(x y) cos
x y
2

2
+ 1 cos
2
(x y) cos
2
x y
2
= 0. (1)
V 1 cos
2
(x y) cos
2
x y
2
0 nn
(1)

2 cos
x +y
2
cos(x y) cos
x y
2
= 0
1 cos
2
(x y) cos
2
x y
2
= 0

2 cos
x +y
2
cos(x y) cos
x y
2
= 0
1 cos
2
(x y)
1 + cos(x y)
2
= 0

2 cos
x +y
2
cos(x y) cos
x y
2
= 0
cos
2
(x y) + cos
3
(x y) = 2

cos(x y) = 1
2 cos
x +y
2
= cos
x y
2
(2)
V cos(x y) = 1 2 cos
2
x y
2
1 = 1 cos
2
x y
2
= 1 nn
(2)

cos(x y) = 1
2 cos
x +y
2
= 1

cos(x y) = 1
cos
x +y
2
=
1
2

cos(x y) = 1
cos
2
x +y
2
=
1
4

cos(x y) = 1
1 + cos(x + y) =
1
2

cos(x y) = 1
cos(x +y) =
1
2
.
345
S dng hm ngc sng tc mt s phng trnh, h phng
trnh.
y l mt phng php sng tc v gii phng trnh, h phng trnh kh hay. Gi s
hm s y = f(x) c hm s ngc l y = g(x). Nu v th ca hai hm s trn cng mt h
trc to -cc vung gc th hai th y i xng nhau qua ng phn gic th nht.
Do nu hm s y = f(x) c hm s ngc l y = g(x) th vic gii phng trnh f(x) = g(x)
quy v gii phng trnh f(x) = x (hoc g(x) = x). Tm li, trong mt s trng hp ta s
dng mnh sau.
Mnh . Cho y = f(x) l hm ng bin, c hm ngc y = f
1
(x) v tp xc nh ca
phng trnh f(x) = f
1
(x) bng D
f
D
f
1. Ta c php bin i sau
f(x) = f
1
(x)

f(x) = x
x D
f
D
f
1
(1)
hoc
f(x) = f
1
(x)

f
1
(x) = x
x D
f
D
f
1.
(2)
Nh vy vic gii phng trnh f(x) = f
1
(x) ta thay th c bi cc h phng trnh tng
ng (1) hoc (2).
Chng minh.
Gi s f(x) = f
1
(x). Khi x = f (f(x)). Nu x > f(x) th do f ng bin nn f(x) >
f (f(x)) f(x) > x (v l). Nu x < f(x), tng t ta cng suy ra v l. Vy x = f(x).
Gi s x = f(x) v x D
f
D
f
1. Khi

x = f
1
(x)
x = f(x)
f(x) = f
1
(x).
Vy (1) c chng minh.
Ch 1. Cho hm s y = f(x) c hm ngc y = f
1
(x). Khi f ng bin f
1
ng
bin.
Ch 2. Nu hm s f lin tc v n iu nghim ngt trn khong (a; b) th tn ti hm
s ngc f
1
.
V d 51. Xt hm s y = f(x) = 2x
2
+ 4x 1 trn (1; +). Hm s ngc ca f(x) l
y =

x + 3
2
1 xc nh trn (3; +). Khi
2x
2
+ 4x 1 =

x + 3
2
1 2x
2
+ 4x =

x + 3
2
.
Ta c bi ton sau.
Bi 64 ( ngh OLYMPIC 30/04/2007). Gii phng trnh
2x
2
+ 4x =

x + 3
2
, vi x 1. (*)
Gii
346
iu kin x 1 (gi thit).
Cch 1. Xt hm s y =

x + 3
2
1, x [1; +) . Khi
y + 1 =

x + 3
2

y 1
y
2
+ 2y + 1 =
x + 3
2

y 1
x = 2y
2
+ 4y 1.
Suy ra hm s y = 2x
2
+ 4x 1 l hm ngc ca hm y =

x + 3
2
1 trn [1; +). Mt
khc hm s y = 2x
2
+ 4x 1 ng bin trn [1; +) nn
2x
2
+ 4x 1 =

x + 3
2
1 2x
2
+ 4x 1 = x

x =
3 +

17
4
x =
3

17
4
.
Kt hp vi iu kin suy ra x =
3 +

17
4
l nghim duy nht ca (*). 2
Nhn xt: V sao li tm hm ngc ca hm y =

x + 3
2
1, x [1; +)? l do ta
gi s
ay +b =

x + 3
2
a
2
y
2
+ 2aby + b
2
=
x + 3
2
x = 2a
2
y
2
+ 4aby + 2b
2
3.
So snh vi phng trnh cho thy rng cn chn a, b sao cho

2a
2
= 2
4ab = 4

a = 1
b = 1

a = 1
b = 1.
Vy nn ta mi xt hm s y =

x + 3
2
1, x [1; +). Ngoi ra ta cn c th gii thch
nh sau: T phng trnh cho thy rng cn xt hm s c tp gi tr l [1; +) v c
dng y = a

x + 3
2
1

vi a > 0

. Khi
y
2
+ 2y + 1 = a
2
.
x + 3
2
x =
2
a
2
y
2
+
4
a
2
y +
2
a
2
3

2
a
2
;
4
a
2

= (2; 4) a = 1 y =

x + 3
2
1.
Cch 2. t y + 1 =

x + 3
2
, iu kin y 0. Ta c h

(y + 1)
2
=
x + 3
2
2x
2
+ 4x = y + 1

2y
2
+ 4y = x + 1 (1)
2x
2
+ 4x = y + 1. (2)
Ly (1) tr (2) theo v ta c
2(y
2
x
2
) + 4(y x) = x y 2(y x)(y +x) + 5(y x) = 0

y x = 0
2(y +x) + 5 = 0

y = x
y =
5 2x
2
.
347
Khi y = x, thay vo (2) ta c
2x
2
+ 4x = x + 1 2x
2
+ 3x 1 = 0

x =
3 +

17
4
x =
3

17
4
(loi).
Khi y =
5 + 2x
2
, thay vo (2) ta c
4x
2
+ 10x + 3 = 0

x =
5

13
4
(loi)
x =
5 +

13
4
.
Vi x =
5 +

13
4
ta c
y =
5 + 2x
2
=
5 +

13
4
< 0 (khng tho mn y 0)
Kt hp vi iu kin suy ra x =
3 +

17
4
l nghim duy nht ca (*). 2
V d 52. Xt hm s y = f(x) = x
2
3x trn

3
2
; +

. Hm s ngc ca f(x) l
y = f
1
(x) =
3 +

9 + 4x
2
xc nh trn

9
4
; +

. Khi
3 +

9 + 4x
2
= x
2
3x

9 + 4x = 2x
2
6x 3.
Ta c bi ton sau.
Bi 65. Gii phng trnh

9 + 4x = 2x
2
6x 3. ()
Gii
iu kin x
9
4
. t 2y 3 =

9 + 4x, iu kin y
3
2
. Ta c

(2y 3)
2
= 9 + 4x
2y 3 = 2x
2
6x 3

4y
2
12y = 4x
2x
2
6x = 2y

2y
2
6y = 2x (1)
2x
2
6x = 2y (2)
Ly (1) tr (2) theo v ta c
2(y
2
x
2
) 6y + 6x = 2x 2y 2(y
2
x
2
) + 4(x y) = 0
(y x)(y +x 2)

y = x
y = 2 x.
Khi y = x, thay vo (1) ta c
2x
2
6x = 2x x
2
4x = 0

x = 0
x = 4.
348
Khi x = 0 th y = 0 (loi). Khi x = 4 th y = 4.
Khi y = 2 x, thay vo (2) ta c
x
2
3x = 2 x x
2
2x 2 = 0

x = 1

3
x = 1 +

3.
Khi x = 1 +

3 th y = 1

3, khng tho mn iu kin.


Khi x = 1

3 th th vo phng trnh cho thy tho mn. Vy phng trnh cho c


hai nghim x = 4, x = 1

3. 2
Nhn xt: Phng trnh () c tp xc nh l

9
4
; +

, trong khi ta c
D
f
D
f
1 =

3
2
; +

, v vy (*) nhn x = 1

3 lm nghim l iu d hiu, v gii phng


trnh (*) l tm x thuc tp xc nh

9
4
; +

v tho mn (*).
V d 53. Hm s y =
3

x + 3 l hm s ngc ca hm s y = x
3
3 trn R. Do ta c
bi ton sau.
Bi 66. Gii phng trnh
3

x + 3 = x
3
3
Gii
Tp xc nh R.
t y =
3

x + 3. Khi ta c h

y
3
= x + 3 (1)
x
3
= y + 3 (2)
Ly (2) tr (1) theo v ta c x
3
y
3
= y x x
3
+x = y
3
+ y. (3)
Xt f(t) = t
3
+ t. V f

(t) > 0, t R nn (3) f(x) = f(y) x = y. Do


x
3
= x + 3 x
3
x = 3 (4)
Gi s x =
2

3
t, thay vo (4) ta c
4t
3
3t =
9

3
2
. (5)
cho gn, k hiu m =
9

3
2
> 1. Xt
m =
1
2

3
+
1

2
2m
3
+ 1 = 0
3
= m

m
2
1.
Do , nu t =
3

m+

m
2
1 =
3

3
2
+

239
4
=
3

3 +

239
2
th
m =
1
2

3
+
1

,
1

=
3

239
2
.
rng
1
2

3
+
1

= 4

1
2

+
1

3
3

1
2

+
1

.
349
Vy
t
0
=
1
2

+
1

=
1
2

3 +

239
2
+
3

239
2

l mt nghim ca phng trnh (5). Ta s chng minh t


0
l nghim duy nht ca phng
trnh. Ta c [t
0
[ > 1 v
4t
3
3t = 4t
3
0
3t
0
(t t
0
)(4t
2
+ 4t
0
t + 4t
2
0
3) = 0.
rng phng trnh 4t
2
+ 4t
0
x + 4t
2
0
3 = 0 v nghim do

= 4t
2
0
4(4t
2
0
3) = 12 12t
2
0
= 12(1 t
2
0
) < 0 (v t
2
0
> 1).
Vy t =
1
2

m+

m
2
1 +
3

m
2
1

l nghim duy nht ca phng trnh (5). Do


phng trnh cho c nghim duy nht l
x =
1

3 +

235
2
+
3

235
2

. 2
Bi 67. Cho trc s thc a. Gii phng trnh (n l x)
a
2011
+ x =
2011

a x.
Gii
Tp xc nh R. Xem a nh n s. Xt hm y = f(a) = a
2011
+x. Ta c
y = a
2011
+ x a
2011
= y x a =
2011

y x.
Vy hm s y =
2011

a x l hm ngc ca hm y = a
2011
+ x, a R. Mt khc v
(a
2011
+x)

= 2011a
2010
0, a R nn y = a
2011
+x l hm ng bin trn R, do
a
2011
+x =
2011

a x

a
2011
+ x = a
a R
x = a a
2011
.
Vy phng trnh cho c nghim duy nht x = a a
2011
.
sng tc h phng trnh
gii h phng trnh th phng php th v phng php cng l thng dng nht, l
phng php c bn nht. T bi hc v lng v h phng trnh c phng php ny.
Tuy nhin phng php ny vn thng xut hin nhng k thi ln, nhng k thi ch dnh
cho nhng hc sinh xut sc. c nhiu ti liu v h phng trnh cp n phng php
ny, do vy sau y ta ch trnh by mt s bi ton kh v i su hn vo vic phn tch k
thut gii cng nh k thut sng tc.
Bi 68. Gii h phng trnh

x
2
(y + z)
2
= (3x
2
+x + 1) y
2
z
2
y
2
(z +x)
2
= (4y
2
+y + 1) z
2
x
2
z
2
(x + y)
2
= (5z
2
+y + 1) x
2
y
2
.
350
Gii
D thy rng (x; 0; 0) , (0; y; 0) , (0; 0; z) tho mn h cho. Tip theo xt xyz = 0. Khi
chia cc phng trnh ca h cho x
2
y
2
z
2
ta c

1
z
+
1
y

2
= 3 +
1
x
+
1
x
2

1
x
+
1
z

2
= 4 +
1
y
+
1
y
2

1
y
+
1
x

2
= 5 +
1
z
+
1
z
2
.

1
z
2
+
2
zy
+
1
y
2
= 3 +
1
x
+
1
x
2
1
x
2
+
2
zx
+
1
z
2
= 4 +
1
y
+
1
y
2
1
y
2
+
2
xy
+
1
x
2
= 5 +
1
z
+
1
z
2
.
Cng li ta c

1
x
+
1
y
+
1
z

1
x
+
1
y
+
1
z

12 = 0

1
x
+
1
y
+
1
z
= 3
1
x
+
1
y
+
1
z
= 4.
Vi
1
x
+
1
y
+
1
z
= 3, ta c

3 +
1
x

2
= 3 +
1
x
+
1
x
2

3 +
1
y

2
= 4 +
1
y
+
1
y
2

3 +
1
z

2
= 5 +
1
z
+
1
z
2

5
x
= 6
5
y
= 5
5
z
= 4

x =
5
6
y =
5
5
z =
5
4
.
Vi
1
x
+
1
y
+
1
z
= 4, ta c

4
1
x

2
= 3 +
1
x
+
1
x
2

4
1
y

2
= 4 +
1
y
+
1
y
2

4
1
z

2
= 5 +
1
z
+
1
z
2

9
x
= 13
9
y
= 12
9
z
= 11

x =
9
13
y =
9
12
z =
9
11
.
Cc nghim ca h l
(x; y; z) =

5
6
; 1;
5
4

, (x; y; z) =

9
13
;
3
4
;
9
11

,
(a; 0; 0) , (0; b; 0) , (0; 0; c)

vi a, b, c R, tu

. 2
Bi 69. Gii h phng trnh

x
2
(y z) =
5
3
(1)
y
2
(z x) = 3 (2)
z
2
(x y) =
1
3
. (3)
Gii
351
Ta c hng ng thc sau :

x
2
y
2
x
2
z
2

z
2
y
2
x
2
y
2

x
2
z
2
y
2
z
2

= 0.
T nhn (1) vi y +z, nhn (2) vi z + x, nhn (3) vi x +y ta c

5
3
(y +z) + 3 (z + x) +
1
3
(x +y) = 0
5y 5z + 9z + 9x + x + y = 0
4y = 10x 4z y =
5x + 2z
2
y z =
5x
2
. (4)
T (4), (1) c
5x
2
=
5
3x
2
, suy ra x, th vo (2), (3) c h hai n. Cng vic tip theo tr
nn n gin.
Bi 70. Gii h phng trnh

xz +y = 7z (1)
yz +x = 8z (2)
x +y +z = 12. (3)
Gii
T (1) c y = 7z zx. Thay vo (2) v (3) c

(7z zx) z +x = 8z
x + (7z zx) + z = 12

x (1 z
2
) = 8z 7z
2
(4)
x (1 z) = 12 8z. (5)
R rng z = 1 khng tho (5), do (5) x =
12 8z
1 z
. Thay vo (4) c
(12 8z) (1 +z) = 8z 7z
2
z
2
+ 4z 12 = 0

z = 2
z = 6.
Vi z = 2 ta c x = 4, y = 6. Vi z = 6 ta c x =
60
7
, y =
66
7
.
Kt lun: h c hai nghim (4; 6; 2) ,

60
7
;
66
7
; 6

. 2
Bi 71 (HSG tnh Qung Bnh, nm hc 2010-2011). Gii h

x y 1 = 1 (1)
y
2
+ x + 2y

x y
2
x = 0. (2)
Gii
iu kin x 0 v x y 1 0. Phng trnh (1) tng ng

x =

x y 1 + 1 x = x y + 2

x y 1 y = 2

x y 1
y
2
= 4 (x y 1) (y + 2)
2
= 4x y + 2 = 2

x.
Phng trnh th hai ca h tng ng vi
y
2
+ x + 2y

x y
2
x = 0

y +

2
= xy
2
y +

x = y

x.
352
Ta thu c

y + 2 = 2

x
y +

x = y

y + 2 = 2

x
2y + (y + 2) = (y + 2) y.
T tm c nghim ca h l (x; y) =

1
4
; 1

, (4; 2),

17
8
;

17 5
2

. 2
Bi 72. Gii h phng trnh sau :

4x
2
+y
4
4xy
3
= 1 (1)
4x
2
+ 2y
2
4xy = 2. (2)
Gii
Ly (1) tr (2), v theo v, ta c :
y
4
2y
2
4xy
3
+ 4xy + 1 = 0

y
2
1

2
4xy

y
2
1

= 0

y
2
1

y
2
1 4xy

= 0 y = 1 hoc y = 1 hoc y
2
1 + 4xy = 0.
Nu y = 1, thay vo phng trnh (1) ta c :
4x
2
+ 1 4x = 1 x (x 1) = 0 x = 0 hoc x = 1.
Nu y = 1, thay vo phng trnh (1) ta c :
4x
2
+ 1 + 4x = 1 x (x + 1) = 0 x = 0 hoc x = 1.
Nu y
2
1 + 4xy = 0 x =
1 y
2
4y
(d thy trong trng hp ny y = 0), thay vo phng
trnh u tin ta c:
4

1 y
2
4y

2
+y
4
4

1 y
2
4y

y
3
= 1

1 y
2

2
+ 4y
6
4y
4

1 y
2

= 4y
2

1 y
2

2
4y
4

1 y
2

= 4y
2

1 y
4

1 y
2

1 y
2

4y
4
4y
2

1 + y
2

= 0

1 y
2

8y
4
+ 5y
2
1

= 0
Cng vic n y tr nn n gin.
Nhn xt: y l mt dng h phng trnh a thc kh kh, r rng nu phng trnh th
hai, ngi ta chia hai v cho 2 th kh c th nhn bit gi tr ny m nhn vo ri tr tng v
nh trn. Vic pht hin ra gi tr 2 nhn vo c th dng cch t tham s ph ri la chn.
Bi 73. Gii h phng trnh

y
3
x x
4
= 28
xy
2
+ 2x
2
y +x
3
= 18

2
Gii
Ta bin i tng ng

x(y
3
x
3
) = 28 (1)
x(y +x)
2
= 18

2. (2)
353
Suy ra (x; y) l nghim th y > x > 0. T (2), rt y theo x v th vo (1) c
x

3
4

x
x

3
x
3

= 28. (3)
t t =

x (t > 0) v (3) thnh :
t
9
(3
4

8 t
3
)
3
+ 28t = 0. (4)
Xt hm f(t) = t
9
(3
4

8 t
3
)
3
+ 28t, vi t > 0.
Ta c f

(t) > 0 vi mi t > 0.


Chng t hm f ng bin trn (0; +), nn (4) c nghim th nghim duy nht.
T h c nghim (x
0
; y
0
) th nghim l duy nht. D thy y = 2x th t (1), ta c: x
4
= 4
hay x =

2. Suy ra y = 2

2.
Th li (

2; 2

2) tha mn (2).
Tm li h cho c nghim duy nht l (x; y) = (

2; 2

2). 2
KINH NGHIM GII MT S BI H PHNG TRNH
Khi gii h phng trnh, d c dng cch g bin i i chng na th mc ch cui cng ca
ta cng chuyn v phng trnh mt bin v gii phng trnh va thu c. cng l suy
ngh t nhin, vic lm gim bin l quy lut khng ch trong ton hc m c trong cuc sng
chng ta vn thng lm. Tm li, khi gii h phng trnh th chng ta phi tm cch lm
gim s n ca h thun li trong vic gii n. Sau y l mt s kinh nghim m nhm
bin son thu c trong qu trnh hc tp v ging dy, xin c chia s cng bn c.
Nu trong phng trnh ca h m c mt n xut hin di dng bc nht, th ta c th rt
n theo n cn li v th vo phng trnh th hai ca h v phng trnh thu c c bc
khng nh nhng tng gii l rt r rng.
Bi 1: Gii h phng trnh

2x
3
+ y(x + 1) = 4x
2
5x
4
4x
6
= y
2
Gii
Cch 1:
V phng trnh th nht ca h cha nhiu y nn ta ngh n vic rt y theo x v th vo
phng trnh th hai ca h.
Ta c y =
2x
2
(2 x)
x + 1
(do x = 1 khng l nghim ca h).
354
Thay vo phng trnh th hai ca h ta c :
x
4

5 4x
2

=
4x
4
(2 x)
2
(x + 1)
2

x = 0
(5 4x
2
)(x
2
+ 2x + 1) = 4(4 4x +x
2
)

x = 0
4x
4
+ 8x
3
+ 3x
2
26x + 11 = 0

x = 0
(x 1)(2x 1)(2x
2
+ 7x + 11) = 0

x = 0 y = 0
x = 1 y = 1
x =
1
2
y =
1
2
Vy h cho c nghim: (x; y) = (0; 0), (1; 1),

1
2
;
1
2

2
Nhn xt: Cch gii ny c mt u im l khng cn phi mnh khe g c m ch cn bin
i ht sc bnh thng. Tuy nhin, n c mt nhc im l n ch gip chng ta gii quyt
bi ton thi, cn con ng sng tc ra bi ton th cch gii trn khng th lm r
c. hiu r c ngun gc ca bi ton v l cch m tc gi sng tc bi ton
trn, ta cng xem qua cch gii sau:
Cch 2:
Ta vit li h nh sau:

2x
3
+y(x + 1) = 4x
2
y
2
+ 4x
6
= 5x
4
Nhn thy x = 0 y = 0, hay (x; y) = (0; 0) l mt nghim ca h.
Vi x = 0 ta c h

2x +
y
x
2
(x + 1) = 4

y
x
2

2
+ 4x
2
= 5
t a = 2x, b =
y
x
2
ta c c h:

a +b

a
2
+ 1

= 4
a
2
+b
2
= 5
y l h i xng loi I. Vic gii h ny khng my kh khn.
Nhn xt: Qua li gii trn, ta thy con ng ch tc ra nhng h kiu ny l xut pht
t mt h bit thut gii, chng ta thay th hnh thc ca cc bin c mt trong h v bin
i rt gn ta thu c mt h c hnh thc hon ton xa l vi ci h ban u.
Chng hn: T h

x +y +xy = 5
x
2
+y
2
= 5
(lu h ny c nghim (1; 2))
Ta thay th x bng
y
2x
3
v y bng y
2
th ta c h:

y
2x
3
+ y
2
+
y
3
2x
3
= 5
y
2
4x
6
+ y
4
= 5

y(y
2
+ 2x
3
y + 1) = 10x
3
y
2
(1 + 4x
6
y
2
) = 20x
6
355
Vy ta c h phng trnh sau:

y(y
2
+ 2x
3
y + 1) = 10x
3
y
2
(1 + 4x
6
y
2
) = 20x
6
Bi 2: Gii h phng trnh

x
2
2xy +x +y = 0 (1)
x
4
4x
2
y + 3x
2
+y
2
= 0 (2)
Gii
Nhn thy phng trnh th nht ca h l phng trnh bc nht i vi x nn ta rt x theo
y v th vo phng trnh th hai c phng trnh mt n.
T (1) suy ra y =
x
2
+x
2x 1
( do x =
1
2
khng l nghim ca h), thay vo (2) ta c
x
4
4x
2
x
2
+x
2x 1
+ 3x
2
+

x
2
+ x
2x 1

2
= 0

x = 0
f(x) = 0
Vi f(x) = x
2
(2x 1)
2
4(x
2
+x)(2x1) +3(2x 1)
2
+(x + 1)
2
= 4x
4
12x
3
+10x
2
6x+4
Ta c f(x) = 0 2x
4
6x
3
+ 5x
2
3x + 2 = 0 (x 1)(x 2)(2x
2
+ 1) = 0 x = 1, x = 2
Vy h cho c 3 nghim (x; y) = (0; 0), (1; 2), (2; 2) 2
Nhn xt: Cng nh v d 1, cch gii trn ch gii quyt c bi ton ch khng phi l
con ng sng tc bi ton . iu ny thi thc chng ta i tm mt li gii khc cho
bi ton trn. S xut hin x
2
2xy v x
4
4x
2
y gi cho ta ngh n cc hng ng thc. Vy
ta th vit li h nh sau:

(x y)
2
+x +y y
2
= 0
(x
2
y)
2
+ 3x
2
3y
2
= 0
Vic lm ny cng khng my kh quan, v khi nhn vo h chng ta cng cha pht hin c
mi lin h no. Theo cch lm v d 1 ta bin i:
Nu x = 0 y = 0 l nghim ca h.
Nu x = 0, ta c h

x 2y + 1 +
y
x
= 0
x
2
4y +
y
2
x
2
+ 3 = 0

x +
y
x
= 2y + 1

x +
y
x

2
= 6y 3
Suy ra (2y + 1)
2
= 6y 3. n y th bi ton tr nn n gin.
Vi cch gii trn, ta c th to ra c rt nhiu h phng trnh khc nhau. y chng ta
ch rng vic gii h cui cng quy v gii cc phng trnh bc hai nn chuyn cc h s
nhn nhng gi tr no khng quan trng.
Chng hn t h

x +
2y
x
= 4x
2
+ 4

x +
2y
x

2
= x
2
3
qua khai trin ta c h

2y = 4x
3
x
2
+ 4x
4x
2
y + 4y
2
= 3x
2
hai bi trn chng ta gii theo phng php th. Du hiu nhn thy l vic xut hin ca
mt phng trnh l phng trnh bc nht i vi mt n. By gi chng ta chuyn qua xt
mt s h m chng ta thc hin rt th m phng trnh i vi mt n trong mt phng
356
trnh no khng phi l phng trnh bc nht.
Bi 3: Gii h phng trnh

x
3
8x = y
3
+ 2y (1)
x
2
3 = 3(y
2
+ 1) (2)
Gii
Cch 1:
T (2) ta suy ra: x
2
= 3(y
2
+ 2) (3) , thay vo (1) ta c:
x
3
8x = y(y
2
+ 2) =
yx
2
3
x(3x
2
xy 24) = 0

x = 0
y =
3x
2
24
x
Nu x = 0 thay vo (3) ta thy phng trnh v nghim.
Nu y =
3x
2
24
x
thay vo (3) ta c
x
2
= 3

3x
2
24
x

2
+ 6 13x
4
213x
2
+ 864 = 0

x
2
= 9
x
2
=
96
13

x = 3 y = 1
x =

96
13
y =

78
13
Vy h c 4 cp nghim l: (x; y) = (3; 1),

96
14
;

78
13

2
Nhn xt: Chng ta ngh n php th do phng trnh th nht ch cha y
3
v y; phng
trnh th hai ca h li cha y
2
nn nu ta thay y
2
vo phng trnh th nht th phng trnh
th nht ca h tr thnh phng trnh bc nht i vi n y v ta thc hin rt y nh trn.
Tuy nhin, c l y cng khng phi l con ng ch tc bi ton trn. T nhn xt trn, ta
thy phng trnh th nht hai bin x, y lch bc nhau 2 bc ( x
3
v x; y
3
v y), ng thi
phng trnh th hai cng lch bc nhau 2 bc ( x
2
, y
2
v hng s).
iu ny gi ta to ra s ng bc nh sau:
Cch 2:
H cho tng ng

x
3
y
3
= 8x + 2y
6 = x
2
3y
2
6(x
3
y
3
) = (8x + 2y)(x
2
3y
2
)
y l phng trnh ng cp bc 3. Vic cn li gii quyt h khng cn kh khn na.
Bi 4: Gii h phng trnh

x
3
+ 3xy
2
= 49 (1)
x
2
8xy +y
2
= 8y 17x (2)
Gii
357
Bi ton ny c hai cch gii bng h s bt nh v n ph tng - hiu. Sau y ta cng
xem qua hai cch gii khc:
Cch 1:
Ta thy x = 0 khng phi l nghim ca h nn t (1) suy ra y
2
=
x
3
+ 49
3x
()
Th vo phng trnh (2) ta c
x
2
8xy
x
3
+ 49
3x
= 8y 17 24y(x
2
+ x) = 2x
3
+ 51x
2
49
24xy(x + 1) = (x + 1)(2x
2
+ 49x 49)

x = 1
y =
2x
2
+ 49x 49
24x
Nu x = 1 th vo (*) y = 4.
Nu y =
2x
2
+ 49x 49
24x
th vo (*), ta c:

x
3
+ 49
3x
=

2x
2
+ 49x 49
24x

2
192x(x
3
+ 49) = (2x
2
+ 49x 49)
2
Bin i rt gn ta c:
4x
4
+ 4x
3
+ 45x
2
+ 94x + 49 = 0 (x + 1)
2
(4x
2
4x + 49) = 0 x = 1
Vy h c nghim: (x; y) = (1; 4) 2
Cch 2:
V x = 0 khng l nghim ca h nn ta t y = tx.
Khi h tr thnh:

x
3
(1 + 3t
2
) = 49
x
2
(1 8t +t
2
) = x(8t 17)
Ta vit li h di dng

x
3
=
49
1 + 3t
2
=
49
49 + 3(t
2
16)
=
49
49 + 3a
x =
8t 17
t
2
8t + 1
=
8t 17
(t
2
16) (8t 17)
=
b
a b
vi a = t
2
16; b = 8t 17

49
49 + 3a
=
b
3
(a b)
3
49

b
3
+ (a b)
3

+ 3ab
3
= 0
a

49

b
2
b(a b) + (a b)
2

+ 3b
3

= 0

a = 0
49

b
2
b(a b) + (a b)
2

+ 3b
3
= 0 (*)
Vi a = 0 t
2
= 16, thay vo h ta c x = 1 y = 4.
Xt (*), khai trin v rt gn, ta c:
() 49t
4
+ 360t
3
+ 547t
2
360t + 304 = 0 (t + 4)
2
(49t
2
32t + 19) = 0 t = 4
358
Nhn xt: Hai cch gii trn i hi tnh ton nhiu, v cha th ni ln tng ca ngi
ra . i vi bi ny cch gii hay nht l dng h s bt nh ( c cp trong chng
H phng trnh). Xut pht t nghim x = 1, ta s to ra cc phng trnh cha nhn t
(x + 1), t khai trin c mt h mi. V d nh hai bi sau:
Bi 4*: Gii h phng trnh :

x
3
+ 2xy
2
= 5
2x
2
+xy +y
2
= 4x +y
Bi 4**: Gii h phng trnh :

x
3
+y
2
= (x y)(xy 1)
x
3
x
2
+ y + 1 = xy(x y + 1)
Bi 5: Gii h phng trnh :

1 + x
3
y
3
= 19x
3
y + xy
2
= 6x
2
Gii
V x = 0 khng l nghim ca h nn h tng ng

1
x
3
+y
3
= 19
y
1
x
2
+ y
2
1
x
= 6

a
3
+y
3
= 19
a
2
y +y
2
a = 6
(vi a =
1
x
)
t S = a +y, P = ay. Khi ta c

S(S
2
3P) = 19
SP = 6

S = 1
P = 6

a = 3
y = 2

a = 2
y = 3
Vy nghim ca h l: (x; y) = (
1
3
; 2), (
1
2
; 3) 2
Nhn xt: Ngoi cch gii trn, ta c th gii theo cch sau:
Do x = 0 khng l nghim ca h, ta vit li h di dng

6(1 + xy)(1 xy +x
2
y
2
) = 6.19x
3
19xy(1 + xy) = 19.6x
2
Cng hai phng trnh ca h li ta c: (1 + xy)(6x
2
y
2
+ 13xy + 25) = 0 n y, bi ton
tr nn n gin.
Nhn xt:Cng tng , ta c bi ton sau:
Bi 5*: Gii h phng trnh

y + xy
2
= 6x
2
1 + x
2
y
2
= 5x
2
Bi 6: Gii h phng trnh

(x
2
+ 1)y
4
+ 1 = 2xy
2
(y
3
1)
xy
2
(3xy
4
2) = xy
4
(x + 2y) + 1
Gii
359
H cho tng ng

x
2
y
4
+ 2xy
2
+ 1 + y
4
2xy
5
= 0
3x
2
y
6
2xy
2
x
2
y
4
2xy
5
1 = 0

x
2
+ 2
x
y
2
+
1
y
4
2xy = 1
3x
2
y
2

2x
y
2
x
2
2xy
1
y
4
= 0

x +
1
y
2

2
2xy = 1
3x
2
y
2
2xy

x +
1
y
2

2
= 0
( do y=0 khng l nghim ca h)
t a = x +
1
y
2
, b = xy, ta c h

a
2
2b = 1
a
2
3b
2
+ 2b = 0

a
2
= 2b 1
3b
2
4b + 1 = 0

b = 1
a = 1
Nu a = b = 1 :

x +
1
y
2
= 1
xy = 1

x =
1
y
y
2
y 1 = 0

x =

5 1
2
y =
1 +

5
2
v

x =
1 +

5
2
y =
1

5
2
Nu a = 1; b = 1 :

x +
1
y
2
= 1
xy = 1

x =
1
y
y
2
+y + 1 = 0
(v nghim)
Vy nghim ca h cho l: (x; y) =

5
2
;
1

5
2

2
Bi 7: Gii h phng trnh

(x
2
1)
2
+ 3 =
6x
5
y
x
2
+ 2
3y x =

4x 3x
2
y 9xy
2
x + 3y
Gii
T phng trnh th hai, ta c

3y x
x + 3y = 0
H cho tng ng vi:

(x
2
2x + 4)(x
2
+ 2) = 6x
5
y
(3y x)
2
=
4x 3x
2
y 9x
2
y
2
x + 3y

x
6
+ 8 = 6x
5
y
9y
2
6xy +x
2
=
4x
x + 3y
3xy

x
6
+ 8 = 6x
5
y
(x
2
3xy + 9y
2
)(x + 3y) = 4x

x
6
+ 8 = 6x
5
y
x
3
+ 27y
3
= 4x
V x = 0 khng l nghim ca h, nn ta c

1 +
8
x
6
=
6y
x
1 +
27y
3
x
3
=
4
x
2
360
t a =
2
x
2
> 0, b =
3y
x
, ta thu c h

1 + a
3
= 2b
1 + b
3
= 2a

1 + a
3
= 2b
(a b)(a
2
+ab +b
2
+ 2) = 0

a = b
a
3
2a + 1 = 0

a = b
(a 1)(a
2
+ a 1) = 0

a = b
a = 1, a =
1 +

5
2
Vi a = b = 1, ta c:

2
x
2
= 1
3y
x
= 1

x =

2
y =

2
3
v

x =

2
y =

2
3
Vi a = b =
1 +

5
2
ta c:

2
x
2
=

5 1
2
3y
x
=

5 1
2

x =

5 + 1
y =

5 + 1

5 1

6
v

x =

5 + 1
y =

5 + 1

5 1

6
Vy nghim ca h cho l (x; y) =

2;

2
3

5 + 1;
(

5 1)

5 + 1
6

2
Bi 8: Gii h phng trnh

6x
4

x
3
x

y
2
(y + 12) x
2
= 6
5x
4

x
2
1

2
y
2
11x
2
= 5
Gii
Nhn thy, x = 0 khng l nghim ca h nn chia hai v ca mi phng trnh cho x
2
ta c:

x
2
+
1
x
2

x
1
x

y
2
y 12 = 0
5

x
2
+
1
x
2

x
1
x

2
y
2
11 = 0

x
1
x

x
1
x

y
2
y = 0
5

x
1
x

x
1
x

2
y
2
1 = 0
t a = x
1
x
, ta c h

6a
2
ay
2
y = 0
5a
2
a
2
y
2
1 = 0
Chia hai v ca h cho a
2
= 0 ta c:

y
2
a
+
y
a
2
= 6
y
2
+
1
a
2
= 5
t u =
1
a
, ta c h

y
2
u +u
2
y = 6
u
2
+ y
2
= 5

uy(u +y) = 6
(u +y)
2
2uy = 5

u + y = 3
uy = 2
361
T ta tm c:

u = 1
y = 2
hoc

u = 2
y = 1
Vi u = 1 a = 1 x
1
x
= 1 x
2
x 1 = 0 x =
1

5
2
Vi u = 2 a =
1
2
x
1
x
=
1
2
2x
2
x 2 = 0 x =
1

17
4
Vy nghim ca h cho l (x; y) =

5
2
; 2

17
4
; 1

2
Bi tp t luyn
Gii cc h phng trnh sau:
1)

x +

y +

y = 2

y +

x = 1
2)

x
2
xy +y
2
= 3(x y)
x
2
+xy +y
2
= 7(x y)
2
3)

x +y +

x y = 8
y

x y = 2
4)

x
3
+ 3xy
2
= 6xy 3x 49
x
2
8xy +y
2
= 10y 25x 9
5)

x
4
+ 2x
3
y + x
2
y
2
= 2x + 9
x
2
+ 2xy = 6x + 6
6)

x
2
+ y +x
3
y +xy
2
+xy =
5
4
x
4
+y
2
+ xy(1 + 2x) =
5
4
7)

xy +x +y = x
2
2y
2
x

2y y

x 1 = 2x 2y
8)

x
4
x
3
y +x
2
y
2
= 1
x
3
y x
2
+xy = 1
9)

x +y 4
(x
4
+ y
4
)(x
7
+ y
7
) = x
11
+ y
11
10)

y +
2
3

x
2
12y + 1 =
1
12
(x
2
+ 17)
x
2
8y
+
2x
3
=

x
3
3y
+
x
2
4

y
2
11)

y
6
+y
3
+ 2x
2
=

xy x
2
y
2
4xy
3
+y
3
+
1
2
2x
2
+

1 + (2x y)
2
Pnu Luc i: OII TOAN DNO PHNO TRNH -
H PHNO TRNH
Bi 1 (IMO 1967): Trong 1 cuc thi u th thao, tng s huy chng l m, c pht
trong n ngy thi u. Trong ngy th nht, ngi ta pht 1 huy chng v
1
7
s huy chng
cn li. Ngy th hai, ngi ta pht 2 huy chng v
1
7
s huy chng cn li. Nhng ngy
cn li tng t. Ngy sau cng cn li n huy chng pht. Tnh m, n?
Gii
Gi s s huy chng cn li khi bt u ngy thi u th r l m
r
.
Khi m
1
= m, m
n
= n v k < n :
6(m
k
k)
7
= m
k+1
Ta bin i:
m = 1+2.
7
6
+3.

7
6

2
+...+n.

7
6

n1
= 36

1 (n + 1).

7
6

n
+n

7
6

n+1

= 36+
7
n
(n 6)
6
n1
Do (6; 7) = 1 n 6
.
.
. 6
n1

[n 6[ [6
n1
[
n 6 = 0
D thy ch c n = 6 tho. Vy m = 36. 2
Bi 2 (APMOPS 2004): Mt ng c pht trin theo mt tc khng i:
200 con cu n ht c trong 100 ngy.
150 con cu n ht c trong 150 ngy.
Hi 100 con cu n ht c trong bao nhiu ngy?
Gii
Gi s mi con cu n ht 1 ming c mi ngy, vy trong 100 ngy 200 con cu s n ht
20000 ming c, tng t, 150 con cu s n ht 22500 ming c trong 150 ngy.
t x (n v: ming c) l lng c ng c ban u, a l tc mc c (n vi: ming
c/ngy), t l thi gian 100 con cu n ht ng c (n vi: ngy), t gi thit, ta c h:

x + 100a = 20000 (1)


x + 150a = 22500 (2)
x +at = 100t (3)
T (1) v (2) ta suy ra x = 15000, a = 50, vy t y ta d dng suy ra t = 300 2.
362
363
Bi 3: Mt ngi cha khi cht li di chc phn chia gia ti cho cc con l cc ng tin
vng nh sau : con c uc 100 ng v
1
10
s cn li , ngi con th hai c 200 ng
v
1
10
s cn li, ngi con th ba c 300 ng v
1
10
s cn li,......C nh vy cho n
khi gia ti c chia ht th s tin mi con bng nhau. Hi ngi cha c bao nhiu ng
tin vng, c bao nhiu con v mi ngi c chia bao nhiu ng tin vng?
Gii
Gi s tin ca cha l x (ng) (x > 0)
Con c c 100 +
x 100
10
Con th hai c 200+
x 300
x 100
10
10
Cho 100 +
x 100
10
= 200 +
x 300
x 100
10
10
Gii phng trnh trn ta tm c x = 8100 nn c 9 ngi con v mi ngi con c 900
ng. 2
Bi 4: Mt nhm gm 21 ngi i du lch n cc nc Anh, Php,, trong mi
ngi i t nht 1 nc v khng ai i c 3 nc, bit rng:
S ngi i c 2 nc v Anh gp i s ngi i c c 2 nc Php v .
S ngi i c Php v li gp i s ngi i Anh v Php.
S ngi ch i (khng i Php v Anh) hn s ngi ch i Anh (khng i Php v )
l mt ngi v bng s ngi i Php.
Hi:
a/ Hy tm s ngi i ng 1 nc.
b/ Hy tm s ngi i t nht 1 trong 2 nc Anh v Php.
Gii
Gi A, P, Y l tp hp s ngi i n cc nc Anh, Php, . Gi x, y, z l s ngi ch i
Anh, Php, . Gi t l s ngi i c 2 nc Anh v Php, th s ngi i Php v l 2t,
cn s ngi i 2 nc v Anh l 4t.
Theo bi ta c h phng trnh:

x +y +z + 7t = 21 (1)
z = x + 1 (2)
z = y + 3t (3)
T (1) 7t < 21 t < 3
T (1); (2); (3) 3z + 4t = 22 (t 1)
.
.
. 3
M 0 t < 2 t = 1
Khi t = 1 th:

x +y +z = 14
z = x + 1
z = y + 3
364
Do (x; y; z) = (5; 3; 6)
Vy c 14 ngi ch i ng 1 nc, t nht 15 ngi i t nht 1 trong 2 nc Anh v Php. 2
Bi 5: Mt ngi i xe p in dc ng tu in v nhn thy rng: c 7 pht c 1
on tu chy cng chiu vi anh ta v c 5 pht c 1 on tu chy ngc li. Hi cch
nhau bao nhiu pht th ga u tin c 1 on tu khi hnh? Bit rng, cc on tu
khi hnh sau nhng khong thi gian nh nhau v chuyn ng vi vn tc khng i,
khng dng li bt k ni no trn ng v ngi i xe p in cng i vi vn tc khng
i.
Gii
t t l khong thi gian xut pht gia 2 tu in (tnh bng pht), x, y ln lt l vn tc
ca tu in v xe p (n v di/pht, x > y), ta c khong cch gia 2 tu in l xt.
Gi s khi gp mt tu in, th khi xe p s cch tu i ngay sau l xt, theo cng thc
khi 2 xe i ngc chiu, ta c
xt = 5(x + y) (1)
Tng t, ta cng c
xt = 7(x y) (2)
T (1) v (2) ta suy ra c x = 6y, th vo (1), ta li c 6yt = 7(6y y) t =
35
6
Vy 2 tu khi hnh cch nhau
35
6
pht 2.
Bi 6: Mt h nc c cung cp bi 3 vi nc. Bit rng nu tng vi cung cp nc
cho h th vi th nht lm y h nhanh hn vi th hai l 5 gi, vi th ba li lm y
h nhanh hn vi th nht l 4 gi. Cn nu vi th nht v th hai cng cung cp nc
cho h th thi gian chng lm y h bng thi gian ca vi th ba. Hi nu c 3 vi cng
cung cp nc cho h th chng s lm y h trong bao lu?
Gii
Gi thi gian vi th ba lm y h l t (gi), t > 0.
Thi gian vi th nht lm y h l t + 4 (gi).
Thi gian vi th hai lm y h l t + 9 (gi).
Trong 1 gi, vi th nht v th hai chy c
1
t + 4
+
1
t + 9
h nc v vi th ba chy c
1
t
h nc.
Theo bi,ta c phng trnh:
1
t + 4
+
1
t + 9
=
1
t
Tng ng:
t(t + 9) + t(t + 4) = (t + 4)(t + 9)
Ta tm c t = 6.
Vy trong 1 gi, c ba vi chy c:
1
t + 4
+
1
t + 9
+
1
t
=
1
3
365
Vy nu c ba vi cng cung cp nc th sau 3 gi h s y. 2
Bi 7: C 3 i xy dng cng lm chung mt cng vic. Lm chung c 4 ngy th i
III c iu ng lm vic khc, hai i cn li cng lm thm 12 ngy na th hon
thnh cng vic. Bit rng nng sut i I cao hn nng sut i II, nng sut i III l
trung bnh cng ca nng sut i I v II. Nu mi i lm mt mnh mt phn ba cng
vic th mt 37 ngy mi xong. Hi nu mi i lm mt mnh th bao nhiu ngy xong
cng vic trn?
Gii
Gi thi gian cc i I, II, III lm ring xong cng vic ln lt l x, y, z (n v: ngy).
Theo bi,ta c h phng trnh:

1
x
+
1
y
+
1
z

+ 12

1
x
+
1
y

= 1 (1)
1
x
>
1
y
(2)
1
z
=
1
2

1
x
+
1
y

(3)
x + y +z
3
= 37 (4)
T (1) v (3) ta c: 4

2
z
+
1
z

+ 12.
2
z
= 1 z = 36
T (3) ta suy ra
1
x
+
1
y
=
1
18
, kt hp vi (4) ta s c:

x + y = 75
xy = 1350
Do x, y l nghim ca phng trnh: x
2
75x + 1350 = 0 x
1
= 30 v x
2
= 45
Vy thi gian cn cc i I, II, III hon thnh cng vic ln lt l 30 ngy, 45 ngy, 36 ngy.2
Bi 8: Trong k thi hc sinh gii ca trng, nu sp mi phng thi 22 hc sinh th tha
1 em, nu gim 1 phng thi th s hc sinh c chia u cho mi phng. Tnh s hc sinh
tham gia k thi bit rng mt phng thi khng cha qu 40 hc sinh.
Gii
Gi s phng thi l x (x N, n v: phng).
Theo bi, nu sp mi phng thi 22 hc sinh th tha 1 em nn tng s hc sinh l 22x + 1
(em).
S phng sau khi gim l x 1 (phng).
Do lc ny s hc sinh chia u cho mi phng nn 22x + 1
.
.
. x 1
M 22x + 1 = 22(x 1) + 23 23
.
.
. x 1 x = 2 hoc x = 24
Ti x = 2, s hc sinh c l 45 em, khi y s hc sinh mi phng sau khi gim 1 phng l 45
em (tri vi gi thit).
Ti x = 24, s hc sinh c l 529 em, iu ny ph hp vi bi.
Vy c tt c 529 hc sinh. 2
Pnu Luc : PHNO TRNH VA CAC NHA TOAN
HC NI TINO
i. lch s pht trin ca phng trnh
C my cch gii phng trnh bc hai?
Lch s phng trnh bc hai bt ngun t nn vn minh Babilon c i (khong 1800 nm
trc cng nguyn). Lc h bit cch gii tt c cc phng trnh bc hai nhng khng
din t trong tp s thc.
Vo khong 1500 nm trc cng nguyn,trong mt tc phm ca ngi Ai Cp v cc bi
ton c th c nhng v d v gii phng trnh bc hai.
Trng phi Pythagores (th k VI trc cng nguyn) gii phng trnh bc hai bng hnh
hc v v sau ngi ta gi l phng php Pythagores.
th k III trc cng nguyn,ngi Hy Lp c i bin vic gii phng trnh bc hai
thnh c s cho ton b hnh hc ca h v c th lm vic trong tp hp s thc,h thay
th cc tnh ton ca ngi Babilon bng cc php dng hnh bng thc thng v compa.Tuy
vy h ch tnh ton tp hp s hu t dng,cho nn c nhiu phng trnh bc hai h khng
gii c.Phi ch n th k XVI,khi xut hin s phc th mi gii c tt c cc phng
trnh bc hai.
Trung Hoa c i,cch gii phng trnh bc hai cng c trnh by trong b sch Sch
ton chn chng,trong cun Trng Khu Kin ton kinh v trong cun S th cu chng.
Ngi Hindu tha nhn mt phng trnh bc hai c li gii thc th c hai nghim hnh
thc.H thng nht php gii i s cc phng trnh bc hai bng phng php b sung bnh
phng quen thuc,do vy ngy nay phng php ny c gi l phng php Hindu.
Nh thi c i,ngi Babilon,ngi Ai Cp,ngi Hy Lp,ngi trung Hoa,... bit cch
gii phng trnh bc hai nhng cng thc nghim th mi n nm 825 nh ton hc Al-
Khowarizmi mi lp c.ng gii phng trnh bc hai bng i s v hnh hc.ng vit:
x
2
+px +q = 0 thnh

x +
p
2

2
=
p
2
4
q
T ng tm c nghim:
x =
p
2

p
2
4
q
366
367
V sau ng li gii theo cch khc,ng t z = x +
p
2
v thay vo phng trnh u c
z
2

p
2
4
q

= 0
Vic gii phng trnh ny tr thnh n gin.
Trong chng trnh i s lp 9 c cng thc tnh nghim ca phng trnh bc hai dng
chnh tc ax
2
+bx+c = 0 bng cch t = b
2
4ac Nu < 0 th phng trnh v nghim.
Nu = 0 th phng trnh c nghim kp:
x
1
= x
2
=
b
2a
Nu > 0 th phng trnh c hai nghim phn bit:

x
1
=
b +

2a
x
2
=
b

2a
Francois Viete (1540-1603) ngi Php a ra h thc ca hai nghim ny:

x
1
+ x
2
=
b
a
x
1
x
2
=
c
a
H thc ny v sau c mang tn ng (h thc Viete).
Francois Viete (1540-1603)
Ngoi ra cn c cc cch gii phng trnh bc hai bng hnh hc sau y: cch ca Sir
John Leslie (1766-1832). ngi Anh trong cun Cc c s ca hnh hc, cch ca Thomas
Carlyde (1795-1881) ngi Anh, hc tr ca Leslie,...
Phng trnh v nh bc hai 2 n y
2
= ax
2
+ 1 trong a l s nguyn dng khng chnh
phng c Brahma Gupta v Bhaskara ngi n gii. Ngi ta gi l phng trnh Pell.
368
L thuyt y v phng trnh c Joshep-Louis Lagrange (1736-1813), ngi Php hon
tt vo 1766-1769.
Cuc thch chn ng th gii ton hc
Nh chng ta bit, vic tm li gii cho phng trnh bc 3 l kh phc tp v cng thc
nghim l kh cng knh.
Trong mt bn Babilon tm thy c cc gi tr ca n
3
+ n
2
vi n = 1 n n = 30 v nh vy
chng ta cng tm c nghim ca 30 phng trnh bc 3 c bit. Nh ton hc Neugebauer
tin rng ngi Babilon hon ton c th quy mt phng trnh bc 3 tng qut v dng chun
x
3
+x
2
= c.
Trung Hoa, b sch Chc c ton kinh ca Vng Hin Chng vit vo u thi ng
(th k VII) c nu cch gii phng trnh bc 3 tng qut bng i s. Trong b sch S th
cu chng, Tn Cu Thiu cng a ra phng trnh bc 3. Nhng sau , ngi Trung Hoa
qu ch n phng php i s trn bn tnh, m t ch s dng cc k hiu i s
nh ang dng, do vy t th k VII nn ton hc Trung Hoa bt u lc hu so vi Phng Ty.
Omar Khayyam (1048-1122) Khorasan gii phng trnh bc 3 bng hnh hc mt cch
c o.
Sau Al-Khowarizmi c nhiu nh ton hc i tm cch gii phng trnh bc 3 mt cch
mit mi. Nhng tri qua sut 7 th k, ngoi vic tm c cch gii ca nhng phng trnh
khc th khng c bc tin no cho vic gii phng trnh bc 3. V vy c ngi t ra
chn nn, cho rng phng trnh bc 3 c th khng c thc. Nhng gio s Scippionel del
Ferro (1465-1526) ngi Italia, Trng i hc Tng hp Bologna th khng nh vy. ng vn
tip tc con ng ca mnh l tm mit mi nghin cu vn hc ba lc by gi l gii
phng trnh bc 3. Tri khng ph ng, vo mt ngy khng th tin c, ng tm ra
bc t ph v n nm 1505, ng tuyn b tm ra cch gii c bit cho phng trnh:
x
3
+mx = n (vi m, n > 0)
Vo thi , mi ngi u gi b mt cch gii ca mnh, v vy vic Ferro gi kn bo bi
cng l iu khng c g l. Nhng ng tic l, ng khng c dp no cng b thnh
tu ca mnh. Mi n khi sp qua i, ng mi li b mt ny cho ngi con r tin cn l
Anabel Nova. Nhng v sau, mt mn sinh ca Ferro l Antonio Fior ly cp bo bi ca ng.
Trong trng phi Italia th Fontana (1500-1557) cng i tin phong trong vic tm cch gii
phng trnh bc 3.
369
Niccolo Tartaglia (1499-1557)
By gi li ni n mt nh ton hc khc, l Niccolo Tartaglia (1499-1557). Thi th
u ca ng tri qua tht nng n. ng sinh ra Brescia min bc Italia nn c gi l Niccolo
ca Brescia v thng c gi l Tartagli v lc nh ng b qun Php lm hi d man. Lc
ng mi 13 tui th qun Php trn vo Brescia. ng cng vi ngi cha (lc l ngi a
th) v dn chng chy trn vo ngi nh th n nhng qun Php rt theo v thm ha
xy ra: ngi cha b git cht cn cu b b chm vo hm v ming. Vt thng vm
ming lm ng ni nng rt kh khn nhng ng c t cht thng minh li ham hc. Sau khi
m ng qua i, ng phi t i tm ng sng cho mnh. ng hc vt l, ton hc v t r ti
nng rt sm, c nhiu ngi thi knh phc.
Vo nm 1530 mt nh ton hc a ra cho ng hai cu hi mang tnh thch thc, nhm
h uy tn ca ng:
1. Tm mt s m lp phng cng vi 3 ln bnh phng th bng 5.
2. Tm ba s m trong s th hai ln hn s th nht l 2, s th ba ln hn s th hai l
2 v tch ca chng l 1000.
y thc cht l tm nghim ca phng trnh bc 3, vi phng trnh cu hi 1 l:
x
3
+ 3x
2
5 = 0 v cu hi 2 l: x
3
+ 6x
2
+ 8x 1000 = 0 ng tm c nghim ca
c hai phng trnh ny nn ng ngy cng ni ting.
Cc mn sinh ca Ferro cng tuyn b gii c phng trnh bc 3. Khng ai chu ai nn
cui cng cc nh ton hc Italia quyt nh m cuc thch u gia hai bn, mi bn a ra
30 bi ton lm trong 2 gi.
Sp n ngy thi, Tartaglia cm thy nao nng v ng l ngi t hc, s khng bo v c
cch gii ca mnh. ng suy ngh rt nhiu, a ra nhiu phng n khc nhau v trc ngy
thi 8 ngy, ng tim ra phng php mi gii phng trnh bc 3.
ng hc thuc cch gii mi v ngh ra 30 bi ton m ch c cch gii ny mi thc hin c.
Vo ngy 22/2/1535, cc nh ton hc ko v thnh ph Milan d cuc so ti. Ba chc bi
ton m mi bn a ra u l phng trnh bc 3. Tartaglia gii 30 bi ton m i phng
a ra trc gi quy nh, trong khi nhm mn sinh Ferro khng gii c bi no trong s
30 ton ca Tartaglia. Nh vy cuc thi kt thc v phn thng tuyt i thuc v Tartaglia.
370
Girolamo Cardano (1501-1576)
Tin tc c truyn i lm chn ng c gii ton hc. thnh ph Milan c mt ngi ng
ngi khng yn, l Girolamo Cardano (1501-1576). ng khng ch l thy thuc ni ting
khp chu u, m cn l mt nh ton hc ti ba, dy ton v c nhiu cng trnh nghin cu
v ton hc. ng chuyn tm nghin cu phng trnh bc 3 nhng cha c kt qu. Cho nn
khi nghe tin Tartaglia gii c phng trnh bc 3, ng hy vng s c hng mt phn
thnh tu ca Tataglia.
Lc ny Tartaglia ni ting khp chu u nhng ng li khng mun cng b rng ri cng
trnh ca mnh. ng ch vit li trong tc phm Nguyn tc hnh hc, cho nn trong t sch
ca nhiu ngi kh lng c c tc phm ca Tartaglia.
Vi thi chn tnh, hiu hc, sau nhiu ln Cardano ngh, nm 1539 Tartaglia ng
truyn li nhng b quyt cho Cardano. Nhng Cardano khng tn trng li ha, nm 1545
gii thiu cch gii phng trnh bc 3 vi li gii thch ca mnh trong cun Ars magna.
Cardano vit:
Khong 30 nm trc, Ferro tm c phng php gii ny, truyn li cho ngi khc
v tng tranh lun vi Tartaglia, Tartaglia cng pht hin c phng php ny. Ti
nhiu ln ngh thit tha v cui cng Tartaglia truyn t cho ti phng php gii
nhng li khng ch cho ti phng php chng minh, v vy buc ti phi tm ra nhiu cch
chng minh. V n rt kh, ti xin m t n nh sau:. . .
Sau cuc tranh ci v bn quyn cch gii ny n ra gay gt nhng cui cng l phi
chin thng v ngi ta cng nhn li gii l ca Tartaglia. Tuy vy Cardano vn ni ting
nh cng b cch gii ny.
V sau Cardano cn a ra cch gii khc cho phng trnh x
3
+ mx = n vi m, n > 0 nh
sau: t x = u +v thay vo phng trnh c:
u
3
+v
3
+ (3uv +m)(u + v) n = 0
Vic gii phng trnh trn tng ng vic gii h phng trnh:

3uv +m = 0
u
3
+v
3
n = 0
371
hoc

2
n
m
3
27
= 0
Trong mt cun sch ca Trung Quc li ni rng, sau 2 nm Cardano cng b cch gii
phng trnh bc 3 th trong bi Nhng cu hi v pht minh, Tartaglia ph phn thi
thiu trung thc ca Cardano v yu cu thnh ph Milan t chc tranh lun cng khai vi
Cardano. n ngy gp nhau tranh lun th khng phi Cardano m li l mt hc tr ti
ba ca Cardano, l Lodovico Ferrari (1522-1565). Ferrari khng nhng nm c cch gii
phng trnh bc 3 m cn gii c phng trnh bc 4 nn Tartaglia chu tht bi. T
Tartaglia nh b vt thng lng v m hn n lc cht.
Trong cun Lch s ton hc ca Howard Eves ngi M xut bn nm 1969 li vit: . . .
nhng li phn i mnh lit ca Tartaglia n tai Ferrari nn ngi ny lp lun cho thy
ca mnh rng, Cardano c c thng tin cn cho mnh t Ferro qua mt ngi th ba v ln
n Tartaglia l n cp t cng mt ngun . . .
Cho d cc li n thi nh th no nhng cui cng li gii c lu truyn n ngy nay
vi tn gi chung l cng thc Cardano-Tartaglia:
x =
3

n
2
+

n
2

2
+

m
3

n
2

n
2

2
+

m
3

3
Cng cn ni thm rng, cng thc trn l cng thc nghim ca phng trnh bc 3 cha y
. Tuy nhin, t phng trnh bc 3 y (chnh tc hay hon chnh):
ax
3
+bx
2
+cx +d = 0
n phng trnh x
3
+mx = n ch cn t:
y = x +
b
3a
Nm 1572, Bombelli cho cng b mt cun sch i s gp phn ng k vo vic gii phng
trnh bc 3. Trong cc sch gio khoa v l thuyt cc phng trnh cho bit rng, nu

n
2

2
+

m
3

3
l m th phng trnh x
3
+ mx = n c 3 nghim thc. Nhng trong trng
hp ny, cc nghim c biu th bng hiu ca hai cn bc 3 ca cc s phc lin hp.
iu tng chng bt thng ny gi l trng hp bt kh quy ca phng trnh bc 3
lm bn tm ng k cc nh i s hc thi xa. Bombelli ch ra tnh thc ca cc nghim
thot nhn l khng thc trong trng hp bt kh quy.
Trong cun Canon mathematicus seu ad triangula ca Viete xut bn nm 1579, tc gi c
gi mt cch gii bng lng gic cho trng hp bt kh quy ca cc phng trnh bc 3.
Trong lun vn ca Viete thy c cch gii rt p sau y cho phng trnh bc 3:
x
3
+ 3ax = 2b
Phng trnh trn l mt dng m phng trnh bc 3 no cng c th quy v c. Nu t:
x =
a
y
y
372
th phng trnh trn tr thnh:
y
6
+ 2by
3
= a
3
y l phng trnh bc 2 ca y
3
, ta c th tm ra y ri sau l x.
V phng trnh x
3
+ 3ax = 2b th Isaac Newton (1642-1727) cng c cch gii.
Nhng vinh quang sau khi qua i
chu u n th k XVI, khoa hc t nhin pht trin rt nhanh chng. Truyn thuyt
tn gio cho rng,Thng sinh ra th gii v Tri t c hnh vung. Nhng pht kin a
l ca Christophe Columbus (1451-1506), Fernand de Magellan (1480-1521) v nhng ngi
khc chng minh y rng Tri t c hnh cu, l iu khng th chi ci c.
Pht minh v a l ca Galileo em li cho nhn loi nhng nhn thc mi v v tr. Ton
hc c suy tn l n hong ca khoa hc t nhin. T th k XVI n th k XVIII xut
hin hng lot cc nh ton hc kit xut, h a nn ton hc ln mt nh cao mi. S
xut hin phng php ta , ng dng ca s phc, sng to ra vi-tch phn,... kt hp
nghin cu th gii khch quan trong trng thi tnh v ng.Vo thi gian , c chu u
gn nh b li ng sau s tr tr ca thi Trung C. Trong tin trnh pht trin ,ton
hc lun i tin phong.
Nhng nhnh phng trnh i s th tnh hnh li khng hon ton nh vy. Do cuc thch
chn ng c th gii ton hc vo u th k XVI nn ngi ta tm c cch gii
phng trnh bc ba, bc bn. T gia th k XVI tr i, ngi ta bt u i su nghin cu
phng trnh bc nm. Cc nh ton hc phn tch t m cch gii phng trnh t bc hai
n bc bn. Nu mt phng trnh c nghim vit c bng cng thc i s ca cc h s
th c gi l phng trnh gii c bng cn thc. Trc thi Bombelli v Viete ngi ta
xc nh c cng thc tng qut tnh nghim ca phng trnh t bc 1 n bc bn.
Khng bao lu, sau khi gii c phng trnh bc ba th phng trnh bc bn tng qut
cng c cch gii i s. Vic gii phng trnh bc bn tng qut quy v vic gii mt phng
trnh bc ba lin kt.
Nm 1540, Zuanne de Tonini da Coi ngi Italia ngh Cardano gii bi ton dn n
phng trnh bc bn nhng Cardano khng gii c, m hc tr l Ferrari li gii c v
sau Cardano cng b cch gii ny trong cun Ars magna ca ng.
Cch gii ca Ferrari vit gn theo cch k hiu hin nay nh sau: bin i n gin s quy
mt phng trnh bc bn chnh tc (y ) v dng:
x
4
+ px
2
+qx +r = 0
T phng trnh trn, bin i c
x
4
+ 2px
2
+p
2
= px
2
qx r + p
2
hoc
(x
2
+ p)
2
= px
2
qx r + p
2
373
Vi y bt k, t phng trnh trn ta c:
(x
2
+ p +y)
2
= px
2
qx r +p
2
+ 2y(x
2
+p) + y
2
= (p + 2y)x
2
qx + (p
2
r + 2py + y
2
)
By gi ta chn y v phi ca phng trnh trn l mt bnh phng.
y l trng hp khi: 4(p + 2y)(p r 2py + y) q = 0
y l phng trnh bc ba ca y nn tm c cch gii. Mt gi tr ca y nh vy s quy
phng trnh bc bn lc u v vic ch phi ly cc cn bc hai.
Mt cch khc bng i s c Viete xut v mt cch na do Descartes a ra nm 1637
nhng trong nhiu sch gio khoa i hc c th cch gii ca Viete cng ging nh cch
gii ca Ferrari.
Nh ton hc Vanmec ngi Ty Ban Nha cng gii c phng trnh bc bn nhng tn
bo cha Tuocmacvada thiu cht ng v theo hn, ng lm tri Tri: phng trnh
bc bn khng hp vi kh nng ca ngi trn tc- l mun ca Tri.
V vic gii phng trnh bc bn tng qut c thc hin ty thuc vo vic gii phng
trnh bc ba lin kt, nn nm 1750 Euler c gng lm iu tng t l quy vic gii phng
trnh bc nm tng qut v php gii phng trnh bc bn lin kt nhng tht bi.
Rt nhiu nh ton hc cng vt c tm kim cch gii phng trnh bc nm nhng khng
i n kt qu. Trc tnh hnh , ngi ta bt u hoi nghi l liu c tn ti hay khng
mt cng thc tnh nghim ca phng trnh bc nm tng qut.
Nm 1778, nh ton hc Lagrange m c mt u mi quan trng. ng tp trung
tm kim cng thc chung gii cc phng trnh t bc hai n bc bn, v ng cho rng,
nu tm c cng thc chung th s suy lun tm c cch gii phng trnh bc nm.
Nhng cui cng ng pht hin thy nghim ca mt phng trnh bit c th c biu
th bng mt hm s i xng ca mt phng trnh h tr khc. Phng trnh h tr ny
c ng gi l cch gii d kin. Dng cch gii d kin ny, ng s dng gii phng
trnh bc ba, bc bn nhng khi n phng trnh bc nm th ng nh chu. Mt tia sng
li le trong u ng: cng thc nh vy l khng th tn ti c, nhng ng li khng th
chng minh iu ny.
Joshep-Louis Lagrange (1736-1813)
374
Phng php tng qut bin i mt phng trnh a thc bc n i vi x thnh mt phng
trnh bc n i vi y, trong cc h s ca y
n1
, y
n2
u bng 0 c Tschirnhausan (1651-
1708) a ra. V sau php bin i phng trnh bc nm nh vy, trong cc h s ca
y
4
, y
3
, y
2
u bng 0 c Brings (1736-1798) a ra nm 1786. iu ny ng vai tr quan
trng trong vic gii phng trnh bc nm bng hm eliptic. Nm 1834, Jerrard (mt 1863)
li chng minh c h s ca y
n3
cng bng 0.
Trong cc nm 1803, 1805, 1813, nh vt l Paolo Ruffini (1765-1822) ngi italia a ra
mt cch chng minh v mt iu m lc by gi c coi l mt s kin, l nghim ca
phng trnh tng qut bc nm hoc bc cao hn u khng th biu th c bng cc cn
thc theo cc h s ca phng trnh .
Loi ngi ng trc s thch thc v tr tu ht i ny n i khc, cho n khi xut
nhin nh ton hc tr tui Niels Henrik Abel (1802-1829) ngi Na Uy. ng l ngi rt can
m. Ngay t thu nh, ng bt u tm li gii cho phng trnh bc nm. Nh quyt tm
chin thng n cng nn vo nm 1824 (lc ng 22 tui), ng chng minh c mt cch
tng qut rng, phng trnh bc t 5 tr ln l khng th c cch gii kiu cn thc. Vi tr
tu ca tui tr, ng tuyn b vi th gii mt cch chn l: tr tu ca con ngi l bt
kh chin thng.
Nhng cn ng thnh cng ca Abel ht sc gp gnh. Tuy c nhng thnh cng trong thi
gian ngn ngi nhng nhng g l au kh cn nng n hn. Nhng vinh quang ca ng hu
ht n sau khi ng qua i.
ng sinh ra trong mt gia nh mc s nng thn. Cha m ca ng li ng con v ngho.
Trong s 7 anh ch em, ng l ngi anh th 2. Nm 13 tui, ng c a vo trong trng
dng. Ngay t u ng rt hng th vi ton hc. Nm 1817, trong trng xy ra mt
s kin c bit, trong mt m thay i c cuc i ng. Thy dy ton ca ng, do ngc
i hc tr nn b sa thi. Thay thy gio c l thy Homlboe (sinh nm 1795) mi 22 tui.
Thy Holmboe nhanh chng pht hin ti nng ton hc ca Abel. Khi Abel nu ra quyt
tm tn cng vo phng trnh bc nm, nhiu ngi ch diu, cho rng ch nhi lm sao
li i n tht thin nga. Vy m Holmboe li rt ng h Abel, ng vin ng c gng vn ln.
Niels Henrik Abel (1802-1829)
Nm 1821, Abel thi u vo trng i hc. thc hin tng ca mnh, ng theo hc
thy Gauss. Ban u ng hc theo cch ca nhng ngi i trc, i tm cch gii p mt
cch chnh din. Sau nhiu ngy thng mit mi, nm 1824 ng chng minh l phng
trnh tng qut bc nm khng th c cng thc nghim bng cn thc. iu lm bn tm
buc loi ngi phi suy ngh sut 2 th k, cui cng c mt thanh nin khng c
375
ting tm gii quyt. Tuy nhin, iu Abel nu ra khng c cc tp ch ton hc ng
ti, buc ng phi t b tin ra in n. Nhng nhng au kh ca ng khng v th m gim bt.
Nm 1825, Abel n nhiu nc chu u v g ca nhiu ni nhng khng u coi ng ra
g c, k c nhng ni c gi l vng quc ton hc. Cui cng ng n Berlin. Rt may
mn, y ng c k s Klaye hiu c tng ca ng. Tuy Klaye khng hiu ht ni
dung m Abel trnh by nhng Klaye li hiu c nng lc to ln ca Abel. Nm 1826 Klaye
gip Abel cho ra i tp ch L lun v ton hc. Ba s u ca tp ch ng 22 bi
pht biu ca Abel, gii thiu cc cng trnh nghin cu ton hc ca Abel. Nhng thnh tu
xut sc ca Abel dn dn thu ht s ch ca gii ton hc chu u. Chnh v vy m tp
ch ny ni ting cho n tn ngy nay.
Thng 5/1827, vi tm lng thng nh qu hng T quc, Abel tr v th Otslo. Tuy
nhin, qu hng th ng li khng tm c cng vic g thch hp. Thng 9/1828 bn vin
s hn lm khoa hc Php yu cu vua Sal XIV gip vt cht, to iu kin Abel
nghin cu khoa hc. Nhng do vt v qu m bnh lao ca ng li ti pht, e da n
tnh mng ca ng. Ngy 6/4/1829, ngi sao sng rc trn bu tri tt ln.
Ngy 9/4/1829, ngi thn trong gia nh nhn c mt bc th t Berlin gi n vi ni
dung:
ng Abel knh mn!
Trng chng ti quyt nh tn vinh ng l gio s ton hc ca trng.
Chc mng vinh d ca ng!
Nhng bc th ny n chm, ng qua i trc 3 ngy.
Ngy 28/6/1830, Vin hn lm khoa hc Php trao gii thng ln cho Abel.
l nhng vinh quang sau khi ng qua i.
Li c cu chuyn v vic Abel gii phng trnh c bc rt ln nh sau: v i s H Lan c
ln ni dc vi vua Henry IV rng, nc Php khng c nh ton hc no c th gii c
bi ton do Adrianus Romanus (1561-1615) ngi H lan t ra nm 1593, v bi ton ny i
hi phi bit cch gii mt phng trnh bc 45. Viete c triu n. Sau khi xem xt bi
ton, ng nhn ra mi lin h lng gic c bit v a ra 2 nghim, sau thm 21 nghim
na.
376
tiu s mt s nh ton hc ni ting
Mt cuc i trn bia m
Ngi ta khng bit g nhiu v cuc i ca nh ton hc c Hy Lp l i--phng (Diophante):
ng sng th k III trc Cng nguyn, sinh A-lch-xng-ri, tm nghin cu su v
phng trnh v tm tt cuc i mnh bng nhng hng ch trn bia m nh sau:
Hi du khch! Ni y yn ngh mt ngi tn l i--phng. V nhng con s nhim mu
c th s ni cho bn bit v nhng thng ngy di ca i ng. ng sng th ngy trong
mt phn su cuc i. Mt phn mi hai cuc i na, cm ng ln phn ru. Thm mt
phn by cuc i, ng mang nhn ci trn tay v 5 nm sau, c mt a con trai xinh
xn. Than i, d rt c thng yu, ngi con ny cht khi anh ta va bng na tui th
ca cha. Qu au kh, ngi cha bt hnh ch sng thm bn nm sau ci cht ca con.
Bn hy ni i: ng ta th bao nhiu tui v cuc i ng ra sao?.
Theo ngn ng phng trnh th khi t tui th ca i--phng l n s x, phin dch tng
cu trong li trn bia m, ta s c phng trnh sau:
x
6
+
x
12
+
x
7
+ 5 +
x
2
+ 4 = x
Gii phng trnh ny, bn d dng tm c x = 84. Vy i--phng sng 84 nm. Thi
nin thiu ca ng (84 : 6) l 14 nm, V 84 : 12 = 7 nn ng c ru lc 14 + 7 = 21 (tui).
Li do 84 : 7 = 12 nn ng ly v lc 21 + 12 =33 (tui). ng c con trai lc 38 (tui). V
ngi con ch sng bng na tui th ca cha nn anh ta cht lc 42 tui. Khi , ngi cha
38 + 42 = 80(tui). V ng nhm mt la i lc 80 + 4 = 84 (tui).
Tht l mt tm bia c o!
Ch v l sch qu hp!
Ngay t thi c ngi ta bit tam gic c ba cnh 3, 4, 5 (n v di) l tam gic vung v
3
2
+ 4
2
= 5
2
ngoi ra, c v s s nguyn dng khc tha mn phng trnh x
2
+y
2
= z
2
.
Chng hn cc b ba s 6; 8; 10 hay 5; 12; 13. . . Ta u bit y l cc s Pi-ta-go. T phng
trnh ny, ny ra ngh: liu c tm c cc b ba s nguyn x, y, z tha mn phng
trnh x
n
+ y
n
= z
n
(n l s t nhin ln hn 2) hay khng. ngh ngy ln u tin c
i--phng xem xt k, v vy phng trnh c tn gi l phng trnh i--phng.
Ngi ta chng minh c l vi tt c cc s n t 3, 4. . . n 4000, chng c cc s nguyn
x, y, z no nghim ng phng trnh i--phng c. Ring Pi-e Phc-ma (Pierre Fermat, 1601-
1665) mt nh ton hc Php, ngi tng cng vi Pa-xcan tm ra php tnh xc sut,
mnh dn nghi ng: Phng trnh i--phng s lun lun v nghim, d n l bt c s t
nhin no.
Nm 1637, Phc-ma vit bn l cung sch ca i--phng rng ng tm c cch chng
minh rt k diu iu nghi ng ni trn ca mnh l ng, nhng khng ghi ra c ch v. . .
l cun sch qu hp.
T n nay, hn ba trm nm tri qua, nhiu nh ton hc tn bao cng sc ri vn
chu b tay trong vic i tm chng minh ca Phc-ma.
377
(Pierre Fermat (1601-1665)
iu nghi ng ni trn tr thnh mt bi ton lch s ni ting v phng trnh i--phng
v c tn gi bi ton Phc-ma. Nm 1993, nh ton hc ng-ry Oai (Andrew Wiles) ngi
Anh chng minh c Bi ton Phc-ma vi cc cng c ton hc hin i v vi hn . . .
200 trang giy.
Hai gng mt tr
Lch s ca i s trong nhng trang nghin cu v phng trnh u gi mi bng dng ca
hai chng trai tr thin ti: Ni-en hen-rch A-ben (Niels Henrik Abel, 1802-1829) nh ton hc
Na Uy v E-va-rt Ga-loa (Evariste Galois, 1811-1832) nh ton hc Php. Cng sng hi u
th k XIX, cuc i ngn ngi ca c hai ngi cng li nhng tc phm v gi cho ton
hc hin i.
(Evariste Galois (1811-1832)
C A-ben ln Ga-loa u bc l ti nng v ton ngay t lc cn ngi trn gh nh trng,
c hai cng quan tm n vic gii cc phng trnh i s c bc ln hn hai. Tuy khng h
bit n vic lm ca nhau, c hai cng gi cng trnh ca mnh n Vin hn lm khoa hoc
Php, v. . . cng gp ri ro: Cng trnh ca A-ben b ct vo t lu tr, mi lc anh cht
mi c ngi v tnh c v in ra, cn Ga-loa ba ln kin nhn gi ti mi n ln th ba mi
c nhn tr li cng trnh ca mnh vi li ph Khng th hiu c!. A-ben cht m thm
trong ngho tng, nhng li tn tui trong cc phng trnh A-ben v nhm A-ben ca
l thuyt nhm. Cn Ga-loa cht trong cuc quyt u v danh d vi bn khiu khch, li
60 trang th anh vit trong m cui cng, trong trnh by ngn gn mi kt qu quan
trng nht anh tm ra khi nghin cu phng trnh. L th di ny c vit bng nt ch
v cng hi h, xen gia nhng cng thc ton l nhng cu Ti vi qu. Ch cn my ting
378
ng h na thi, Ti khng kp. . .
Vy m 60 trang th m ra cho cc nh ton hoc th gii v s hng i, v li mt
l thuyt v phng trnh i s mt n s mang tn l thuyt Ga-loa.
Bn hy nhn li nm sinh v nm mt ca A-ben v Ga-loa: hai gng mt thin ti lun
ng cnh cnh ca m vo lu i ton hc hin i, mi mi tr trung.
Sng hay cht
Vo nm 1927, Erwin Schrodinger vit ra mt phng trnh cho nhng sng lng t. N
ph hp vi nhng th nghim mt cch tuyt vi ng thi v nn mt bc tranh ca mt th
gii rt khc l, trong nhng ht s cp nh electron khng phi l vt th r rng, m l
nhng m my xc sut. Spin ca electron ging nh mt ng tin c th na sp na nga
cho n khi n ri xung bn. Khng lu sau , cc nh l thuyt li lo lng khng yn trc
mi tnh cht l lng t, v d nhng con mo va sng va cht, v nhng v tr song song
trong Adolf Hitler l k chin thng cuc chin tranh th gii ln th hai.
C hc lng t khng b rng buc vi nhng b n trit l nh vy. Hu nh mi vt dng
hin i my vi tnh, in thoi di ng, my chi game, xe hi, t lnh, l vi sng u
cha nhng con chip nh gc transistor, dng c c s hot ng da trn c hc lng t
ca cht bn dn. Nhng cng dng mi cho c hc lng t xut hin gn nh hng tun.
Cc chm lng t - nhng ming nh xu ca mt cht bn dn c th pht ra nh sng
thuc mi mu sc v c s dng ghi nh sinh hc, trong chng thay th cho nhng
cht nhum truyn thng, thng l c hi. Cc k s v nh vt l ang c gng pht minh
ra my vi tnh lng t, mt dng c c th thc hin song song nhiu php tnh khc nhau,
ging ht nh con mo va sng va cht.
Laser l mt ng dng na ca c hc lng t. Chng ta s dng chng c thng tin t
nhng l nh li ti trn a CD, DVD v a Blu-ray. Cc nh thin vn s dng laser o
khong cch t Tri t n mt trng. Thm ch c th phng nhng tn la v tr ln t
Tri t vi sc y l mt chm laser mnh.
Chng cui trong cu chuyn ny c xut x t mt phng trnh gip chng ta hiu ngha
ca sng. N bt u vo nm 1807, khi Joseph Fourier ngh ra mt phng trnh cho dng
nhit. ng gi mt bi bo v ni v n n Vin hn lm Khoa hc Php nhng b t chi.
Vo nm 1812, vin hn lm Php a vn nhit thnh ti ca gii thng hng nm
ca vin. Fourier li gi mt bi bo di hn, c hiu chnh v git gii.
Ci hp dn nht ca bi bo ginh gii thng ca Fourier khng phi l phng trnh, m
l cch ng gii n. Mt bi ton in hnh l tm xem nhit dc theo mt thanh mng
thay i nh th no theo thi gian, cho bit trc c im nhit ban u. Fourier c th
gii phng trnh ny mt cch nh nhng nu nh nhit bin thin nh mt sng hnh sin
dc theo chiu di thanh. V th, ng biu din mt c trng phc tp hn l s kt hp ca
nhng ng hnh sin vi bc sng khc nhau, gii phng trnh cho mi ng cong hnh sin
thnh phn, v cng tt c nhng nghim ny li vi nhau. Fourier khng nh phng php
ny ng cho mi c trng nhit bt k, thm ch ng c vi trng hp trong nhit
379
c gi tr nhy cc. Tt c nhng g phi lm l cng gp mt s v hn nhng ng gp
t nhng ng cong hnh sin vi tn s ln dn.
Joseph Fourier (1768-1830)
Du vy, bi bo mi ca Fourier b ch trch l khng cht ch, v mt ln na vin
hn lm Php t chi ng ti. Vo nm 1822, Fourier pht l mi phn i v cho cng b
l thuyt ca ng di dng mt quyn sch. Hai nm sau , ng t b nhim mnh lm th
k ca vin hn lm, d mi ca ng vo nhng k ch trch ng, v cho ng bi bo gc ca
ng trong tp san ca vin. Tuy nhin, nhng ngi ch trch cha chu dng li. Cc nh ton
hc bt u nhn ra rng nhng chui v hn l nhng th nguy him; chng khng lun lun
hnh x ging nh nhng tng hu hn, p . Vic gii quyt nhng vn ny ha ra l
ht sc kh khn, nhng phn quyt cui cng l quan im ca Fourier c th c lm cho
cht ch bng cch ngoi suy ra nhng c trng rt khng u. Kt qu l php bin i
Fourier, mt phng trnh xem mt tn hiu bin thin theo thi gian l tng ca mt chui
nhng ng cong hnh sin thnh phn v tnh ra bin v tn s ca chng.
Ngy nay, php bin i Fourier nh hng n cuc sng ca chng ta theo v s kiu. Chng
hn, chng ta c th s dng n phn tch tn hiu dao ng to ra bi mt trn ng t
v tnh ra nhng tn s m nng lng truyn bi mt t chn ng l ln nht. Mt bc
tin ti xy dng nhng cng trnh chu c ng t l m bo rng tn s ring ca cng
trnh khc vi tn s ca ng t.
Nhng ng dng khc bao gm vic loi tp m ra khi nhng bn ghi m c, tm kim cu
trc ca ADN bng nh chp tia X, ci thin s thu nhn v tuyn v ngn cn nhng dao
ng khng mong mun xe hi. Thm mt ng dng na m a s mi ngi chng ta s
dng thng xuyn m khng , l chp nh k thut s.
Nu bn tnh xem cn bao nhiu thng tin biu din mu sc v sng ca mi pixel trong
mt bc nh k thut s, bn s pht hin ra rng mt chic camera k thut s nhi nht vo
th nh ca n lng d liu nhiu gp mi ln ci th nh c th cha. Camera lm cng
vic ny bng cch s dng s nn d liu JPEG gm nm bc nn khc nhau. Mt trong s
chng l mt phin bn k thut s ca php bin i Fourier, n hot ng vi mt tn hiu
khng thay i theo thi gian m thay i t u ny qua u kia bc nh. C s ton hc
hu nh l ging ht. Bn bc cn li tip tc lm gim d liu thm na, n khong bng
mt phn mi lng ban u.
y mi ch l by trong nhiu phng trnh m chng ta bt gp hng ngy, nhng khng
nhn ra chng ang hin din y. Nhng s tc ng ca nhng phng trnh ny i vi
lch s th su sc hn nhiu. Mt phng trnh tht s mang tnh cch mng c th c s tc
ng i vi s tn ti ca loi ngi ln hn c mi nh vua v hong hu c mu chon
380
y nhng quyn s hc ca chng ta.
C (hoc c th c) mt phng trnh, trn ht thy, m cc nh vt l v nh v tr hc t
ht nim tin yu vo y: mt l thuyt ca tt c thng nht c hc lng t v thuyt tng
i. Ni ting nht trong s nhiu ng c vin l l thuyt siu dy. Nhng nh mi ngi chng
ta u bit, cc phng trnh ca chng ta cho th gii vt cht c l ch l nhng phin bn
n gin ha khng bt gi c cu trc su sc ca thc ti. Ngay c nu t nhin c tun
theo nhng nh lut vn vt, th chng c th khng c biu din di dng nhng phng
trnh.
Mt s nh khoa hc ngh rng n lc chng ta t b nhng phng trnh truyn thng
theo ui nhng thut ton nhng cng thc khi qut hn tnh ton mi th, k c
vic ra quyt nh. Nhng cho n nhng ngy y, nu c, s hiu bit su sc nht ca chng
ta v cc nh lut ca t nhin s tip tc c dng thc nhng phng trnh, v chng ta s
hc cch tm hiu chng v thch ng vi chng. Cc phng trnh c thnh tu ca chng,
Chng tht s lm bin chuyn th gii v chng s li tip tc lm th gii bin chuyn.
TAI LIU THAM KHO
1. Nguyn Vn Mu ; Nguyn Vn Tin
Mt s chuyn i s bi dng hc sinh gii. NXB Gio dc 2010.
2. Thanh Sn
Mt s chuyn hnh hc phng bi dng hc sinh gii. NXB Gio dc 2010.
3. Nguyn Vn Mu ; Trn Nam Dng ; Nguyn ng Pht ; Nguyn Thu Thanh
Chuyn chn lc: S phc v p dng. NXB Gio dc 2009
4. V Dng Thy ; Nguyn Vn Nho
40 nm Olympic Ton hc quc t. NXB Gio dc 2006.
5. on Qunh ; Don Minh Cng ; Trn Nam Dng ; ng Hng Thng
Ti liu chuyn ton i s 10. NXB Gio dc 2010.
6. Nguyn Vn Mu
Phng php gii phng trnh v bt phng trnh. NXB Gio dc 2010.
7. Thanh Sn
Mt s chuyn hnh hc khng gian bi dng hc sinh gii. NXB Gio dc 2010.
8. V Hu Bnh
Nng cao v pht trin ton 9. NXB Gio dc 2006.
9. Tp ch Ton hc v tui tr.
10. Tuyn tp 10 nm thi Olympic 30/4. NXB Gio dc 2006.
11. Cc bi thi Olympic Ton THPT Vit Nam. NXB Gio dc 2007.
12. Cc ti liu trn Internet:
Nguyn Tt Thu
Chuyn phng trnh- bt phng trnh, t n ph a v h phng trnh
Nguyn Thnh Vn ; Nguyn Phi Hng
Phng php t n ph gii phng trnh v t
Trng H Minh Duy
Mt phng php gii phng trnh v t
L S Ging
H s bt nh trong a thc
Din n Math.vn
Tuyn tp 60 bi h phng trnh
381
382
Phm Kim Chung
Cc phng php gii h phng trnh
ng nh Sn
ng dng nh l Lagrange
13. Din n http://mathscope.org.
14. Din n http://onluyentoan.vn.
15. Din n http://www.artofproblemsolving.com/Forum.
16. Bch khoa ton th m Wikipedia.

You might also like